You are on page 1of 226

TOMO I

2 º
Guía didáctica del docente

FÍSICA
Nathalie Oyola Espinoza
MEDIO

EDICIÓN ESPECIAL PARA EL


MINISTERIO DE EDUCACIÓN
PROHIBIDA SU COMERCIALIZACIÓN
Guía didáctica del docente • TOMO I

FÍSICA 2 º
MEDIO

Nathalie Francisca Oyola Espinoza


Licenciada en Educación en Física y Matemática
Profesora de Estado en Física y Matemática
Universidad de Santiago de Chile
La Guía Didáctica del Docente de Física 2° Medio, es una creación del Departamento
de Estudios Pedagógicos de Ediciones SM, Chile

Dirección editorial Diseño de portada


Arlette Sandoval Espinoza Estudio SM
Coordinación editorial Ilustración de portada
María José Martínez Cornejo Estevan Silveira
Coordinación área Ciencias Naturales Diseño y diagramación
Andrea Tenreiro Bustamante Gabriela de la Fuente Garfias
Karina Riquelme Riquelme
Edición
Nathalie Oyola Espinoza Fotografía
Archivo fotográfico SM
Autoría Pixabay
Nathalie Oyola Espinoza Shutterstock
Asesoría pedagógica Jefatura de producción
Guadalupe Álvarez Pereira Andrea Carrasco Zavala
Corrección de estilo y pruebas
Cristian Oyarzo Barrientos
Coordinación de diseño
Gabriela de la Fuente Garfias

Esta guía corresponde al Segundo año de Educación Media y ha sido elaborado conforme al Decreto
Supremo N° 614/2013, del Ministerio de Educación de Chile.
©2018 – Ediciones SM Chile S.A. – Coyancura 2283 piso 2 – Providencia
ISBN: 978-956-363-307-8 / Depósito legal: 280717
Se terminó de imprimir esta edición de 3.553 ejemplares en el mes de noviembre del año 2018.
Impreso por A Impresores.
Quedan rigurosamente prohibidas, sin la autorización escrita de los titulares del “Copyright”, bajo
las sanciones establecidas en las leyes, la reproducción total o parcial de esta obra por cualquier
medio o procedimiento, comprendidos la reprografía y el tratamiento informático, y la distribución en
ejemplares de ella mediante alquiler o préstamo público.
Índice

Guía Didáctica del Docente


Tomo I Tomo II
Fundamentación general................................................4 Fundamentación general............................................110
Articulación de la propuesta...........................................6 Articulación de la propuesta....................................... 112
Fundamentación diseño instruccional..........................8 Visión global................................................................ 114
Estructura de las unidades de aprendizaje.................. 11 Unidad 3: Trabajo y energía...............................118
Elementos clave de la propuesta..................................14 Planificación de la unidad..........................................120
Visión global..................................................................18 Inicio de la unidad....................................................... 122
Reproducción del Texto del estudiante Desarrollo de la unidad...............................................124
Cierre de la unidad......................................................148
Unidad 1: Movimiento.........................................22
Profundización disciplinar.........................................150
Planificación de la unidad............................................24
Profundización didáctica............................................ 151
Inicio de la unidad.........................................................26
Ficha de refuerzo......................................................... 152
Desarrollo de la unidad.................................................28
Desafío complejo.........................................................158
Cierre de la unidad....................................................... 46
Evaluación de la unidad............................................. 160
Profundización disciplinar.......................................... 48
Solucionario.................................................................162
Profundización didáctica............................................. 49 3
Reproducción del Texto del estudiante
Ficha de refuerzo.......................................................... 50
Desafío complejo...........................................................56 Unidad 4: Universo…........................................168
Evaluación de la unidad................................................58 Planificación de la unidad..........................................170
Solucionario...................................................................62 Inicio de la unidad....................................................... 172
Reproducción del Texto del estudiante Desarrollo de la unidad............................................... 174
Cierre de la unidad......................................................186
Unidad 2: Fuerzas…….........................................70
Profundización disciplinar.........................................188
Planificación de la unidad............................................72
Profundización didáctica............................................189
Inicio de la unidad.........................................................74
Ficha de refuerzo........................................................ 190
Desarrollo de la unidad.................................................76
Desafío complejo.........................................................194
Cierre de la unidad....................................................... 86
Evaluación de la unidad..............................................196
Profundización disciplinar.......................................... 88
Solucionario.................................................................198
Profundización didáctica............................................. 89
Reproducción del Texto del estudiante
Ficha de refuerzo ......................................................... 90
Guía Didáctica del Docente
Desafío complejo.......................................................... 94 RDC……......………………………………………………..............203
Evaluación de la unidad............................................... 96 Anexos……………………………………………………..............204
Solucionario.................................................................. 98 Bibliografía……………………………………………….............209
Reproducción del Texto del estudiante Reproducción del Texto del estudiante

Bibliografía.................................................................. 104

Física • 2.º Medio


Fundamentación general
Cada segundo que vivimos es un momento nuevo y único en el universo, un momento
que nunca volverá a ser de nuevo. ¿Y qué enseñamos a nuestros niños? Les enseña-
mos que dos más dos son cuatro, y que París es la capital de Francia. ¿Cuándo les
enseñamos también lo que son? Debemos decir a cada uno de ellos: ¿sabes lo que
eres? Eres una maravilla. Eres único. En todos los años que han pasado, nunca ha
habido un niño como tú. Tus piernas, tus brazos, tus inteligentes dedos, la manera
en que te mueves… Puede ser que te conviertas en un Shakespeare, un Miguel Ángel
o un Beethoven. Tienes capacidad para todo. Sí, eres una maravilla1.
Pablo Casals

El texto escolar es un instrumento que busca apoyar la permiter al docente adecuarse a la realidad de su clase y
implementación curricular, a través de instancias de para las diversas formas de aprender de los estudiantes. Se
aprendizaje y orientaciones diseñadas para favorecer la labor compone de dos elementos: Textos del estudiante y Guía
del docente y enriquecer las interacciónes dentro de la sala Didáctica del Docente (GDD).
de clases.
En el modelo del Texto del estudiante y GDD, estos principios
Las Bases Curriculares, como elementos articulador de
se expresan en la manera en que sus partes se organizan e
la propuesta, reafirman en su concepción el sentido de
interactúan. Los mismos principios que sirven para organizar
contribuir al desarrollo completo e integral de los niños y
las Unidades, pueden verificarse en la organización de sus
niñas en sus dimensiones espiritual, ética, moral, afectiva,
distintas secciones. Por ejemplo, tanto las unidades como sus
intelectual, artística y física mediante el cultivo de valores,
divisiones se estructuran de modo que, al comienzo los y las
conocimientos y destrezas.
estudiantes exploran sus aprendizajes previos y proyectan
4 La estructura de esta propuesta refleja una concepción del el trabajo mediante la generación de metas y estrategias;
aprendizaje como un proceso activo, consciente, basado en durante los procesos, evalúan y reelaboran sus estrategias de
las experiencias y aprendizajes previos de los alumnos y las forma constante; al cierre, vuelven a evaluar, definen su nivel
alumnas, y desencadenado por la motivación. Para el logro de desarrollo y proyectan sus aprendizajes futuros.
de estos objetivos la propuesta pedagógica considera recursos
que se relacionan con los intereses de los y las estudiantes
y que proponen instancias de aprendizaje diversas para

Texto escolar

Componentes y su
relación
Guía didáctica

Modelo de aprendizaje
Estilo de aprendizaje
Bases curriculares
Guía Didáctica del Docente

Evaluación para el aprendizaje


Bases sobre las cua-
Modelo de persona les se construye la
Realidad nacional
propuesta
Contexto cultural
1 Educación democrática. En https://
Etapa desarrollo biológico
educaciondemocratica.wordpress.
com/2013/01/14 hacer-el-mundo- niños y niñas
digno-de-sus-hijos-pau-casals/

Fundamentación general
El aprendizaje es un proceso altamente complejo y sus defini- adecuadas para generar un buen aprendizaje si no se consi-
ciones varían dependiendo del contexto y de la perspectiva. deran otros campos como las competencias emocionales, y las
En educación, el aprendizaje puede definirse como “la adqui- partes del cerebro que intervienen en ellas.
sicion y la integración de nuevos conocimientos, con el fin de
En la monografía de la OCDE La comprensión del cerebro
poder reutilizarlos funcionalmente”.
(2007)2, se puede leer: “… lo emocional es en parte responsable
Sabemos que aprendemos a aprender; aprendemos cómo re- del dominio cognitivo global presente en los niños y los adultos,
sulta más fácil la construcción del conocimiento, aprendemos y debemos tenerlo en cuenta como se merece…Tradicionalmente
a conocer lo que nos motiva, a seguir aprendiendo por placer, dichos estudios se han concentrado sobre los aspectos cogniti-
por necesidad o para crecer como personas. Aprendemos y vos del aprendizaje. Hemos sido negligentes al no efectuar esos
desaprendemos constantemente, nuestro cerebro está siempre análisis sobre las zonas asociadas a lo emocional y a lo afectivo,
trabajando. Sin embargo, gracias a los estudios desarrollados pues el papel que los mencionados aspectos desempeñan den-
a nivel internacional, se tiene la certeza de que, si bien lo cog- tro de la funciones cognitivas no se reconocía.”
nitivo es importante, no se pueden encontrar las respuestas

Ciclo del aprendizaje

Hoy en día la Neurodidáctica o Neuropedagagogía, como la de- A partir de lo anterior, el modelo del Texto escolar considera es-
nomina el neurólogo Francisco Mora 3, o Neuroeducación, como tas fases en su estructura. Las instancias de aprendizaje han sido
la llama el psicopedagogo Roberto Paterno, vienen a ser una desarrolladas pasando por tres momentos didácticos:
suerte de eslabón que permite reunir lo que la Neurología, las
Inicio /Desarrollo / Cierre
Ciencias cognitivas, la Psicología del aprendizaje y la Pedagogía
han intentado comprender desde siempre, es decir, cómo apren- Esta estructura de trabajo se reitera en cada unidad, en las
demos, cuáles son las mejores maneras de aprender y, en conse- propuestas por subunidad y en el desarrollo de cada uno de
cuencia, cómo organizar las mejores maneras de enseñar. El ciclo los temas. 5
del aprendizaje que plantea David Kolp considera cómo trabaja
el cerebro en el proceso de aprendizaje. Este ciclo se basa en que
el aprendizaje se origina a partir de una experiencia concreta.

Actividades de motivación.
Contextos lúdicos y atrayentes.
Ideas previas o implícitas.
Definición de metas y motivaciones.
Aplicación de los Actividades e instancia de aprendizaje:
aprendizajes. Desarrollo de pensamiento crítico.
Actividades de evaluación. Desarrollo pensamiento metacognitivo.
Proyectos interdisciplinarios. Experiencia Concreta Trabajo individual y/o colaborativo.
Experimentación Activa En esta etapa captamos nueva informa- Observación Reflexiva
En esta etapa comprendemos la ción (percibimos) sintiendo, es decir, a En esta etapa procesamos la experien-
nueva información (la procesamos) través de los sentidos, del contacto con cia observando; otorgamos sentido ob-
haciendo, implicándonos en nuevas los aspectos tangibles de las experiencias. servando la experiencia y reflexionando
experiencias y experimentando en sobre la conexión entre lo que hicimos y
forma activa para comprender. las consecuencias de nuestras acciones.

Conceptualización Abstracta Guía Didáctica del Docente


En esta etapa obtenemos nueva informa-
ción (percibimos) pensando; por medio del Mapas conceptuales.
pensamiento obtenemos nuevos conceptos, Esquemas/ definiciones.
ideas y teorías que orienten la acción.

2. OCDE (2002), Understanding the Brain: Towards a New Learning Science. OCDE. Paris.
3. Mora, S (2008). Fundamentos biológicos del aprendizaje. En Estilos de Aprendizaje (Facultades

de Medicina). Escanero. Mora S y otros. Prensas Universitarias de Zaragoza. Zaragoza. España.

Física • 2.º Medio


Articulación de la propuesta
Los componentes de la propuesta editorial: Texto del estudiante (TE) y Guía Didáctica del
Docente (GDD) se articulan a partir de un hilo conductor que cruza los distintos momentos
didácticos y establece una secuencia y progresión que da cuenta de los Objetivos de
Aprendizaje (OA) y responde a sus respectivos Indicadores de Evaluación (IE).

Unidad 1:Movimiento
Lección 1: Descripción del
movimiento
Lección 2: Análisis de los
movimientos horizontales
Lección 3: Análisis de los
movimientos verticales

Unidad 2: Fuerzas
Lección 1: Comprendiendo las
6 fuerzas y sus efectos
Lección 2: Explicando las leyes de
Newton

Unidad 3: Trabajo y energía


Lección 1: Trabajo y potencia
mecánica
Lección 2: Energía y su
conservación
Se organiza que son:
Unidades Lección 3: Impulso y cantidad de
movimiento

que están construidas en base


a tres momentos didácticos: Unidad 4: Universo
Lección 1: Origen y evolución del
universo
Guía Didáctica del Docente

Lección 2: Las leyes del universo

Inicio Desarrollo Cierre

Articulación de la propuesta
Para dar cuenta de los OA, las habilidades y las actitudes, en el modelo didáctico del Texto
y de la GDD, se proponen las siguientes instancias:

Sugerencias para Orientaciones Actividades


abordar los errores metodológicas que complementarias para
frecuentes. responden a las apoyar el desarrollo
secciones del Texto. de los aprendizajes.
Desde
la guía
Ampliación y Fichas de refuerzo, Instancias de
profundización de de profundización evaluación,
tipo disciplinar y y actividades rúbricas, pautas y
didáctica. complementarias. solucionarios.

Instancias para la Actividades que Actividades de síntesis,


motivación, activación apoyan el desarrollo aplicación y consolidación
y el registro de los e integración de de las habilidades y los
Desde el texto aprendizajes previos y los contenidos, aprendizajes adquiridos
el establecimiento de habilidades y y revisión de las metas y
metas y estrategias. actitudes. estrategias.

Proceso metacognitivo continuo que


permite monitorear la evaluación y
regular su autonomía.

Guía Didáctica del Docente


Actividades digitales
complementarias a
los contenidos.

Física • 2.º Medio


Fundamentación Diseño instruccional
El enfoque comunicativo

Los distintos recursos que conforman esta propuesta conclusiones basadas en las evidencias registradas. Estos
buscan entregar a las profesoras y los profesores un saberes se insertan en el vasto cuerpo de conocimiento
conjunto articulado de experiencias de aprendizaje que que ha acumulado la Física a lo largo de su historia y, a su
los ayuden a desarrollar en sus estudiantes aquellos vez, se construyen en un determinado contexto cultural,
aprendizajes prescritos por el Currículum nacional. En social y político. La asignatura de Física permite despertar
cuanto a la asignatura de Física, esta disciplina se apoya en el estudiantado el asombro por conocer el mundo
en la perspectiva epistemológica distintiva del quehacer que lo rodea, comprenderlo y utilizar metodologías para
científico y presupone la existencia de fenómenos en el estudiarlo. Asimismo, le otorga la posibilidad de aplicar
entorno susceptibles de ser estudiados mediante diversas una mirada científica a su aproximación a la naturaleza. En
metodologías, que están consensuadas y que son sometidas esta línea, la asignatura promueve una actitud de respeto
a similares estándares en todo el mundo. El estudio de hacia las pruebas o evidencias, un contacto reflexivo con
dichos fenómenos implica un proceso de razonamiento el mundo natural y una actitud flexible para reconsiderar
lógico, que incluye hipótesis, inferencias, explicaciones y ideas carentes de sustento empírico4.

Elementos estructurales

Proyecto del Texto: una mirada transversal del conocimiento


Como instancia de aplicación, al inicio del Texto, se presenta una actividad grupal basa-
da en la metodología de proyectos. Según Sergio Tobón, “La metodología de proyectos
8 tiene gran valor en la educación, ya que en ella se integran aspectos fundamentales en
la formación, tales como observación de fenómenos naturales y/o sociales, análisis de
documentos, consulta bibliográfica, interpretación de situaciones, construcción de ins-
trumentos de conocimiento, desarrollo de la creatividad, afianzamiento de las compe-
tencias comunicativas y trabajo en equipo. Los proyectos estimulan la automotivación
en la medida en que se relacionan con los intereses de los estudiantes y posibilitan el
despliegue de la creatividad, la inventiva y la generación de ideas y acciones. Mediante
esta metodología, las y los estudiantes toman sentido de pertenencia con su profesión,
asumen retos y problemas reales de esta y realizan actividades en equipo, en las cuales
es esencial uno para todos y todos para uno. El trabajo por proyectos dentro del currí-
culo posibilita que los estudiantes aprendan a emprender con base en la solución de
problemas reales, con la facilitación metodológica y conceptual del docente y de otras
personas de la comunidad. El aprendizaje se da en entornos con sentido para las y los
estudiantes, y esto promueve el aprendizaje significativo”.5

Ciencia, Tecnología y Sociedad: un elemento esencial


Al inicio y cierre de cada lección se presentan estudios científicos, adelantos tecnológi-
cos y su impacto, todo lo cual, desde el enfoque CTS (Ciencia, Tecnología y Sociedad),
pretende constituirse en el motor que moviliza a los estudiantes a querer descubrir y
explorar las temáticas que se abordarán en cada unidad, además de detectar conoci-
mientos e ideas previas. Hodson (1992) manifiesta que uno de los principales objetivos
Guía Didáctica del Docente

de la enseñanza de las ciencias es la comprensión del importante papel de las interac-


ciones CTS (incluyendo aquí las cuestiones económicas, políticas, éticas, históricas,
filosóficas y sociológicas de la ciencia y la tecnología), debiendo ser este uno de los
componentes esenciales de la alfabetización científica y tecnológica para todas las 4 Mineduc (2012). Bases curriculares

personas. “[...] una premisa básica del movimiento CTS es que, al hacer más pertinente Educación Básica. Disponible en: http://
la ciencia para la vida cotidiana de los estudiantes, estos pueden motivarse, interesarse www.cirriculumenlineamineduc.
cl/605/w3-article-14598.html
más por el tema y trabajar con más ahínco para dominarlo. Otro argumento a su favor 5 Tobón, S. (2006). Método de trabajo por

es que, al darle relevancia social a la enseñanza de las ciencias, se contribuye a formar proyectos. Madrid: Uninet

Fundamentación Diseño instruccional


buenos ciudadanos; es decir, al concienciar a las y los estudiantes de los problemas
sociales basados en la ciencia, estos se interesan más por la propia ciencia”.6

Actividades de modelamiento de estrategias


para el desarrollo de habilidades
En cada unidad, se ofrecen actividades a través de las cuales se modelan la o las habi-
lidades científicas que se pretende que las y los estudiantes desarrollen. En las situa-
ciones de enseñanza y aprendizaje, como lo propuesto en las actividades Indaguemos
y en el Desarrollo de estrategias, la tarea principal que debe llevar a cabo el alumnado
es, en un sentido amplio, aprender antes, durante y después de participar en las dis-
tintas actividades o tareas escolares. De ahí que el aprendizaje autorregulado apunta
al proceso mediante el cual los aprendices dirigen sistemática y controladamente sus
pensamientos, sentimientos y acciones hacia la consecución de sus metas. Requiere,
por lo tanto, de la activación y mantenimiento de cogniciones y conductas dirigidas a
metas y de actividades mentales como la atención, el repaso, el uso de estrategias de
aprendizaje y el control de la comprensión, así como creencias sobre la autoeficacia,
sobre los resultados obtenidos y el valor que se atribuye al aprendizaje (Schunk, 2000).
Todo ello remite a una concepción activa del alumno-aprendiz como constructor de su
propio aprendizaje, capaz de ajustar sus acciones y estrategias de forma intencional
para alcanzar el éxito. Desde un punto de vista metacognitivo, el aprendizaje autorregu-
lado requiere que el aprendiz conozca las demandas de la tarea, sus propias cualidades
personales y las estrategias necesarias para realizarla. Desde esta perspectiva, las es-
trategias de aprendizaje se consideran planes orientados hacia la realización con éxito
de las tareas, lo que incluye actividades de selección y organización de la información,
repaso, integración de lo nuevo en lo dado, incremento de la significación del material 9
y creación de un clima de trabajo adecuado.7

Taller de habilidades científicas: una instancia


de trabajo colaborativo
En cada unidad se presenta una actividad grupal, a partir de la cual se pretende que las
y los estudiantes desarrollen habilidades científicas y valoren la importancia del traba-
jo colaborativo, comprendiendo además que el rigor, la creatividad y la perseverancia
son actitudes indispensables en el quehacer científico. A partir de contextos cercanos,
se espera que el alumnado pueda aplicar lo aprendido en situaciones de la vida coti-
diana, de modo tal que aprender ciencias tenga un sentido y no se constituya en un fin
en sí mismo. Lo anterior nace del hecho de que la ciencia ocupa hoy un rol protagónico 6 Acevedo, J., Vázquez, A. y Manassero,
en la sociedad. El conocimiento científico ha trascendido prácticamente a todos los M. (2013). Papel de la educación CTS
aspectos de lo cotidiano, y se vuelve indispensable no solo para la comprensión del en una alfabetización científica y
medio en que estamos inmersos, sino también para participar de manera fundamen- tecnológica para todas las personas.
Revista Electrónica de Enseñanza de las
tada en una sociedad democrática. Es así que la Conferencia Mundial sobre la Ciencia
Ciencias, 2 (2), 80-111.
para el siglo XXI (año 1999), auspiciada por la UNESCO, planteó que “el acceso al saber 7 Barca, A., Almeida, L., Porto, A.,

científico con fines pacíficos desde una edad muy temprana forma parte del derecho a Peralbo, M. y Brenlla, J. (2012).
la educación que tienen todos los hombres y mujeres, y que la enseñanza de la ciencia Motivación escolar y rendimiento:
impacto de metas académicas,
es fundamental para la plena realización del ser humano, para crear una capacidad de estrategias de aprendizaje y Guía Didáctica del Docente
científica endógena y para contar con ciudadanos activos e informados”. Y proclama: autoeficacia. Anales de psicología, 28,
“La enseñanza científica, en sentido amplio, sin discriminación y que abarque todos los (3), 848-859.
8 González, C., Martínez, M., Martínez,
niveles y modalidades, es un requisito previo esencial de la democracia y el desarrollo C., Cuevas, K. y Muñoz, L. (2009). La
sostenible”. (Declaración de Budapest, UNESCO-ICSU, 1999).8 educación científica como apoyo a
la movilidad social: desafío en torno
al rol del profesor secundario en la
implementación de la indagación
científica como enfoque pedagógico.
Estudios Pedagógicos, XXXV (1), 63-78.

Física • 2.º Medio


La CIENCIA se construye: el carácter dinámico
del conocimiento científico
En las páginas de cierre se presentan hitos relacionados con alguna temática de la
unidad, que consideran tanto el punto de vista de los aportes de mujeres y hombres
de ciencia, como diversos adelantos científicos y tecnológicos, con la finalidad de re-
levar la construcción colectiva y dinámica del conocimiento científico. La historia de
la ciencia alerta sobre la necesidad de una aproximación fenomenológica de las repre-
sentaciones, concepciones y creencias: los estudiantes necesitan saber con qué y cómo
se relacionan dichos modelos teórico-conceptuales y contrastarlos con situaciones de
su vida cotidiana, o mejor aún con situaciones de la vida real en otros momentos de la
evolución de la ciencia misma (Vidal, Solar y Quintanilla, 2007; García, 2003).
Algunas propuestas para incorporar el análisis histórico en la enseñanza de las ciencias son:
» Explicar historias contextualizadas, que pueden ser utilizadas desde un punto de
vista didáctico no normativo: para introducir conceptos, para motivar, para promover
determinadas actitudes, para relacionar conocimientos de diferentes áreas de la cien-
cia, para fundamentar actividades interdisciplinares, para ayudar a concebir unitaria-
mente la génesis del conocimiento científico de diferentes disciplinas (Quintanilla y
otros, 2007).
Repetir prácticas relevantes, haciendo ver cuáles eran las ideas científicas en el tiempo
en que se postularon, las posibilidades de interpretación que existían y la utilidad de
las mismas, superando las limitaciones de un análisis centrado en si “eran verdad o
no lo eran”.

10 Identificar y describir instrumentos antiguos mediante láminas o esquemas obtenidos


de reproducciones o de sitios en internet. Reflexionar sobre los materiales con que
fueron elaborados, cómo se divulgaron, qué aportaron, las ideas que suscitaban o las
polémicas que atenuaban, etc.9
• Dar a conocer personajes históricos que muestren los aspectos humanos de
las ciencias y el conjunto de valores (individuales y sociales) en los cuales se
desarrollan.
• Promover el análisis de “entramados” histórico-políticos; histórico-geográficos;
históricosociales o histórico-económicos, que favorecieron o no el desarrollo y la
divulgación de la ciencia, sus problemas, sus instrumentos, etc.
Síntesis: una herramienta para organizar los aprendizajes
Al cierre de cada unidad se destacan las nociones esenciales de la unidad, y se muestra
cómo estas se relacionan entre sí mediante el uso de organizadores gráficos. Siguiendo
a Campos (2005), un organizador gráfico puede definirse como una representación vi-
sual y gráfica que establece relaciones jerárquicas y paralelas entre conceptos amplios
e inclusivos, y otros más específicos. Los organizadores gráficos permiten trabajar con
ideas, identificar los principales tópicos de un determinado contenido, organizar, ges-
tionar y ordenar la información según su importancia, establecer relaciones jerárqui-
cas, integrar nuevos aprendizajes, etc., todo lo cual hace que puedan ser considerados 9 Uribe, M., Quintanilla, M., y Solosona,
como eficaces estrategias didácticas para la adquisición del conocimiento, habilidades N. (2010). Aplicación del modelo
Guía Didáctica del Docente

y actitudes de una manera significativa. En este mismo sentido, Ausubel considera que, de Stephen Toulmin a la evolución
para que se produzca un aprendizaje considerado como significativo, es necesario esta- conceptual del sistema circulatorio:
perspectivas didácticas. Ciencia y
blecer una vinculación lógica y una relación coherente con los conocimientos previos educación,16, 1.
ya existentes en el estudiante, de modo que tenga lugar una interacción entre el nuevo 10 Villalustre, L. y Del Moral, M.

conocimiento y el que él ya posee, en la cual ambos se modifican. De este modo, se (2012). E-actividades apoyadas en
organizadores gráficos: aprendizaje
concibe el aprendizaje como un proceso dinámico, activo, en el que la estructura cog- significativo en el contexto virtual
nitiva está constantemente en reestructuración y cuyo resultado es la determinación de ruralnet. Innovación educativa, 22,
explícita de diferencias y similitudes entre las ideas relacionadas (Ausubel, 2002).10 29-141.

Fundamentación Diseño instruccional


Estructura de las unidades de aprendizaje
Esta estructura se basa en la presentación de una secuencia didáctica y temática
que permita la construcción de aprendizajes significativos en los y las estudiantes
a través de la presentación de situaciones, contextos y actividades atractivas,
cercanas y desafiantes. El motor de partida será la motivación, la activación, el
registro y la toma de conciencia de aprendizajes previos e ideas implícitas, ya
que existe consenso en considerar el desarrollo del pensamiento metacognitivo
como una de las claves para la adquisición de un pensamiento profundo y de
calidad (pensamiento crítico, creativo y reflexivo). Estos procesos metacognitivos
serán los responsables del monitoreo, la evaluación y la regulación autónoma de
los aprendizajes. La unidad se divide en tres momentos que reflejan las etapas
de cualquier acción estructurada: inicio, desarrollo y cierre. Esta secuencia
permite organizar la actividad, de modo que alumnos y alumnas puedan hacerse
conscientes de sus modos de aprender, de sus progresos y de sus dificultades,
revisando constantemente el proceso. De esta forma, el trabajo de la unidad se
articula como una verdadera experiencia formadora.

Inicio de unidad TE GDD


La unidad comienza con distintos recursos gráficos Para apoyar el inicio de la unidad, la GDD incluye
y actividades asociadas a ellos, que buscan motivar recursos y actividades especialmente diseñados
a las y los estudiantes y activar sus aprendizajes, para la motivación, la evaluación y la activación
generando curiosidad y compromiso. En este de aprendizajes previos, como:
momento, alumnas y alumnos son invitados a • Orientaciones al docente.
mirar atrás y buscar, en sus experiencias y sus • Ventanas de profundización didáctica. 11
aprendizajes, cuáles pueden ser pertinentes para • Sugerencias de evaluación.
relacionarse de forma significativa con los temas y
aprendizajes de la unidad.
Al mismo tiempo, se los motiva a mirar adelante, a
proyectar cómo enfrentarán el trabajo de la unidad,
generando metas y estrategias.

Guía Didáctica del Docente

Física • 2.º Medio


Inicio de unidad TE GDD
Cada unidad se subdivide en lecciones, y estas Proporciona un conjunto de recursos didácticos y
a su vez en temas. El desarrollo de la unidad disciplinares, y de actividades complementarias,
comprende diversas experiencias de aprendizaje, orientados al desarrollo de nuevos aprendizajes:
que se organizan en distintos tipos de página que
• Planificación de la lección.
abordan los diferentes aspectos de la asignatura. • Orientaciones al docente.
Al igual que en la unidad completa, cada una • Actividades complementarias.
de sus principales divisiones se organiza en una • Ventanas de profundización.
estructura de tres momentos didácticos. De esta • Instrumentos de evaluación.
manera, se busca formar hábitos metacognitivos
en las y los estudiantes, de modo que puedan Articuladas al desarrollo del contenido, se presentan activi-
adueñarse de sus capacidades, generando y dades aplicadas, como el Indaguemos, Taller de habilidades
monitoreando sus estrategias. ciencias y Desarrollo de estrategia.

Cada lección se inicia con una lectura científica, cuya finalidad


es permitir el reconocimiento de ideas previas y aproximarse a
los nuevos conceptos.

12
Guía Didáctica del Docente

La sección Integro lo que


aprendí es una oportunidad
para que las y los estudiantes
evalúen cómo han incorpora-
do los nuevos aprendizajes.

Estructura de las unidades de aprendizaje


Cierre de unidad TE GDD
Se presentan recursos y actividades enfocadas Para el cierre, la GDD entrega un conjunto de
principalmente en analizar, aplicar y crear a partir recursos didácticos y disciplinares, orientados a la
de los aprendizajes de la unidad. Este conjunto consolidación de los aprendizajes. Por ejemplo:
de elementos se presenta contextualizado y su • Orientaciones al docente.
propósito es que las y los estudiantes evalúen • Ventanas de profundización.
sus niveles de desempeño en habilidades, • Actividades complementarias fotocopiables.
conocimientos y actitudes. También se entregan • Instrumentos de evaluación fotocopiables.
espacios para la síntesis de los aprendizajes de la • Solucionarios.
unidad y para una reflexión sobre su proceso de
aprendizaje, mediante la determinación de cómo
resultó la aplicación de sus estrategias y cómo
pueden mejorarlas.

13

Guía Didáctica del Docente

Física • 2.º Medio


Elementos clave de la propuesta
a. Aprendizaje significativo: ¿Por qué aprender a pensar?

Las estrategias de pensamiento pretenden, de forma sistemática y eficaz, fomentar


el aprendizaje significativo, la auténtica comprensión a través del pensamiento
profundo, y la aplicación o transferencia de los conocimientos y del pensamiento
a la vida. Se trata de uno de los cambios más significativos del enfoque educativo
del siglo XXI. Según este planteamiento, el o la estudiante deja de ser un mero
receptor pasivo y se convierte en un agente dinámico que busca, procesa y utiliza
la información para solucionar sus problemas mediante el pensamiento crítico y
creativo.
Estrategias de aprendizaje

Son un conjunto de procesos, herramientas, destrezas y hábitos de pensamiento, es


decir, distintas formas de mejorar el proceso de pensar. La propuesta considera la uti-
lización de una serie de estrategias. A continuación, se incluyen algunos ejemplos de
organizadores visuales, faros del pensamiento y metacognición, que corresponden a
las tres fases del proceso de aprendizaje significativo: la organización de la informa-
ción o de las ideas, su profundización y la autorregulación, respectivamente, para ser
utilizados de forma transversal a lo largo del año.

Organizadores visuales Metacognición

14
Los organizadores visuales son La metacognición tiene como objetivo el conocimiento profundo
estrategias o herramientas que nos de qué sabemos y cómo lo hemos aprendido para mantener las
permiten ordenar espacialmente estrategias que nos funcionan y detectar cuáles deberíamos
las informaciones, ideas, modificar. Es la capacidad de autorregularnos en la competencia
conocimientos o pensamientos de aprender a aprender para conseguir una mayor autonomía en el
sobre las que trabajamos o que proceso de aprendizaje. Su práctica constante favorece:
son objeto de nuestro proceso • el aprender de la experiencia consolidando los modos de actuar exito-
de aprendizaje. Son eficaces, en sos y descartar los que no funcionan;
especial, para los estudiantes que • corroborar lo que se conoce y la forma en que se ha aprendido;
aprenden mejor de manera visual. • la transferencia a la vista cotidiana de los conocimientos adquiri-
Se pueden utilizar para acompañar dos, porque la práctica de la metacognición permite distanciarse del
explicaciones orales, para sintetizar contenido concreto y poner foco en las estrategias de pensamiento y
lo trabajado, para investigar o su valor.
reflexionar. Es deseable disponer, al final de cada actividad, de unos minutos
para pensar sobre los procesos de aprendizaje.

En el siguiente enlace puede descargar


explicaciones y ejemplo de mapas concep- En el siguiente enlace, puede descargar explicaciones y ejemplos de estrategias,
tuales, mapas mentales y cronogramas que como Diario de pensar; Pensaba, pienso; Pregunta estrella; Pasos reflexionados; Qué
puede utilizar a lo largo del Texto. aprendo, para qué; Mente dispuesta.
http://codigos.auladigital.cl. Ingresando el http://codigos.auladigital.cl. Ingresando el código 18GF2M014b.
Guía Didáctica del Docente

código 18GF2M014a.

Elementos clave de la propuesta


b. Aprendizaje colaborativo

Ante la diversidad que se observa en el aula, el aprendizaje cooperativo es una


fórmula mediante la cual todas y todos los estudiantes puedan participar y
aprender en los mismos escenarios educativos, independientemente de sus
características cognitivas, afectivas y sociales. El aprendizaje cooperativo es una
metodología cuyo objetivo es la construcción de conocimiento y la adquisición
de competencias y habilidades sociales. Fomenta valores como la tolerancia, el
respeto y la igualdad. Se basa en el trabajo en equipo y, según Spencer Kagan,
promueve la responsabilidad individual, la interdependencia positiva con iguales,
la interacción simultánea y la participación igualitaria. El aprendizaje cooperativo
constituye una metodología que se deriva de los aportes de autores de gran
prestigio en el ámbito de la educación y la psicología, tales como Piaget, Vygotski
o Ausubel, entre otros. Son muchos los teóricos que defienden que la formación
del alumnado no debe reducirse al desarrollo de sus capacidades intelectuales,
sino que es imprescindible atender a su desarrollo integral como personas.
Investigadores como Vygotski, Bruner o los hermanos Johnson basan sus teorías
en la importancia que tienen la mediación social y el lenguaje como mecanismos
de desarrollo y de aprendizaje. Si consideramos que nuestra sociedad es cada vez
más diversa, y que está conformada por gente de distintos orígenes culturales,
con diferentes intereses y maneras de ver el mundo, el respeto por la diferencia
debe ser un valor esencial para una buena convivencia. Por tanto, el aprendizaje
cooperativo valora de forma positiva la heterogeneidad de las y los estudiantes,
convirtiéndola en un recurso de aprendizaje. Todo ello, en consonancia con el 15
deseo de una sociedad más equitativa, democrática e inclusiva.
Algunas de las características que se necesita considerar para que una actividad sea
realmente cooperativa son:
a. el clima de aula;
b. la creación de los equipos;
c. normas de funcionamiento;
En el Texto se proponen de forma integrada actividades que propician el trabajo cola-
borativo y que son apoyadas desde la GDD con orientaciones y pautas de evaluación
que permiten a las y los estudiantes ir mejorando sus competencias. A continuación
se proponen algunas orientaciones o alternativas para enriquecer y potenciar las acti-
vidades propuestas.

Estructuras cooperativas Actividad de clima y cohesión Instrumento de evaluación


de equipo
Alternativas de organización de los Normas de funcionamiento. Habili-
equipos de trabajo. Puede descar- Puede descargar un ejemplo y ma- dades sociales y actitudes a poner
gar un ejemplo y material impri- terial imprimible en: en juego. Puede descargar un ejem-
mible en: plo y material imprimible en: Guía Didáctica del Docente
http://codigos.auladigital.cl, ingre-
http://codigos.auladigital.cl, ingre- sando el código 18GF2M015b. http://codigos.auladigital.cl, ingre-
sando el código 18GF2M015a. sando el código 18GF2M015c.

Bibliografía. Kagan, S. (2001). Kagan Structures and Learning Together. What is the Difference? Kagan on line Magazine. Disponible en:
www.kaganonline.com/KaganClub/FreeArticles.html.

Física • 2.º Medio


c. Evaluación para el aprendizaje

La propuesta didáctica concibe la evaluación en función del aprendizaje, en otras pala-


bras, como un medio y no como un fin. En consonancia con los principios antes men-
cionados, las actividades de evaluación se presentan contextualizadas en situaciones
problema y apuntan a habilidades superiores. Su objetivo es entregar, a estudiantes
y docentes, información útil para la mejora de las prácticas y el logro cabal de los
aprendizajes. Para esto, proveen de instrumentos que explicitan los indicadores que
les permitirán conocer sus niveles de logro. Este trabajo se entrecruza con el proceso
de reflexión metacognitiva antes descrito, ya que, estrictamente se trata de un mismo
proceso. Si decimos que se trata de un proceso es porque es posible distinguir una se-
cuencia de acciones que debería ser constitutiva de la evaluación para el aprendizaje.
Estas son:
“Búsqueda de indicios: ya sea a través de la observación o de ciertas formas de medición
se obtiene información […] de modo no caprichoso sino sistemático y planificado […].
Forma de registro y análisis: a través de un conjunto variado de instrumentos se re-
gistran estos indicios, este conjunto de información permitirá llevar a cabo la tarea
de evaluación […].
Criterios: elementos a partir de los cuales se puede establecer la comparación respec-
to del objeto de evaluación o algunas de sus características […].
Juicio de valor: íntimamente vinculado con el anterior, pero constituyendo el com-
ponente distintivo de todo proceso de evaluación, se encuentra la acción de juzgar,
16 de emitir o formular juicios de valor […].
Toma de decisiones: por último, la toma de decisiones es un componente inherente al
proceso de evaluación y que lo diferencia de otro tipo de indagación sistemática. Las
acciones evaluativas cobran sentido en tanto soporte para la toma de decisiones […]” .11
Estas etapas pueden constituirse en una forma de definir la evaluación, pero su ade-
cuada aplicación requiere algunas precisiones. La definición de criterios es una activi-
dad que implica ciertos riesgos: por un lado, caer en un normativismo que atente contra
la motivación de las y los estudiantes y, por otro, renunciar a la elaboración de criterios
en nombre de una aproximación más intuitiva. Consideramos que ambos riesgos deben
ser evitados y, en este sentido, recomendamos enfáticamente poner en común con
el estudiantado los instrumentos y criterios de evaluación desde el comienzo de las
actividades. En cuanto a los juicios de valor, es fundamental no perder de vista que
generan impacto emocional e inciden en la motivación de las y los estudiantes. Para
evitar la carga negativa que pueden arrastrar los juicos, además de tener criterios claros
y comunes, es necesario “desmitificar el error”, pues sin este elemento el aprendizaje es
imposible. En este sentido, puede decirse que: “Estas elecciones equivocadas se llaman
adecuadamente ‘error’ cuando son de tal clase que puedan proporcionar al organismo
una información que contribuya a su futura destreza”. Si se comprende que el “error”,
por definición, es condición necesaria del aprendizaje, se entenderá la importancia de
formar juicios razonablemente fundados. En el conjunto de recursos que conforman la
Guía Didáctica del Docente

propuesta se ha optado por preferir conceptos con un impacto emocional menor, como
“problema” o “inadecuación”.

11 Ministerio de Educación. (2006). Evaluación Para el Aprendizaje: Enfoque y materiales prácticos para lograr que

sus estudiantes aprendan más y mejor. Santiago: Unidad de Currículum y Evaluación.

Elementos clave de la propuesta


d. Motivación para el aprendizaje

“Motivación” y “emoción” son términos estrechamente las emociones, ya que estas últimas constituyen la forma en
ligados. Estas palabras derivan del verbo latino movere, que el cerebro evalúa si actúa o no sobre las cosas. Por lo
que significa ‘moverse’, ‘poner en movimiento’, ‘estar listo tanto, es posible formular la hipótesis de que los sistemas
para la acción’. La motivación es lo que nos impulsa a emocionales crean motivación. Las emociones y las
actuar para conseguir un objetivo, un estado interno que motivaciones son los verdaderos impulsores de cualquier
excita, dirige y sostiene el comportamiento. aprendizaje humano.
En el ámbito escolar se entiende que la motivación es El profesor o la profesora deberían controlar el estado
un proceso interno que activa, dirige y mantiene una de ánimo de sus estudiantes, puesto que este va a ser
conducta hacia un aprendizaje concreto y en el que determinante en la motivación, en la elaboración de
participan, obviamente, variables biológicas, psicológicas, contenidos docentes, en la comprensión y en la ejecución
de personalidad, sociales y cognitivas. El desarrollo y de los mismos. El cerebro dispone de un sistema muy
equilibrio de estos procesos favorecerá una motivación bueno de motivación que consiste en la búsqueda de
intrínseca, gracias a la cual los alumnos y las alumnas novedades; el niño o la niña se motivan si tiene nuevos
podrán presentar un conducta positiva frente al aprendizaje estímulos que explorar o conocer. En un ambiente aburrido,
sin necesidad de recurrir a la motivación extrínseca estable, sin contrastes, anodino, el cerebro del niño o niña
relacionada con refuerzos exteriores, recompensas, se vuelve pasivo por falta de motivación. Por el contrario,
castigos, notas, etc. en un ambiente enriquecido, lleno de estímulos y con
perspectivas de futuro, el cerebro se activa enormemente.
La motivación es crucial para el aprendizaje exitoso y se
encuentra vinculada muy de cerca con la comprensión y Pero los estados de motivación y desmotivación no son
las emociones. La motivación puede ser descrita como la permanentes, por lo que durante un día de clase, las y los
fuerza resultante de los componentes emocionales y refleja estudiantes pueden pasar por varios estados sucesivos. 17
hasta dónde está preparado un organismo para actuar Según Jensen (2004)12, puede haber tres razones
física y mentalmente de una manera focalizada. De acuerdo diferentes por las que el alumnado no está motivado
con esto, la motivación está íntimamente relacionada con temporalmente:
1. Las asociaciones del pasado, que le pueden provocar un estado negativo o apático. La memoria emocional es aso-
ciativa y puede dispararse ante un determinado estímulo que recuerde una situación negativa.
2. Estilos de enseñanza poco o nada participativos, carencia de recursos, temperatura inadecuada, etc.
3. Falta de objetivos claramente definidos o creencias limitantes respecto de su aprendizaje, lo cual no les permite
activar la parte izquierda de la corteza frontal, asociada a las emociones positivas.
Podemos recomendar cinco estrategias que propone Jensen (2004) para ayudar a los alumnos y las alumnas a motivarse.
1. Eliminar la amenaza. Es importante que el o la docente descubra y neutralice los problemas que pueden existir en
el aula y ser percibidos por los y las estudiantes como amenazas.
2. Fijar objetivos. Estos deben ser conocidos por las y los estudiantes y relacionados con sus vidas e intereses.
3. Influir positivamente. Es necesario influir sobre las creencias de alumnos y alumnas respecto a sí mismos y sus
capacidades para desarrollar su autoconfianza.
4. Gestionar sus emociones. Se debe enseñar la regulación emocional; también puede promoverse mediante el traba-
jo de rituales, celebraciones, movimiento, etcétera.
5. Generar un proceso de retroinformación continua. Tanto si se da entre docentes y estudiantes, como entre compa- Guía Didáctica del Docente
ñeros y compañeras, el feedback enriquece la experiencia de aprendizaje.

12 Jensen, E. (2004). Cerebro y aprendizaje: Competencias e implicaciones educativas. Madrid: Narcea.

Física • 2.º Medio


Visión global
Visión Global Tomo I
Unidad 1 Movimiento
Tiempo estimado: 20 horas pedagógicas
Objetivo de aprendizaje: Analizar, sobre la base de la experimentación, el movimiento rectilíneo uniforme y acelerado de un objeto
respecto de un sistema de referencia espacio-temporal, considerando variables como la posición, la velocidad y la aceleración en
situaciones cotidianas.
Lección Tiempo Actitud OFT Habilidades de investigación
científica

1 Descripción 6 Mostrar curiosidad, creatividad e interés Dimensión • Observar y plantear.


del por conocer y comprender los fenómenos cognitiva- • Planificar y conducir una investigación.
movimiento del entorno natural y tecnológico, intelectual • Procesar y analizar la evidencia.
disfrutando del crecimiento intelectual Proactividad y • Evaluar una investigación.
que genera el conocimiento científico trabajo • Comunicar una investidación.
y valorando su importancia para el
Tecnologías de la
desarrollo de la sociedad.
2 Análisis 10 información y la
de los Trabajar responsablemente en forma comunicación
movimientos proactiva y colaborativa, considerando
horizontales y respetando los variados aportes del
equipo y manifestando disposición a
entender los argumentos de otros en las
soluciones a problemas científicos.
3 Análisis 4 Usar de manera responsable y efectiva
de los las tecnologías de la comunicación para
movimientos favorecer las explicaciones científicas y
18 verticales el procesamiento de evidencias, dando
crédito al trabajo de otros y respetando la
propiedad y la privacidad de las personas.

Indicadores de Evaluación
• Demuestran, con experimentos sencillos, por qué es necesario el uso de sistemas de referencia y de coordenadas en la descripción
del movimiento de un objeto.
• Utilizan las fórmulas de adición de velocidades de Galileo en situaciones simples y cotidianas, como la de vehículos que se mueven
unidimensionalmente.
• Explican conceptos de cinemática, como tiempo transcurrido, posición, desplazamiento, distancia recorrida, velocidad media e
instantánea y aceleración, entre otros, asociados al movimiento rectilíneo de un objeto.
• Identifican características de la cinemática del movimiento rectilíneo, en situaciones cotidianas, como ocurre con vehículos, entre
otros ejemplos.
• Analizan, con conceptos de cinemática y herramientas gráficas y analíticas, el movimiento rectilíneo de un objeto en situaciones
cotidianas.
• Explican el concepto de aceleración de gravedad, y consideran su uso en situaciones de caída libre y lanzamientos verticales.
• Obtienen conclusiones, en relación con conceptos de cinemática, a partir de investigaciones experimentales sobre objetos con movi-
miento rectilíneo con aceleración constante (nula o no nula).
Guía Didáctica del Docente

Visión global Tomo I


Unidad 2 Fuerzas
Tiempo estimado: 18 horas pedagógicas
Objetivo de aprendizaje: Explicar, por medio de investigaciones experimentales, los efectos que tiene una fuerza neta sobre un objeto,
utilizando las leyes de Newton y el diagrama de cuerpo libre.
Lección Tiempo Actitud OFT Habilidades científicas

1 Comprendiendo 8 Esforzarse y perseverar en el trabajo Proactividad y • Observar y plantear.


las fuerzas y sus personal entendiendo que los logros trabajo. • Planificar y conducir una investigación.
efectos se obtienen solo después de un trabajo Dimensión • Procesar y analizar la evidencia.
riguroso, y que los datos empíricamente física y • Evaluar una investigación.
confiables se obtienen si se trabaja con Dimensión • Comunicar una investidación.
precisión y orden. moral.
Demostrar valoración y cuidado Dimensión
por la salud y la integridad de las sociocultural y
2 Explicando las 10 personas, evitando conductas de riesgo, ciudadana.
leyes de Newton considerando medidas de seguridad y
tomando conciencia de las implicancias
éticas de los avances científicos y
tecnológicos.
Demostrar valoración e interés por
los aportes de hombres y mujeres al
conocimiento científico y reconocer que
desde siempre los seres humanos han
intentado comprender el mundo.

Indicadores de Evaluación 19
• Identifican una fuerza como la interacción entre dos cuerpos y su carácter vectorial, entre otras características.
• Realizan investigaciones experimentales para obtener evidencias de la presencia de fuerzas como peso, roce y normal, que actúan
sobre un cuerpo, en situaciones cotidianas, describiéndolas cualitativa y cuantitativamente.
• Aplican las leyes de Newton en diversas situaciones cotidianas, como cuando un vehículo frena, acelera o cambia de dirección su
movimiento, entre otras.
• Encuentran, con un diagrama de cuerpo libre, la fuerza neta o resultante sobre un objeto en el que actúa más de una fuerza.
• Analizan el efecto que provoca la fuerza neta o resultante en el movimiento de un objeto.
• Aplican la ley de Hooke en diversas investigaciones experimentales y no experimentales en las que se utilizan resortes u otros
materiales elásticos.

Guía Didáctica del Docente

Física • 2.º Medio


Visión Global Tomo II
Unidad 3 Trabajo y energía
Tiempo estimado: 22 horas pedagógicas
Objetivo de aprendizaje
• Describir el movimiento de un objeto, usando la ley de conservación de la energía mecánica y los conceptos de trabajo y potencia
mecánica.
• Analizar e interpretar datos de investigaciones sobre colisiones entre objetos, considerando:
• La cantidad de movimiento de un cuerpo en función del impulso que adquiere.
• La ley de conservación de cantidad de movimiento (momento lineal o momentum).
Lección Tiempo Actitud OFT Habilidades científicas

1 Trabajo y 6 Esforzarse y perseverar en el trabajo Proactividad y • Observar y plantear.


potencia personal entendiendo que los logros trabajo • Planificar y conducir una investigación.
mecánica se obtienen solo después de un trabajo Dimensión • Procesar y analizar la evidencia.
riguroso, y que los datos empíricamente sociocultural y • Evaluar una investigación.
confiables se obtienen si se trabaja con ciudadana • Comunicar una investidación.
2 Energía y su 8 precisión y orden.
Dimensión
conservación Reconocer la importancia del entorno cognitiva-
natural y sus recursos, y manifestar intelectual
conductas de cuidado y uso eficiente
de los recursos naturales y energéticos
en favor del desarrollo sustentable y la
3 Impulso y 8 protección del ambiente.
cantidad de Manifestar una actitud de pensamiento
20 movimiento crítico, buscando rigurosidad y
replicabilidad de las evidencias para
sustentar las respuestas, las soluciones o
las hipótesis.

Indicadores de Evaluación
• Determinan el trabajo mecánico realizado por una fuerza en situaciones unidimensionales diversas y cotidianas, como cuando se
arrastra o levanta un objeto, o cuando este cae, entre otras.
• Describen la energía mecánica de un objeto en términos de su energía cinética, potencial gravitatoria y potencial elástica, según
corresponda.
• Aplican la ley de conservación de la energía mecánica en situaciones cotidianas, como en el movimiento de un objeto en caída libre
y, cualitativamente, en una montaña rusa, entre otras.
• Evalúan el efecto del roce en el movimiento de un objeto, en relación con la ley de conservación de la energía mecánica.
• Aplican el teorema del trabajo y la energía en situaciones unidimensionales simples y cotidianas.
• Determinan la potencia mecánica desarrollada por una fuerza en situaciones cotidianas, como ocurre en el funcionamiento de una
grúa o un ascensor, entre otras.
• Evalúan la facilidad o dificultad que existe para cambiar el estado de movimiento de un objeto, de acuerdo a su cantidad de
movimiento.
• Describen el impulso que adquiere un objeto en términos de la variación de su cantidad de movimiento y lo relacionan con la
segunda ley de Newton.
• Aplican la ley de conservación de la cantidad de movimiento en un sistema cerrado, en colisiones entre objetos que se mueven en la
Guía Didáctica del Docente

misma dirección.
• Distinguen colisiones elásticas e inelásticas o plásticas entre dos objetos que se mueven en la misma dirección.
• Explican que en una colisión elástica, entre dos objetos que se mueven en una misma dirección, se conserva la energía cinética.
• Explican que los efectos de una colisión entre dos objetos pueden ser diferentes para cada uno de ellos.
• Analizan resultados experimentales obtenidos en colisiones entre dos objetos que se mueven en la misma dirección.

Visión global Tomo II


Unidad 4 Universo
Tiempo estimado: 16 horas pedagógicas
Objetivo de aprendizaje
• Demostrar que comprenden que el conocimiento del universo cambia y aumenta a partir de nuevas evidencias, usando modelos
como el geocéntrico y el heliocéntrico, y teorías como la del Big Bang, entre otros.
• Explicar cualitativamente por medio de las leyes de Kepler y la de gravitación universal de Newton:
• El origen de las mareas.
• La formación y dinámica de estructuras cósmicas naturales, como el sistema solar y sus componentes, las estrellas y las galaxias.
• El movimiento de estructuras artificiales, como sondas, satélites y naves espaciales.
Lección Tiempo Actitud OFT Habilidades científicas

1 Origen y 8 Demostrar valoración e interés por Dimensión • Observar y plantear.


evolución del los aportes de hombres y mujeres al sociocultural y • Planificar y conducir una investigación.
universo conocimiento científico y reconocer que ciudadana • Procesar y analizar la evidencia.
desde siempre los seres humanos han Dimensión • Evaluar una investigación.
intentado comprender el mundo. cognitiva- • Comunicar una investidación.
Mostrar curiosidad, creatividad e interés intelectual
por conocer y comprender los fenómenos
del entorno natural y tecnológico,
2 Las leyes del 8 disfrutando del crecimiento intelectual
universo que genera el conocimiento científico
y valorando su importancia para el
desarrollo de la sociedad.
Manifestar una actitud de pensamiento
crítico, buscando rigurosidad y 21
replicabilidad de las evidencias para
sustentar las respuestas, las soluciones o
las hipótesis.

Indicadores de Evaluación
• Explican diversos modelos que han intentado describir el universo desde la Antigüedad hasta inicios del siglo XX, como el geocén-
trico y el heliocéntrico, postulados por Ptolomeo y Copérnico respectivamente, entre otros.
• Identifican virtudes y limitaciones de los modelos del universo para explicar su dinámica.
• Distinguen a científicos como Galileo, Brahe y Newton, entre otros, por sus aportes en la concepción de modelos del universo.
• Explican cualitativamente la evolución del universo según la teoría del Big Bang.
• Describen características de las cosmogonías de culturas que habitan Chile, como el origen y los elementos que componen el uni-
verso, entre otros aspectos.
• Relacionan el desarrollo tecnológico con la evolución de los modelos que describen el universo.
• Explican cualitativamente, con las leyes de Kepler, las características del movimiento de los cuerpos del sistema solar.
• Explican cualitativamente el fenómeno de las mareas con la ley de gravitación universal.
• Explican cualitativamente, con la ley de gravitación universal, el movimiento de traslación que ocurre en sistemas planetarios,
satelitales, galácticos y de estructuras artificiales espaciales, entre otros.
• Describen estructuras cósmicas, como planetas, estrellas, sistemas estelares y galaxias, entre otras, a partir del colapso gravitacional.
• Explican las ventajas y desventajas de los campos gravitacionales en la navegación espacial y en la instalación de sondas y satéli-
tes, entre otros dispositivos tecnológicos. Guía Didáctica del Docente

Física • 2.º Medio


Texto del estudiante

FÍSICA 2 º
MEDIO

Nathalie Francisca Oyola Espinoza


Licenciada en Educación en Física y Matemática
Profesora de Estado en Física y Matemática
Universidad de Santiago de Chile

Carolina Tobar González


Licenciada en Ciencias Exactas
Profesora de Educación Media en Matemática y Física
Universidad de Chile
El Texto del estudiante Física 2.° Medio es una creación del Departamento de Estudios
Pedagógicos de Ediciones SM, Chile.

Dirección editorial Diseño y diagramación


Arlette Sandoval Espinoza Rossana Allegro V.
Mauricio Fresard L.
Coordinación editorial José Luis Jorquera Dölz
María José Martínez Cornejo
Iconografía
Coordinación área Ciencias Naturales Vinka Guzmán Tacla
Andrea Tenreiro Bustamante
Ilustraciones
Edición Javier Bahamonde Otárola
Nathalie Oyola Espinoza Fernando Vergara Piña
Asistente de edición René Moya Vega
Carolina Tobar González Fotografías
Autoría Archivos fotográficos SM
Nathalie Oyola Espinoza Shutterstock
Carolina Tobar González Pixabay
NASA
Consultor ESO
Rolando Díaz Delgado Gestión de derechos
Asesoría pedagógica Loreto Ríos Melo
Guadalupe Álvarez Pereira Jefatura de producción
Corrección de estilo y pruebas Andrea Carrasco Zavala
Cristian Oyarzo Barrientos

Coordinación de Diseño
Gabriela de la Fuente Garfias

Diseño de portada
Estudio SM

Ilustración de portada
Estevan Silveira

Este texto corresponde al Segundo año de Educación Media y ha sido elaborado conforme al
Decreto Supremo N° 614/2013, del Ministerio de Educación de Chile.
©2018 – Ediciones SM Chile S.A. – Coyancura 2283 piso 2 – Providencia
ISBN: 978-956-363-290-3 / Depósito legal: 280278
Se terminó de imprimir esta edición de 216.755 ejemplares en el mes de noviembre del año 2018.
Impreso por A Impresores.
Quedan rigurosamente prohibidas, sin la autorización escrita de los titulares del “Copyright”, bajo las sanciones establecidas
en las leyes, la reproducción total o parcial de esta obra por cualquier medio o procedimiento, comprendidos la reprografía y
el tratamiento informático, y la distribución en ejemplares de ella mediante alquiler o préstamo público
Presentación
El texto que tienes en tus manos es una herramienta elaborada pensando
en ti. Tú serás el protagonista de tu propio aprendizaje y el texto será el
vehículo que, junto con tu profesor o profesora, te oriente y te acompañe
en la adquisición de los contenidos y el desarrollo de las habilidades,
actitudes y procedimientos propios de la Física.

¿Qué es la Física?
Es una ciencia que busca explicar el entorno que nos rodea. Para ello se
vale de la observación y de la experimentación, con el fin de establecer
leyes, principios y teorías que den cuenta del mundo en el que vivimos.
Es una ciencia en constante construcción y, gracias a sus logros, hoy en
día podemos ser testigos de incontables avances tecnológicos.

¿Qué aprenderé?
En este texto aprenderás acerca de fenómenos relacionados
con el movimiento y la fuerza. También conocerás las
diferentes formas en que la energía se manifiesta y explorarás
los orígenes y la evolución del universo. Finalmente,
esperamos que puedas reconocer que, en la ciencia, el
conocimiento se construye de forma dinámica y colaborativa.

¿Cómo aprenderé?
El texto promueve el desarrollo de habilidades científicas y actitudes como
un elemento central del proceso de aprendizaje. Para ello, se presenta
una serie de estrategias, actividades, proyectos y procedimientos prácticos
que te permitirán razonar, argumentar y experimentar en torno a los
fenómenos que se producen en la naturaleza.

Este proyecto es una propuesta integral, que busca ¿Para qué?


contribuir a tu formación como ciudadano activo,
crítico, reflexivo, capaz de integrarte y dejar huella Para que logres acercarte a esta
en la sociedad. Te invitamos a recorrer tu texto y disciplina científica con gusto y
asombrarte con lo que eres capaz de lograr. motivación, a fin de que conozcas más
tu entorno desde el prisma de la Física.
Te invitamos a ser protagonista de tu aprendizaje y a
tomar un lugar activo para construir un mundo cada
vez mejor.

¿Qué espero yo?


Índice
Conoce tu texto......................................................................................6
Proyecto del texto...............................................................................12
Ruta del aprendizaje..........................................................................14

Movimiento.. .................................................. 16 Fuerzas. . .........................................................74

Evaluación inicial...................................................................................18 Evaluación inicial...................................................................................76

Lección 1 Descripción del movimiento.................................22 Lección 1 Comprendiendo las fuerzas y sus efectos...... 80
Tema 1 ¿Cuándo nos movemos?....................................................24 Tema 1 Características generales de las fuerzas........................82
Tema 2 Analizando la relatividad del movimiento................... 30 Tema 2 Identificando las fuerzas en la vida cotidiana............ 86

Integro lo que aprendí Evaluación de proceso........................36 A poner en práctica mediante un taller


de habilidades científicas................................................................. 90
Lección 2 Análisis de los movimientos horizontales....... 38 Integro lo que aprendí Evaluación de proceso....................... 94
Tema 1 Describiendo el Movimiento
Rectilíneo Uniforme (MRU)............................................. 40 Lección 2 Explicando las leyes de Newton......................... 96

A poner en práctica mediante un taller Tema 1 Estableciendo la primera ley de Newton..................... 98


de habilidades científicas................................................................. 44 Tema 2 Aplicando la segunda ley de Newton......................... 100
Tema 2 Movimiento Rectilíneo
Tema 3 Estableciendo la tercera ley de Newton..................... 104
Uniformemente Acelerado (MRUA)............................. 46
Integro lo que aprendí Evaluación de proceso..................... 108
Integro lo que aprendí Evaluación de proceso........................52
La ciencia se construye......................................................110
Lección 3 Análisis de los movimientos verticales............. 54
Síntesis..................................................................................112
Tema 1 ¿Qué características posee la caída libre?....................56
A poner en práctica mediante un taller Evaluación final....................................................................114
de habilidades científicas..................................................................58
Tema 2 ¿Qué características posee el
lanzamiento vertical?........................................................ 60
Integro lo que aprendí Evaluación de proceso....................... 64
La ciencia se construye....................................................... 66

Síntesis...................................................................................68

Evaluación final .................................................................... 70

4 Ïndice
Trabajo y energía...................................... 118 Universo. . .................................................... 178

Evaluación inicial................................................................................ 120 Evaluación inicial................................................................................ 180

Lección 1 Trabajo y potencia mecánica............................ 124 Lección 1 Origen y evolución del universo....................... 184
Tema 1 ¿Cuándo se realiza trabajo mecánico?.........................126 Tema 1 Origen del universo...........................................................186
Tema 2 ¿Cómo se relaciona el trabajo mecánico Tema 2 Sistemas planetarios......................................................... 190
con la potencia?............................................................... 130
Integro lo que aprendí Evaluación de proceso......................198
Integro lo que aprendí Evaluación de proceso.......................134
Lección 2 Las leyes del universo..........................................200
Lección 2 Energía y su conservación................................ 136
Tema 1 Las leyes de Kepler............................................................ 202
Tema 1 ¿Qué es la energía?...........................................................138
Tema 2 La ley de gravitación universal de Newton...............206
Tema 2 ¿Qué es la energía potencial?........................................142
A poner en práctica mediante un taller
A poner en práctica mediante un taller de habilidades científicas...............................................................208
de habilidades científicas................................................................146 Tema 3 Los efectos de la fuerza de
Tema 3 ¿Qué es la energía mecánica?...................................... 148 atracción gravitacional................................................... 210
Integro lo que aprendí Evaluación de proceso.......................154 Integro lo que aprendí Evaluación de proceso......................214

Lección 3 Impulso y cantidad de movimiento................ 156 La ciencia se construye......................................................216


Tema 1 Describiendo el impulso...................................................158 Síntesis..................................................................................218
Tema 2 Cantidad de movimiento................................................ 160 Evaluación final................................................................... 220
Integro lo que aprendí Evaluación de proceso.......................168

La ciencia se construye...................................................... 170 Material de apoyo


Anexo.................................................................................................... 224
Síntesis...................................................................................172
Índice temático................................................................................... 234
Evaluación final.....................................................................174
Glosario................................................................................................ 236
Bibliografía.......................................................................................... 238
Notas..................................................................................................... 239

Física • 2.° Medio 5


Conoce tu texto

PROYECTO DEL TEXTO


Este texto se centra en una de las grandes ideas de las ciencias:
El movimiento de un objeto depende de las interacciones en
que participa.
Para desarrollar esta idea, te proponemos un Proyecto que se
subdivide en 10 misiones, las cuales encontrarás en cada una
de las lecciones que estudiarás en el texto.

RUTA DEL APRENDIZAJE


En estas páginas te
presentamos los principales
contenidos que serán
abordados en el texto para que
puedas identificar y registrar
el estado de avance de tus
aprendizajes.

6 Conoce tu texto
Inicio de
Unidad
ENTRADA DE UNIDAD
Se presenta una imagen central
cuya misión es motivar el estudio de
los aprendizajes de la unidad. Para
ello, en la actividad individual, se
proponen preguntas relacionadas
con la situación propuesta.

EVALUACIÓN INICIAL
Para despertar tu curiosidad, se
proponen, en esta instancia, una
serie de preguntas relacionadas
con las principales nociones que
se desarrollarán en la unidad.

¿QUÉ? ¿CÓMO? ¿PARA QUÉ?


En estas páginas se presentan los
aprendizajes propuestos en el texto.
En ¿Qué aprenderás y descubrirás
en la unidad? sabrás lo que vas a
estudiar, junto con el cómo y para
qué lo aprenderás.
En ¿Cómo te gustaría protagonizar tu
propio aprendizaje?, se presenta una
serie de preguntas que pretenden
orientar la planificación de tu proceso
de aprendizaje.
Finalmente, ¿Cómo lograr mis
metas? te permitirá implementar una
estrategia de aprendizaje para el
logro de tus propias metas.

Física • 2.° Medio 7


Conoce tu texto
Presentación de la lección

COMIENZO DE LECCIÓN
Cada lección comienza con la
sección Ciencia, tecnología y
sociedad. Aquí, te presentamos
artículos relacionados con
la ciencia y desarrollos
tecnológicos asociados a los
aprendizajes de la unidad.

INICIO DE LA MISIÓN
INDAGUEMOS En cada lección del texto, te
proponemos una misión que se
Cada tema inicia mediante una
relaciona con el Proyecto del texto.
actividad, cuya finalidad es que
reconozcas y registres tus ideas previas,
además de acercarte a los nuevos
conceptos a través de la observación y
el planteamiento de preguntas.

DESARROLLO DE ESTRATEGIAS
En las lecciones se presentan actividades en los que se te
enseñará, paso a paso, cómo realizar un procedimiento
propio de las ciencias, para luego poner a prueba tus
aprendizajes en el paso 5: Aplico lo aprendido.

CIERRE DE LECCIÓN
Al finalizar cada lección,
encontrarás nuevamente la
sección Ciencia, tecnología y
sociedad.

REFLEXIONO SOBRE LO QUE APRENDÍ CIERRE DE LA MISIÓN


En esta sección, podrás evaluar tu proceso Aquí, podrás evaluar el trabajo
de aprendizaje mediante una serie de individual y colectivo realizado en la
preguntas asociadas a los contenidos, Misión. Además, deberás establecer si
habilidades, estrategias y actitudes esta se incluirá en el Proyecto del texto.
desarrolladas en la lección.

8 Conoce tu texto
A PONER EN PRÁCTICA
A lo largo del desarrollo de cada
lección, se presentan talleres
experimentales en los que podrás
vivenciar las distintas etapas de
una investigación científica.

ETAPAS DE LA INVESTIGACIÓN CIENTÍFICA


Para llevar a cabo una investigación científica, se emplea un
método preciso y cuidadoso con el propósito de estudiar y
OBSERVAR comprender los fenómenos que acontecen en la naturaleza.
Para ello, es posible distinguir las siguientes etapas, las cuales
Muchas veces es casual, pero comúnmente es no necesariamente siguen una secuencia determinada.
activa y dirigida a partir de la curiosidad. De esta
manera, lo observado se analiza, se relaciona con
conocimientos científicos anteriores y se registra
por su potencial relevancia. ANALIZAR Y CONCLUIR
Es la explicación de los resultados
obtenidos. Se interpretan tanto los
resultados esperados como los no
PLANTEAR UN PROBLEMA esperados.
Y FORMULAR UNA HIPÓTESIS
En esta etapa, se plantea una interrogante o
problema a partir de las observaciones con EVALUAR
el objetivo de delimitar el fenómeno que se Se refiere a la revisión del procedimiento
investigará. realizado. Para ello, es necesario considerar
aspectos como la selección de materiales, la
rigurosidad en la ejecución de los pasos, en
las mediciones y en el análisis, la identificación
EXPERIMENTAR y corrección de los errores y la calidad de las
En esta se planifica y se desarrolla fuentes de información utilizadas.
un procedimiento experimental
que permita responder la pregunta
planteada. Para ello, es necesario
relacionar las variables de estudio. COMUNICAR
Consiste en dar a conocer los resultados de la
investigación científica y las conclusiones obtenidas
REGISTRAR Y ORGANIZAR a partir de ella. En esta etapa, se deben explicar los
nuevos conocimientos adquiridos y los procesos
Es la recolección y registro de los datos que emprendidos mediante un lenguaje claro y
surgen durante el procedimiento experimental preciso, que incluya la explicación de los conceptos
aplicado. Los resultados deben ser organizados, de mayor complejidad.
entre otros recursos, en tablas de datos,
gráficos, figuras y esquemas.

Física • 2.° Medio 9


Conoce tu texto

INTEGRO LO QUE APRENDÍ


Es una instancia evaluativa que te permite
saber cómo te encuentras en tu proceso
de aprendizaje respecto de las nociones
esenciales de la lección.

¿CÓMO VOY?
Aquí podrás identificar el nivel de
logro obtenido en la evaluación. A
partir de este, podrás valorar los
aprendizajes adquiridos respondiendo
las preguntas de la sección Reflexiono
sobre mi desempeño.

Evaluación para
el aprendizaje

LA CIENCIA SE CONSTRUYE
Esta instancia permite conocer
cómo se ha construido el
conocimiento científico a lo
largo del tiempo y, a su vez,
relacionar distintos avances que
se han producido en el estudio
de la ciencia con diferentes
eventos históricos.

10 Conoce tu texto
SÍNTESIS
Corresponde a una instancia en
la que se destacan las nociones
esenciales de la unidad. En esta,
se propone como herramienta
de síntesis la construcción de
un organizador gráfico que
te permitirá esquematizar
los contenidos, habilidades y
actitudes de la unidad.

EVALUACIÓN FINAL
Para cerrar la unidad, se propone
una instancia evaluativa de los
contenidos, en la que se miden,
principalmente, habilidades de
orden superior, como analizar,
aplicar y evaluar.

Actitudes
En las diversas actividades y evaluaciones del texto, encontrarás preguntas
actitudinales que pretenden identificar tu disposición por aprender. Estas pueden
incluir componentes afectivos, cognitivos y valorativos.

Material de apoyo
Al final del texto, se presenta una serie de herramientas, cuyo objetivo es apoyar tu trabajo
escolar a lo largo del año. Acá encontrarás el índice temático, el glosario, anexos y bibliografía
que podrás utilizar para estudiar.

MÁS ALLÁ DEL TEXTO


Para descubrir nuevas actividades y profundizar en los aprendizajes, pídele ayuda a tu profesor(a)
para acceder a los recursos digitales que se sugieren en el texto. A lo largo de tu texto, también
encontrarás códigos que podrás ingresar en la página http://codigos.auladigital.cl para ver los sitios
web sugeridos.

Física • 2.° Medio 11


Proyecto del texto

Los aprendizajes propuestos en este texto El movimiento de un objeto


se relacionan de manera transversal con la
siguiente gran idea de la ciencia: depende de las interacciones
en que participa

A partir de esta, te proponemos realizar, junto con la ayuda de tu profesor o


profesora, el gran proyecto del texto, en el que deberás elaborar un stand
científico que explique la importancia de esta gran idea. Para ello, a lo largo
este libro, te proponemos diversas misiones, de las cuales deberás seleccionar
7 para que sean parte de este proyecto. Así, cada vez que termines una de las
10 misiones propuestas, vuelve a estas páginas para justificar tu elección.

Primer semestre
UNIDAD LECCIÓN MISIÓN ¿POR QUÉ INCLUIRÉ ESTA MISIÓN EN EL PROYECTO?

Grabar un video que ejemplifique


1
la relatividad del movimiento.

Confeccionar una maqueta que


1 2 muestre el movimiento de un
cuerpo con velocidad variable.

Crear un paracaídas que permita


3
frenar la caída de un huevo.

Confeccionar un afiche que


muestre un caso hipotético que
1
simule lo que ocurriría si no
existieran las fuerzas.
2

Escribir la letra de una canción que


2
explique las leyes de Newton.

12 Proyecto del texto


SEGUNDO semestre
UNIDAD LECCIÓN MISIÓN ¿POR QUÉ INCLUIRÉ ESTA MISIÓN EN EL PROYECTO?

Crear un juego de mesa cuyo


1 propósito permita establecer la
relación entre trabajo y potencia.

Confeccionar una presentación en


Power Point o Prezi que plantee
3 2
una estrategia de ahorro de
energía para tu escuela.

Confeccionar una maqueta que te


3 permita ejemplificar tres tipos de
colisiones.

Crear un cortometraje que explique


1
el origen y evolución del universo.

Explicar una de las consecuencias


2
de las leyes que rigen el universo.

A continuación, te invitamos a descubrir


la Ruta del aprendizaje que te mostrará el
camino que deberás seguir para adquirir los
aprendizajes propuestos en este texto.

Física • 2.° Medio 13


Ruta del aprendizaje

Aprender forma parte de tu vida en todo momento, no solo en el colegio, sino


también en tu casa, cuando juegas y cuando compartes con otros. Se trata de un
proceso constante. Por eso, si estás atento a él, podrás sacarle el mayor provecho. ¿Qué preguntas debes
A continuación, te presentamos un esquema que te ayudará a conocer la plantearte para
secuencia de aprendizajes propuestos en el texto, junto con una serie de
preguntas que te ayudarán a reconocer tu propia manera de aprender y aprender?
mejorarla.

Primer semestre
UNIDAD 1: MOVIMIENTO UNIDAD 2: FUERZAS
Antes de iniciar la unidad

¿Qué me gustaría aprender? ¿Qué actitudes debería poner en práctica para alcanzar
¿Cómo me gustaría aprenderlo? mis metas?

Durante el desarrollo de la unidad

Lección 1: Descripción del movimiento Lección 1: Comprendiendo las fuerzas y sus efectos
¿Qué sé sobre los movimientos? ¿Por qué es importante aprender sobre las fuerzas?

Lección 2: Análisis de los movimientos horizontales


¿Qué inquietudes tengo y me gustaría resolver al estudiar Lección 2: Explicando las leyes de Newton
este tema? ¿Con cuál de mis conocimientos previos se relaciona lo
que estoy aprendiendo?

Lección 3: Análisis de los movimientos verticales


¿Con qué puedo relacionar los aprendizajes de esta
lección?

Al finalizar la unidad

¿Cómo puedo aplicar lo que aprendí en mi vida? ¿Qué aprendizajes incorporé a los que ya tenía?
¿Qué estrategia me resultó más efectiva para aprender? ¿Qué temas me gustaron más? ¿Por qué?

EL MOVIMIENTO DE UN OBJETO DEPENDE


¿De qué manera el desarrollo de las ¿De qué manera el estudio de las fuerzas se relaciona
Proyecto misiones de la unidad me permitió con la idea de que el movimiento de un objeto
comprender mejor los movimientos? depende de las interacciones en que participa?

14 Ruta del aprendizaje


Segundo semestre
UNIDAD 3: TRABAJO Y ENERGÍA UNIDAD 4: UNIVERSO
Antes de iniciar la unidad

¿Qué sé sobre este tema?


¿Qué inquietudes tengo y me gustaría resolver al estudiar?
¿Con qué lo puedo relacionar?

Durante el desarrollo de la unidad

Lección 1: Trabajo y potencia mecánica Lección 1: Origen y evolución del universo.


¿Qué actitudes debería poner en práctica para alcanzar ¿Por qué es importante aprender sobre el universo?
mis metas?

Lección 2: Energía y su conservación Lección 2: Las leyes del universo


¿Hay algún tema que no estoy comprendiendo? ¿Qué me gustaría aprender sobre las leyes del universo?

Lección 3: Impulso y cantidad de movimiento


¿Qué inquietudes tengo y me gustaría resolver al estudiar
este tema?

Al finalizar la unidad

¿Qué me generó mayores dificultades? ¿Cómo lo resolví? ¿Cómo valoro la importancia de lo que aprendí?
¿Qué nuevos desafíos tengo al iniciar un próximo tema? ¿Cuál fue mi actitud a lo largo de la unidad?

DE LAS INTERACCIONES EN QUE PARTICIPA


¿De qué manera la gran idea de la ciencia me
¿De qué manera el desarrollo de estas misiones me
permitió explicar los fenómenos naturales que
permitió comprender mejor el mundo que nos rodea?
ocurren en el universo?

¿Cuáles de todas las estrategias utilizadas en las misiones del Ahora te invitamos a descubrir el
año me resultaron más significativas?, ¿por qué? apasionante mundo de la Física siguiendo
esta ruta del aprendizaje.

Física • 2.° Medio 15


Movimiento

Propósito de la unidad
La presente unidad tiene como hilo conductor el movi- asignando especial importancia a los conceptos de posi-
miento, de manera que las actividades que se desarrollan ción, tiempo, desplazamiento, velocidad media, rapidez y
en el texto se articulan de acuerdo con los contenidos, ha- aceleración. Dentro de este estudio, se incluyen también los
bilidades y actitudes que se derivan del estudio de este. Su movimientos provocados por la acción de la aceleración de
principal finalidad es que los y las estudiantes describan el gravedad, como la caída libre y el lanzamiento vertical.
movimiento de objetos en una dimensión por medio de las Por su parte, la unidad de la Guía didáctica tiene como pro-
22 magnitudes que los caracterizan y establezcan relaciones en- pósito apoyar, desde la labor docente, la adquisición de los
tre estas últimas. aprendizajes, habilidades y actitudes por parte de los y las
Se espera que las y los estudiantes comprendan que, para estudiantes. Para ello, se entrega una serie de orientaciones
describir el movimiento de un cuerpo, se requiere de un didácticas, actividades complementaras e instancias de apo-
sistema de referencia, que se escoge de manera arbitraria y yo para la evaluación. Por esa razón, con la presente unidad
según conveniencia. Asimismo, se espera que identifiquen, del Texto del estudiante y de la Guía didáctica, se espera pro-
en situaciones cotidianas y cercanas, las características mover y apoyar el desarrollo de los siguientes contenidos,
de los movimientos rectilíneo uniforme y uniformemente habilidades, actitudes, Objetivos Fundamentales Transver-
acelerado, tanto en forma analítica como en forma gráfica, sales y grandes ideas de las ciencias.

Contenidos Habilidades de Investigación Científicas (HIC)


• Sistema de referencia y sistema de coordenadas. En esta unidad aplicarán las siguientes habilidades de inves-
• Relatividad clásica y adición de velocidades de Galileo. tigación científicas:
• Conceptos de trayectoria, distancia recorrida, desplaza- • Observar y plantear preguntas. (HIC 1)
miento, rapidez y velocidad. • Planificar y conducir una investigación. (HIC 2)
• Diferenciación entre rapidez de velocidad, rapidez • Procesar y analizar la evidencia. (HIC 3)
media de rapidez instantánea y velocidad media de • Evaluar una investigación. (HIC 4)
velocidad instantánea.
• Comunicar una investigación. (HI 5)
• Descripción analítica y gráfica del movimiento rectilí-
Guía Didáctica del Docente

neo uniforme (MRU).


• Descripción analítica y gráfica del movimiento rectilí-
neo uniforme acelerado (MRUA).
• Confección de gráficos para el MRU y para el MRUA, de
posición en función del tiempo, de velocidad en fun-
ción del tiempo y de aceleración en función del tiempo.
• Interpretación de gráficos del MRU y MRUA.

Unidad 1 • Movimiento
Objetivos Fundamentales Transversales (OFT)
En la unidad se promueve el logro de los siguientes OFT:
• Dimensión cognitiva-intelectual. (OFT 1)
• Proactividad y trabajo (OFT 3)
• Tecnologías de la información y la comunicación (TIC) (OFT 5)

Actitudes
En las actividades propuestas se promueven las siguientes actitudes:
• Mostrar curiosidad, creatividad e interés por conocer y comprender los fenómenos del entorno na-
tural y tecnológico, disfrutando del crecimiento intelectual que genera el conocimiento científico y
valorando su importancia para el desarrollo de la sociedad. (OA A)
• Trabajar responsablemente en forma proactiva y colaborativa, considerando y respetando los
variados aportes del equipo y manifestando disposición a entender los argumentos de otros en las
soluciones a problemas científicos. (OA C)
• Usar de manera responsable y efectiva las tecnologías de la comunicación para favorecer las expli-
caciones científicas y el procesamiento de evidencias, dando crédito al trabajo de otros y respetan-
do la propiedad y la privacidad de las personas. (OA E)

Grandes ideas
• Los organismos tienen estructuras y realizan procesos para satisfacer sus necesidades y responder
al medio ambiente. (GI 1)
• El movimiento de un objeto depende de las interacciones en que participa. (GI 7) 23

Organización de los contenidos de la unidad del Texto del estudiante


Los contenidos de la unidad se organizan en tres lecciones. Ello, con el fin de comprender las caracte-
rísticas de los movimientos rectilíneos uniformes y acelerados. El siguiente esquema muestra, en una
panorámica general, cómo se organizan los contenidos en la unidad del Texto del estudiante.

Movimiento

Lección 1: Descripción Lección 2: Análisis de los Lección 3: Análisis de los


del movimiento movimientos horizontales movimientos verticales

¿Cúando nos movemos? Describiendo el ¿Qué características


Movimiento Rectilíneo posee la caída libre?
Uniforme (MRU)
Guía Didáctica del Docente
¿Qué características
Analizando la Movimiento Rectilíneo posee el lanzamiento
relatividad del Uniformemente vertical?
movimiento Acelerado (MRUA)

Física • 2.º medio


Planificación de la unidad Tiempo estimado: 20 horas pedagógicas

La siguiente propuesta de planificación considera los Aprendizajes Esperados (AE), Objetivos de Apren-
dizaje (OA) y los Indicadores de Evaluación (IE) asociados a cada uno de ellos para cada lección del Texto
del estudiante.

Lección Tiempo Actitud OFT Aprendizajes esperados

1 6 Mostrar curiosidad, creatividad Dimensión Reconocer la importancia de establecer


e interés por conocer y compren- cognitiva-intelectual un sistema de referencia y un sistema
der los fenómenos del entorno de coordenadas para describir el movi-
Proactividad y trabajo
natural y tecnológico, disfrutan- miento de un objeto.
do del crecimiento intelectual Tecnologías de la informa-
Analizar de manera cualitativa y cuan-
que genera el conocimiento cien- ción y la comunicación
titativa variadas situaciones cotidianas
tífico y valorando su importancia
de movimiento rectilíneo: uniforme y
para el desarrollo de la sociedad.
uniforme acelerado.
2 10 Trabajar responsablemente en
Analizar de manera cualitativa y cuan-
forma proactiva y colaborativa,
titativa la caída libre y los lanzamientos
considerando y respetando los
verticales.
variados aportes del equipo y
manifestando disposición a en-
tender los argumentos de otros
en las soluciones a problemas
científicos.

3 4 Usar de manera responsable y


efectiva las tecnologías de la
comunicación para favorecer
24 las explicaciones científicas y el
procesamiento de evidencias,
dando crédito al trabajo de otros
y respetando la propiedad y la
privacidad de las personas.

Notas:
Guía Didáctica del Docente

Unidad 1 • Movimiento
ad

unid
1

Objetivos de aprendizaje Indicadores de Evaluación

Analizar, sobre la base de la experimentación, el movimien- IE 1. Demuestran, con experimentos sencillos, por qué es necesario
to rectilíneo uniforme y acelerado de un objeto respecto de el uso de sistemas de referencia y de coordenadas en la descripción
un sistema de referencia espacio-temporal, considerando del movimiento de un objeto.
variables como la posición, la velocidad y la aceleración en
IE 2. Utilizan las fórmulas de adición de velocidades de Galileo en si-
situaciones cotidianas. (OA 9)
tuaciones simples y cotidianas, como la de vehículos que se mueven
unidimensionalmente.
IE 3. Explican conceptos de cinemática, como tiempo transcurrido,
posición, desplazamiento, distancia recorrida, velocidad media e
instantánea y aceleración, entre otros, asociados al movimiento
rectilíneo de un objeto.
IE 4. Identifican características de la cinemática del movimiento
rectilíneo, en situaciones cotidianas, como ocurre con vehículos,
entre otros ejemplos.
IE 5. Analizan, con conceptos de cinemática y herramientas gráficas
y analíticas, el movimiento rectilíneo de un objeto en situaciones
cotidianas.
IE 6. Explican el concepto de aceleración de gravedad y consideran
su uso en situaciones de caída libre y lanzamientos verticales.
25
IE 7. Obtienen conclusiones, en relación con conceptos de cinemá-
tica, a partir de investigaciones experimentales sobre objetos con
movimiento rectilíneo con aceleración constante (nula o no nula).

Notas:

Guía Didáctica del Docente

Física • 2.º medio


Inicio de la unidad Páginas 16 a 19

¿Por qué es importante analizar las características de los Motivación para el aprendizaje
movimientos?
Páginas 16 y 17 “Teniendo en cuenta la diversi-
dad de alumnos existente, ¿de
Con el fin de motivar a sus estudiantes en el estudio de los aprendizajes de la uni- qué modo afecta la motivación de
dad, se propone el análisis de una conversación que entablan dos adolescentes cada uno de ellos? ¿Atribuyen los
en torno a las características del movimiento que describe un cuerpo en caída alumnos a las distintas pautas de
libre. A partir del diálogo que inician Romina y Miguel, se pueden activar algunos actuación docente que configuran
conocimientos previos de las y los estudiantes. Para ello, solicite que, a partir los diferentes entornos de apren-
de la información que se entrega en la ilustración, respondan las preguntas de la dizaje el mismo valor motivador
actividad de la página 17. que les atribuyen los modelos teó-
ricos? ¿En qué grado la diferente
orientación motivacional de los
¿Quién está en lo correcto Romina o Miguel? alumnos previa a la instrucción
Páginas 18 y 19 determina el grado en que esta,
cuando se diseña sobre la base
Con el propósito de seguir indagando en los aprendizajes previos de las y los de los modelos descritos, mejo-
estudiantes, en estas páginas se propone una secuencia de procedimiento que ra el interés y la motivación por
permitan establecer cuál de los adolescentes está en lo correcto. Para ello, sus aprender? Responder estas cues-
estudiantes deberán anticipar los resultados de cada experiencia a partir de sus tiones es importante para poder
conocimientos previos. Es importante destacar que en esta instancia todas las res- mejorar la eficiencia motivacional
puestas son útiles, ya que se trata de una etapa de exploración; por eso, más allá de los entornos de aprendizaje y
de centrarse en lo correcto o incorrecto de las respuestas entregadas, estimúlelos ajustarlos a las distintas clases de
a expresar sus ideas. alumnos”.
26
Para finalizar con esta etapa de motivación, señale a sus estudiantes que en el J. Alonso Tapia (2005)
Texto tendrán la oportunidad de validar sus respuestas.
Guía Didáctica del Docente

Evaluación de conocimientos previos

Utilice estas páginas para evaluar los conocimientos previos de sus estudian-
tes. Para ello, solicite que respondan individualmente las preguntas planteadas;
luego, motive a quienes lo deseen a exponer sus respuestas para analizar las
situaciones descritas.

Unidad 1 • Movimiento
ad

unid
1
Qué aprenderás y descubrirás en la unidad? Metacognición
Página 20
La secciones propuestas en estas
Esta sección entrega a las y los estudiantes las principales metas que se espera páginas (20 y 21) están orienta-
que alcancen en la unidad, las cuales se enfocan en los aprendizajes propuestos. das al proceso metacognitivo de
A su vez, para cada meta establecida, en ¿Cómo alcanzarlas? se señalan las princi- sus estudiantes. Considere que la
pales acciones que deberán desarrollar, a lo largo de la unidad, considerando las interrogación metacognitiva se
estrategias que les permitirán el logro de estos aprendizajes. Por último, ¿Para qué entiende como la conciencia del
alcanzarlas? destaca la importancia que tiene para las y los estudiantes el logro autoconocimiento del aprendiza-
de los aprendizajes propuestos, teniendo en cuenta las actitudes que promueve la je, de manera que las preguntas
unidad. Para finalizar esta sección, motívelos a establecer sus propias metas, consi- ayuden a tomar decisiones que
derando las estrategias que emplearán para lograrlas y el propósito que promueve faciliten la asimilación de nuevos
cada una de ellas considerando el trabajo metacognitivo. aprendizajes. Considere que las
fases de la interrogación metacog-
nitiva son planificación, ejecución
¿Cómo te gustaría protagonizar tu propio aprendizaje? y evaluación. Las preguntas que
Página 21 se plantean en cada una de estas
fases deben ser distintas, ya que
Esta sección permite recoger las diferentes motivaciones de las y los estudiantes. están dirigidas a trabajar diferen-
Se espera que, a partir de ellas, planifiquen su trabajo en función de sus aspiracio- tes procesos de la metacognición.
nes y estrategias de estudio. En este caso en particular, la motivación está enfocada Como el trabajo de estas páginas
en los aprendizajes del movimiento, de tal manera que ellos mismos logren men- está enfocado en la etapa de plani-
cionar los contenidos, estrategias y actitudes que desean desarrollar a lo largo de ficación, puede sugerir a sus estu-
la unidad, anticipando, además, las principales dificultades a las cuales se podría diantes que respondan preguntas
enfrentar. Invítelos a compartir con el resto del curso sus motivaciones, estimulan- como: 27
do el respeto por las ideas de otro y la expresión de las mismas.
• ¿Qué conceptos aprendidos
con anterioridad me pue-
¿Cómo lograr mis metas? den servir para satisfacer
Página 21
las inquietudes de mis
motivaciones?
En esta instancia, se propone a las y los estudiantes una estrategia para el logro de • ¿Qué manera de acercarme
sus metas. Para ello, guíelos para establecer los pasos que les permitirán aplicar a un nuevo conocimiento
con éxito la estrategia y motívelos a que su propuesta sea presentada cuando la me resulta más cómoda y
desarrollen. Como en este caso la estrategia es escribir la letra de una canción, los natural?
pasos que se pueden considerar son los siguientes: • ¿Qué métodos son las óptimos
1. Escoger una canción con la melodía que más le guste. para conseguir los objetivos
2. Escribir la historia de la canción, considerando las metas establecidas. que me planteo?
3. Hacer coincidir la letra con la melodía de canción, para luego cantarla. • ¿Qué técnicas respetan mi
estilo personal de trabajo?
Recuerde que las respuestas de sus
estudiantes en esta fase de la in-
terrogación metacognitiva pueden
ser retomadas una vez que se haya
avanzado en la unidad; ello, con el Guía Didáctica del Docente
objetivo de revisarlas y replantear-
las, si fuese necesario.

Física • 2.º medio


Desarrollo de la unidad Páginas 22 a 65

LECCIÓN 1: Descripción del movimiento

Las actividades propuestas en el Texto del estudiante tienen como propósito el logro de
los Aprendizajes Esperados (AE), abordando con diferentes estrategias los Indicadores
de Evaluación (IE) e incorporando en cada una de ellas el trabajo con habilidades,
actitudes y Objetivos Fundamentales Transversales (OFT).
AE IE Actividad HIC Actitud/OFT
Reconocer la importancia IE 1-2 Inicio de la misión (P. 23) HIC 2 OA A/OFT 1
de establecer un sistema IE 1 Indaguemos (P. 24) HIC 1-2 OA A/OFT 1
de referencia y un sistema IE 3-4 Determinemos la posición (P. 25) HIC 3 OA C/OFT 3
de coordenadas para
describir el movimiento de IE 3 Indaguemos (P. 26) HIC 1-2 OA A/OFT 1
un objeto. IE 3-4 Determino la distancia recorrida y el desplazamiento (P. 27) HIC 3 OA A/OFT 1
IE 3 Indaguemos (P. 28) HIC 1-2-3 OA A/OFT 1
IE 3 Aplico la ecuación de rapidez (P. 28) HIC 3 OA A/OFT 1
IE 3 Desarrollo de estrategias (P. 29) HIC 3 OA A/OFT 1
IE 1 Establezcamos un sistema de referencia (P. 31) HIC 3 OA A/OFT 1
IE 1 Indaguemos (P. 32) HIC 1-3 OA A/OFT 1
IE 1 Desarrollo de de la misión (P. 33) HIC 2-5 OA A/OFT 1
IE 2 Indaguemos (P. 34) HIC 1 OA A/OFT 1
IE 1-2 Cierre de de la misión (P. 35) HIC 4 OA A/OFT 1
IE 1-2-3-4 Integro lo que aprendí (P. 36 y 37) HIC 3 OA A/OFT 1

28
Propósito de la lección A continuación se presenta una serie de orientaciones para
trabajar los temas, actividades y secciones más importantes
En esta lección se trabajan los conceptos que permiten des- presentes en la lección del Texto del estudiante. Junto con
cribir el movimiento de los cuerpos, según lo propuesto en ello actividades e información complementarias, entre otros
los Aprendizajes Esperados. Lo anterior se desarrolla a par- recursos, presentes en la Guía didáctica.
tir de los recursos y las actividades presentes en el Texto del
estudiante y en la Guía didáctica, en función del logro de
los Indicadores de Evaluación considerando, además del Ciencia, tecnología y sociedad P. 22
contenido, las habilidades y acticudes.
En el Texto del estudiante, las diferentes temáticas se desa- Esta sección tiene por objetivo promover la alfabetización
rrollan con una propuesta didáctica que trabaja los conteni- científica de los estudiantes sobre los contenidos de la lec-
dos comenzando con actividades Indaguemos que buscan la ción, utilizando como insumo los textos El sistema de Posicio-
activación de los conocimientos previos para posteriormente namiento Global y Alertas de tsunamis más rápidas y efectivas
formalizarlos en el contenido tratado en el texto. Luego, se gracias al GPS. En estas, se intencionan preguntas para co-
presentan actividades que abarcan distintas habilidades, enfo- menzar una discusión con los estudiantes, de manera de pro-
cándose principalmente en las de orden superior, ciclo que se mover la argumentación de las ideas a partir de la escritura o
repite a lo largo de la lección. De forma articulada al desarrollo la oralidad. Según Sardà y Sanmartí (2000), en la construc-
del contenido, en los Desarrollo de estrategias se entregan he- ción del conocimiento científico, la argumentación es crucial
rramientas para resolver problemas. Asimismo, a lo largo de en el proceso de negociación de significados. En este sentido,
la lección se incorpora el trabajo de las actitudes con los OFT. es fundamental en el intercambio de diversas ideas en el aula
Guía Didáctica del Docente

por medio de argumentos, que posibiliten la construcción del


En la Guía didáctica se presentan pautas para poder utilizar conocimiento y con un uso adecuado del lenguaje científico.
algunas de las actividades del Texto del estudiante como ins- Luego de leer ambos textos, formule las siguientes preguntas:
trumentos de evaluación, fichas de trabajo para los dife- ¿Cuáles son los principales usos del GPS? ¿Cómo impacta el
rentes ritmos de aprendizaje, actividades complementarias desarrollo de esta tecnología en la sociedad? ¿Cómo creen
y evaluaciones, con sus respectivos solucionarios. que hubiera sido el impacto de la tecnología GPS si esta no
fuera de uso público? ¿Cómo creen que sería la sociedad si
más tecnologías fueran de uso libre?, ¿qué ejemplos pueden
mencionar?
Unidad 1 • Movimiento
ad

unid
1
Inicio de la misión (P. 23)
Ciencia al día P. 23 ¿Cómo conducir una investigación?
Esta sección, es parte de Ciencia, tecnología y sociedad por lo Esta sección es parte del gran proyecto propuesto para el
que de la misma manera, pretende la construcción del cono- texto, en el cual se propone que, en cada lección, las y los es-
cimiento científico, pero destacando el desarrollo científico tudiantes desarrollen una misión. En este caso en particular,
contemporáneo y el rol de chilenos. De forma adicional a la misión tiene como finalidad integrar los contenidos actitu-
las preguntas presentadas en el texto, puede plantear a sus dinales propuestos para esta unidad a través de la grabación
estudiantes las siguientes: ¿Qué científicos chilenos cono- de un video que ejemplifique la relatividad del movimiento.
cen? ¿Cómo consideran que es la divulgación de las investi- Para ello, se realiza un trabajo interdisciplinar en que se con-
gaciones que se realizan en Chile?, ¿a qué creen que se deba? jugan otras asignaturas, como Tecnología y el uso de las TIC
¿De qué manera se podría destacar el aporte de científicos (tecnologías de la información y la comunicación). Se sugie-
chilenos en el conocimiento colectivo? re organizar esta actividad con el encargado de informática
del colegio con el fin de realizar un trabajo interdisciplinar y
Centros de investigación en Chile consignar los espacios necesarios para llevarlo a cabo.
En esta instancia, las y los estudiantes deben planificar su
Según el Consejo Superior de Investigaciones Científi- trabajo. Para ello, guíe su labor considerando que para esta
cas CSIC de España, la Comisión Nacional de Científica investigación científica deben considerar lo siguiente:
y Tecnológica CONICYT es el segundo mejor centro de in-
vestigación del país. CONICYT fue creada en 1967 como
• Seleccionar un plan de acción que permita solucionar
una problemática.
organismo asesor de la Presidencia en materias de desa-
rrollo científico. Desde hace más de 40 años, CONICYT • Explicar el propósito y el procedimiento de cada parte
ha estado presente en cada una de las iniciativas de apo- de la secuencia de actividades propuestas en su plan
yo al fortalecimiento de la ciencia y tecnología en Chile, de trabajo.
orientando sus esfuerzos al objetivo final de contribuir al • Exponer cómo implementarán la estrategia solicitada
progreso económico, social y cultural del país. para comunicar los resultados. 29

En 1982 se crea el Fondo Nacional de Desarrollo Científico Considere que este plan de trabajo se desarrollará cuando las
y Tecnológico (Fondecyt), como el principal fondo público y los estudiantes estudien todos los aprendizajes necesarios
de apoyo a la investigación individual en Chile. Diez años de la lección. En este caso en particular, el desarrollo de la
después, nace el Fondo de Fomento al Desarrollo Científi- misión se llevará a cabo en la página 33 del texto.
co y Tecnológico (Fondef), que busca promover la vincu-
lación y asociatividad entre instituciones de investigación
y empresas, con el objetivo de desarrollar proyectos de Ventana de profundización didáctica
investigación aplicada, desarrollo precompetitivo, interés Los proyectos
público y transferencia tecnológica.
Los proyectos son actividades que invitan a las y los es-
Actualmente, CONICYT cuenta con un total de 13 progra- tudiantes a problematizar la realidad. Generalmente re-
mas para el apoyo y desarrollo de la investigación de las quieren de un tiempo largo para su realización y de cierta
ciencias dentro de las cuales se encuentra: autonomía. Además, les permiten relacionarse de manera
• EXPLORA: Programa Nacional de Divulgación y directa con el mundo natural y socio-tecnológico del que
Valoración de la Ciencia y la Tecnología. son parte. Cuando los niños y niñas tienen la oportuni-
• FONDAP: Fondo de Financiamiento de Centros de dad real de desarrollar un proyecto a largo plazo, se está
Investigación en Áreas Prioritarias. promoviendo en ellos el interés por conocer más acerca
de un determinado fenómeno. Al mismo tiempo, estas ac-
• REGIONAL: Programa Regional de Investigación tividades son altamente motivadoras, ya que consideran
Científica y Tecnológica.
las experiencias previas, respondiendo así a los intereses
• FONDEQUIP: Fondo de Equipamiento Científico y y fortaleciendo el compromiso de las y los estudiantes con
Guía Didáctica del Docente
Tecnológico. el medio ambiente y la sociedad. Mediante este método,
Fuente: http://www.conicyt.cl/ se estimula el espíritu crítico, fundamentalmente a par-
tir de la formulación de preguntas sobre el mundo en que
vivimos.
Finalmente, estas actividades desarrollan una variedad
de habilidades cognitivas y metacognitivas, tales como

Física • 2.º medio


Desarrollo de la unidad
investigar, planificar y comunicar. Dichas habilidades son Puede evaluar las habilidades y actitudes científicas de la
indispensables para el proceso de enseñanza-aprendizaje actividad realizada, usando la siguiente rúbrica:
en las Ciencias Naturales, por lo que los proyectos son un
Rúbrica de evaluación de habilidades y actitudes
aporte significativo para el área.
Niveles de logro
Fuente: La Cueva, A. (1998). La enseñanza por proyectos: Indicadores
L ML PL
¿mito o reto? Revista Iberoamericana de Educación, 16, 165-190.
(Adaptación) Conducen adecuadamente la experiencia.
Describen detalladamente las características
del suceso observado.
Tema 1: ¿Cuándo nos movemos? Páginas 24 a 34
Definen el concepto de movimiento a partir
Para que sus estudiantes sean capaces de describir los mo- de las características de la experiencia
vimientos, deben comprender los parámetros con los cuales realizada.
es posible hacerlo: la posición, el desplazamiento, la distan- Argumentan la importancia de las habi-
cia, la rapidez y la velocidad. Puede proponerles desarrollar lidades y el conocimiento para explicar
un esquema con estos conceptos y con las ideas previas que fenómenos del entorno.
manejan en torno a ellos, para volver a revisarlo al final de L = Logrado; ML = Medianamente logrado; PL = Por lograr.
la lección, completándolo con los conocimientos adquiridos.
Actividad complementaria
Activación de conocimientos previos
Lee la siguiente situación:
Para comenzar con el estudio de este tema, puede preguntar
a sus estudiantes: ¿qué variables físicas se deben considerar Patricia se encuentra hablando por teléfono en el balcón
si se quiere describir el movimiento de un cuerpo?, ¿cómo se de su departamento. En el piso inferior, Sebastián saca una
le pueden asignar cualidades a un determinado movimien- caja y, más abajo, Camila deja una toalla para que se seque.
to? La idea es que utilicen sus propios términos para respon- Tal como se muestra en la siguiente imagen.
30
der, lo que le permitirá detectar los aprendizajes de entrada.
Habilidades y actitudes científicas
Las actividades Indaguemos propuestas en el Texto del estu-
diante pretenden que los y las estudiantes conozcan, desde
su propia experiencia, lo que implica la actividad científica;
es decir, que adquieran habilidades de investigación cien-
tífica mediante la práctica. Se espera que de este modo,
comprendan también cómo se genera el conocimiento cien-
tífico cuando se efectúa de forma integral, permitiendo la
comprensión y la reflexión sobre lo aprendido, generando
nuevos conocimientos a partir de los conceptos previos.
Actividad del texto (P. 24)
Indaguemos
Es importante que considere que en todas las actividades del
Texto del estudiante, con un carácter exploratorio y/o expe-
rimental, se declaran el objetivo, las habilidades y actitudes En relación con la situación descrita, responde:
que en ella se trabajan. De forma adicional, se explicita el a. ¿Qué objeto representado en la imagen se encuentra
tiempo aproximado en el que se espera que sean desarrolla- debajo de Patricia, frente a Sebastián y arriba de
das. La actividad propuesta busca que los y las estudiantes Camila?
Guía Didáctica del Docente

reconozcan sus ideas previas sobre los sistemas de referen-


cia necesarios para localizar un objeto en el espacio. Por otra b. ¿Cómo señalarías la antena parabólica que se
parte, los invita a identificar las habilidades necesarias para muestra en la escena?
construir nuevos aprendizajes con el fin de que comprendan c. ¿Qué elementos son necesarios al momento de
su propio proceso de aprendizaje. Al finalizar la actividad, se describir la ubicación de un objeto?
sugiere realizar una sesión plenaria para comparar las res-
d. ¿Se podría determinar la ubicación de un objeto si no
puestas y discutirlas, valorando los distintos puntos de vista.
existiera ningún tipo de referencia? Explica.

Unidad 1 • Movimiento
ad

unid
1
Actividad del texto (P. 25) terminología se utilizará a lo largo de la unidad y les permi-
Determinemos la posición de las personas tirá distinguir las magnitudes que describen el movimiento,
por ejemplo, la rapidez de la velocidad. Antes de revisar los
Con el desarrollo de esta actividad, se espera que sus estu-
contenidos de estas páginas haga una introducción en que
diantes puedan determinar la posición de algunas personas
diferencie claramente una magnitud escalar de una vecto-
a partir de diferentes sistemas de referencia. Es importante
rial, indicando las características de cada una.
que, a partir de esta, comprendan que la descripción de cual-
quier movimiento siempre está sujeta al sistema de referen- Actividad del texto (P. 26)
cia que se utiliza. Indaguemos
Puede evaluar la actividad realizada, usando la siguiente La actividad propuesta busca que los y las estudiantes
rúbrica: identifiquen la diferencia entre la distancia recorrida y el
desplazamiento a partir de sus ideas previas. Como esta ex-
Rúbrica de evaluación
periencia necesita de un insecto, sería interesante realizar la
Niveles de logro actividad al aire libre, considerando que la física es la ciencia
Indicadores
L ML PL que pretende explicar todos los fenómenos que ocurren en
Determinaron la posición de cada persona el entorno natural.
de acuerdo a cada sistema de referencia Puede evaluar las habilidades y actitudes científicas de la
establecido.
actividad realizada, usando la siguiente rúbrica:
Señalaron correctamente la importancia
de establecer un sistema de referencia para Rúbrica de evaluación de habilidades y actitudes
describir un movimiento. Niveles de logro
Indicadores
Identificaron los aprendizajes que utilizaron L ML PL
para realizar la actividad.
Conducen adecuadamente la experiencia.
L = Logrado; ML = Medianamente logrado; PL = Por lograr.
Identifican correctamente la representa-
ción gráfica de la distancia recorrida y el 31
Actividad complementaria desplazamiento.
Tres amigos, Paula, Esteban y Catalina, se encuentran sen- Señalan la importancia del estudio del movi-
tados en las gradas del gimnasio de su colegio, mientras miento para la comprensión de conceptos.
Daniel los observa, tal como se muestra en la imagen.
L = Logrado; ML = Medianamente logrado; PL = Por lograr.
Paula

Esteban Actividad del texto (P. 27)


Z (m)
3 Determino la distancia recorrida y el desplazamiento

David
Catalina
2 Esta actividad permite poner en práctica los aprendizajes ad-
quiridos acerca de la distancia recorrida y el desplazamiento.
1 Por tratarse de una actividad individual, es necesario guiar a
las y los estudiantes en la interpretación del contexto para que
X (m) puedan determinar lo solicitado. Si lo considera necesario, re-
8
5 6 7 suelva esta actividad en la pizarra, en conjunto con ellos. Es
Y (m) 1 4
2 3 2
3 importante que, en esta instancia, sus estudiantes puedan evi-
4 1
denciar numéricamente la diferencia entre el desplazamiento
Si el sistema de referencia se sitúa en los pies de Daniel, y la distancia recorrida, ya que, a partir de estos conceptos, se
responde: obtienen otras variables que permiten describir el movimiento.
a. ¿Cuál es la posición de cada amigo?1 Para profundizar estos contenidos, puede realizar la siguiente
pregunta: ¿de qué manera se podría calcular el desplazamien- Guía Didáctica del Docente
b. ¿Quién se encuentra más lejos de Daniel?
to en un plano o en el espacio? Si bien la pregunta excede los
¿Cuál es la diferencia entre distancia recorrida y des- propósitos de aplicación de la lección, permite que los y las
plazamiento? (P. 26) estudiantes conozcan las limitaciones del modelo unidimen-
sional. Puede complementar con preguntas como la siguiente:
Los conceptos de desplazamiento y distancia, de la página
¿Con qué sistema de coordenadas se podría describir el mo-
27, introducirán a sus estudiantes el concepto de vector y
vimiento de un jugador por una cancha de fútbol, un pájaro
escalar. Es muy importante que comprendan en esta ins-
por el aire o de un equilibrista que camina por la cuerda floja?
tancia la diferencia entre ambos conceptos, ya que esta

Física • 2.º medio


Desarrollo de la unidad
Puede evaluar la actividad realizada, usando la siguiente Actividad del texto (P. 28)
rúbrica: Aplico la ecuación de rapidez
Rúbrica de evaluación Guíe a sus estudiantes en el trabajo con la expresión que des-
Niveles de logro cribe la rapidez, ya que despejar las diferentes variables que
Indicadores la componen puede implicar una dificultad para el posterior
L ML PL
cálculo. Indíqueles los criterios que comúnmente se utilizan
Determina correctamente la distancia con este fin y señáleles que los pueden aplicar luego en otras
recorrida. expresiones.
Determina correctamente el desplazamiento.
Desarrollo de estrategias (P. 29)
Establece las características actitudinales
para mantener la curiosidad y el interés por Realice el esquema del problema en la pizarra y resuélva-
aprender. lo paso a paso con sus estudiantes; ello, con el objetivo de
L = Logrado; ML = Medianamente logrado; PL = Por lograr. incorporar este tipo de estrategia para resolver problemas,
de modo que puedan aplicarla luego en el paso 5 aplico lo
¿Cuál es la diferencia entre rapidez y velocidad? (P. 28) aprendido. Es importante que, en esta instancia, sus estu-
diantes puedan evidenciar la diferencia entre rapidez y des-
Confundir los conceptos de rapidez y velocidad suele ser un plazamiento, considerando principalmente que la primera
error frecuente entre las y los estudiantes; por ello, es im- depende de la distancia recorrida, por lo que es una magni-
portante que, en el tratamiento de estos, se evidencien sus tud escalar, mientras que la segunda depende del desplaza-
diferencias, de modo que puedan ser empleados posterior- miento, por lo que es una magnitud vectorial.
mente sin mayor dificultad.
Actividad del texto (P. 28) Actividad complementaria
Indaguemos Jorge sale de su casa (punto P de la imagen) a las 15:45 h
En esta actividad, para reemplazar los rieles solicitados, pue- rumbo a la casa de su amiga Carolina (señalada por el
32 den utilizarse listones de maderas o reglas unidas por cinta punto C), recorriendo el camino indicado en la imagen.
adhesiva. Lo fundamental es que sean superficies rígidas y 200 m
que permitan que las bolitas tengan un solo grado de libertad
para desplazarse. El tratamiento de datos de la actividad no
debería implicar dificultad, pero sí su obtención. Aproveche
100 m 100 m
esta oportunidad para promover el desarrollo de aspectos acti-
tudinales relacionados con el trabajo científico, recalcándoles
que debe existir una buena comunicación al interior del grupo
de trabajo y ser muy ordenados en sus mediciones, ya que
estas actitudes les permitirán obtener mejores resultados. P C

Puede evaluar las habilidades y actitudes científicas de la Al llegar a su destino, su reloj marca las 15:50 h. A partir
actividad realizada, usando la siguiente rúbrica: de esta situación responde:
Rúbrica de evaluación de habilidades y actitudes a. ¿Cuál fue la rapidez media de Jorge? (Expresa el
Niveles de logro resultado en m/s)
Indicadores
L ML PL b. ¿Cuál fue la velocidad media de Jorge? (Expresa el
Conducen adecuadamente la experiencia. resultado en (m/s)?
Reconocen las características que determi-
nan la rapidez de un cuerpo.
Definen correctamente el concepto de
Guía Didáctica del Docente

rapidez.
Establecen sus inquietudes al analizar los
resultados de la experiencia.
L = Logrado; ML = Medianamente logrado; PL = Por lograr.

Unidad 1 • Movimiento
ad

unid
1
Actividad del texto (P. 32)
Tema 2: Analizando la relatividad Indaguemos
del movimiento Páginas 30 a 34
Para impulsar la alfabetización científica y la naturaleza de
Alfabetización científica las ciencias, que se promueven en esta lección, antes de reali-
zar la actividad, pida a sus estudiantes que formulen hipótesis
En esta lección se trabajan los conceptos que permiten com- respecto de cómo verán la pelota. ¿Pueden afirmar que desde
prender que el movimiento de los cuerpos depende siempre ambas posiciones observarán lo mismo? Invítelos a expresar
del sistema de referencia utilizado. Para ello, se presenta un sus ideas, evitando el juicio crítico, guiándolos para que en las
contexto cotidiano con el propósito de que las y los estudian- hipótesis observen la relación de las trayectorias del objeto
tes adquieran los conceptos e ideas básicas de la relatividad según los posibles observadores. También es posible variar la
para comprender las experiencias y situaciones cercanas. actividad, pidiendo que un o una estudiante deje caer la pelo-
Trabaje en conjunto con sus estudiantes la situación que se ta mientras corre y observen qué ocurre, comparando esto con
describe en las páginas 30 y 31, respondiendo las preguntas lo observado en el caso anterior; ello, con el fin de verificar
que ahí se plantean. Para enfatizar la discusión, puede rea- que la relatividad del movimiento depende del observador.
lizar las siguientes preguntas: las aves ¿cómo verán al chofer
der bus: en reposo o en movimiento?, ¿cómo verá el chofer Puede evaluar las habilidades y actitudes científicas de la
a los árboles y a las vacas? La idea de estas preguntas es que actividad realizada, usando la siguiente rúbrica:
infieran que todos los movimientos son relativos al observa- Rúbrica de evaluación de habilidades y actitudes
dor y por ende al sistema de referencia. Niveles de logro
Indicadores
Actividad del texto (P. 31) L ML PL
Establezcamos un sistema de referencia
Conducen adecuadamente la experiencia.
Indique a sus estudiantes que, para establecer el sistema de Describen las trayectorias a partir de las
referencia solicitado, se pueden incorporar objetos, animales observaciones realizadas.
o personas a la ilustración, de forma que logren describir el Explican las características de la relatividad 33
movimiento deseado. Por ejemplo, pueden incorporar una de las trayectorias a partir de las evidencias
persona caminando dentro de bus o bien caminando por el observadas.
costado del camino. Señalan la importancia de las observa-
Puede evaluar la actividad realizada, usando la siguiente ciones para la descripción de fenómenos
rúbrica: cotidianos.
L = Logrado; ML = Medianamente logrado; PL = Por lograr.
Rúbrica de evaluación
Niveles de logro Desarrollo de la misión (P. 33)
Indicadores
L ML PL ¿Cómo comunicar una investigación?
Establecen correctamente el sistema de Para llevar a cabo el proyecto de esta lección y de acuerdo al
referencia para cada caso solicitado. trabajo interdisciplinar señalado al inicio del mismo, lleve a
Señalan las características que determinan sus estudiantes a la sala de computación para que puedan
que todos los movimientos son relativos. elaborar el video solicitado. Para asegurarse de que los ejem-
L = Logrado; ML = Medianamente logrado; PL = Por lograr. plos empleados por cada grupo de trabajo sean correctos, ge-
nere la instancia para que cada equipo le muestre su ejemplo
¿Las trayectorias pueden ser relativas? (P. 32) y sugiera ideas de mejoras para la elaboración del producto
final. Cuando cada grupo haya terminado su misión, solicite
Naturaleza de las ciencias
a las y los estudiantes que presenten su video al resto del
A partir de lo que establecen los Objetivos de Aprendizaje curso y en conjunto discutan y analicen cada ejemplo. En
de las Bases Curriculares, se espera que los y las estudian- este punto puede formular preguntas como las siguientes: Guía Didáctica del Docente
tes adquieran un conjunto de ideas sobre la naturaleza de ¿Cómo se evidencia el movimiento relativo en este ejemplo?
las ciencias. En esta lección en particular, las actividades ¿Qué pasaría si se cambia el observador, hubieran tenido el
Indaguemos pretenden promover el conocimiento científico mismo resultado? ¿De qué manera el trabajo audiovisual
basado en evidencia empírica de manera que, a partir del permite comprender mejor los fenómenos cotidianos? ¿To-
trabajo práctico, las y los estudiantes logren adquirir los nue- dos los movimientos son relativos?
vos conocimientos científicos.

Física • 2.º medio


Desarrollo de la unidad
Puede evaluar la actividad realizada, usando las siguientes
rúbricas: Mujeres en la historia de la ciencia P. 35
Rúbrica de evaluación de habilidades y actitudes Aproveche esta sección para destacar el rol de la mujer en
Niveles de logro sociedades conservadoras, en las cuales históricamente esta
Indicadores
L ML PL se centró en las labores del hogar o en el trabajo manual.
Seleccionan un plan de trabajo coherente Puede plantear algunos temas de discusión con preguntas
con la misión encomendada. como la siguiente: ¿qué importancia ha tenido el acceso
Justifican el procedimiento realizado para igualitario a la educación, en equiparar las posibilidades
llevar a cabo su trabajo. de ambos sexos, de acceso al mundo del conocimiento y el
Implementan adecuadamente la estrategia trabajo calificado? Promueva el respeto por las opiniones
para ejemplificar lo solicitados. divergentes y el derecho a expresarse por turnos. Si lo esti-
Trabajan de forma responsable y ordenada ma conveniente, solicite a sus estudiantes que realicen una
siguiendo el plan establecido. investigación sobre el aporte de las mujeres a la ciencia con-
L = Logrado; ML = Medianamente logrado; PL = Por lograr. temporánea y que escriban una reseña de la mujer que les
resulte más interesante.
Rúbrica de evaluación para la comunicación de resultados
Niveles de logro
Indicadores Ventana de profundización didáctica
L ML PL
Informan sobre la investigación realizada con
La importancia de la retroalimentación
los recursos comunicacionales utilizados.
Comúnmente la retroalimentación al trabajo realizado por
Expresan la realización de su investigación
las y los estudiantes es entregada en forma de calificación
argumentando las razones de la elección de
su ejemplo. o a partir de algún juicio de valor utilizando categorías de
Presentan su trabajo utilizando lenguaje
bueno o malo. Un estudio realizado por Ruth Butler en
científico apropiado. 1988 demostró que la retroalimentación con comentarios
34
L = Logrado; ML = Medianamente logrado; PL = Por lograr. realizados por el profesor o profesora se tradujo en ma-
yores logros para las y los estudiantes participantes de la
Actividad del texto (P. 34) investigación. Por su parte, esta investigadora logró demos-
Indaguemos trar que la retroalimentación que consistía en la entrega de
una nota y comentarios de los docentes, resultó ser poco
Guíe a sus estudiantes para que infieran sobre qué sucede efectiva, ya que aquellos tendían a concentrarse en la nota,
cuando se observa un cuerpo en movimiento desde un sis- haciendo caso omiso de los comentarios que los invitaban
tema de referencia que también está en movimiento. Puede a mejorar su trabajo. Desde esta perspectiva, se concluye
que el contexto de la actividad no les resulte cercano. En ese que los comentarios utilizados como retroalimentación son
caso, plantee otras situaciones cercanas en que puedan evi- efectivos para optimizar el proceso de aprendizaje, ya que
denciar la velocidad relativa, por ejemplo, el paradero y un los estimula a revisar su trabajo a partir de una corrección
automóvil en movimiento, un vehículo y una moto, la Tierra cualitativa, que les da la posibilidad de mejorar. Con el fin
y la Luna, entre otros. de que estos comentarios tengan el efecto deseado, es im-
Puede evaluar las habilidades y actitudes científicas de la portante que sean evaluaciones que partan desde aspectos
actividad realizada, usando la siguiente rúbrica: positivos sin utilizar juicios de valor.
Rúbrica de evaluación de habilidades y actitudes Fuente: http://www.rmm.cl/sites/default/files/
Niveles de logro evaluacion_para_el_aprendizaje.pdf (Adaptación)
Indicadores
L ML PL
Conducen adecuadamente la experiencia.
Guía Didáctica del Docente

Relacionan la experiencia con los conteni-


dos trabajados anteriormente.
Establecen la velocidad de un cuerpo desde
un sistema de referencia establecido.
Establecen una pregunta de interés respecto
a la situación analizada.
L = Logrado; ML = Medianamente logrado; PL = Por lograr.

Unidad 1 • Movimiento
ad

unid
1
Cierre de la misión (P. 35)
Integro lo que aprendí P. 36 y 37
¿Cómo evaluar una investigación?
En esta instancia de la misión, se espera que las y los estu- Invite a sus estudiantes a poner en práctica los aprendizajes
diantes evalúen su desempeño en el trabajo realizado, ana- adquiridos en la lección respondiendo las preguntas que se
lizando su trabajo personal como colectivo, con el propósito proponen en esta evalución de proceso. Una vez que hayan
de que puedan calificarlo a partir de la retroalimentación. respondido las preguntas, pida a sus estudiantes que desa-
Para ello, además de las preguntas planteadas en el Texto rrollen la sección ¿Cómo voy? y registren su nivel de desem-
del estudiante, se sugiere implementar las siguientes pautas peño según las respuestas correctas que hayan tenido. De
de evaluación: acuerdo al nivel de desempeño se propone el trabajo con ac-
tividades complementarias. El propósito de estas actividades
Pauta de auto-evaluación es trabajar según los diferentes ritmos de aprendizaje. Si
Aspectos a evaluar Sí No sus estudiantes obtuvieron un nivel de desempeño logrado,
Realicé aportes para la ejemplificación del movi- invítelos a realizar la Ficha de ampliación de la página 51.
miento relativo. A quienes obtuvieron un desempeño correspondiente a Por
lograr y Medianamente logrado, solicíteles realizar la Ficha
Fui responsable en mis tareas dentro del grupo de
trabajo.
de refuerzo de la página 50.
Trabajé con conciencia e interés en el desarrollo de Para apoyar el desarrollo de la metacognición de sus estu-
la misión. diantes solicite que respondan las preguntas planteadas en
Respeté las decisiones del grupo con el propósito de la sección Reflexiono sobre mi desempeño, enfóquese en moti-
desarrollar satisfactoriamente el proyecto. varlos, junto con verificar si pudieron relacionar lo estudiado
con la vida diaria. Asimismo, para verificar la conexión con
la vida cotidiana, pregúnteles: ¿en qué situaciones se puede
Pauta de co-evaluación
observar el movimiento relativo? Respecto de la motivación,
Aspectos a evaluar Sí No puede realizar las siguientes preguntas:
35
Hubo buena disposición para el trabajo colaborativo. • ¿Cuál fue mi actitud durante la lección?
Los distintos miembros del grupo colaboraron con • ¿Cómo influyó mi actitud en mi aprendizaje?
ideas. • Si no puedo resolver un problema, ¿qué siento?
Se respetaron distintas ideas para llevar a cabo el • ¿Cómo enfrento el éxito?
procedimiento.
Se comunicaron de una manera clara y adecuada.

Reflexiono sobre lo que aprendí (P. 35)


Para orientar esta sección, invite a sus estudiantes a que
analicen su proceso de aprendizaje en la lección, invitándo-
los a responder las preguntas plateadas. Es importante que,
en esta instancia, las y los estudiantes identifiquen además
de los contenidos abordados las habilidades, estrategias y
actitudes que se desarrollaron durante la lección.

Guía Didáctica del Docente

Física • 2.º medio


Desarrollo de la unidad

LECCIÓN 2: Análisis de los movimientos horizontales

Las actividades propuestas en el Texto del estudiante tienen como propósito el logro de los Aprendizajes
Esperados (AE), abordando con diferentes estrategias los Indicadores de Evaluación (IE) e incorporando
en cada una de ellas el trabajo con habilidades, actitudes y Objetivos Fundamentales Transversales (OFT).
AE IE Actividad Habilidad Actitud/OFT
Analizar de manera cualita- IE 4 Inicio de la misión (P. 39) HIC 2 OA A-C/OFT 1-3
tiva, cuantitativa, algebraica IE 4-5 Indaguemos (P. 40) HIC 1-2-3 OA C-E/OFT 3-5
y gráficamente, variadas
situaciones cotidianas de mo- IE 5 Reflexiono sobre el gráfico de posición (P. 41) HIC 3 OA A/OFT 1
vimiento rectilíneo: uniforme IE 4-5 Interpretemos gráficos del MRU (P. 42) HIC 3 OA C/OFT 3
y uniforme acelerado.
IE 4-5 Desarrollo de estrategias (P. 43) HIC 3 OA A/OFT 1
IE 4-5 A poner en práctica (P. 44 y 45) HIC 1-2-3 OA C/OFT 3
IE 4-5 Indaguemos (P. 46) HIC 1-2-3 OA C/OFT 3
IE 4 Aplico la ecuación de aceleración (P. 47) HIC 3 OA A/OFT 1
IE 5 Desarrollo de estrategias (P. 48) HIC 3 OA A/OFT 1
IE 4 Desarrollo de de la misión (P. 48) HIC 2-5 OA A/OFT 1
IE 4 Desarrollo de estrategias (P. 50) HIC 3 OA A/OFT 1
IE 4 Cierre de la misión (P. 51) HIC 4 OA A/OFT 1

Propósito de la lección A continuación se presenta una serie de orientaciones pa-


36 ra trabajar los temas, actividades y secciones presentes en
En esta lección se trabajan los conceptos que permiten anali- la lección del Texto del estudiante. Además, se acompañan
zar el movimiento de los cuerpos a partir de las característi- actividades e información complementarias, entre otros re-
cas que poseen cuando describen una trayectoria horizontal, cursos, en la Guía didáctica.
según lo propuesto en los Aprendizajes Esperados. Lo an-
terior se desarrolla a partir de los recursos y las actividades
presentes en el Texto del estudiante y en la Guía didácti- Ciencia, tecnología y sociedad P. 38
ca, en función del logro de los Indicadores de Evaluación
considerando, además del contenidos, las habilidades y Con la intención de activar las ideas previas y la alfabetiza-
actitudes. ción científica de las y los estudiantes sobre los contenidos
En el Texto del estudiante, las diferentes temáticas se desarro- de esta lección, se utiliza el texto El rol de la velocidad en los
llan con una propuesta didáctica que trabaja los contenidos accidentes de tránsito para comenzar una discusión con los
comenzando con actividades Indaguemos que buscan la activa- estudiantes, de manera de promover la argumentación de
ción de los conocimientos previos para posteriormente forma- las ideas a partir de la escritura y la oralidad. Este sentido,
lizarlos en el contenido tratado en el texto. Luego, se presentan es fundamental que el intercambio de diversas ideas en el
actividades que abarcan distintas habilidades, enfocándose aula, por medio de argumentos, posibilite la construcción del
principalmente en las de orden superior, ciclo que se repite a lo conocimiento. Luego de leer el texto, formule las siguientes
largo de la lección. De forma articulada al desarrollo del conte- preguntas: ¿De qué manera influye la velocidad en los acci-
nido, en los Desarrollo de estrategias se entregan herramientas dentes de tránsito? ¿Por qué es importante conocer las con-
para resolver problemas y en A poner en práctica se trabajan las secuencias que tiene el exceso de velocidad? ¿Cómo influye
habilidades científicas propuestas, poniendo énfasis en el desa- este conocimiento en la sociedad? ¿Consideran que este tipo
Guía Didáctica del Docente

rrollo de procesos científicos. Asimismo, a lo largo de la lección de campañas ayuda a generar conciencia en la conducción?,
se incorpora el trabajo de las actitudes con los OFT. ¿por qué?

En la Guía didáctica se presentan pautas para poder utilizar


algunas de las actividades del Texto del estudiante como ins-
trumentos de evalución, fichas de trabajo para los diferen-
tes ritmos de aprendizaje, actividades complementarias y
evaluaciones, con sus respectivos solucionarios.

Unidad 1 • Movimiento
ad

unid
1
Actividad del texto (P. 40)
Ciencia al día P. 39 Indaguemos
Esta sección es parte de Ciencia, tecnología y sociedad, por Esta actividad es la primera instancia en que el estudiante
lo que de la misma manera pretende la construcción del co- será capaz de registrar datos. Para ello, es importante fomen-
nocimiento científico a partir del análisis de la información. tar la actitud de realizar trabajos en forma ordenada y rigu-
En este caso en particular el texto El secreto de la VELOCIDAD rosa, ya que el resultado de la experiencia depende de ello.
de Usain Bolt promueve el estudio del movimiento descrito Puede evaluar las habilidades y actitudes científicas de la
por el atleta introduciendo los aprendizajes que se adquiri- actividad realizada, usando la siguiente rúbrica:
rán durante la lección. Una vez leído el texto, formule las
siguientes preguntas: ¿Qué parámetros se consideraron en Rúbrica de evaluación de habilidades y actitudes
el estudio para analizar la velocidad del atleta? ¿De qué ma- Niveles de logro
nera, las condiciones externar influyen en la velocidad que Indicadores
L ML PL
adquiere una persona? ¿Qué importancia tiene el estudio de
Conducen adecuadamente la experiencia.
la velocidad para las acciones que realizamos a diario?
Registran los datos de forma responsable.
Inicio de la misión (P. 39)
Determinan correctamente la velocidad.
¿Cómo conducir una investigación?
Analizan correctamente los resultados de la
En esta lección, los estudiantes tendrán la misión de con- experiencia.
feccionar una maqueta que muestre el movimiento de un
Trabajan de forma ordenada y colaborativa.
cuerpo con velocidad variable. En este caso, el trabajo in-
terdisciplinario se puede llevar a cabo con el profesor de L = Logrado; ML = Medianamente logrado; PL = Por lograr.
tecnología. Para comenzar con esta misión, motive a los
Actividad del texto (P. 41)
estudiantes indicándoles que al momento de conocer los
Reflexiono sobre el grafico de posición
aprendizajes necesarios, deberán llevar a cabo su proyecto
(página 48 del texto). En esta instancia, solo deben planifi- Aproveche la oportunidad para hacer notar que las represen- 37
car su trabajo. Para ello, oriente el trabajo de los estudiantes taciones gráficas permiten la síntesis de mucha información,
considerando que: por lo que dentro del lenguaje de la ciencia son representa-
• Forman equipos de trabajo respetando las habilidades ciones cargadas de significado que se pueden interpretar.
y competencias de cada integrante. En este caso en particular, los estudiantes deben identificar
que la pendiente de la recta en un gráfico de posición-tiempo
• Reconocen que las responsabilidades individuales en
representa la rapidez del movimiento.
la ejecución de su investigación.
• Seleccionan un plan de acción para diseñar su in- Actividad del texto (P. 42)
vestigación científica que permita solucionar su Interpretemos gráficos del MRU
problemática. La interpretación y construcción de gráficos es esencial para
• Utilizan procedimientos, software y plataformas de la comprensión de los aprendizajes de esta lección; por ello
análisis de textos durante la búsqueda de información es fundamental que las y los estudiantes realicen satisfacto-
en su investigación científica. riamente esta actividad. Para asegurar el propósito de esta,
puede realizar la actividad de forma grupal con todo el curso.
Para destacar la importancia de los gráficos en la síntesis
Tema 1: Describiendo el Movimiento Rectilí- de mucha información, puede plantear preguntas como las
neo Uniforme (MRU) Páginas 40 a 45 siguientes: ¿se podría decir de otra manera la información
contenida en el gráfico?, y si es así, ¿cuáles son las ventajas
Es importante que trabaje previamente con sus estudiantes y desventajas de realizar una representación gráfica?
los elementos que conforman un gráfico, su construcción
y su interpretación. Además, el cálculo de la pendiente de Guía Didáctica del Docente
un gráfico y el área bajo la curva, ya que aquello tendrá un
significado físico particular, según sean las magnitudes re-
presentadas en los ejes.

Física • 2.º medio


Desarrollo de la unidad
Puede evaluar las habilidades y actitudes científicas de la Para ello, formule las siguientes preguntas: ¿Cómo influyen
actividad realizada, usando la siguiente rúbrica: los desperdicios que desechamos al medio ambiente? ¿De
qué manera se pueden reducir, reutilizar y/o reciclar los
Rúbrica de evaluación de habilidades y actitudes
materiales de la actividad?
Niveles de logro
Indicadores Puede evaluar las habilidades y actitudes científicas de esta
L ML PL
actividad experimental, usando la siguiente rúbrica:
Conducen adecuadamente la experiencia.
Rúbrica de evaluación de habilidades y actitudes
Determinan el desplazamiento y la veloci-
dad media de cada gráfico. Niveles de logro
Indicadores
Construyen correctamente los gráficos de L ML PL
velocidad en función del tiempo. Formulan una hipótesis para la pregunta de
Analizan las características del movimiento investigación.
representado en cada gráfico. Conducen adecuadamente el procedimiento
Estipulan las ventajas que tienen los gráfi- experimental.
cos para el estudio del movimiento. Realizan estudios de confiabilidad para
L = Logrado; ML = Medianamente logrado; PL = Por lograr. validar los datos obtenidos.
Comparan las inferencias e interpretaciones
Desarrollo de estrategias (P. 43) formuladas con la hipótesis de trabajo, para
hallar coherencia y consistencia entre ellos.
En el paso 5 aplico lo aprendido, indique a sus estudiantes
que usen el mismo paso a paso desarrollado en el problema Plantean conclusiones basándose en las
propuesto con el fin de que reconozcan la utilidad de aplicar evidencias y los resultados formulando re-
laciones entre las variables y las inferencias
los modelos. Es importante que, en esta instancia, puedan
formuladas.
poner en práctica todos los aprendizajes de la lección.
Señalan las dificultades del procedimiento
38
experimental.
A poner en práctica P. 44 y 45 Evalúan el trabajo individual y colectivo.
L = Logrado; ML = Medianamente logrado; PL = Por lograr.
¿Cómo conducir una investigación científica?
Esta actividad de aprendizaje representa la oportunidad para
trabajar habilidades científicas, que en este caso promueve Tema 2: Movimiento Rectilíneo Uniforme
principalmente Organizar datos cualitativos con precisión. Acelerado (MRUA) Página 46 a 50
De forma transversal al objetivo de esta actividad práctica,
se trabajan otras habilidades: Conducir rigurosamente in- En este tema se introduce por primera vez el concepto de
vestigaciones científicas; Analizar y explicar los resultados aceleración. Es importante que en esta instancia quede en
de una investigación científica experimental, y Explicar y evidencia su relación con la variación de la velocidad y el
argumentar con evidencias. tiempo. Para ello, puede utilizar diversos ejemplos en los que
Para llevar a cabo esta experiencia, guíe en todo momento se evidencie el cambio de velocidad que provoca la acelera-
los pasos establecidos, de forma que las y los estudiantes lo- ción como: la carrera de Usain Bolt, el lanzamiento de un
gren el propósito de la actividad, considerando las actitudes pelota, entre otros.
que esta también promueve. Para comenzar, solicíteles leer Actividad del texto (P. 46)
la Situación problema y, a partir de ella, elaborar una hipóte- Indaguemos
sis a la siguiente pregunta de investigación: ¿Cómo varía el
nivel del agua de la botella a medida que el líquido sale por La actividad requiere rigurosidad en torno a las observa-
el orificio? Una vez establecida la hipótesis de la pregunta, ciones, las interpretaciones y las conclusiones que se des-
Guía Didáctica del Docente

respalde el procedimiento experimental que llevan a cabo prendan de ella. Oriente a sus estudiantes para que noten
las y los estudiantes. los efectos de la fuerza aplicada sobre la bolita en cuanto
a su velocidad. Invite a sus estudiantes a contrastar los re-
Considere dar énfasis a la cápsula Reduce, Reutiliza, Reci- sultados obtenidos, con el objetivo de crear una conclusión
cla de manera que los estudiantes desarrollen una de las general para la actividad.
grandes ideas de la ciencia propuestas en la unidad: “Los
organismos tienen estructuras y realizan procesos para sa-
tisfacer sus necesidades y responder al medio ambiente”.

Unidad 1 • Movimiento
ad

unid
1
Puede evaluar las habilidades y actitudes científicas de la Desarrollo de la misión (P. 48)
actividad realizada, usando la siguiente rúbrica: ¿Cómo comunicar una investigación?
Rúbrica de evaluación de habilidades y actitudes Para llevar a cabo el proyecto de esta lección y de acuerdo al
Niveles de logro trabajo interdisciplinar señalado al inicio del mismo, solicite
Indicadores a sus estudiantes que desarrollen la maqueta que les permita
L ML PL
evidenciar un movimiento con velocidad variable. Para con-
Conducen adecuadamente la experiencia. firmar que las ideas planificadas por cada grupo de trabajo
Describieron los cambios en el estado del sean correctas, genere la instancia para que cada equipo le
movimiento a partir de sus observaciones. muestre el diseño de su prototipo y sugiera ideas de mejoras
Fueron rigurosos en sus observaciones y
para la elaboración del producto final.
descripciones. Puede evaluar la actividad realizada, usando las siguientes
L = Logrado; ML = Medianamente logrado; PL = Por lograr. rúbricas:
Rúbrica de evaluación de habilidades y actitudes
Actividad del texto (P. 47)
Aplico la ecuación de aceleración Niveles de logro
Indicadores
L ML PL
Esta actividad permite aplicar la ecuación de aceleración ba-
jo un contexto cotidiano de acuerdo al texto de Usain Bolt de Forman equipos de trabajo respetando
la sección CTS del inicio de la lección. Como esta es la prime- las habilidades y competencias de cada
integrante.
ra instancia en que las y los estudiantes aplicarán este mo-
delo matemático, es importante guiar su trabajo individual. Reconocen que las responsabilidades indivi-
duales en la ejecución de su investigación.
Desarrollo de estrategias (P. 48)
Seleccionan un plan de acción coherente
En el paso 5 Aplico lo aprendido, indique a sus estudiantes para diseñar su investigación científica.
que usen el mismo paso a paso desarrollado en el problema Utilizan procedimientos, software y pla- 39
propuesto con el fin de que reconozcan la utilidad de aplicar taformas de análisis de textos durante la
los modelos. búsqueda de información.
Utilizan conocimientos científicos en solu-
Actividad complementaria ciones de problemas cotidianos.
Un tren cuya velocidad inicial es de 8 m/s acelera de ma- Argumentan la importancia de las habi-
nera constante a una razón de 4 m/s2. lidades y conocimientos científicos para
resolver diferentes problemas del entorno.
L = Logrado; ML = Medianamente logrado; PL = Por lograr.

Rúbrica de evaluación para la comunicación de


resultados
Niveles de logro
Indicadores
L ML PL
Informan sobre la investigación realizada
con los recursos más adecuados.
Expresan la realización de su investigación
argumentando las razones de la elección de
su trabajo.
a. ¿Qué velocidad tiene el tren a los 20 segundos de viaje? Presentan su trabajo utilizando lenguaje Guía Didáctica del Docente
científico apropiado.
b. ¿Qué distancia abrá recorrido el tren en ese tiempo?
L = Logrado; ML = Medianamente logrado; PL = Por lograr.

Física • 2.º medio


Desarrollo de la unidad
Las TIC (P. 49)
Para trabajar de manera más lúdica y dinámica la construc-
ción de gráficos del MRUA invite a sus estudiantes a trabajar
con el recurso digital: El Hombre Móvil.

Fuente: https://phet.colorado.
edu/es/simulation/
legacy/moving-man

Este permite aprender sobre los gráficos de posición, velocidad ¿Cómo evaluar una investigación?
y aceleración utilizando una interfaz que permite mover a un
Para que los estudiantes evalúen su desempeño en el trabajo
hombre de ida y vuelta fijando los parámetros de su movimien-
realizado, analizando su desempeño personal como colecti-
to. Al determinar la posición, la velocidad y/o la aceleración, la
40 vo, pueden calificarlo a partir de la retroalimentación. Para
simulación permite observar el movimiento del hombre en el
ello, además de las preguntas planteadas en el Texto del es-
mismo instante que va construyendo los gráficos de posición,
tudiante, se sugiere implementar las siguientes pautas de
velocidad y aceleración que describen el movimiento.
evaluación:
Desarrollo de estrategias (P. 50)
Pauta de auto-evaluación
Explique a sus estudiantes que, para extraer información del Aspectos a evaluar Sí No
gráfico, deben identificar las características del mismo. Para
Realicé aportes para la confección de la maqueta.
ello, lo primero que deben identificar son las variables del
gráfico: en este caso el eje X representa el tiempo y el eje Y Fui responsable en mis tareas dentro del grupo de
la velocidad. Luego, en el paso 5 aplico lo aprendido, las y los trabajo.
estudiantes deberán aplicar los modelos trabajados en los Trabajé con conciencia e interés en el desarrollo de
pasos que se modelaron en la actividad. la misión.

Pauta de co-evaluación
Ciencia tecnología y sociedad P. 41
Aspectos a evaluar Sí No
Mencione a sus estudiantes que la curiosidad del ser huma- Tuvieron una buena disposición para realizar el
no por conocer y predecir fenómenos naturales lo ha llevado trabajo de forma colaborativa.
a desarrollar nuevas tecnologías a partir del estudio de los Respetaron los tiempos asignados para realizar cada
animales como los descritos en esta sección. Esto ha traí- una de las tareas.
do grandes avances científicos e innovaciones tecnológicas Se respetaron distintas ideas para llevar a cabo el
Guía Didáctica del Docente

impensadas, como autos de carrera y aviones supersónicos. procedimiento.


Cierre de la misión (P. 51) Se comunicaron de una manera clara y adecuada.

Unidad 1 • Movimiento
ad

unid
1
Ventana de profundización didáctica Integro lo que aprendí P. 52 y 53
Metacognición y motivación
Explique a sus estudiantes que esta sección constituye una
Las actividades que apuntan al desarrollo del pensamien- instancia evaluativa en la cual pueden medir el logro de sus
to metacognitivo son fundamentales para motivar a las y aprendizajes. Pídales que completen la sección ¿Cómo voy?,
los estudiantes. Este tipo de recursos les entrega un cierto para identificar el logro de los aprendizajes propuestos.
grado de elección, estimulando su noción de autodeter-
Para trabajar los diferentes ritmos de aprendizaje de sus es-
minación y la autorregulación del aprendizaje. En este
tudiantes, le proponemos utilizar las siguientes actividades
sentido, las actividades enfocadas en potenciar el pensa-
complementarias: si el nivel de desempeño de un estudiante
miento metacognitivo motivan al estudiante hacia la ta-
es Logrado, pídale completar la Ficha de ampliación de la
rea, validándolo como principal responsable del proceso
página 53 de la Guía didáctica. Si el o la estudiante obtuvo
de enseñanza-aprendizaje.
un desempeño Medianamente logrado o Por lograr, solicítele
Fuente: https://educrea.cl/la-metacognicion-en-la-escuela- trabajar en la Ficha de refuerzo de la página 52.
la-importancia-de-ensenar-a-pensar/ (Adaptación).
Puede complementar el trabajo metacognitivo de sus es-
Reflexiono sobre lo que aprendí (P. 51) tudiantes en la sección Reflexiono sobre mi desempeño con
Para profundizar en el pensamiento metacognivo de las y las siguientes preguntas: ¿qué conceptos que no conocías al
los estudiantes respecto de su trabajo en esta lección, puede principio de la lección podrías definir con tus propias pala-
realizar preguntas como las siguientes: ¿De qué manera el bras ahora?, ¿cuál es el proceso que tiene mayor dificultad
estudio científico permite comprender mejor el mundo que para ti?, ¿qué es lo que debes ejercitar para superarlo? Plan-
nos rodea? ¿Cómo pueden mejorar su actitud frente al apren- tea una pregunta relacionada con el movimiento que aún
dizaje de la física? ¿De qué manera es importante compren- no puedas responder e investiga qué conocimientos sería
necesario dominar para hacerlo.
der los modelos para explicar fenómenos cotidianos?
41
Notas:

Guía Didáctica del Docente

Física • 2.º medio


Desarrollo de la unidad

LECCIÓN 3: : Análisis de los movimientos verticales

Las actividades propuestas en el Texto del estudiante tienen como propósito el logro de los Aprendizajes
Esperados (AE), abordando con diferentes estrategias los Indicadores de Evaluación (IE) e incorporando
en cada una de ellas el trabajo con habilidades, actitudes y Objetivos Fundamentales Transversales (OFT).
AE IE Actividad Habilidad Actitud/OFT
Analizar de manera cualita- IE 6-7 Inicio de la misión (P. 55) HIC 2 OA A-C/OFT 1-3
tiva, cuantitativa, algebraica IE 6-7 Indaguemos (P. 56) HIC 1-2-3 OA A-C-E/OFT 1-3-5
y gráfica la caída libre y los
lanzamientos verticales. IE 6-7 A poner en práctica (P. 58 y 59) HIC 1-2-3-5 OA A/OFT 1
IE 7 Desarrollo de la misión (P. 59) HIC 2-3-5 OA C/OFT 3
IE 7 Indaguemos (P. 60) HIC 1-2-3 OA A/OFT 1
IE 7 Desarrollo de estrategias (P. 62) HIC 3 OA A-C-E/OFT 1-3-5
IE 7 Cierre de de la misión (P. 63) HIC 4 OA A/OFT 1

Propósito de la lección Ciencia al día P. 54


En esta lección se trabajan los conceptos que permiten ana-
lizar el movimiento de los cuerpos a partir de las caracterís- En esta sección se expone La física en el parque de diversiones.
ticas que poseen cuando describen una trayectoria vertical, En particular se analiza una de las entretenciones principales
según lo propuesto en los Aprendizajes Esperados. Lo an- del parque de diversiones Fantasilandia ubicado en Santia-
terior se desarrolla a partir de los recursos y las actividades go. La idea de este artículo es evidenciar que el estudio de la
presentes en el Texto del estudiante y en la Guía didácti- física, y en particular el movimiento, se aplica a diversos apa-
42 ca, en función del logro de los Indicadores de Evaluación ratos tecnológicos como el Xtreme fall. Si sus estudiantes no
considerando, además del contenido, las habilidades y conocen esta atracción, analice el movimiento en caída libre
actitudes. de cualquier otro tipo de juegos que se pueden encontrar en
otros parques de entretenciones. Para guiar el análisis formu-
En el Texto del estudiante, las diferentes temáticas se desarro-
le preguntas como: ¿Qué factores debe considerar la ciencia
llan con una propuesta didáctica que trabaja los contenidos
para elaborar este tipo de juegos? ¿De qué manera impacta
comenzando con actividades Indaguemos que buscan la ac-
este tipo de entretención en la sociedad?
tivación de los conocimientos previos para posteriormente
formalizarlos en el contenido tratado en el texto. Luego, se
presentan actividades que abarcan distintas habilidades, en- Ciencia tecnología y sociedad P. 55
focándose principalmente en las de orden superior, ciclo que
se repite a lo largo de la lección. De forma articulada con el En esta sección se presenta una breve reseña sobre la cons-
desarrollo del contenido, en los Desarrollo de estrategias se en- trucción del paracaídas a lo largo de la historia en El PA-
tregan herramientas para resolver problemas y en A poner en RACAÍDAS en contra de las caídas. Aquí, se espera que los
práctica se trabajan las habilidades científicas propuestas en estudiantes analicen el carácter dinámico de las ciencias
el análisis de experimentos clásicos. Asimismo, a lo largo de y como su evolución depende del contexto histórico y social.
la lección se incorpora el trabajo de las actitudes con los OFT. Para enfatizar el análisis en el proceso de construcción de co-
En la Guía didáctica se presentan pautas para utilizar algu- nocimiento científico, formule preguntas como las siguientes:
nas de las actividades del Texto del estudiante, como instru- ¿Cómo influye el contexto histórico en el desarrollo de nue-
mentos de evaluación, fichas de trabajo para los diferentes vas tecnologías como la elaboración del paracaídas? ¿Cómo
ritmos de aprendizajes, actividades complementarias y evoluciona el diseño del paracaídas en función de los nuevos
Guía Didáctica del Docente

evaluaciones, con sus respectivos solucionarios. conocimientos? ¿Es posible que se siga estudiando sobre la
forma y las características del paracaídas?, ¿por qué?
A continuación, se presenta una serie de orientaciones para
trabajar los temas, actividades y secciones presentes en la
lección del Texto del estudiante. Junto con ello, se presen-
tan actividades e información complementarias, entre otros
recursos, en la Guía didáctica.

Unidad 1 • Movimiento
ad

unid
1
Inicio de la misión (P. 55)
Ventana de profundización didáctica
¿Cómo conducir una investigación? Motivación para el aprendizaje
Esta lección, las y los estudiantes tendrán la misión de crear
un paracaídas que permita frenar la caída de un huevo cuan- A veces no es que los alumnos no aprendan porque no
do se deja car desde un segundo piso. En este caso, el trabajo estén motivados, sino que no están motivados porque no
deberá ser guiado directamente por el profesor de física, permi- aprenden, y no aprenden porque su modo de pensar, al
tiéndole reconocer todas las características necesarias para su afrontar las tareas, es inadecuado, impidiendo la expe-
elaboración, considerando los materiales requeridos, la forma riencia satisfactoria que supone sentir que se progresa,
implementada y las mediciones que deberá llevar a cabo para la experiencia que activa la motivación. El hecho de que es-
confección de su prototipo, considerando que en esta instancia to ocurra se ve producido, de acuerdo con Dweck y Elliot
deberán sólo planificar su trabajo. Para comenzar con esta mi- (1983), si el alumno, al afrontar una tarea, se fija sobre to-
sión, señale que, al momento de conocer los aprendizajes nece- do en la posibilidad de fracasar en lugar de aceptarla como
sarios, deberán llevar a cabo su proyecto (página 59 del texto). un desafío y de preguntarse cómo puede hacerla, es decir,
se centra en los resultados más que en el proceso que le
permite alcanzarlos y considera los errores como fracasos
Tema 1: ¿Qué características posee la caída y no como ocasiones de las que es posible aprender.
libre? Páginas 40 a 45 Fuente: Tapia, J. (2005) Motivación para el aprendizaje:
la perspectiva de los alumnos.
Para comenzar el análisis de este tipo de movimiento, puede
solicitar a sus estudiantes que recuerden las primeras pági- Actividad complementaria
nas de la unidad (Páginas 16 y 17), en las cuales se muestra
A continuación se presenta un gráfico de velocidad en
a una niña lanzándose en bungee y cómo esta caída genera
función del tiempo para un cuerpo que es lanzado verti-
la discusión de sus amigos Romina y Miguel. Plantéeles que
calmente hacia arriba.
en la siguiente actividad, podrán poner en práctica las mis-
mas experiencias que ellos realizaron, para determinar las Gráfico de velocidad-tiempo 43
características de la caída libre. Velocidad (m/s)

Actividad del texto (P. 56) 30


Indaguemos
20
Para llevar a cabo esta actividad, señale a sus estudiantes que el 10
uso de instrumentos tecnológicos, como el teléfono celular, pa-
ra el registro de videos es una herramienta útil para el análisis −10 1 2 3 4 5 6 Tiempo (s)
de evidencias cuando se utilizan de manera responsable. Para
−20
ello, guíelos en la implementación de este tipo de herramientas
para analizar los resultados de la experiencia y así establecer las −30
conclusiones que permiten caracterizar la caída de los cuerpos.
A partir de la información, responde:
Puede evaluar las habilidades y actitudes científicas de la
actividad realizada, usando la siguiente rúbrica: a. ¿A qué se debe que la recta corte el eje del tiempo?
Rúbrica de evaluación de habilidades y actitudes b. ¿Qué ocurre con el cuerpo en el instante t = 3 s?
Niveles de logro c. ¿Qué valor tiene la pendiente de la recta?, ¿qué
Indicadores
L ML PL representa este valor?
Conducen adecuadamente la experiencia.
Describen correctamente las características A poner en práctica P. 58 y 59 Guía Didáctica del Docente
de la caída libre.
Comparan sus resultados con las ideas pre- ¿Cómo conducir una investigación científica?
vias planteadas al inicio de la unidad.
El taller de habilidades científicas propuesto en esta actividad
Manipulan responsablemente las herra-
tiene como finalidad analizar un experimento clásico de ma-
mientas tecnológicas para el análisis de los
resultados. nera que las y los estudiantes logren valorar el conocimiento
científico a partir del análisis e interpretación de resultados,
L = Logrado; ML = Medianamente logrado; PL = Por lograr.
así como la elaboración de conclusiones. Se espera que al

Física • 2.º medio


Desarrollo de la unidad
finalizar este taller, logren comunicar los resultados de la ex- Rúbrica de evaluación para la comunicación de resultados
periencia utilizando alguna herramienta TIC.
Niveles de logro
Puede evaluar la actividad realizada, usando la siguiente Indicadores
L ML PL
rúbrica:
Informan sobre la investigación realizada
Rúbrica de evaluación para la comunicación de resultados con los recursos más adecuados.
Niveles de logro Expresan la realización de su investigación
Indicadores argumentando las razones de la elección de
L ML PL
su trabajo.
Diseñan una estrategia comunicacional para
informar los resultados de la investigación. Presentan su trabajo utilizando lenguaje
científico apropiado.
Seleccionan los recursos comunicacionales
más apropiados para ser utilizados según L = Logrado; ML = Medianamente logrado; PL = Por lograr.
el público receptor al que vaya dirigida la
información.
Evalúan la publicación que comunicarán Tema 2: ¿Qué características posee el
examinando la coherencia del lenguaje
lanzamiento vertical? Páginas 60 a 62
empleado y la consistencia con los objetivos
de una investigación.
En este tema, se analizará como último tipo de movimiento el
L = Logrado; ML = Medianamente logrado; PL = Por lograr.
lanzamiento vertical: hacia arriba y hacia abajo. Es importante
señalar que en este solo se considera el movimiento en una di-
Desarrollo de la misión (P. 59)
mensión (vertical) por lo que el lanzamiento de proyectil, que
¿Cómo comunicar una investigación?
se desarrolla en dos dimensiones, no es parte de este estudio.
Solicite a sus estudiantes que evalúen la planificación ini-
Actividad del texto (P. 60)
cial de su proyecto, descrito en la página 55 del Texto del
Indaguemos
44 estudiante y que, a partir de sus nuevos aprendizajes, esta-
blezcan las mejoras al prototipo de paracaídas. Una vez que En esta actividad, el uso de instrumentos tecnológicos, como
cada equipo de trabajo confeccione el paracaídas, llévelos el teléfono celular para el registro de videos es una herra-
al segundo piso del establecimiento y, a partir de un traba- mienta útil para el análisis de evidencias. Para ello, guíe
jo sincronizado, permita que pongan a prueba su modelo, a sus estudiantes en la implementación de este tipo de he-
de manera que uno de los integrantes de cada grupo puede rramientas para analizar los resultados de la experiencia y
filmar la caída del huevo, para luego evaluar y analizar los así establecer las conclusiones que permiten caracterizar el
resultados de la experiencia. Se espera que en esta instancia, lanzamiento vertical hacia arriba. Es importante que esta ac-
los mismos estudiantes evalúen las ventajas de desventajas tividad, la herramienta tecnológica les permita verificar que
del modelo propuesto considerando si cumplió o no con el el lanzamiento realizado fue efectivamente vertical y en una
objetivo de mantener intacto al huevo. dimensión.
Puede evaluar la actividad realizada, usando la siguiente Puede evaluar la actividad realizada, usando la siguiente
rúbrica: rúbrica:
Rúbrica de evaluación de habilidades y actitudes Rúbrica de evaluación de habilidades y actitudes
Niveles de logro Niveles de logro
Indicadores Indicadores
L ML PL L ML PL
Planificaron satisfactoriamente su plan de Conducen adecuadamente la experiencia.
trabajo.
Señalan la relación entre de velocidad ini-
Consideraron los materiales necesarios para cial y la altura que alcanza la pelota.
la confección de su prototipo
Guía Didáctica del Docente

Determinan correctamente el tipo de movi-


Confeccionaron satisfactoriamente su mode-
miento que describe el lanzamiento vertical
lo de paracaídas.
a partir de las evidencias.
Obtuvieron los resultados esperados, de
forma que el huevo no se rompió al caer. Utilizan responsablemente la tecnología
para analizar evidencias.
Tomaron responsablemente registro audiovi-
sual del funcionamiento de su paracaídas. L = Logrado; ML = Medianamente logrado; PL = Por lograr.
L = Logrado; ML = Medianamente logrado; PL = Por lograr.

Unidad 1 • Movimiento
ad

unid
1
Las TIC Para ello, además de las preguntas planteadas en el texto del
estudiante, se sugiere implementar las siguientes pautas de
Para trabajar de manera más dinámica este contenido, se
evaluación:
sugiere utilizar el recuso digital Movimiento de un proyectil.
Pauta de auto-evaluación
Aspectos a evaluar Sí No
Trabajé con motivación durante todos los procesos
de la misión.
Planteé mis ideas de manera respetuosa.
Escuché las opiniones de mis compañeros.

Pauta de co-evaluación
Aspectos a evaluar Sí No
Todos los integrantes participaron activamente en la
Fuente: https://phet.colorado.edu/sims/projectile-motion/ distribución de las tareas.
projectile-motion_es.html
Establecieron los materiales que se requirieron para
Este recurso permite visualizar la trayectoria de un proyectil la elaboración del paracaídas.
indicando la altura y el tiempo que tarda en realizar todo su Siguen los pasos propuestos en la planificación de su
recorrido. Para cada movimiento, se pueden establecer los proyecto.
parámetros del ángulo de lanzamiento (que para esta lección Analizaron los resultados de su experiencia a partir
siempre será 90°), la velocidad inicial, la masa y el diámetro del registro audiovisual obtenido.
del cuerpo. Además, en de la simulación, se pueden eviden- Estipularon ventajas y desventajas del prototipo
ciar las consecuencias que genera el roce con el aire en este diseñado.
movimiento. 45
Desarrollo de estrategias (P. 62) Reflexiono sobre lo que aprendí (P. 63)

Realice el esquema del problema en la pizarra y resuélvalo Para profundizar el pensamiento metacognivo de las y los
paso a paso con sus estudiantes, con el objetivo de incor- estudiantes respecto de su trabajo en esta lección, puede
porar este tipo de estrategia para resolver problemas y que realizar preguntas como las siguientes: ¿qué conceptos que
ellos puedan aplicarla luego en el paso 5 aplico lo aprendido. no conocías al principio de la lección podrías definir ahora?,
¿cuál es el proceso que tiene mayor dificultad para ti?, ¿qué
es lo que debes ejercitar para superarlo?
Ciencia tecnología y sociedad P. 63

Continuando la temática de la unidad, en esta sección se Integro lo que aprendí P. 64 y 65


mencionan las características de los deportes extremos, co-
mo el paracaidismo y el wingsuit, y su relación con la caída Explique a sus estudiantes que esta sección constituye una
libre de los cuerpos. Para resaltar la importancia del desarro- instancia evaluativa en donde pueden medir el logro de sus
llo de la ciencia y la tecnología y su impacto en la sociedad, aprendizajes. Pídales que completen la sección ¿Cómo voy?,
responda en conjunto con sus estudiantes las preguntas que para identificar el logro de los aprendizajes propuestos.
se plantean en el texto, dando mayor énfasis a las actitudes Para trabajar los diferentes ritmos de aprendizaje de sus
y OFT que fomentan el autocuidado. estudiantes, le proponemos que utilice las complementarias.
Cierre de la misión (P. 63) Si el nivel de desempeño de un estudiante es Logrado, pídale
completar la Ficha de ampliación de la página 55, de la Guía
¿Cómo evaluar una investigación? didáctica. Si su nivel de desempeño fuese Medianamente Guía Didáctica del Docente
Para que los estudiantes evalúen su desempeño en el trabajo logrado o Por lograr, solicítele trabajar la Ficha de refuerzo
realizado, analizando su desempeño personal como colec- de la página 54. Puede complementar el trabajo metacog-
tivo, pueden calificarlo a partir de una retroalimentación a nitivo de sus estudiantes en la sección Reflexiono sobre mi
partir del análisis del registro visual que obtuvieron de su desempeño con las siguientes preguntas: ¿Qué actitudes he
proyecto. desarrollado de forma satisfactoria? ¿De qué manera el uso
de herramientas tecnológicas permite comprender mejor los
fenómenos del entono?

Física • 2.º medio


Cierre de la unidad Páginas 66 a 73

La CIENCIA se construye
Páginas 66 y 67

Esta sección tiene la finalidad de enfatizar el carácter diná-


mico del proceso de construcción del conocimienot cientí-
fico. En este caso en particular, se muestra la evolución de la
construcción del reloj a lo largo de la historia.
Para trabajar estas páginas, puede mencionar que la evolu-
ción del conocimiento desde la observación hasta la elabo-
ración de modelos más completos es un fenómeno inherente
a todas las ciencias.
Trabajo con la información
Para indagar en esta sección, lea la información junto con
sus estudiantes de manera que puedan evidenciar esta evo-
lución considerando el contexto social e histórico. Para ello,
además de las preguntas que se proponen en el texto, puede
realizar preguntas como: ¿Cómo era el contexto histórico en
cada época señalada? ¿Qué conocimientos científicos creen
que se tenía en cada época? ¿Cómo influye el contexto social
en el desarrollo del conocimiento científico?

Síntesis
46 Páginas 68 y 69

¿Cómo trabajar con los organizadores gráficos?


En la actividad Síntesis de estas páginas, se promueve la con-
fección de un organizador gráfico. Para ello, se guía el proce-
so de construcción de un árbol gráfico en función de cinco
pasos de tal manera que al ir siguiéndolos los estudiantes
logren su confección de forma independiente. De esta mane-
ra, se espera que en la página 69 del Texto, el o la estudian-
te complete el árbol gráfico representando los contenidos,
habilidades y actitudes desarrolladas en la unidad. Si lo
desea, puede solicitar previamente los materiales, de modo
que las y los estudiantes puedan elaborar este mismo organi-
zador de forma más extendida y con mayores recursos en un
pliego de cartulina e incorporando, fotos, tablas, imágenes,
entre otros recursos.
Para finalizar, mencione a sus estudiantes que también
pueden utilizar diferentes herramientas tecnológicas dispo-
nibles en Internet para realizar un organizador gráfico, por
ejemplo: al introducir el código 18GF2M046a en el sitio web
Guía Didáctica del Docente

codigos.auladigital.cl encontrará una herramienta digital en


la que pueden crear mapas conceptuales, exportarlos como
imagen y compartirlos a través de una dirección URL. Ade-
más, permite crear mapas de forma colaborativa.

Unidad 1 • Movimiento
ad

unid
1
Evaluación final Metacognición
Páginas 70 a 73
Es necesario impulsar y animar a
El propósito de estas páginas es evaluar los contenidos, las habilidades y las ac- los y las estudiantes hacia prác-
titudes trabajadas en la unidad. Sugiera a los estudiantes que trabajen de manera ticas metacognitivas, ayudán-
individual de modo que puedan identificar aquellos contenidos y/o habilidades dolos a establecer criterios de
que no han alcanzado. referencia a través de preguntas
Para complementar las preguntas planteadas en la sección Reflexiono sobre lo que que desencadenen una reflexión
aprendí, puede plantear las siguientes: ¿qué estrategias utilicé en la unidad para fructífera sobre sus propios proce-
abordar los contenidos?, ¿fueron efectivas?, ¿qué debería cambiar para ser más sos de aprendizaje. Se requiere un
eficaz en el estudio de la próxima unidad? estilo de pregunta flexible y articu-
lada, que genere diálogo. Puede ser
De forma adicional, le proponemos trabajar con la evaluación propuesta en las necesario que los sujetos se ofrez-
páginas 58 a 61, de la Guía didáctica. Esta evaluación presenta preguntas de se- can a sí mismos como modelos en
lección múltiple, cuyas respuestas las encuentrá en la sección Solucionario de la la búsqueda de sus motivaciones
Guía didáctica, página 66. y de las razones que se esconden
tras sus decisiones.

Pregunta de integración con otras asignaturas

Esta pregunta tiene la finalidad de propiciar la integra-


ción de los aprendizajes con otras asignaturas, como Edu-
cación Artística. En este caso en particular, se presenta 47
la obra de Salvador Dalí: La perspectiva de la memoria o
relojes blandos. A partir de esta imagen, se espera que el
estudiante elabore una hipótesis respecto de la interpre-
tación de esta obra a partir de la representación que da el
artista sobre los relojes. El análisis propuesto en esta pre-
gunta puede resultar complejo para las y los estudiantes;
sin embargo, es importante considerar que toda hipótesis
es válida y que la interpretación de las obras de arte en
particular siempre depende del observador. De esta ma-
nera, puede resaltar el hecho de que todo es relativo y
nada es absoluto, tal como se estudió en la unidad.

Guía Didáctica del Docente

Física • 2.º medio


ad

unid
1
Profundización disciplinar

Relatividad del tiempo


Newton, en su teoría, postuló la existencia de un sistema una transformación que deja invariante las ecuaciones
de referencia en el cual el universo estaría en reposo: un de Maxwell, no solo cuando se cambia de posición, sino
espacio absoluto, en el que todos los otros sistemas de re- que también cuando se cambia de instante. Sin embargo,
ferencia podían existir y desde el cual el comportamiento tuvo que recurrir a ideas que no eran entendidas en su
global de los componentes del universo es observable de época, como que el tiempo transcurría de forma diferente
manera mucho más simple que desde la Tierra. Además en sistemas de referencia distintos. Einstein postuló que
de un espacio absoluto, que funcionaba como el sistema las ecuaciones de Maxwell deben tener la misma forma
referencial universal, Newton propuso un tiempo abso- en cualquier sistema de referencia inercial y que, por lo
luto, no porque le hiciera falta para explicar sus teorías, tanto, es imposible distinguir, a partir de experimentos
sino porque parecía lógico, de acuerdo a nuestra expe- electromagnéticos, un sistema de referencia inercial de
riencia cotidiana, que existiera una única forma de medir otro. Para que este principio de relatividad se cumpla, es
el tiempo. Afirmar que el tiempo transcurría de diferente necesario que las transformaciones de Lorentz sean físi-
forma en un lugar u otro del universo era considerado camente válidas; en consecuencia, propuso que el tiem-
imposible en la época de Newton o al menos lo era has- po medido entre dos sucesos depende del movimiento de
ta que apareció Einstein. Uno de los problemas que más quien lo mide. Es imposible determinar de forma única
le interesaba a Einstein era la incapacidad que tenían la duración de un fenómeno, ya que, de acuerdo con la
la relatividad de Galileo y la teoría electromagnética de teoría de la relatividad, el tiempo de ese reloj no coincide
Maxwell para pasar de un sistema de referencia a otro sin con el que marca otro reloj que se mueve con respecto al
48 cambiar la forma de las ecuaciones, a diferencia de la me- primero, pero la relación entre los dos tiempos se puede
cánica newtoniana. Luego, Lorentz demostró que existe determinar perfectamente.

Fuente: Hacyan, S. (1996). Relatividad para principiantes. México D.F.: Fondo de Cultura Económica.

Los límites de los modelos: la caída libre


Los modelos que describen la caída libre han sido puestos respuesta está dada por la fuerza de roce, que tiene distin-
a prueba en los últimos años por Felix Baumgartner en to sentido al movimiento que se produce cuando ellos van
2012 y por Alan Eustace en 2014, ambos saltando desde apartando las moléculas de aire de su camino, disipando
la estratósfera a 39 068 y 41 150 m de altura, respectiva- energía que, de otra forma, hubiera aumentado su veloci-
mente. Según los modelos, se podría esperar que Eustace dad. El problema es que esta fuerza no es constante, sino
alcanzara una mayor velocidad durante su caída, ya que que depende de múltiples factores, como la viscosidad, la
se lanzó desde mayor altura; sin embargo, esto no fue densidad y el régimen de fluido del aire en determinada
así: Baumgartner alcanzó una velocidad máxima de 1343 zona, la resistencia aerodinámica y la velocidad que lleva
km/h y Eustace de 1322 km/h. Son muchas las variables el cuerpo. La fuerza de roce en ambos casos depende de
que se deben considerar en estos saltos. Si se supone a variados factores, lo que influyó en la velocidad que am-
Baumgartner y a Eustace como partículas esféricas y al bos experimentaron. Por otra parte, también existen fac-
medio como vacío, entonces Eustace hubiese sido el ven- tores humanos que intervienen en la velocidad alcanzada
Guía Didáctica del Docente

cedor en velocidad, aunque no hubiera importado el para- durante este tipo de saltos, por ejemplo, el momento de
caídas, ya que ambos nunca hubiesen dejado de acelerar apertura del paracaídas y cómo se acomodaron durante
y el desenlace hubiese sido mortal, no obstante estas con- la caída. En consecuencia, siempre se debe recordar que
diciones no se cumplen en ninguno de los dos saltos. ¿A los modelos de predicción son solo aproximaciones de la
qué se debe que el modelo no se ajuste a lo sucedido? La realidad en condiciones ideales.

Fuentes: Fuentes: Mohazzabi, P. y Shea, J. (1996). High-altitude free fall.


Disponible en: <http://www.jasoncantarella.com/downloads/AJP001242.pdf

Unidad 1 • Movimiento
ad

unid
1
Profundización didáctica

Técnicas de estudio, un camino a la autorregulación


“Es importante enseñar a estudiar por cuenta propia, a buscar por cuenta propia, a asombrarse”.

◾◾ Reflexione
¿Sabe con seguridad si sus estudiantes estudian de forma correcta?
◾◾ Empatice
¿Cómo puede organizarse si nadie le enseña a hacerlo?
◾◾ Decida
¿Por qué no enseñarles técnicas de estudio a sus estudiantes?

Podemos aprender mucho del fracaso. En este caso, es 2. Lectura de exploración. Corresponde a una lectura del
importante tener en cuenta que se está trabajando con texto en detalle. El estudiante debe tomarse todo el
jóvenes en una edad en que la frustración es un tema tiempo necesario, diferenciando claramente lo que
sensible. Por esto, es esencial que haga lo posible para comprende de lo que no. Si no entiende algo, debe
que sus estudiantes no tengan una sensación de fraca- seguir leyendo; a veces un concepto o idea posterior
so. Esto no es sinónimo de bajar la exigencia, pero sí de puede aclarar algo que previamente no se comprendía.
enseñarles a autorregularse creando una actitud positiva Todas las dudas al final de esta segunda lectura deben
frente al estudio. Antes de hablar de técnicas de estudio, ser marcadas para luego ser aclaradas. 49
es recomendable evaluarlos. El estudio es una actividad
exigente que requiere de un cuerpo y una mente sana, por 3. Lectura de segunda exploración. En esta lectura el estu-
lo tanto, sus estudiantes deben dormir, comer y sentirse diante marcará las zonas del libro en donde esté la in-
bien. Debe estar alerta a cualquier indicio de que algunas formación explicita o de la que pueda hacer inferencias
de estas condiciones no se estén cumpliendo. Por ello, lo para contestar las preguntas que no pudo responder
primero que se les debe enseñar es elegir un lugar de es- con lo visto en clases o en las dos lecturas anteriores.
tudio, silencioso, bien iluminado y a una temperatura que También debe subrayar la idea más importante y las
les sea cómoda. Lo segundo es la constancia y el manejo secundarias, idealmente con distintos colores. En esta
de los tiempos, lo ideal es que los intervalos de estudio etapa se deben resolver las actividades y preguntas en
sean de veinticinco a treinta minutos, con descansos de el texto. Usted como docente también puede proponer
algunos minutos para relajarse. Además, mencióneles actividades complementarias.
que estudiar de noche no es una buena opción, ya que el Es importante que, al comenzar a utilizar esta técnica de
cerebro se acostumbra a rendir intelectualmente durante estudio, guíe a sus estudiantes. Probablemente, durante
ese período y no durante el día, que es el horario en que su trabajo les surgirán dudas, frente a las cuales debe es-
se dan las pruebas o exámenes. Hay bastantes técnicas tar atento para ayudarlos. Se espera que sus estudiantes
de estudio; la memorización, el resumen, los diagramas en lo sucesivo puedan aplicar esta técnica de estudio de
en V, los esquemas, etcétera. Pero profundizaremos en las forma autónoma. Por el momento, al menos al principio,
técnicas de lectura para un estudio eficiente. incentívelos continuamente a estudiar.
1. Lectura de barrido. Es una lectura rápida del texto, sin Guía Didáctica del Docente
detenerse en los problemas u ejercicios. Con ella el es-
tudiante se hace una idea de lo que va a estudiar.
Fuentes:
– Carrell, P. (1998). Can Reading Strategies be Successfully Taught?
– González, R. (2008). Técnicas de estudio.
– Tuero, F. (2009). Técnicas de estudio.

Física • 2.º medio


ad

unid
Ficha de refuerzo
1
Lección 1

Para reforzar los aprendizajes de la lección, realiza las siguientes actividades en tu


cuaderno.

1. ¿Cuál es la posición de cada gato en relación con el siguiente sistema de coordenada?

A B

–2 –1 0 1 2 X (cm)
Material fotocopiable

2. Cuando viajas en un auto: ¿qué es lo que se mueve, tú o lo que te rodea?, ¿según cuál sis-
tema de referencia? Explica.
3. Javiera se encuentra inicialmente en la esquina de una plaza, como se muestra en el si-
guiente esquema. Si ella rodea la plaza por el borde, hasta llegar al punto desde el cual
partió, ¿cuál es la distancia que recorrió y su desplazamiento?
50
100 m
Javiera

100 m 100 m

100 m
4. Francisca y Felipe corren por el parque en línea recta, Francisca a 5 m/s hacia la derecha y
Felipe a 7 m/s hacia la izquierda. A partir de esta situación, responde:
a. ¿Francisca se acerca o se aleja de Felipe?
b. ¿Cuál es la velocidad de Felipe con
respecto a Francisca?
c. ¿Cuál es la velocidad de Francisca con
respecto a Felipe?

5. Frente a ti pasa un tren en línea recta a velocidad constante, y en el interior se mueve hacia
la cola del tren el cobrador de boletos. Dibuja la trayectoria del cobrador de boletos desde
el punto de vista del conductor del tren y desde tu punto de vista.
6. ¿Qué quiere decir que la velocidad relativa entre dos objetos que se encuentran en movi-
miento sea cero? Explica.
7. En una carrera de caballos, uno de ellos se mueve a 60 km/h y otro a 60,5 km/h. ¿Cuál es
la velocidad de este último con respecto al primero?

Nombre:
ad

unid
Ficha de ampliación
1
Lección 1

Para reforzar los aprendizajes de la lección, realiza las siguientes actividades en tu


cuaderno.

1. ¿Cuál es la posición de cada gato en relación con el siguiente sistema de coordenada?

Y (cm)

2
A

1
B

Material fotocopiable
(0 ,0) X (cm)
–2 –1 12

2. La Tierra gira en torno a su propio eje y además se traslada alrededor del Sol. Si sitúas un
sistema de referencia en el Sol y uno en la Tierra:
a. ¿En qué caso tú te estarías moviendo y en qué caso la Tierra se movería?
b. ¿El Sol se movería en alguno de estos casos? 51

3. ¿Qué características debe tener el movimiento de una persona para que el valor del despla-
zamiento sea igual al de la distancia recorrida?
4. Un auto se mueve a 10 km/h con respecto a un bus que viaja en la misma dirección y sen-
tido. Al pasar frente a un control policial de velocidad se determinó que el bus excedía la
velocidad permitida de 120 km/h. ¿El auto excedía la velocidad permitida? Explica.
5. En el borde de la rueda de un automóvil se marca un punto, como el que se muestra en el
esquema. Si el auto comienza a moverse hacia la derecha por un camino: ¿cómo será la
trayectoria que sigue el punto desde el punto de vista de un observador que se encuentra
inmóvil al costado del camino?, ¿y desde el punto de vista de un observador que se mueve
al costado del auto con la misma velocidad que este?

6. ¿Cómo podrías explicar a un compañero el concepto de velocidad relativa con un ejemplo


de la vida cotidiana? Escríbelo.
7. Dos atletas, Sebastián y Romina, durante una práctica corren por una pista recta en diferen-
tes sentidos, pero en la misma dirección. Si Romina se mueve a 32 km/h hacia la derecha y
Sebastián a 35 km/h en el sentido contrario, ¿cuál es la velocidad de Romina con respecto
a Sebastián?

Nombre:
ad

unid
Ficha de refuerzo
1
Lección 2

Para reforzar los aprendizajes de la lección, realiza las siguientes actividades en tu


cuaderno.

1. ¿Cuáles de las siguientes características corresponden a la velocidad y cuáles a la rapi-


dez? Indica con una V para las que corresponden a la velocidad y con una R para las de la
rapidez.
Es una magnitud escalar.
Es una magnitud vectorial.
Corresponde al desplazamiento por unidad de tiempo.
Corresponde a la distancia por unidad de tiempo.
Material fotocopiable

2. Jorge camina en línea recta por una plaza, cruzándola de forma diagonal, recorriendo 60 m
en 3 min. A partir de esta situación, responde:
a. ¿Cuál es la distancia que recorre?
b. ¿Cuál es el módulo del desplazamiento?
c. ¿Qué valor tendrá su rapidez y el módulo de su velocidad?
3. ¿Cuál o cuáles de los siguientes gráficos describe correctamente el movimiento de un au-
52 tomóvil que se mueve en línea recta con velocidad constante por una carretera? Justifica
tu elección.
Gráfico posición-tiempo Gráfico velocidad-tiempo Gráfico aceleración-tiempo
1 x (m)
2 v (m/s)
3 a (m/​s​2​)

1 1 10

0,5 0,5

0 0 0
0,5 1 1,5 2 t (s) 0,5 1 1,5 2 t (s) 0,5 1 1,5 2 t (s)

4. A continuación se grafica la velocidad de una lancha que se mueve en línea recta.


Gráfico velocidad-tiempo
v (m/s)

B C
20

10

D
A
45 t (s)
10 20 30 40

a. ¿En cuál o cuáles tramos se mueve con velocidad constante?


b. ¿En qué tramos la lancha acelera y en cuáles desacelera?
c. ¿Cómo puedes determinar la distancia que recorrió en total la lancha? Determínala.
5. Describe el movimiento rectilíneo uniforme y el uniformemente acelerado utilizando grá-
ficos de posición, velocidad y aceleración en función del tiempo.

Nombre:
ad

unid
Ficha de ampliación
1
Lección 2

Para reforzar los aprendizajes de la lección, realiza las siguientes actividades en tu


cuaderno.

1. Busca un ejemplo de la vida cotidiana que muestre claramente la diferencia entre rapidez
y velocidad. Explícalo mediante un esquema, de forma que lo puedan entender tus com-
pañeros o compañeras que no comprenden la diferencia.
2. Pídele a un compañero o una compañera que camine libremente desde un extremo de la
cancha de tu colegio hasta el extremo opuesto. A medida que camina, debe ir marcando con
tiza su trayectoria. Mide con un cronómetro el tiempo que tardó en recorrer dicha distancia.
A partir de esta situación, responde:
a. ¿Cuál es el desplazamiento de tu compañero(a)?

Material fotocopiable
b. ¿Cuál es la distancia que recorrió?
c. ¿Cuál es el valor de su rapidez y el de su velocidad?
d. ¿Qué debería ocurrir para que el módulo de la velocidad de tu compañero(a) fuera
igual a su rapidez?
3. Un móvil describe un movimiento rectilíneo uniforme y su posición en el tiempo se pre-
senta en la siguiente tabla.
X (m) 0 10 20 30 40 50 53

t (s) 0 1 2 3 4 5
a. Realiza un gráfico de posición en función del tiempo.
b. A partir del gráfico construido, determina el módulo de la velocidad del móvil.
c. ¿Qué forma tendría el gráfico de posición en función del tiempo si el móvil fuera au-
mentando la velocidad a medida que avanza? Realiza un esquema.
4. El siguiente gráfico muestra cómo varía la posición de una nadadora en el tiempo.
Gráfico posición-tiempo
x (m)
2

0
1 2 3 4 5 6 7 t (s)

-2

a. ¿Qué velocidad media experimenta la nadadora entre los 5 y 6 s?, ¿y entre los 6 y 7 s?
b. ¿Cómo será el movimiento de la nadadora entre los 5 y 6 s? Descríbelo.
5. Confecciona un gráfico de posición, uno de velocidad y otro de aceleración en función del
tiempo de un corredor que comienza su movimiento con aceleración es de 1 m/ s2. Supón
que el movimiento es rectilíneo.

Nombre:
ad

unid
Ficha de refuerzo
1
Lección 3

Para reforzar los aprendizajes de la lección, realiza las siguientes actividades en tu


cuaderno.

1. Josefina lanza una pelota verticalmente hacia arriba y esta alcanza una altura de 3 m.
Luego, su amiga Bernardita lanza la misma pelota hacia arriba, pero esta solo alcanza una
altura de 2 m. A partir de esta situación, responde:
a. ¿De qué depende, despreciando el roce con el aire, la altura que alcanza la pelota en
ambos casos?
b. ¿Qué características debería tener el lanzamiento para que la pelota llegue a una altura
de 5 m?
Material fotocopiable

2. Una piedra demora 10 s en alcanzar su máxima altura, si se considera g = 10 m/s2 y despre-


ciables los efectos de roce. ¿Con qué velocidad fue lanzada hacia arriba la piedra?
3. Describe la caída libre y el lanzamiento vertical utilizando gráficos de posición, velocidad
y aceleración en función del tiempo.
4. Una persona situada en la terraza de un edificio de 5 pisos, sostiene en sus manos dos es-
feras macizas, una de plumavit y la otra de acero. Si “suprimimos el aire”, en el momento
en que suelta simultáneamente las dos esferas, entonces:
a. ¿Qué tipo de movimiento describen ambas esferas?
54
b. ¿Cómo es la caída de ambas esferas? Describe.
c. ¿Influye la masa en la caída de las esferas? Fundamenta.
5. Javier lanza verticalmente hacia arriba una piedra desde un punto O que considera como
origen del sistema de referencia. A partir de esto, afirma que en el punto más alto que al-
canza la piedra:
I. Su rapidez es nula.
II. Su aceleración es nula.
III. Su velocidad es nula.
IV. Su posición es nula.
¿Cuál(es) de las afirmaciones es correcta(s)? Justifica tu elección.
6. Confecciona un organizador gráfico que relacione los siguientes contenidos aprendidos
en esta lección. Considera incorporar las características de la caída libre y el lanzamiento
vertical en este.

Caída libre MRUA Ecuaciones de movimiento

Velocidad Lanzamiento vertical Aceleración de gravedad

Posición Tiempo Representación gráfica

Nombre:
ad

unid
Ficha de ampliación
1
Lección 3

Para reforzar los aprendizajes de la lección, realiza las siguientes actividades en tu


cuaderno.

1. Javier quiere lanzar una pelota verticalmente hacia arriba para llegar al balcón de su de-
partamento, que se encuentra a 4 m de altura, pero no sabe con qué velocidad inicial mí-
nima hacerlo. Dos amigos, Gabriel y Gerardo, le ayudan a calcular esta velocidad, como se
muestra a continuación:
Gabriel Gerardo
1 ​v​  2f​ ​ = ​v​  2i​  ​ + 2a ⋅ Δx
d = ​v0​ ​ t + ​ _  ​ g​t2​ ​
2 Si ​v​ ​ = 0
1
​v0​ ​ =d + ​ _  ​  g = 8,9 m/s
f
______
2 ​v​i​ = ​√2g ⋅ Δx ​ 

Material fotocopiable
____
​ m/s ≈ 8,5 m/s
= ​√ 78,4  

¿Quién está en lo correcto? Justifica tu elección.


2. Diseña un procedimiento experimental para determinar la aceleración que experimenta
una piedra cuando se la deja caer desde cierta altura.
3. Confecciona un gráfico que de posición, uno de velocidad y otro de aceleración en función
del tiempo de un proyectil que se lanza verticalmente hacia arriba con una velocidad de
50 m/s.
4. Imagina que se sueltan simultáneamente desde la misma altura una pluma y un martillo. 55
Si se desprecia la resistencia del aire, un estudiante afirma que:
I. El martillo llega primero al suelo.
II. El martillo tiene mayor aceleración que la pluma.
III. Ambos impactan en el suelo con igual rapidez.
¿Cuál(es) de las afirmaciones es correcta(s)? Justifica tu elección.
5. A 100 metros sobre la superficie de un lago, Javier lanza verticalmente hacia arriba, con
una resortera, una piedra que demora 20 segundos en impactar en la superficie del agua.
Despreciando la resistencia del aire y considerando g = 10 m/s2, determina:
a. La velocidad inicial de la piedra.
b. La altura máxima que alcanzó la piedra.
6. Confecciona un organizador gráfico que relacione los contenidos, habilidades y actitudes
aprendidos en esta lección.

Caída libre Creatividad Ecuaciones de movimiento

Evidencias Aplicación de modelos Aceleración de gravedad

Uso responsable Tiempo Representación gráfica


de las TIC

Nombre:
ad

unid
Desafío complejo
1
Objetivo: Describir el movimiento Habilidades: Identificación de pro- Actitudes: Manifestar interés
de un insecto y representar los da- blemas, hipótesis, procedimientos por conocer más de la reali-
tos obtenidos. experimentales, inferencias y con- dad y utilizar sus conocimien-
clusiones en investigaciones cientí- tos al estudiar los fenómenos
ficas clásicas o contemporáneas. abordados en la unidad.

¿Cómo se mueven los insectos?


INICIO
Lee atentamente la siguiente situación.
Material fotocopiable

Vicente es un aficionado de los insectos y quiere investigar sobre las


características del movimiento de los chanchitos de tierra. ¿Cuál será la mejor
forma de representar los datos recopilados durante su investigación?

DESARROLLO CIERRE
Realiza la siguiente actividad, la cual te permitirá A partir de la actividad experimental y lo aprendido
responder la pregunta inicial. en la lección, responde las siguientes preguntas.
56 1. Formen grupos de cuatro integrantes y reúnan los a. Si en el papel milimetrado realizaras un eje de
siguientes materiales: papel milimetrado, cronó- coordenadas cartesiano, ¿cómo podrías indicar
metro, lápiz, regla y chanchitos de tierra. las posiciones inicial y final del chanchito de
2. Marquen con un lápiz, en la hoja de papel mili- tierra durante su movimiento? Explica.
metrado, la posición inicial desde donde comen- b. Señala las posiciones inicial y final de los tres
zará el movimiento del chanchito de tierra. chanchitos de tierra.
3. Tomen con mucho cuidado uno de los chanchitos c. ¿Conoces otra forma de representar posiciones?,
de tierra, dejándolo en el lugar marcado como po- ¿cuál?
sición inicial.
4. Midan el tiempo con un cronómetro y marquen d. ¿Qué etapas del procedimiento experimental te
en la hoja de papel milimetrado con un lápiz la dificultó más llevar a cabo?, ¿por qué? Señala
posición final del chanchito de tierra, luego de una estrategia para mejorarlo.
transcurridos unos 10s. e. ¿Podrías determinar la velocidad y rapidez de los
5. De forma adicional pueden realizar una graba- chanchitos de tierra?, ¿cómo? ¿Su velocidad será
ción con un celular del movimiento del chanchito la misma si la consideraras con respecto a otro
de tierra. sistema de referencia, por ejemplo, con respecto
6. Repitan los pasos anteriores con otros dos chan- a otro chanchito de tierra que se moviera en el
chitos de tierra. Es importante que sean muy cui- mismo papel milimetrado?
dadosos al manipular estos insectos y que, una f. ¿Qué fue lo que más te llamó la atención al reali-
vez finalizada la actividad, los devuelvan al lugar zar esta actividad?
en que los encontraron. g. ¿Qué otros movimientos podrías describir usan-
do la misma técnica?

Nombre:
ad

unid
Desafío complejo
1
Objetivo: Describir el movimiento Habilidades: Organizar e interpre- Actitudes: Desarrollar actitudes
usando un software de análisis de tar datos, y formular explicaciones. de creatividad e innovación.
movimientos. Identificar las limitaciones que
presentan los modelos.

¿Cómo utilizar un software para


describir un movimiento?
INICIO

Material fotocopiable
Lee atentamente la siguiente situación.

Durante una competencia, los atletas se mueven con distintas velocidades, tardan
diferentes tiempos en completar la carrera, aceleran de diferentes maneras, es decir, cada
uno describe un movimiento con diferentes características. ¿Has sentido curiosidad alguna
vez acerca de cómo sería tu desempeño en una competencia de alta exigencia?

DESARROLLO Definan el objeto que quieren analizar, en este ca-


Realiza la siguiente actividad, la cual te permitirá so el estudiante. Para esto, deben hacer clic en el 57
responder la pregunta inicial. botón Creary Masa puntual, marcando Trayectoria
automática. Luego, seleccionen un punto del estu-
1. Reúnanse en grupos de cinco estudiantes y consi- diante, que se diferencia del fondo, presionando
gan los siguientes materiales: celular con cámara las teclas Ctrl y Shift. Presionen el botón Search
de video, 6 banderines, huincha de medir y un para buscar la masa puntual seleccionada en todo
silbato. el video. Automáticamente se generan datos de la
2. Busquen un lugar donde puedan correr en línea posición en el tiempo, los cuales se grafican.
recta, como mínimo 50 m. Ubiquen un banderín CIERRE
en el punto que considerarán como origen del sis-
tema de referencia y otro cada 10 m hasta los 50 m. A partir de la actividad y lo aprendido en la lección,
3. Para el análisis del movimiento descrito por cada responde las siguientes preguntas.
estudiante, utilicen el software gratuito llamado a. Construye una tabla con los datos del recorrido
Tracker. Realicen el recorrido de forma indivi- de cada estudiante; el tiempo en que recorrió los
dual, grabando cada caso. 50 m, su velocidad promedio en cada tramo y su
4. En el programa deben abrir uno de los videos aceleración.
grabados. Seleccionen solo la parte del video que b. A partir de los datos obtenidos, ¿qué caracterís-
muestre el movimiento del estudiante en los 50 m. ticas tienen los movimientos descritos por los
Esto lo pueden hacer con el botón Ajustes de cor- integrantes del grupo? Compáralos.
te. Inserten los ejes de coordenadas y ubiquen el
origen en el punto de partida del movimiento. c. ¿Cómo fue tu desempeño al recorrer los 50 m?
Haciendo clic en Herramientas de calibración, ¿Los resultados fueron los que esperabas?
inserten una Vara de calibración, marcando la d. ¿En qué otras situaciones podrías usar este
distancia entre dos banderines como referencia. software?

Nombre:
Evaluación de la unidad
I. Selección múltiple 3 ¿Cuál es la rapidez de Francisca?
1 Mauricio se encuentra parado sobre el suelo A. 0,0125 m/min D. 80 m/min
de su cocina, como se muestra en la siguien- B. 1,33 m/min E. 100 m/min
te imagen. C. 5 m/min
y (m)
4 Desde una avioneta que viaja hacia la de-
recha, se deja caer una pesa. ¿Cómo verá la
2 trayectoria de la peso Pedro, que se encuen-
tra en reposo sobre la Tierra?
1

0 x (m)
Material fotocopiable

1 2 3 4

Si se mueve al punto (1, 2), ¿cuál será su


desplazamiento?
A. −2 m, en dirección del eje Y.
B. 2 m, en dirección del eje Y. A. D.
C. 2 m, en dirección del eje X.
D. −2 m, en dirección del eje X.
58 E. 2 m, alejándose del origen. B.
C. E.
A partir del siguiente enunciado, responde las
preguntas 2 y 3.
Francisca corre desde su casa a la de su amiga,
tardando 5 min en recorrer 4 cuadras, como se
muestra en el siguiente esquema:
Casa amiga 5 Fernando calcula la velocidad de un auto
100 m durante un tramo recto de una carretera.
Si quiere expresar el resultado en unidades
del Sistema Internacional, ¿en qué unidad
100 m

debería expresarla?
100 m
A. m D. m/s
100 m

B. s E. km/s
C. km/h
Casa de Francisca
6 Dos automóviles se mueven con distinta
2 ¿Cuál es la distancia recorrida por Francisca velocidad por un camino recto, como se
Guía Didáctica del Docente

y su desplazamiento, respectivamente? muestra en el siguiente esquema:


___
A. 400 m y 200 ​​√ 2 ​​  m
___ 100 km/h
B. 200 √​​  2 ​​ m y 400 m
___ ___ A 90 km/h
C. 200 √ ​​  2 ​​ m y 200 ​​√ 2 ​​  m
D. 400 m y 400 m B
___
E. 400 m y 400 ​​√ 2 ​​  m

Unidad 1 • Movimiento
ad

unid
1
Si se considera que ambos viajan en sentido 10 Un atleta (Sistema A`) pasa frente a un
positivo, ¿cuál es la velocidad del auto B con semáforo (Sistema A) a una velocidad v con
respecto a la del auto A? respecto al semáforo.
A. 190 km/h D. 90 km/h
¿Cuál(es) de las siguientes afirmaciones es o
B. −10 km/h E. 190 km/h
son correcta(s)?
C. 10 km/h
I. En un instante t, las coordenadas del
7 Un tren viaja de Temuco a Santiago y al atleta con respecto al semáforo, serían
pasar por la estación de San Fernando lo los siguientes: X' = X + v⋅t ;Y'= Y; Z' = Z,
hace a una velocidad de 80 km/h. Si en la donde son paralelos los ejes X con X' e Y
estación Juan observa pasar el tren mientras con Y' y son perperpendiculares Z con Z'.
camina hacia el sur a 50 m/min, ¿cuál es la II. Para aplicar la transformación de
velocidad de Juan con respecto a los pasaje- Galileo es necesario que los tiempos

Material fotocopiable
ros del tren? Considera el sentido positivo cero de ambos sistemas coincidan.
del movimiento hacia el norte. III. La transformación de Galileo permite
A. −77 km/h D. 77 km/h describir el movimiento de un cuerpo
B. −30 km/h E. −83 km/h con respecto a un sistema que se mueve
con velocidad constante.
C. 30 km/h
A. Solo I. D. Solo I y III.
8 Marcela se sienta en una banca mientras B. Solo II. E. I, II y III.
frente a ella pasan dos niños corriendo,
C. Solo I y II. 59
Pedro hacia la derecha con una velocidad
de 3 m/s y Daniel hacia la izquierda a −2 11 Un cubo de madera es lanzado verticalmen-
m/s. ¿Cuál es la velocidad de Pedro con te hacia arriba con una rapidez inicial de
respecto a Marcela y con respecto a Daniel, 10 m/s. Si se considera g = 10 m/s2, ¿qué
respectivamente? altura alcanza el cubo?
A. 5 m/s y 3 m/s D. 1 m/s y 5 m/s A. 2 m D. 50 m
B. 3 m/s y 5 m/s E. 3 m/s y 0 m/s B. 5 m E. 100 m
C. 3 m/s y 1 m/s C. 15 m
9 La velocidad de un automóvil con respecto 12 El siguiente gráfico muestra la posición en
a un bus es de −20 km/h. Si la velocidad el tiempo de un ciclista que se mueve en
del bus con respecto a un observador que se línea recta y en una misma dirección.
encuentra en reposo a la orilla del camino
Gráfico posición-tiempo
es de 120 km/h, ¿cuál es la velocidad del
x (m)
automóvil con respecto al observador? 12
10
A. −100 km/h D. -80 km/h 8

B. −20 km/h E. 20 km/h 6

C. 80 km/h
4 Guía Didáctica del Docente
2

–2 –1 0 1 2 3 4 t (s)
–2
–4

Física • 2.º medio


Evaluación de la unidad
¿Cuál(es) de las siguientes afirmaciones es Con respecto al gráfico anterior, ¿cuál de las
(son) correcta(s)? siguientes afirmaciones es correcta?
I. La rapidez del ciclista es constante. A. En el tramo 1 la rapidez del automóvil es
II. La aceleración del ciclista es nula. constante.
III. Transcurridos 3 s el ciclista se ha B. En el tramo 2 la rapidez del automóvil se
desplazado 3 m. mantiene constante.
A. Solo I. D. Solo I y III. C. La rapidez del automóvil en el tramo 1 es de
B. Solo II. E. Solo II y III. 20 m/s.
C. Solo I y II. D. La rapidez del automóvil aumenta en el
tramo 3.
13 El gráfico muestra la posición de un carro E. La rapidez del automóvil en el tramo 3 es de
en función del tiempo. 100 m/s.
Material fotocopiable

Gráfico posición-tiempo
x (m)
15 La rapidez de un atleta es graficada en fun-
200 ción del tiempo.
180
160 Gráfico velocidad-tiempo
140 v (m/s)
120
100 40
80 30
60
20
40
20 10 A B C

60 0 2 4 6 8 10 12 0 1 2 3 4 5 6 7 8 9 10 11 12 t (s)
t (s)

¿Cuál de las siguientes afirmaciones es co-


¿Qué característica tiene el movimiento del rrecta con respecto al movimiento del atleta?
carro?
A. El atleta comienza a moverse a 30 m/s.
A. Velocidad constante.
B. La distancia recorrida en el tramo B es de
B. Posición constante. 60 m.
C. Movimiento acelerado. C. La rapidez alcanzada en el tramo A es de
D. Movimiento uniforme. 45 m/s.
E. Aceleración nula. D. En el tramo B el atleta no se mueve.
14 El siguiente gráfico muestra la rapidez de un E. El atleta recorre en total 330 m.
automóvil que se mueve en una misma direc- 16 La aceleración de un automóvil de carrera
ción por un tramo recto de una carretera. es graficada en función del tiempo, como se
Gráfico velocidad-tiempo muestra a continuación.
v (m/s) Gráfico aceleración-tiempo
30
2 a (m/​s2​ ​)
55
50
Guía Didáctica del Docente

1
20 45
3 40
35
10 30
25
20
15
0 20 40 60 80 100 t (s)
10
5

0 1 2 3 4 5 6 7 8 9
t (s)

Unidad 1 • Movimiento
ad

unid
1
¿Cuál de las siguientes afirmaciones es Gráfico posición-tiempo
correcta con respecto al movimiento del y

automóvil?
H
A. Entre los 0 y 9 s la rapidez del automóvil
aumenta en 225 m/s.
B. El automóvil recorre en 9 s una distancia de
225 m.
C. La velocidad del automóvil es constante t
​t​​ ​t​​
entre los 0 y 9 s. 1 2

D. El automóvil tiene un desplazamiento nulo ¿Cuál(es) de las siguientes afirmaciones es


pasados los 9 s. (son) correcta(s)?
E. La aceleración del automóvil a los 9 s es de I. La esfera es lanzada hacia arriba desde

Material fotocopiable
225 m/s2. el punto H.
17 Gonzalo deja caer una piedra desde un II. En el instante t1 la esfera inicia su
puente de 40 m de altura sobre el agua. descenso.
Transcurridos 2 s, ¿en qué posición se en- III. La esfera asciende hasta el instante t1
contrará la piedra? Considera la aceleración donde su altura es máxima.
de gravedad igual a 10 m/s2. A. Solo III. D. Solo I y III.
A. A 5 m del puente. B. Solo I y II. E. I, II y III.
B. A 10 m del puente. C. Solo II y III. 61
C. A 20 m del agua.
20 La velocidad de un tren es graficada por
D. A 40 m del puente. Francisca como se muestra a continuación:
E. A 5 m bajo el agua.
Gráfico velocidad-tiempo
v (m/s)
18 ¿Cuál de las siguientes afirmaciones es
40
correcta respecto de la aceleración de
38
gravedad? 36

A. En el lanzamiento vertical hacia arriba su 34

sentido coincide con el movimiento. 32

B. Depende de las características del 30

movimiento vertical.
0 1 2 3 4 5 t (s)
C. Posee un valor aproximadamente constante.
D. En la caída libre se opone al movimiento. Con respecto al gráfico construido por Fran-
cisca, ¿cuál de las siguientes afirmaciones
E. Depende de la masa de los cuerpos en caída
es correcta?
libre.
A. La velocidad del tren permanece constante.
19 La posición de una esfera es graficada como B. La aceleración del tren aumenta Guía Didáctica del Docente
se muestra a continuación. gradualmente.
C. La velocidad inicial del tren es nula.
D. La aceleración del tren permanece constante.
E. La velocidad del tren aumenta
exponencialmente.

Física • 2.º medio


Solucionario
2. Se espera que los estudiantes mencionen algunas de las
Del Texto del estudiante siguientes actitudes: expresar opiniones en torno a los
fenómenos observados; utilizar el conocimiento adqui-
Inicio de unidad (P. 17) rido para solucionar problemáticas cotidianas; formular
1. Romina tiene la razón, ya que la caída de los cuerpos no preguntas interesantes sobre las observaciones del en-
depende de la masa de los mismos. torno natural.
2. La velocidad inicial es cero y la caída se produce por la Indaguemos (P. 28)
aceleración de gravedad. a. Los conceptos de distancia y tiempo.
3. Dejar caer desde la misma altura y simultáneamente dos b. El tiempo.
objetos de diferente masa para observar como es la caída
c. La rapidez corresponde a la distancia que se recorre en
que describen.
un determinad tiempo.
Lección 1: Descripción del movimiento d. Respuestas variadas.
Tema 1: ¿Cuándo nos movemos?
Indaguemos (P. 24) Aplico la ecuación de rapidez (P. 28)
d. En la posición de los botones respecto de la cuadrícula d (m) t (s) vm (m/s)
realizada.
5 5 1
e. Para que un cuerpo se mueva, necesariamente debe
cambiar de posición. El movimiento se define como el 10 5 2
cambio de posición de un cuerpo respecto de otro. 12 3 4
f. Se espera que los estudiantes consideren que la observa- Desarrollo de estrategias (P. 29)
ción permite identificar los conceptos relacionados con
el fenómeno. Además, permite realizar descripciones de v = 2,2 m; →
m
v
​​   ​​  = –2,2 m
m

un suceso a partir de la información que se obtiene me- Tema 2: Analizando la relatividad del movimiento
diante la observación. Establezcamos un sistema de referencia (P. 31)
62 Determinemos la posición de las personas (P. 25) 1. Un pasajero sentado.
1. → ​​  A ​​  = –30 m; →
x x ​​ →
​​  B ​​  = 20 m; x  ​​ = 60 m 2. Un pasajero caminando dentro del bus.
→ ​​  = –90 m; ​​ x→ ​​ = –40 m
C
2. ​​ x 3. Un vehículo que se mueva con la misma velocidad
A B
3. La posición de cada persona es diferente en ambos ca- del bus.
sos, ya que se cambia el sistema de referencia. 4. El letrero del paradero; los árboles, una vaca acostada,
4. Se utilizan los conocimientos de posición, sistema de por ejemplo.
referencias y sistema de coordenadas. 5. Es importante que las y los estudiantes argumenten que
efectivamente todos los movimientos son relativos ya
Indaguemos (P. 26)
que todos ellos dependen del sistema de referencia que
a. El trazo de color azul es una línea sin forma definida, los describe.
mientras que el trazo rojo es una flecha.
Indaguemos (P. 32)
b. El trazo azul representa la distancia recorrida, ya que se
formó al seguir el camino descrito por el insecto en la a. La trayectoria de la pelota desde la posición 1 es para-
hoja. La flecha roja representa el desplazamiento, ya que bólica y el movimiento es de izquierda a derecha. Des-
comienza en el punto de partida y termina en la posición de la posición 2 la trayectoria se observa recta y hacia
final del insecto. el frente.
c. La distancia recorrida es la medida del camino descrito b. La trayectoria se ve diferente, ya que depende del siste-
en un movimiento. El desplazamiento es el cambio de ma de referencias desde el cual se observa.
posición de un cuerpo. c. Las y los estudiantes pueden sugerir inquietudes, por
d. Se espera que las y los estudiantes consideren que es im- ejemplo: ¿Todas las trayectorias son relativas? Además,
portante diferenciar ambos conceptos, porque cada uno pueden establecer la importancia de la observación
Guía Didáctica del Docente

representa una descripción particular del movimiento. considerando que esta permite identificar los conceptos
En uno se representa la distancia de un camino seguido, científicos relacionados con el fenómeno observado y
mientras que en el otro se representa el cambio de una describir un objeto presente en un suceso con la infor-
posición a otra. mación del registro de observaciones. Del mismo modo,
reconocen que dos o más observadores pueden tener
Determino la distancia recorrida y el desplazamiento distintas percepciones de un mismo fenómeno.
(P. 27)
1. Δ​x ​​ ​ → = 120 m; d = 180 m

Unidad 1 • Movimiento
ad

unid
1
Indaguemos (P. 34) c. Se puede decir que es constante la velocidad, ya que en
a. Velocidad media, sistema de referencia y movimiento cada tramo tiene aproximadamente el mismo valor.
relativo. d. Por ejemplo: escuchando las opiniones de cada integran-
b. Si se considera como positivo el sentido hacia abajo de te; considerando las capacidades de cada uno, entre otras.
la escalera: v ​​ ​​  →Sandra
  → 
= 2 m/s; ​​ v ​​ Carlos
= –2 m/s Reflexiono sobre el gráfico de posición (P. 41)
c. Si se considera como positivo el sentido hacia abajo de a. La persona 1.
​​  →Carlos
la v ​​   = –4 m/s b. Porque recorre una mayor distancia en el menor tiempo.
d. Por ejemplo: ¿Con que velocidad se ve Sandra a sí mis- Además, la pendiente de la recta es mayor.
ma? ¿Con qué velocidad ve Sandra a Lucía? Interpretemos gráficos del MRU
Integro lo que aprendí (P. 36 y 37) ​​ →i Javiera = 6 m; x ​​
a. x ​​ ​​ →f  Javiera = 2 m
​​ →  ​​ → 
1. Desde un observador Desde un observador en
b. x ​​ = 1 m; x ​​
i Fernando
=4m i Fernando
dentro del avión: tierra:
c. Δ​x ​​ ​ → Javiera = –4 m; Δ​x ​​
​ → Fernando = 3 m
​​  →​​  
d. v  ​​ → 
= –1,33 m; v ​​
m Javiera
= 0,75 m m Fernando
Gráfico velocidad - tiempo Gráfico velocidad - tiempo
3 3

Velocidad (m/s)

Velocidad (m/s)
Fernando
2 Javiera 2
1 1

2. a. Desde el auto que posee la mayor velocidad. 0


1 2 3 4 5
0
1 2 3 4 5
→ 
b. ​​ v ​​ = –7 m/s
-1
-2
-1
-2
(ciclista/corredor) Tiempo (s) Tiempo (s)
c. Se determina con la ecuación de rapidez.
a. dJaviera = 4 m; dFernando = 3 m
3. X´= 10 m
b. Porque, al describir un movimiento rectilíneo uniforme,
4. No es correcto lo que afirma Jorge, ya que al encontrar-
el módulo del desplazamiento es igual a la distancia
se ambos ciclistas que se mueven con la misma rapidez
recorrida.
pero en sentido contrario, la velocidad relativa de uno 63
c. Por ejemplo: permiten representar información gráfica-
respecto del otro será el doble.
mente; sirven como herramienta de análisis de la infor-
5. a. El movimiento de los astros dependerá del sistema
mación, entre otros.
de referencia.
Desarrollo de estrategias (P. 43)
b. El sistema puede estar en reposo en la Tierra, o bien
describir el mismo movimiento que la Tierra pero ​​  →
v ​​ m = 5 m/s. Llega primero a la meta Andrea ya que se
que parezca en reposo. demora un tiempo de 16 segundos.
6. Posición
A poner en práctica (P. 44 y 45)
b. Gráfico distancia-tiempo
Rapidez
Distancia (cm)

Velocidad

Parámetros Tiempo

Distancia
Movimiento recoririda
Tiempo (s)

Desplazamiento c. El gráfico distancia recorrida en función del tiempo re-


Es relativo
depende del
presenta una línea recta. En consecuencia, se observa
Sistema de que la distancia recorrida es proporcional al tiempo.
referencia d. Se puede conocer la rapidez determinando la pendiente
Lección 2: Análisis de los movimientos horizontales de la recta.
Guía Didáctica del Docente
Tema 2: Describiendo el Movimiento Rectilíneo e. Se puede determinar el valor de tiempo prolongando la
Uniforme (MRU) recta graficada, o bien utilizando la ecuación de rapidez.
Indaguemos (P. 40) f. En un frasco de Mariotte, el descenso del agua describe
a. Que demora aproximadamente el mismo tiempo en re- un movimiento rectilíneo uniforme ya que recorre dis-
correr cada tramo. tancias iguales en tiempos iguales.
b. El módulo de la velocidad es aproximadamente el g. Por ejemplo: dificultad al manipular los materiales, al
mismo. elaborar el montaje o al establecer las funciones de cada
integrante para la obtención de los datos.

Física • 2.º medio


Solucionario
h. Se espera que las y los estudiantes evalúen su trabajo 3. a. vf = 11 m/s
colectivo de forma crítica, coherente y de acuerdo a ar- b. d = 19,5 m
gumentos sólidos. c. Por ejemplo: utilizar casco, tobilleras y rodilleras.
Tema 2: Movimiento Rectilíneo Uniformemente Acele- →
4. a. ​​a ​​m  = –4,16 m/s2
rado (MRUA) b. d = 75,12 m
Indaguemos (P. 46) c. Gráfico velocidad - tiempo
30
a. Los conceptos de fuerza, velocidad, aceleración, posi-

Velocidad (m/s)
25
20
ción, distancia y tiempo. 15
10

b. El estado inicial en ambos casos fue el reposo. 5


0
0 2 4 6 8
c. La velocidad inicial de la bolita era cero; luego comenzó Tiempo (s)

a aumentar debido al impulso de la mano. 5. Contenidos Habilidades Actitudes


d. La velocidad de la bolita comenzó a aumentar producto
de su caída. Aceleración Interpretar Responsabilidad
gráficos
e. Debido a que inicialmente se encontraba en reposo y Ecuaciones de Interés
luego de cada acción se encontraba en movimiento, por movimiento Construir gráficos
lo cual su velocidad era distinta de cero. Lección 3: Análisis de los movimientos verticales
f. Respuestas variadas. Tema 1: ¿Qué características posee la caída libre?
g. Se espera que las y los estudiantes manifiesten que rea- Indaguemos (P. 56)
lizaron un trabajo riguroso y ordenado. 1. Se espera que las y los estudiantes identifiquen errores.
Aplico la ecuación de aceleración (P. 47) 2. Caen producto de la fuerza de gravedad. La masa no in-
​​a→  ​​m  = 1,27 m/s2 fluye en la caída de los cuerpos.
3. La caída libre es el movimiento que describen los cuer-
Desarrollo de estrategias (P. 48)
pos debido a la fuerza de gravedad cuando se los deja
​​a→  ​​m  = 26,66 m/s2 caer.
64 Desarrollo de estrategias (P. 50) 4. Por ejemplo: Porque permitió reproducir varias veces la
vf = 2 + ___ ​​ 1  ​​t  ; vf = 3,66 m/s experiencia para analizar los resultados.
6
Integro lo que aprendí (P. 52 y 53) A poner en práctica
1. a. Tramo 1 Tramo 2 Tramo 3 Tramo 4 Tramo 5
a. t2 (s2) x(cm) x/t2 (m/s2)
MRUA MRU MRUA MRU MRUA 0 0 ––––
b. 1 25 25
Tramo 1 Tramo 2 Tramo 3 Tramo 4 Tramo 5
4 100 25
4 0 –4 0 4
9 225 25
c. Tramo 1 Tramo 2 Tramo 3 Tramo 4 Tramo 5 16 400 25
200 400 150 200 50 25 625 25
d. Respuestas variadas. b. El valor fue 25. Se puede inferir que la aceleración de la
2. a. bolita fue constante.
Posición Velocidad Aceleración
c. Por ejemplo: Las evidencias permiten comprender el en-
inicial media media
torno que nos rodea y, con ello, proponer modelos que
5 5 1 lo describen.
b. d. Por ejemplo: Es importante, ya que gracias al estudio de
Gráfico posición - tiempo
12
Gráfico velocidad - tiempo Galileo se pudo comprender mejor los efectos gravitato-
100
rios. Además, sus resultados permiten que otros cientí-
Velocidad (m/s)

10
Posición (m)

80
60 8
6
ficos puedan realizar sus propias investigaciones para
Guía Didáctica del Docente

40 4
20 2
0
0 1 2 3 4 5 6
0
0 1 2 3 4 5 6 generar nuevos conocimientos.
Tiempo (s) Tiempo (s)

Gráfico aceleración - tiempo Tema 2: ¿Qué características posee el lanzamiento


vertical?
Aceleración (m/s2)

1
Indaguemos (P. 60)
0
0 1 2 3 4 5 6 a. A medida que sube la pelota disminuye la velocidad, ya
Tiempo (s)
que necesariamente debe detenerse para que comience
a bajar.

Unidad 1 • Movimiento
ad

unid
1
b. A medida que baja la pelota, la velocidad aumenta, ya Lanzamiento vertical hacia arriba
que al comenzar a bajar se encontraba inicialmente en Gráfico posición - tiempo Gráfico velocidad - tiempo

Velocidad (m/s)
reposo.

Posición (m)
Tiempo (s)
c. Mientas mayor es la velocidad inicial, mayor es la altura 0 1 2 3 4 5 6 7

que alcanza la pelota.


d. Describe un MRUA, ya que varía la velocidad. Tiempo (s)

e. Por ejemplo: El uso de la tecnología permite analizar con


Gráfico aceleración - tiempo
Tiempo (s)

Aceleración (m/s2)
más detalle la experiencia, ya que se puede retroceder,
avanzar o repetir el suceso las veces que sean necesarias.
Desarrollo de estrategias (P. 62)
Gráfico velocidad - tiempo

0
Tiempo (s) Evaluación final (P. 70 a 73)
1. a. Describe un MRUA.
1 2 3 4 5
–10
Velocidad (m/s)

–20
–30 b. MRUA, desplazamiento, velocidad, aceleración.
–40
–50
2. a. La pendiente es negativa, ya que se considera que el
–60 movimiento descrito por el bloque es negativo
La pelota impacta el suelo con una velocidad de –55 m/s respecto del sistema de referencia establecido.
b. Gráfico aceleración - tiempo
Integro lo que aprendí (P. 64 y 65) Tiempo (s)

Aceleración (m/s2)
1. Grafico 1: Describe un cuerpo con velocidad variable re-
presentando una caída libre, por lo tanto es un MRUA.
Gráfico 2: Representa el lanzamiento vertical hacia
arriba, y luego la caída del cuerpo, por lo tanto también c. El módulo de la aceleración es 2,75 m/s2.
es un MRUA. d. Recorre una distancia de 5,5 m.
Grafico 3: Representa un movimiento con velocidad e. La pendiente del plano. 65
constante en dos modalidades, por lo tanto es un MRU, f. Ninguna.
donde el cuerpo se aleja del sistema de referencia →
3. a. ​​a ​​m  = 0,8 m/s2
durante un tiempo y luego de detiene por un instante
b. d = 40 m
para regresar al origen.
c. d = 160 m
2. a. yf = vit + __ ​​  1  ​​gt2
2 d. d = 200 m
b. vi = 30 m/s
e. Gráfico velocidad - tiempo
c. 50
Gráfico posición - tiempo Gráfico velocidad - tiempo 9
8
Velocidad  (m/s)

45 40 7
40 30
Posición (m)

Velocidad (m/s)

35 6
30 20 5
25 10 4
20 0 3
15 1 2 3 4 5 6 7 2
10 –10 1
5 –20 0
0 –30 0 5 10 15 20 25 30 35
0 1 2 3 4 5 6 7
–40 Tiempo (s)
Tiempo (s) Tiempo (s)

3. v
​​→  ​​f  = –15 m/s 4. a. v = 18 m/s
4. La afirmación no es correcta porque las caídas de los b. v = 18 m/s
cuerpos no describen distancias iguales en tiempos igua- c. v = 18 m/s
les pues afecta la aceleración de gravedad. 5. a. vi = 3 m/s
5. Por tratarse de una caída libre, Javiera puede utilizar la b. t = 1 s
siguiente ecuación para determinar la distancia a la que c. d = 4,5 m
se encuentra del seuelo: Δy = gt2.
d. Δ​x​= 1,5 m
6. Caída libre
Gráfico posición - tiempo Gráfico velocidad - tiempo e. Por ejemplo, Excel, Word.
Guía Didáctica del Docente
6. a. MRU.
Tiempo (s)
Velocidad (m/s)
Posición (m)

0 1 2 3 4

b. v = 1,25 m/s: ​​v→  ​​  = 1,25 m/s: Que la rapidez es


Tiempo (s)
una magnitud escalar y la velocidad es una
Gráfico aceleración - tiempo magnitud vectorial.
Tiempo (s)
c. x = 7 + 1,25t
Aceleración (m/s2)


d. ​​x ​​  = 37 m
e. Por ejemplo: usar caso, rodilleras y coderas.

Física • 2.º medio


Solucionario
7. a. yi ymáx vi g tsubida 4. a. Francisca se acerca a Felipe hasta encontrarse y
luego se aleja de ella.
0 45 30 –10 3
b. Considerando como sentido positivo hacia la
b. y = 30t – 5t2 derecha, la velocidad de Felipe con respecto a
c. t = 3 s Francisca es −22m/s.
d. 50 Gráfico posición - tiempo Gráfico velocidad - tiempo
c. Considerando como sentido positivo hacia la
45 40
40 30
derecha, la velocidad de Francisca con respecto a Felipe
Posición (m)

Velocidad (m/s)
35 20
30
25 10
20
15
0
–10 1 2 3 4 5 6 7
es 22m/s.
10
5
0
0 1 2 3 4 5 6 7
–20
–30 5. Cuando el sistema de referencia es el conductor del tren.
–40
Tiempo (s) Tiempo (s) Cobrador de boletos
8. Por ejemplo, que el tiempo se dilata, o bien, que es Conductor
relativo.

De la Guía didáctica Cuando el sistema de referencia es un observador


inmóvil fuera del tren.
Actividad complementaria (P. 30) Tren
a. La planta, la planta, la planta.
b. Por ejemplo: La antena se encuentra a la derecha de la Cobrador de boletos
planta. Conductor
c. Una referencia.
d. No, ya que siempre es necesario indicar una referencia
para describir la posición de un objeto.
Observador del andén
Actividad complementaria (P. 31)
6. Ambos objetos se mueven en la misma dirección y sen-
66 a. Posición Camila (5,1,1); Posición Esteban (6,4,2); Posi-
tido, y la magnitud de ambas velocidades es la misma.
ción Paula (7,3,3).
7. Considerando que ambos caballos se mueven en la mis-
b. Paula.
ma dirección y sentido, la velocidad del caballo con res-
Actividad complementaria (P. 32) pecto al otro es de 0,5 km/h.
a. v = 1,33 m/s
Ficha de ampliación (P. 51)
b. ​​v→  ​​  = 0,66 m/s
1. Gato A: (−2, 1) cm; Gato B: (2, 0) cm.
Actividad complementaria (P. 39) 2. a. Si el sistema de referencia está situado en el Sol,
a. tú y la Tierra se mueven. Si el sistema de referencia
b. v = 88 m/s está ubicado en la Tierra, ni tú ni la Tierra estarán
c. d = 960 m en movimiento.
Actividad complementaria (P. 43) b. En el caso de que el sistema de referencia esté
a. Se debe a que en ese instante la velocidad es cero. situado en la Tierra, el Sol se moverá.
b. La velocidad es cero y la altura es máxima. 3. Cuando la trayectoria es rectilínea en un mismo sentido.
c. El valor de la pendiente es 10 y representa el valor de la 4. Si tomamos como sistema de referencia el control
aceleración de gravedad. policial, entonces el bus y el auto viajan a exceso de
velocidad.
Ficha de refuerzo (P. 50)
5. Un observador inmóvil a un costado del camino verá
1. Gato A: −1 cm; Gato B: 2 cm.
una trayectoria similar a la siguiente:
2. Si el sistema de referencia es el observador dentro del
auto, el entorno se mueve respecto del auto. Si el sistema
Guía Didáctica del Docente

de referencia es la carretera, el auto se mueve respecto


de ella.
3. La distancia recorrida por Javiera es de 400 m y su des-
plazamiento es 0.

Unidad 1 • Movimiento
ad

unid
1
Un observador que se mueve a un costado del auto con 5. MRU
la misma velocidad que este verá una trayectoria como Gráfico posición - tiempo Gráfico velocidad - tiempo

la siguiente:

Posición (m)

Velocidad (m/s)
Tiempo (s) Tiempo (s)

Gráfico aceleración - tiempo

6. Algunos ejemplos que pueden describir sus estudiantes

Aceleración (m/s2)
son:
• Un bote que se mueve en un río en la misma dirección Tiempo (s)

que la corriente del río: la velocidad con respecto al


río será distinta de la velocidad con respecto de un MRUA
observador en reposo a la orilla. Gráfico posición - tiempo Gráfico velocidad - tiempo

Velocidad (m/s)
Posición (m)
• Lanzar una pelota dentro de un autobús: la veloci-
dad de la pelota con respecto al bus será distinta de
la velocidad con respecto a un observador en reposo Tiempo (s) Tiempo (s)

afuera del bus. Gráfico aceleración - tiempo

7. Considerando como sentido positivo hacia la derecha,

Aceleración (m/s2)
la velocidad de Romina con respecto a Sebastián es de
67Km/h. Tiempo (s)

Ficha de refuerzo (P. 52) Ficha de ampliación (P. 53)


1. R Es una magnitud escalar. 1. Las respuestas de sus estudiantes serán variadas. Un
V Es una magnitud vectorial. ejemplo que pueden dar es el siguiente: Si un corredor
V Corresponde al desplazamiento por unidad de tiempo. da una vuelta completa a una pista de atletismo (400 m) 67
R Corresponde a la distancia por unidad de tiempo. en 100 s, su rapidez habrá sido de 4 m/s, sin embargo,
2. a. Jorge recorre una distancia de 60 m. su velocidad habrá sido 0, pues no hubo desplazamien-
b. El módulo de su desplazamiento es de 60 m. to, debido a que volvió al punto de partida.
c. La rapidez de Jorge es de 0,3 m/s y el módulo de su 2. Los resultados obtenidos por sus estudiantes serán dife-
velocidad es de 0,3 m/s. rentes para cada caso.
3. Los gráficos 1 y 2 describen el movimiento del automó- a. Para determinar el módulo del desplazamiento, debe-
vil; el gráfico 3 corresponde a un movimiento con acele- rán medir la longitud de la recta entre el punto de par-
ración constante, que no es el caso del automóvil. tida y el punto final del recorrido de su compañero.
4. a. En el tramo BC la lancha se mueve con velocidad b. La distancia recorrida será la longitud de la línea
constante. de tiza.
b. En el tramo AB la lancha acelera y en el tramo CD c. La velocidad y la rapidez la deberán determinar con
desacelera. las siguientes expresiones:
c. La distancia que recorrió la lancha en el tramo AD Para la rapidez: v = d/t
se puede determinar calculando el área bajo la Para la velocidad: v = Δ​​x→  ​​/  t
recta del gráfico de velocidad en función del tiempo. d. Para que el módulo de la velocidad sea igual al valor
La lancha recorrió 750 m. de la rapidez, el estudiante debería caminar en línea
recta en un mismo sentido.
3. a. Gráfico N° 1: Posición en función del tiempo.
Gráfico posición - tiempo
Guía Didáctica del Docente
Posición (m)

Tiempo (s)

b. El módulo de la velocidad del corredor se puede


determinar calculando la pendiente de la recta del
gráfico de posición en función del tiempo. En este
caso su valor es 10 m/s.

Física • 2.º medio


Solucionario
c. El gráfico en este caso quedaría: 4. a. Ambas bolitas describen una caída libre (MRUA).
Gráfico N° 2: Posición en función del tiempo. b. Las dos bolitas caen producto de la aceleración de
Gráfico posición - tiempo gravedad, por lo que adquieren la misma velocidad
Posición (m) a medida que caen.
c. La masa no influye en la caída de los cuerpos.
Tiempo (s) 5. Solo es correcta II, ya que a medida que sube la piedra el
4. a. Entre los 5 y 6 s el módulo de la velocidad de la efecto de la gravedad provoca que la velocidad disminu-
nadadora es –2 m/s. Entre los 6 y 7 s el módulo de ya hasta que en el punto más alto es igual a cero, donde
la velocidad de la nadadora es 2 m/s. comienza a bajar. En ese punto, la posición es máxima y
la aceleración de gravedad es constante.
b. La nadadora se encuentra inicialmente en reposo
y comienza a desplazarse de vuelta hacia su punto 6. Aceleración de gravedad: aceleración que experimen-
de origen con velocidad de módulo cada vez mayor, tan los cuerpos debido a la fuerza de atracción gravi-
hasta alcanzar los –2 m/s. tacional. En la Tierra, su valor aproximado cerca de la
superficie es de 9,8 m/s2.
5. 100 Gráfico posición - tiempo Gráfico velocidad - tiempo
12
Caída libre: movimiento que describe un cuerpo pro-
Velocidad (m/s)

10
Posición (m)

80
8
60
40
6
4
ducto de la acción exclusiva de la fuerza de gravedad.
20 2
0
0 1 2 3 4 5 6
0
0 1 2 3 4 5 6
Lanzamiento vertical: movimiento rectilíneo uniforme-
Tiempo (s) Tiempo (s)
mente acelerado, en el que se lanza un cuerpo vertical-
Gráfico aceleración - tiempo
mente con cierta velocidad inicial desde cierta altura y
Aceleración (m/s2)

1
no encuentra resistencia alguna en su camino.
0
MRUA: corresponde al movimiento rectilíneo de un
0 1 2 3 4
Tiempo (s)
5 6
cuerpo cuya velocidad cambia a una tasa constante.
Ficha de refuerzo (P. 54) Rapidez: razón entre una distancia y el tiempo que de-
1. a. La altura que alcanza la pelota en cada caso depende mora un móvil en recorrerla.
68
de la velocidad inicial con que la lanza cada una de Velocidad: magnitud vectorial que representa la
las niñas. variación de la posición de un cuerpo por unidad
b. Para que la pelota alcance una altura de 5 m, de tiempo.
__ debe
ser lanzada con una velocidad inicial de 7​​√ 2 ​​m
  /s. Ficha de ampliación (P. 55)
2. ​​v  ​​i  = 50 m/s
→ 1. Gabriel cometió dos errores: no puede usar la ecuación
3. Caída libre que escogió debido a que tendrá dos datos desconocidos,
Gráfico posición - tiempo Gráfico velocidad - tiempo el tiempo y la velocidad inicial; además cometió un error
Tiempo (s)
algebraico al dividir por el tiempo en ambos lados de la
Velocidad (m/s)
Posición (m)

0 1 2 3 4
ecuación. Gerardo sí está en lo correcto, ya que solo ten-
drá una incógnita en la ecuación que eligió, la velocidad
Tiempo (s)

Gráfico aceleración - tiempo


inicial. Además, desarrolla correctamente la ecuación
Tiempo (s) desde un punto de vista matemático.
Aceleración (m/s2)

2. Por ejemplo: Descargar una aplicación para el celular


que mida la velocidad de los cuerpos y, con ella, deter-
minar la velocidad de la piedra a medida que cae. Si de-
Lanzamiento vertical hacia arriba terminan el tiempo y la velocidad del movimiento, se
Gráfico posición - tiempo Gráfico velocidad - tiempo podrá emplear el modelo matemático para calcular la
aceleración que experimenta la piedra.
Velocidad (m/s)
Posición (m)

Tiempo (s) 3. Gráfico posición - tiempo Gráfico velocidad - tiempo

0 1 2 3 4 5 6 7
Velocidad (m/s)
Guía Didáctica del Docente

Posición (m)

Tiempo (s) Tiempo (s)
0 1 2 3 4 5 6 7
Gráfico aceleración - tiempo
Tiempo (s) Tiempo (s)
Aceleración (m/s2)

Gráfico aceleración tiempo


Tiempo (s)
Aceleración (m/s2)

Unidad 1 • Movimiento
ad

unid
1
4. La afirmación III es la única correcta, ya que la masa no b. El análisis dependerá de las características de los mo-
influye en la caída de los cuerpos; además, al despreciar vimientos de cada integrante del grupo. Sus estudian-
el roce, ambos adquieren la misma rapidez, pues caen tes deben comparar los movimientos realizándose las
producto de la aceleración de gravedad. siguientes preguntas: ¿quién logró la mayor velocidad

5. a. ​​v  ​​i =20 m/s en cada tramo?, ¿quién obtuvo mayor velocidad media
b. ymáx = 20 m/s total?, ¿quién experimentó mayor aceleración?, ¿cuánto
6. Respuestas variadas. Se espera que los organizadores tardaron en lograr la velocidad máxima? Entre otras.
consideren, por ejemplo, los siguientes términos: caída c. Las respuestas de sus estudiantes dependerán de cada
libre, lanzamiento vertical, MRUA, aceleración de grave- grupo y del desempeño de cada integrante.
dad, representaciones gráficas, uso responsable de las d. Las respuestas de sus estudiantes serán variadas. Algu-
TIC, obtención de evidencias, aplicación de modelos, nas opciones son las siguientes: en casos de caída libre,
entre otros. movimiento de un péndulo, entre otros.
Desafío complejo (P. 56) Evaluación de la unidad 1 (P. 58 a 61)
a. La posición inicial y final del chanchito de tierra se 1. D
puede indicar dando el par ordenado según el sistema 2. A
cartesiano. 3. D
b. Las respuestas de sus estudiantes dependerán de los re- 4. D
sultados obtenidos. Deben indicar la unidad de medida 5. D
asociada al sistema de coordenadas cartesiano; puede 6. B
ser en centímetros o metros.
7. E
c. Algunas respuestas de sus estudiantes pueden ser: sis-
8. B
temas referenciales bidimensional o tridimensional.
9. C
Además, podrían conocer el sistema de coordenadas
polares. 10. E
d. Las respuestas dependerán de cada estudiante. 11. B 69

e. Para determinar el módulo de la velocidad, basta con 12. C


conocer la distancia de la línea recta que une el punto 13. C
inicial con el final, y el tiempo transcurrido. Para deter- 14. B
minar la rapidez, se puede seguir la trayectoria del chan- 15. B
chito de tierra con un plumón para medir el recorrido y 16. A
el tiempo transcurrido. 17. C
f. No sería la misma, ya que los dos chanchitos se mueven, 18. C
por lo tanto, la velocidad será diferente en este nuevo 19. E
sistema de referencia.
20. D
g. Las respuestas de sus estudiantes dependerán de sus
intereses.
h. Esta técnica puede utilizarse para describir una infini-
dad de movimientos, por lo tanto, las respuestas de sus
estudiantes serán variadas.
Desafío complejo (P. 57)
a. La tabla debe realizarse con los datos obtenidos a partir
de la experiencia realizada por cada grupo, por lo tanto
serán variadas. Se deben incluir los datos solicitados con
sus respectivas unidades de medida.
Guía Didáctica del Docente

Física • 2.º medio


MOVIMIENTO
¿Por qué es importante
a li za r la s ca racterística s
an
de los movimientos?

¡Qué miedo me da
saltar! ¿Imaginas
la velocidad que se
puede alcanzar?

No te preocupes.
Piensa que yo que
tengo más masa
caeré más rápido.

Pero, Miguel, ¡ambos


caerán con la misma
aceleración!

16
Actividad individual

1 ¿Qué opinas de la conversación entre Romina y Miguel?


¿Quién tiene la razón? Fundamenta.

2 ¿Conoces alguna(s) característica(s) de la caída libre? Men-


ciónala(s).

3 Propón un método para validar alguna de las afirmaciones


propuestas por Romina o Miguel.

17
Evaluación inicial
¿Quién está en lo correcto: Romina o Miguel?

Para comprobar la influencia de la masa en la caída de los cuerpos, Romina y


Miguel desarrollaron las siguientes experiencias.

¿Tienen la misma masa la hoja


extendida y la hoja hecha bolita?
¿Qué crees que ocurre cuando se dejan
caer simultáneamente las dos hojas de
papel idénticas (mismas características)
extendidas?, ¿cómo es la caída?

¿Qué crees que ocurre cuando se dejan


caer simultáneamente? Describe cómo
crees que es la caída de ambos.

18 Unidad 1 • Movimiento
¿Tienen la misma masa una bolita
de acero y otra hecha de papel?

¿Qué crees que ocurre cuando se


dejan caer simultáneamente ambas
bolitas de papel?

¿Qué pasará con la caída de ambas bolitas si se


dejan caer desde la misma altura?, ¿llegarán al
suelo al mismo tiempo? Fundamenta.

Física • 2.° Medio 19


¿Qué aprenderás y descubrirás en la unidad?
Te presentamos las principales metas, estrategias y propósitos de la unidad. Luego, propón las metas
que te gustaría lograr, las estrategias que emplearías para alcanzarlas y el propósito de estas.

Metas ¿Cómo alcanzarlas? ¿Para qué alcanzarlas?

Reconocer la impor tancia ✓ Realizando actividades Para disfrutar del crecimiento


de establecer un sistema de prácticas. intelectual que genera el cono-
referencia y un sistema de ✓ Mostrando interés. cimiento científico y valorar su
coordenadas para describir el ✓ Aplicando modelos. importancia para el desarrollo
movimiento de un objeto. ✓ Desarrollando estrategias. tecnológico y social.
✓ Utilizando las TIC.

Analizar de manera cualita- ✓ Realizando actividades Para buscar soluciones a proble-


tiva, cuantitativa, algebraica prácticas. máticas científicas manifestando
y gráficamente, variadas ✓ Trabajando en equipo. interés por trabajar de forma
situaciones cotidianas de mo- ✓ Organizando datos. responsable y colaborativa.
vimiento rectilíneo: uniforme y ✓ Interpretando gráficos.
uniforme acelerado. ✓ Aplicando modelos.
✓ Elaborando maquetas.

Analizar de manera cualita- ✓ Realizando actividades Para realizar explicaciones


tiva, cuantitativa, algebraica prácticas. científicas, empleando de for-
y gráfica la caída libre y los ✓ Utilizando herramientas ma efectiva las tecnologías de
lanzamientos verticales. tecnológicas. la comunicación.
✓ Analizando experimentos
clásicos.
✓ Procesando evidencias.
✓ Elaborando gráficos.
✓ Construyendo modelos.

Propón tus propias metas para Establece las estrategias Identifica el propósito
esta unidad. que usarás para el logro de de tus metas.
tus metas.






20 Unidad 1 • Movimiento
¿Cómo te gustaría protagonizar tu propio aprendizaje?

Para comprender las características de los


movimientos, en esta unidad aprenderás
conceptos como posición, velocidad, rapidez
y aceleración. Pero ¿qué te gustaría aprender
sobre los movimientos?, ¿por qué?

En esta unidad desarrollarás estrategias que


te permitirán aplicar modelos, interpretar grá-
ficos y sintetizar contenidos. ¿Qué estrategias
te gustaría desarrollar? Propón una alternativa
diferente. ¿Qué dificultades crees que puedes
enfrentar en el estudio del movimiento?

¿Qué actitudes deberías manifestar, a lo largo


de la unidad, para que puedas valorar el cono-
cimiento científico y su importancia en el desa-
rrollo de nuevas tecnologías? ¿De qué manera
fomentas tu creatividad e interés para aprender?

¿Cómo lograr mis metas?


Para alcanzar tus metas, te proponemos como estrategia, escribir la letra de una canción que
se relacione con el cumplimiento de tus propósitos en la unidad. Para ello, define tres pasos
mediante los cuales desarrolles dicha estrategia. Considera, además, utilizar la melodía de una
de tus canciones favoritas y solicitar la ayuda de tu profesor de Artes Musicales.

ESTRATEGIA
Escribir la letra de una canción

Paso 1 Paso 2 Paso 3

Física • 2.° Medio 21


Lección
Descripción del movimiento
Propósito de la lección

¿Podemos asegurar que somos nosotros los En esta lección, aprenderás sobre la rela-
que nos movemos alrededor del Sol y no que tividad del movimiento y cómo siempre es
que es el Sol el que se mueve alrededor de necesario establecer un sistema de referencia
la Tierra? Determinar si algo se mueve o no para describir lo que observamos. Para ello,
parece sencillo, sin embargo, para hacerlo es realizarás actividades que te ayudarán a
necesario comprender ciertos conceptos que comprender mejor los fenómenos del entorno
nos permitan describir de forma apropiada y, natural, de modo que valores su importancia
con ello, responder preguntas como la plan- en el desarrollo de nuevas tecnologías y su
teada anteriormente. impacto en la sociedad.

Ciencia, tecnología y sociedad

El Sistema de Posicionamiento Global


La tecnología GPS (de sus siglas en inglés Global Positioning System) permite co-
nocer, en tiempo real y con gran precisión la posición de un objeto, en cualquier
lugar de la Tierra. Este sistema funciona con 24 satélites que orbitan la Tierra
a una distancia superior a los veinte mil kilómetros, siguiendo trayectorias
sincronizadas gracias a las cuales cubren toda la superficie terrestre. La
gran ventaja es que las señales GPS son de uso público, por lo que no
existen licencias o restricciones para su implementación. ¿Has utiliza-
do el navegador GPS de los celulares? De ser así, ¿para que lo has
empleado? ¿Qué ventajas consideras que tiene para la sociedad
el desarrollo de esta tecnología?

Fuente: www.prometric.com.mx/tecnologiagps.htm
ión
zac
b eti ífica
a t
Alf cien

Alertas de tsunamis más rápidos y


¿Te parece interesante que se
efectivos gracias al GPS puedan utilizar los avances
Según un estudio del Laboratorio Sismológico de la Universidad tecnológicos para optimizar las
alertas de tsunami?, ¿por qué?
de Berkeley, California, EE.UU., las mediciones en tiempo real
del Sistema de Posicionamiento Global (GPS) pueden ser utili-
zadas para mostrar cómo los terremotos importantes desplazan
el fondo oceánico. Estos datos permitirían reducir los tiempos de
alerta de tsunamis de los casi 20 minutos, que demora actual-
mente, a apenas un par de minutos y así potencialmente reducir
los daños a las comunidades costeras.
“Esto no es un despliegue de nuevos instrumentos, solo un
cambio en el pensamiento y el uso de estos instrumentos”, dijo
Diego Melgar, investigador de la Universidad de Berkeley.
Fuente: http://www.uchile.cl/noticias/119299/tecnologia-gps-posibilita-
alertas-de-tsunami-mas-rapidas-y-efectivas

22 Unidad 1 • Movimiento
CIENCIA en

Ciencia al día CHILE


El ingeniero Gonzalo Arroyo construye el
primer péndulo de Foucault CHILENO
Gonzalo Arroyo, ingeniero civil eléctrico de la Universidad Téc-
nica Federico Santa María, fue el encargado de diseñar y cons-
truir un sistema de propulsión que permitiera oscilar el péndulo
de Foucault. Este aparato permite demostrar que el plano de
oscilación de un péndulo es independiente del movimiento
de la plataforma que lo sujeta. De esta manera, al observar el
vaivén del péndulo, luego de varias oscilaciones, se notará un
pequeño cambio de la dirección del plano de oscilación pro-
ducto del movimiento de rotación de la Tierra.
Para lograr el funcionamiento óptimo del péndulo, el ingeniero
desarrolló un sistema de propulsión electrónica que permitiera
la oscilación con amplitud constante, a pesar del roce con el
aire. Este péndulo, construido íntegramente en Chile, es uno
de los pocos que existe en América del Sur y actualmente se
encuentra en las dependencias del colegio San Francisco Ja-
vier, en Puerto Montt. Fotografía de un péndulo de Foucault.

Según el ingeniero, este péndulo “Es un excelente recurso para comprender que
la Tierra es un elemento dinámico en el universo, lo cual resulta increíble, porque
según nuestros sentidos la Tierra nos parece algo estática”.
¿Qué te parece la afirmación del ingeniero? ¿Consideras que, desde nuestra per-
cepción, la Tierra permanece estática?, ¿por qué?

Fuente: http://www.noticias.usm.cl/2012/04/20/sansano-construye-el-primer-
pendulo-de-foucault-completamente-chileno/

Inicio de la misión
Como pudiste leer en estas páginas, el estudio del movi-
miento ha favorecido el desarrollo de diversos avances tec-
nológicos que nos permitan comprender mejor el mundo
que nos rodea. En esta lección, tendrás la misión de grabar
un video que ejemplifique la relatividad del movimiento.
Para ello, deberás solicitar la colaboración de tu profesor de
computación o del encargado de TIC, para que te ayude
a utilizar un software que te permita elaborar un creativo
video.
Para llevar a cabo esta misión, reúnete con dos compañeros
y realicen un plan de trabajo, considerando que al finalizar
esta lección deberán mostrar su video al curso.

Física • 2.° Medio 23


Lección 1

Tema 1 ¿Cuándo nos movemos?


Constantemente, observamos situaciones en las que podemos afirmar que los obje-
¿Qué conceptos utilizas
tos están en movimiento, como en el vuelo de un avión o el tránsito de un automó- frecuentemente para describir
vil por la calle, pero ¿cuándo decimos que un cuerpo se encuentra en movimiento? un movimiento?
En este tema, aprenderás a utilizar el sistema de referencia y el sistema de coorde-
nadas para describir y analizar el estado de movimiento de los cuerpos. Además,
podrás desarrollar diversas actividades que te permitirán aplicar los conceptos que
describen el movimiento. Ello, en el entendido de que estos conocimientos científi-
cos los emplearás en la resolución de problemas cotidianos.

Indaguemos a partir de nuestros aprendizajes previos Actividad grupal

Objetivo: Definir el concepto de Reúnanse en parejas y consigan una hoja de cuaderno, un lápiz, una regla y tres
movimiento. botones de diferente color y realicen el siguiente procedimiento:
Habilidad: Observar y describir las 1. Dibujen una cuadrícula de 6 x 6 en la hoja. Identifiquen las columnas con
características de un suceso.
números del 1 al 6 y las filas con letras de la A a la F.
Actitud: Mostrar curiosidad por conocer
nuevos aprendizajes. 2. Ubiquen los tres botones en diferentes lugares de la cuadrícula, procurando
que queden ubicados al centro de cada cuadrado escogido.
Tiempo: 15 minutos.
3. Uno de los miembros de la pareja cierre los ojos, mientras el otro mueve al-
guno o todos los botones.
4. Al abrir los ojos, trata de determinar qué botones se movieron.
5. Realicen nuevamente la experiencia, pero cambiando los roles.
Luego, respondan las siguientes preguntas en relación con la actividad
realizada.
a. ¿En qué se fijaron para determinar los botones que se movieron?

b. ¿Cuál es la condición necesaria para afirmar que un objeto se ha movido?


¿Cómo definirían el movimiento? Expliquen.

Materiales de la actividad.

c. ¿Qué importancia consideran que tiene la observación para la adquisición


de nuevos aprendizajes? Fundamenten.

Seguramente notaste en la actividad anterior que, para movernos, necesariamente


debemos cambiar nuestra posición respecto de un lugar de referencia. Estos concep-
tos están muy relacionados, pues podemos definir el movimiento como el cambio en
la posición de un cuerpo. Así, si un cuerpo no cambia la posición en la que se encuen-
tra, entonces se encuentra quieto; por el contrario, si un cuerpo se mueve, entonces
su posición cambiará. ¿Qué necesitamos, entonces, para describir un movimiento?

24 Unidad 1 • Movimiento
La posición ​(​x→   ​)​de un cuerpo nos indica su localización respecto de un sistema
de referencia utilizando un sistema de coordenadas.
Por ejemplo, en el siguiente esquema podemos señalar la posición de dos objetos
empleando un sistema de coordenadas en una dimensión (línea recta horizontal).

Sistema de referencia

Sistema de coordenadas
X (cm)
–30 –15 0 15 30

  –30 cm y la posición de la botella es ​​x    30 cm.


Respecto del origen del sistema de coordenadas x = 0, la posición del reloj de arena
es ​​x→ ​​= →
​​=
La posición corresponde a una magnitud vectorial, ya que nos indica la magni- Importante
tud, dirección y sentido a la que se encuentra un objeto respecto a un sistema
Un vector es una herramienta
de referencia. Por ejemplo, en la situación anterior, ambos objetos se encuentran matemática que permite
situados a 30 cm del sistema referencia. Sin embargo, el sentido de cada uno es representar diferentes magnitudes
distinto, dado que el reloj está a la izquierda (señalado con el signo menos) y la físicas. Para profundizar más
botella se encuentra a la derecha (señalado con el signo más). acerca de los vectores, revisa
De esta manera, para describir un movimiento, es necesario establecer un sistema el anexo que se presenta en la
de referencia, que puede ser un lugar o un objeto desde el cual se describe el mo- página 230 del texto.
vimiento, y un sistema de coordenadas, que es un conjunto numérico.
Actividad grupal
Determinemos la posición de las personas
Júntense en parejas y observen la siguiente situación:

A C
B

x (m)

La ilustración no se encuentra a escala.

1. Determinen la posición de las personas A, B y C si el sistema de referencia se


ubica en el origen del sistema de coordenadas.
2. Determinen la posición de las personas A y B si el sistema de referencia es la
persona C.
3. Comparen las respuestas obtenidas en los puntos 1 y 2. Establezcan diferen-
cias y similitudes. ¿Qué pasó al cambiar el sistema de referencia?
4. ¿Qué conocimientos utilizaron para resolver esta actividad?

Física • 2.° Medio 25


Lección 1

¿Cuál es la diferencia entre distancia recorrida y desplazamiento?


¿Qué sabes de los conceptos de distancia y desplazamiento?, ¿en qué se diferencian?
Para responder estas preguntas, realiza la siguiente actividad.

Indaguemos a partir de nuestros aprendizajes previos Actividad grupal

Objetivo: Diferenciar los conceptos de Reunidos en parejas, consigan dos lápices de color (uno azul y uno rojo), una hoja
distancia y desplazamiento. de cuaderno, un insecto (como hormiga o chinita), hilo y regla. Luego, realicen
Habilidad: Establecer comparaciones. el siguiente procedimiento:
Actitud: Valorar la importancia del 1. Marquen con el lápiz azul un punto sobre la hoja y rotúlenlo como el lugar
conocimiento. de partida.
Tiempo: 20 minutos. 2. Ubiquen el insecto sobre ese punto y sigan con el lápiz azul el camino que
recorre, durante algunos segundos.
3. Marquen en la hoja el punto final del camino recorrido por el insecto.
4. Con el lápiz rojo y la regla, tracen una flecha que comience en el punto de
partida y termine en el punto de llegada del movimiento del insecto.
5. Analicen las líneas realizadas por cada lápiz de color. Luego, midan cada una
de las líneas trazadas con los lápices de color y registren sus valores en el
cuaderno. Utilicen el hilo para medir el trazo irregular seguido por el insecto.
Finalmente, respondan las siguientes preguntas:
a. ¿Qué diferencias hay entre cada trazo de color? Expliquen.

b. ¿Cuál trazo creen que representa la distancia recorrida y cuál el desplaza-


miento? Fundamenten.

Materiales de la actividad.
c. ¿Qué diferencias evidenciaron con las mediciones realizadas?

d. ¿Cómo definirían el concepto de distancia y el de desplazamiento?

Advertencia: Manipulen con


precaución el insecto y, cuando e. ¿Qué importancia creen que tiene para el estudio del movimiento diferen-
terminen la actividad, devuélvan- ciar estos conceptos?
lo a su lugar de origen.

Es habitual pensar que la distancia recorrida y el desplazamiento son términos


equivalentes, sin embargo, no lo son. Para entender la diferencia, analizaremos,
en la siguiente página, los resultados obtenidos en la actividad anterior.

26 Unidad 1 • Movimiento
Supongamos que el movimiento de la hormiga entre la posición inicial y la final
es el que se representa a continuación:

x (cm)

0 1 2 3 4 5 6
Posición inicial Posición final

La ilustración no se encuentra a escala.


¿Cuánto caminos posibles hay
entre dos puntos?
El camino realizado por la hormiga entre la posición inicial y la posición final (línea
azul) se denomina trayectoria. La longitud de la trayectoria seguida por la hormi-
ga corresponde a la distancia recorrida (d).
Por otro lado, el desplazamiento (Δ ​​x→   ​​) es la variación entre la posición final y la
inicial. Es decir, en la imagen, el desplazamiento se representa por la flecha roja
que, además, indica que el movimiento comenzó en la posición inicial y terminó
en la posición final. Para determinar el desplazamiento, se utiliza la siguiente ex-
presión matemática:

Δ ​​x    ​​= ​​x→   ​​f – x
​​ →   ​​i

Desplazamiento Posición final Posición inicial

Por ejemplo, si se hace coincidir un sistema coordenado con la dirección del des-
plazamiento de la hormiga, tal como el que se muestra en la imagen, obtendre- Importante
mos que el valor de dicho desplazamiento es Δ ​​x→   ​​= 6 cm – 1 cm = 5 cm.
Una magnitud escalar es aquella
De esta manera, el desplazamiento es una magnitud vectorial, pues tiene mó- que se representa con un valor
dulo, dirección y sentido, a diferencia de la distancia, que solo tiene módulo. Por numérico y su respectiva unidad
esta razón, la distancia corresponde a una magnitud escalar. de medida. Por ejemplo:
• El tiempo: 5 s
Actividad individual
• La temperatura: 25 °C
Determino la distancia recorrida y el desplazamiento
Un estudiante salió de su casa al colegio que se encuentra a 120 metros en línea
recta. Cuando ya había caminado los primeros 30 metros, el estudiante se devol-
vió a buscar unos materiales que se le quedaron en casa, para luego retomar su
camino al colegio.
1. Desde que salió de su casa por primera vez hasta que llegó finalmente al
colegio, ¿cuál fue el desplazamiento y la distancia recorrida por el estudiante?
2. ¿De qué manera has manifestado una actitud que promueva la curiosidad y
el interés por comprender los fenómenos del entorno?

Física • 2.° Medio 27


Lección 1

¿Cuál es la diferencia entre rapidez y velocidad?


Pese a que los conceptos de rápido o lento son subjetivos, es habitual que en
nuestro entorno realicemos una estimación, de forma natural, de la rapidez de
diferentes objetos. Para saber qué parámetros nos permiten comparar la rapidez
de distintos cuerpos, realiza la siguiente actividad.

Indaguemos a partir de nuestros aprendizajes previos Actividad grupal

Objetivo: Reconocer que la rapidez Reúnanse en grupos de tres integrantes y consigan dos rieles de la misma lon-
es una medida de comparación entre gitud, dos bolitas idéntica (material, masa y tamaño) y dos cronómetros (pueden
cuerpos en movimiento. utilizar el del celular). Luego, realicen el siguiente procedimiento:
Habilidad: Analizar los resultados de 1. Sitúen los dos rieles con inclinaciones distintas, tal como se muestra en la
una experiencia.
imagen del costado.
Actitud: Mostrar curiosidad por com-
prender nuevos aprendizajes. 2. Un integrante del grupo deberá soltar simultáneamente ambas bolitas por
cada riel, mientras los otros dos miden, utilizando el cronómetro, el tiempo
Tiempo: 20 minutos. que tarda cada bolita en recorrer el largo del riel.
3. Registren cada valor en sus cuadernos y repitan el procedimiento para validar
los resultados obtenidos.
A continuación, respondan las siguientes preguntas:
a. ¿Qué magnitudes piensan que se relacionan mediante el con-
cepto de rapidez?
b. Si la distancia recorrida por ambas bolitas es la misma, ¿qué mag-
nitud les permite comparar cuál bolita fue más (o menos) rápida?
c. ¿Cómo definirían el concepto de rapidez?

Montaje de la actividad. d. ¿Qué preguntas o inquietudes les surgieron al observar el comportamiento


de las bolitas?

Importante
Una descripción general de qué tan deprisa se mueve un cuerpo es la rapidez
En el Sistema Internacional de uni-
media (​​vm​  ​​​). Esta corresponde a la distancia total recorrida dividida por el tiempo
dades (SI), la unidad de medida de la
total transcurrido y puede ser determinada empleando la siguiente expresión: rapidez es m/s. ¿Qué otras unidades
de medida de la rapidez conoces?
d Distancia recorrida
vm =
Δt Actividad individual
Rapidez media Aplico la ecuación de rapidez
Tiempo empleado
Completa la siguiente tabla con los va-
Ahora, si imaginamos, por ejemplo, que un bus tiene una rapidez media de lores que corresponda.
80 km/h, esto no significa necesariamente que el chofer mantiene dicha rapidez Distancia Tiempo Rapidez
en todo momento, ya que sabemos que un bus realiza continuas detenciones para recorrida (m) empleado (s) media (m/s)
permitir que los pasajeros suban o bajen, o bien, en varias oportunidades se mueve
5 1
más lento o más rápido. Lo anterior hace necesario el uso de otro concepto, el de
rapidez instantánea (v). Esta se refiere a la rapidez que posee un cuerpo en un 10 5
instante determinado (un intervalo de tiempo muy pequeño). 3 4

28 Unidad 1 • Movimiento
Supongamos que un automóvil viaja por una carretera recta a 120 km/h: ¿qué po-
demos decir del vehículo?, ¿se mueve rápido o veloz? Es común que, en nuestro Importante
lenguaje cotidiano, utilicemos los conceptos de rapidez y velocidad sin distinción Al igual que con el concepto de
alguna, sin embargo, estos no representan lo mismo. rapidez, se puede distinguir la

(​​v→ ​​)  (en esta última, el intervalo de


velocidad media de la instantánea
Como vimos anteriormente, la rapidez media nos entrega la medida de qué tan de-
prisa un cuerpo recorre una determinada distancia. En cambio, la velocidad media​​
→ tiempo es muy pequeño).
(​v   ​m)​nos indica el desplazamiento que tuvo un cuerpo dividido por el tiempo total
transcurrido. De esta manera, velocidad media se puede expresar como: En el Sistema Internacional (SI), la

​​​→  ​​  m  ​​​ = ___




velocidad se mide en m/s.
v ​​ Δ​x ​​ ​ 
Δt
Como la velocidad media depende del desplazamiento, es una magnitud vectorial,
por lo que tiene módulo, dirección y sentido, a diferencia de la rapidez, que es una
magnitud escalar.

Desarrollo de estrategias
Aprendiendo a aplicar modelos para determinar la rapidez y la velocidad media.

Situación problema
Casa de Lorena
Lorena sale de su casa para pasear a su pe-
rro, pero cuando se encontraba a 30 m de la
plaza su perro se escapa y se devuelve hasta
la posición 60 m, desde donde reanudan su
camino hasta la plaza. Si en su recorrido total
demoraron 55 s, determina la rapidez y la ve-
locidad media de su perro.

PASO 1 Identifico las incógnitas


Las incógnitas son ​vm​  ​​ y ​​​v→   ​​ m​​​ La ilustración no se encuentra a escala.

PASO 2 Registro los datos



Posición inicial ​​x  ​​ i​​ = 110 m; posición final ​​x→   ​​f = 0 m y tiempo empleado ∆t = 55 s

PASO 3 Utilizo los modelos

Para determinar la rapidez media, utilizamos Para determinar la velocidad media, utiliza-

​ 
la siguiente expresión: mos la siguiente expresión: ​​  ​   ​​​​​​ 
d  ​ 
​vm​  ​​ = ​ ___ → Δ​x→  ​ 
v ​​  m  ​​​= ​ ___
Δt Δt
80 m + 30 m ​​
+ 60 m  
​​v​  m​​​ = _________________
​​    ​​  170 m ​​ 
= ______  
55 s 55 s →​    ​​​ = ___________
​​​ v ​ 0 m – 110
​​    m 
 ​​ –110 m
= ​​ _______ ​​ 

m 55 s 55 s

​​→  ​​m = –2 m/s​​


​vm​  ​​ = 3 m/s​
v

PASO 4 Escribo la respuesta


La rapidez media del perro es 3 m/s​​mientras que su velocidad media es de –2 m/s.
PASO 5 Aplico lo aprendido
Determina la rapidez y la velocidad media del perro cuando camina directo (sin devolverse) desde su
casa hasta la plaza y demora 50 segundos.

Física • 2.° Medio 29


Lección 1

Tema 2 Analizando la relatividad del movimiento ¿Qué crees que es la relativi-


dad?, ¿cómo lo explicarías?
Seguramente en más de alguna oportunidad has escuchado o dicho que un Menciona un ejemplo.
suceso es relativo, pero ¿cuándo evidenciamos que las cosas son relativas?
En este tema aprenderás de qué características depende la relatividad del movi-
miento, de modo que puedas describirlo de forma cualitativa y cuantitativa. Por
ello, analizarás diversas situaciones que te permitirán valorar la importancia de
desarrollar habilidades y conocimientos científicos para comprender el mundo
que nos rodea.
Para comprender la relatividad del movimiento, responde las preguntas, analizando
la situación que se presenta a continuación. Considera que el bus se mueve por la
calle hacia la izquierda, respecto del suelo, tal como muestra la siguiente imagen.

¿Cómo verá el bus este


observador: en reposo o en
movimiento?

Seguramente notaste que, para una persona que se encuentra sentada dentro
de vehículo, el chofer se encuentra en reposo; mientras que para alguien que
se encuentra en la calle, el chofer se encuentra en movimiento. Esto se debe a
que los movimientos dependen del marco de referencia que escogemos para
describirlos, razón por la cual se dice que el movimiento es relativo. Es impor-
tante destacar que ningún movimiento es absoluto, ya que siempre depende
del marco de referencia. Así, cada vez que queremos describir un movimiento,
debemos indicar una referencia, por ejemplo, que el chofer del bus se encuen-
tra en movimiento respecto de la calle.

30 Unidad 1 • Movimiento
¿Cómo verá el chofer al
transeúnte: en reposo o en
movimiento?

¿Cómo verá al conductor


una persona sentada en
¿Qué se puede concluir de
el interior del vehículo: en
las preguntas planteadas: el
reposo o en movimiento?
chofer está en reposo o en
movimiento?, ¿por qué?

¿y cómo verá al transeúnte?

Actividad grupal
Establezcamos un sistema de referencia
Reúnanse en parejas y, a partir de la imagen del bus, establezcan:
1. Un sistema de referencia dentro del vehículo que describa al chofer en reposo.
2. Un sistema de referencia fuera del vehículo que describa al chofer en reposo.
3. Un sistema de referencia fuera del vehículo que describa al chofer en movimiento.
4. ¿Qué opinan sobre la relatividad del movimiento?, ¿será correcto afirmar que todo
movimiento es relativo? Fundamenten.

Física • 2.° Medio 31


Lección 1

¿Las trayectorias pueden ser relativas?


¿En qué situaciones has observa-
Como todos los movimientos dependen del sistema de referencia que los des- do trayectorias relativas?
cribe, la trayectoria que observamos de los cuerpos también depende de esta
referencia, por lo tanto la trayectoria es relativa. Para indagar más acerca de la
forma en que la trayectoria de un cuerpo depende de quien la observe, realiza
la siguiente actividad.

Indaguemos a partir de nuestros aprendizajes previos Actividad grupal

Objetivo: Observar y describir la tra- Reúnanse en grupos de cuatro integrantes, consigan una pelota y realicen el
yectoria de un cuerpo desde diferen-
tes sistemas de referencia.
siguiente procedimiento:

Habilidad: Describir las características 1. Para realizar esta experiencia, dos integrantes del grupo serán los lanzadores,
de un suceso. mientras que los otros dos serán los observadores.
Actitud: Explorar fenómenos desafian- 2. Los lanzadores deberán ubicarse de frente a una distancia aproximada de
tes con los sentidos. un metro. Uno de los observadores deberá ubicarse detrás de uno de los
Tiempo: 20 minutos. lanzadores, mientras que el segundo observador lo hace de frente, tal como
muestra la ilustración.
3. Ahora, los lanzadores deben arrojarse la pelota en forma parabólica, mientras
que cada observador registra en su cuaderno la trayectoria que sigue la pelota.
4. Finalmente, intercambien la posición de los observadores y repitan el proce-
dimiento anterior. Luego, respondan las siguientes preguntas:
a. ¿Cómo fue la trayectoria de la pelota observada desde la posición 1?, ¿y
desde la posición 2?

b. A partir de lo que entienden por movimiento relativo, ¿cómo explicarían


lo observado?

2
c. ¿Qué inquietudes surgieron a partir de sus observaciones?, ¿qué importan-
cia tiene la observación en la exploración del entorno?

Representación de la actividad.

Se dice que un movimiento es relativo y en particular su trayectoria cuando un


fenómeno puede ser descrito desde diferentes sistemas de referencia.
Uno de los primeros investigadores en analizar y explicar la relatividad del movi-
miento fue Galileo Galilei. Para ello, Galileo analizó una situación similar a la que
se representa en el siguiente ejemplo: imagina que estás dentro de un vehículo
que se encuentra detenido y empieza a llover; ¿cómo verás las gotas de lluvia
sobre el vidrio de la ventana? Luego, imagina que el vehículo se pone en movi-
miento: ¿verás de la misma manera las gotas de agua en la ventana?

32 Unidad 1 • Movimiento
Seguramente consideraste que, en el primer caso, las gotas siguen en una tra-
yectoria vertical; sin embargo, cuando el vehículo se encuentra en movimiento,
las gotas de lluvia se inclinan, tal como se muestra en las siguientes imágenes:

¿Cuál es el sentido del


movimiento del vehículo
para que las gotas de agua
describan esa trayectoria?

Trayectoria vertical de las gotas de Trayectoria inclinada de las gotas de


agua cuando el vehículo se encuen- agua cuando el vehículo se encuen-
tra en reposo. tra en movimiento.

Para tratar de explicar la relatividad del movimiento, Galileo introdujo una serie
de ecuaciones que le permitieron describir el movimiento de un cuerpo desde un
sistema de referencia que se mueve con velocidad constante respecto de otro que
está en reposo respecto del suelo. A este cambio de coordenadas se lo denomina
transformación de Galileo y se describe a continuación.
Si un sistema A’ (representado por el ciclista) se mueve respecto de otro A que se
encuentra en reposo respecto del suelo (representado por la joven), entonces, las
coordenadas del sistema A’ respecto de las de A son:
y y’
vx
x’ = x + vx · t
A’
y’ = y
z’ = z A

x x’

z z’

Esto, siempre y cuando el tiempo “cero” de ambos sistemas coincida.


En las expresiones anteriores, vx es la velocidad del sistema A’ respecto de A,
y t es el tiempo.

Desarrollo de la misión
Considerando que las trayectorias también son relativas,
elabora junto a tu equipo de trabajo el ejemplo que quie-
ren representar en su video, utilizando el software esco-
gido. Escriban las fortalezas y debilidades que perciban
durante el desarrollo de esta misión. Escriban las forta-
lezas y debilidades que enfrenten durante el desarrollo
de la misión.

Física • 2.° Medio 33


Lección 1

Determinando la relatividad de las velocidades


Tal como la posición de un cuerpo depende del observador, la velocidad y, por
ende, la rapidez de este también dependen del sistema de referencia desde el
que se describa. Para determinar la velocidad relativa, realiza la siguiente actividad.

Indaguemos a partir de nuestros aprendizajes previos Actividad grupal

Objetivo: Establecer la velocidad Reúnanse en parejas. Luego, lean y analicen la siguiente situación:
media de los cuerpos desde dife-
rentes sistemas de referencia. Sandra desciende por una escalera fija con una velocidad media de 1 m/s, mien-
tras que Carlos asciende por otra escalera con una velocidad de igual magnitud,
Habilidad: Comprender la relativi-
dad de la velocidad.
pero en sentido contrario. En la parte inferior de las escaleras, Lucía se encuentra
detenida, observando a Sandra y a Carlos, tal como se muestra en la imagen del
Actitud: Interés por aprender nue- costado.
vos conocimientos.
Tiempo: 20 minutos.
A partir de esta situación, respondan:
a. ¿Qué conceptos estudiados hasta el momento están involucrados en la situa-
ción descrita?

b. ¿Cuál es la velocidad media de Sandra y Carlos respecto de Lucía?

c. Desde el punto de vista de Sandra, ¿con qué velocidad ve pasar a Carlos?


¿Cómo lo determinaron?

d. ¿Qué pregunta, relacionada con la situación descrita, le plantearías a una com-


pañera o un compañero de grupo?

Representación de la situación.

Tal como determinaste en la actividad anterior, la velocidad depende del sistema


de referencia que lo describe. Para trabajar la relatividad de la velocidad se puede
establecer una relación matemática, de tal manera que la velocidad media de
un sistema A, medida por un observador B que se mueve con cierta velocidad,
queda determinada por la siguiente expresión:

Velocidad de A respecto de B v ​​→   ​​ A ​​ - v


​​  ​​ ⃗   A/B ​​ = v ​​→   ​​ B​​ Velocidad de B respecto del suelo

Velocidad de A respecto del suelo ¿Cuál es la velocidad media de


Sandra respecto de Carlos?

Así, en la situación planteada en la actividad, la velocidad media de Sandra res-


pecto de Lucía, si se considera positivo el sentido hacia abajo de la escalera, es:
→ → →
​​  v   ​​  Sandra/Lucía​​ = ​​  v   ​​  Sandra​​ - ​​  v   ​​  Lucía​​ = 2 m/s – 0 = 2 m/s

34 Unidad 1 • Movimiento
Mujeres en la historia de la ciencia
Mileva Maric (1875-1948), una muy destacada estudiante y posterior física teórica
y matemática, contrajo matrimonio en 1903 con Albert Einstein, el cual se disolvió
legalmente en 1919. Existen bastantes pruebas y testimonios de que Mileva habría
sido coautora de los varios de trabajos de Einstein, entre ellos los relacionados
con el efecto fotoeléctrico, el movimiento browniano y el movimiento relativo.
Se estima que “la insuficiente valoración atribuida al trabajo de Mileva Maric se
explica en el contexto general de los valores de la época en los que se situaba
a la mujer al margen de la ciencia y de la investigación, y cuando se producía
alguna contribución excepcional, esta tendía más a reconocerse en el ámbito
privado que en el oficial”.
Fuente: González, M. (2006). Mileva Einstein-Maric: la madre “olvidada” de la teoría de la
relatividad. Clepsydra, 5. Disponible en: http://publica.webs.ull.es/

¿Qué mujeres científicas conoces?

¿Consideras que el rol de la mujer ha sido menoscabado en la historia de la hu-


manidad?, ¿por qué?
n
ció
e tiza ca
¿De qué manera se puede valorar el aporte de la mujer en la ciencia? ab tífi
Alf cien

Cierre de la misión
Al inicio de esta lección, te dimos la misión de elaborar un
video para ejemplificar la relatividad del movimiento y pre- ¿Esta misión se incluirá en el proyecto del texto?
sentarlo a tu curso. ¿Cómo evaluarías el trabajo realizado? Fundamenta en la página 12.
Fundamenta. ¿Qué ventajas tiene trabajar en equipo?

Reflexiono sobre lo que aprendí


Lee y comenta las siguientes preguntas con tus compañeros y compañeras para
saber si alcanzaste el propósito de la lección.
Contenido Habilidades/Estrategias Actitudes
¿Qué conceptos nuevos aprendiste? A lo largo de esta lección observas- ¿Qué opiniones elaboraste a partir
te, analizaste y evaluaste diversas de los fenómenos que observaste
¿Qué te pareció más relevante del estudio del movi-
situaciones. ¿Qué otra habilidad te en la lección?
miento?, ¿por qué?
gustaría desarrollar?
¿Estás preparado para emplear
¿Consideras que los aprendizajes adquiridos te
¿Qué nuevas estrategias desarrollas- tus conocimientos científicos para
ayudan a comprender mejor los fenómenos que te
te en esta lección?, ¿cuál te pareció resolver problemas cotidianas?, ¿por
rodean?, ¿por qué?
más interesante? qué?
¿Sobré qué temas relacionados con el movimiento te
gustaría aprender? Fundamenta.

Física • 2.° Medio 35


Integro lo que aprendí
Evaluación de proceso

Representa
1 Para combatir los incendios forestales que afectaron
las zonas Centro y Sur de Chile en 2017, se contrató al
Supertanker (un avión de combate de incendios) que
deja caer aproximadamente 72 000 litros de agua en
cada descarga.
Si el avión se mueve con velocidad contante, representa
en la ilustración la trayectoria del agua vista por un ob-
servador en tierra y por otro que está dentro del avión.

Aplica
2 Para estudiar la relatividad del movimiento, Felipe toma una fotografía en la
cual representa la rapidez media, medida respecto del suelo, de un automó-
vil, un ciclista y un corredor, tal como se muestra en la siguiente imagen.

A partir de la información, responde:


a. ¿Respecto de qué observador el corredor se mueve con mayor rapidez?
Justifica.
b. ¿Cuál es la velocidad media del ciclista respecto al corredor?
c. ¿Qué distancia habrá recorrido el ciclista luego de 5 minutos de carrera si
su recorrido fue en línea recta?, ¿cómo lo determinaste?
Analiza
3 Daniela se encuentra en reposo en un sistema de referencia P y Javier sobre
otro sistema H. A partir del instante cero (​​t​ 0​​​), el sistema H comienza a mover-
se en línea recta y con una velocidad constante de 2 m/s respecto del siste-
ma P, tal como se representa en la imagen. ¿Cuáles serán las coordenadas
de H respecto de P después de 3 s?
z z’

P 4m

x x’

y y’

36 Unidad 1 • Movimiento
Evalúa
4 Jorge le explica a Gabriela sobre la velocidad relativa utilizando el siguiente
ejemplo: si un ciclista se mueve a 6 m/s en la misma dirección, pero en senti-
do opuesto a otro que se mueve con igual rapidez, la velocidad del segundo
ciclista respecto del primero será igual a cero. ¿Es correcto el ejemplo dado
por Jorge? Explica.

Evalúa
5 A partir de lo que sabes del sistema solar y la ima-
gen que muestra el planeta Tierra y el Sol, res-
ponde las siguientes preguntas:
a. ¿Quien se mueve: la Tierra o el Sol? Justifica
tu respuesta.
b. ¿Qué características debe poseer un sistema
de referencia para que la Tierra parezca en
reposo? Argumenta.
Sintetiza
Elabora un mapa conceptual utilizando los siguientes
contenidos estudiados en la lección: movimiento, posición,
rapidez, velocidad, sistema de referencia, tiempo, distancia
recorrida, relatividad y desplazamiento.

¿Cómo voy?
Revisa tus respuestas y, según los resultados que hayas obtenido, marca con ✓ el
nivel de desempeño correspondiente. Si es necesario, pídele ayuda a tu profesor
o profesora.
Indicador Ítem Habilidad Nivel de desempeño
Empleé los diferentes parámetros 1y2 Representar y aplicar. L: Dos ítems correctos.
del movimiento relativo.
ML: Un ítem correcto.
PL: Ningún ítem correcto.
Consideré las diferentes ca- 3, 4 y 5 Analizar y evaluar. L: Tres ítems correctos.
racterísticas que describen los
movimientos. ML: Dos ítems correctos.
PL: Uno o ningún ítem correcto.
L = Logrado; ML = Medianamente logrado; PL = Por lograr.

Reflexiono sobre mi desempeño


Según tu apreciación (1: en desacuerdo; 2: ni de acuerdo ni en desacuerdo; 3: de
acuerdo), marca con ✓ las siguientes afirmaciones:
1 2 3
He aplicado con éxito los conceptos que permiten describir los movimientos.
He usado distintas estrategias para aprender los temas más difíciles.
He trabajado responsablemente tanto en las actividades tanto individuales como grupales.
Estoy preparado para continuar aprendiendo sobre los movimientos.

Física • 2.° Medio 37


Lección
Análisis de los movimientos horizontales
Propósito de la lección

Un atleta corriendo los 100 metros planos, un En esta lección, analizarás diversas situa-
auto transitando por la carretera o un perro ciones cotidianas en las que se describen
paseando en el parque: ¿describen el mismo movimientos horizontales. Para ello, deberás
tipo movimiento?, ¿cómo podríamos determi- aplicar modelos matemáticos e interpretar
narlo? En general, los movimientos, se pue- gráficos para comprender y describir las
den clasificar en dos categorías dependiendo características de estos movimientos. Además,
de sí poseen velocidad constante o no. realizarás diversas actividades que te per-
mitirán fortalecer tu responsabilidad y en el
trabajo colaborativo.

Ciencia, tecnología y sociedad

El rol de la velocidad en los accidentes de tránsito


El exceso de velocidad es una de las principales causas de accidentes en Chile. De
hecho, se estima que este factor es relevante en al menos 1 de cada 5 accidentes
y en 1 de cada 3 accidentes fatales1 lo que se explica, principalmente, porque:
• El exceso de velocidad reduce el tiempo de reacción que
tiene un conductor ante un imprevisto en la vía.
• Al superar el límite de velocidad para la que fue diseñada
la vía, se ponen en juego aspectos como la estabilidad del
vehículo, la visibilidad del conductor, entre otros.
• Mientras mayor sea la velocidad de un vehículo que se
acerca a un peatón u otro conductor, más difícil es juzgar
la distancia que los separa.
Durante el año 2015 y para reducir las cifras de accidentes
de tránsito por exceso de velocidad, la Comisión Nacional
de Seguridad de Tránsito (Conaset) realizó una campaña
publicitaria que pretendía mostrar lo importante que es res-
petar la velocidad máxima estipulada en zonas urbanas e
interurbanas.
La campaña pretende dar cuenta de cómo influye la rapidez Eslogan de la campaña: “Chocar a 70 km/h es igual que caer
en la distancia de reacción y de frenado ante la ocurrencia desde un séptimo piso”.
de un accidente y la gravedad que este ocaciona. Fuente: http://www.mtt.gob.cl/archivos/12776
¿Qué opinas sobre esto?
En Chile, el 40 % de los conductores excede el límite establecido en zonas ur-
banas, mientras que el 50 % lo hace en zonas interurbanas. ¿De qué manera se
deben establecer medidas de seguridad?

n
ció
e tiza ca
ab tífi
Alf cien
1
Fuente: Conaset. Se advierte que esta es una estimación, dada la dificultad de establecer la
rapidez del vehículo cuando se produce el accidente.

38 Unidad 1 • Movimiento
Ciencia al día
El secreto de la VELOCIDAD de Usain Bolt
Usain Bolt es el más grande velocista de la historia. Su marca de 9,58 se-
gundos en la carrera de los 100 metros, durante el Campeonato Mundial
de Berlín en 2009, es el actual récord mundial. Por ello, los Científicos de la
Universidad Nacional Autónoma de México (UNAM) realizaron un estudio
analizando sus características.
De acuerdo con el modelo matemático propuesto, en estudio publicado
en la revista especializada European Journal of Physics, el tiempo de 9,58
segundos que Bolt consiguió en Berlín se logró alcanzando una velocidad
de 12,2 metros por segundo, equivalente a unos 44 kilómetros por hora.
John Barrow, profesor de Ciencias Matemáticas de la Universidad de Cam-
bridge, analizó anteriormente cómo Bolt pudo superarse a sí mismo.
Para Barrow, la velocidad de Bolt se debe en parte a la “extraordinaria
longitud de su zancada” a pesar de tener un tiempo de reacción más lento
al disparo de largada. El científico dijo que Bolt aún tiene margen para batir
su propio récord, pero para eso debería ser más rápido en el inicio, y correr
con un viento a favor un poco más fuerte y a una mayor altitud, donde hay
menos resistencia. Según explicó Barrow, para el récord se puede correr
con un viento a favor de hasta dos metros por segundo, por lo tanto, “Bolt
tiene un gran margen para mejorar sin tener que ser más rápido”.
¿De qué marera el estudio científico permite comprender mejor las cuali-

Kaliva / Shutterstock.com
dades de un atleta?, ¿cómo la realización de este estudio permite que un
atleta pueda ser más rápido en el deporte que practica?

Fuente: http://www.bbc.com/mundo/noticias/2013/07/130729_ciencia_usain_bolt_
velocidad_matematica_np

Inicio de la misión
Como pudiste leer en estas páginas, el estudio de la ve-
locidad es importante en la descripción del movimiento.
En esta lección, tendrás la misión de confeccionar una
maqueta que muestre el movimiento de un cuerpo con
velocidad variable. Para ello, puedes solicitar la colabo-
ración de tu profesor o profesora de Educación Tecnoló-
gica, para que te ayude a implementar nuevas técnicas
y recursos.
Para llevar a cabo esta misión, reúnete con tres compañe-
ros y realicen un plan de trabajo, considerando que, al fi-
nalizar esta lección, deberán mostrar su maqueta al curso.

Física • 2.° Medio 39


Lección 2

Tema 1 Describiendo el Movimiento Rectilíneo Uniforme (MRU)


En este tema, estudiaremos movimientos cotidianos que se efectúan con velo-
¿Qué significa que un movimiento
cidad constante. Para ello, aprenderás a utilizar ecuaciones de movimiento que posee velocidad constante?
te permitirán adquirir estrategias para confeccionar e interpretar gráficos que
describan los movimientos.

Indaguemos a partir de nuestros aprendizajes previos Actividad grupal

Objetivo: Determinar el módulo de Reúnete con dos compañeros, y consigan una tiza, una huincha de medir
la velocidad. y un cronómetro (pueden usar el de un celular). Luego, realicen el siguiente
Habilidad: Analizar resultados de procedimiento:
una experiencia. 1. Salgan al patio y busquen un sector amplio donde puedan dibujar una línea
Actitud: Ser riguroso en el trabajo de 30 metros de largo y trácenla con la tiza.
colectivo.
2. Sobre la línea, dibujen una marca cada 5 metros.
Tiempo: 30 minutos.
3. Un integrante del equipo camine sobre la línea con paso firme y regular. Mien-
tras tanto, los otros integrantes registrarán, en la siguiente tabla, el tiempo
cuando pase por cada marca sobre la línea.
Distancia (m) 5 10 15 20 25 30
Tiempo (s)
Velocidad (m/s)

4. Determinen el módulo de la velocidad seguida en cada tramo y registren su


valor en la tabla. ¿Qué modelo matemático utilizarán?
A partir de los datos obtenidos, respondan las siguientes preguntas:
a. ¿Qué características tiene el movimiento descrito por el integrante del grupo?

b. ¿Cómo es el módulo de la velocidad en cada tramo? Comparen.

c. ¿Permanece constante la velocidad media?, ¿cómo lo establecieron?

d. ¿Cómo organizaron y distribuyeron las tareas en equipo?, ¿consideraron las


Materiales de la actividad. habilidades de cada integrante?

Si un objeto en movimiento describe una trayectoria rectilínea, como en la acti-


Importante
vidad anterior, y además posee una velocidad media constante, entonces descri-
be un Movimiento Rectilíneo Uniforme (MRU). Todo MRU posee las siguientes En el MRU siempre se cumple que
características: el módulo de la velocidad coinci-
de en todo momento con el valor
• La trayectoria es una línea recta.

∣​​v→  ​​∣  = v
de la rapidez, por lo tanto:
• La distancia recorrida es igual al módulo del desplazamiento.
• La rapidez es constante a lo largo de todo el movimiento, es decir, la rapidez
media y la instantánea tienen el mismo valor en todo momento.
• El módulo de velocidad coincide en todo momento con el valor de la rapidez.
• No acelera.

40 Unidad 1 • Movimiento
Representación gráfica de un MRU
Analicemos la situación de un automóvil que se mueve en línea recta con veloci-
dad constante, es decir, describe un MRU. Cuando esto sucede, por cada unidad
de tiempo, el automóvil recorre la misma distancia y no cambia el sentido de su
movimiento.

0 s 1 s 2 s 3 s

5 m 5 m 5 m


0 5m 10 m 15 m

Observando el esquema, podemos concluir que, por cada segundo que transcurre,
el automóvil avanza una distancia de 5 metros, de tal manera que:
Tiempo (s) 0 1 2 3
Distancia (m) 0 5 10 15

Si graficamos este movimiento obtenemos lo siguiente:

Gráfico de posición en función del tiempo

Gráfico de posición-tiempo Gráfico de posición-tiempo


x (m) x (m)
15 15

10 10

∆x

5 5
∆t

0 1 2 3 t (s) 0 1 2 3 t (s)

1 En el gráfico se sitúan los puntos que indican la posición del mó- 3 Se puede determinar la rapidez del automóvil determinando la
vil en cada instante de tiempo respecto del sistema de referencia. pendiente del gráfico. Para ello, utilizamos la siguiente expresión:
2 Como en un MRU la velocidad media es constante, se puede Δx  
v = ​​ ___ ​​  10 m – 5 m
​​ = __________    = 5 m/s
​​ 
trazar una recta que une todos los puntos. Δt 2s–1s
El signo que resulta indica el sentido del movimiento. En este
caso, el automóvil se mueve con una velocidad media de
Actividad individual 5 m/s en sentido positivo respecto del sistema de coordenadas.
Reflexiono sobre el gráfico de posición
Gráfico de posición-tiempo

Posición (m)
Persona 1
Tres personas se mueven describiendo un MRU, mediante las
Persona 2 características se representaron en el gráfico posición en fun-
ción del tiempo adjunto.
Persona 3
1. ¿Qué persona se mueve a mayor velocidad?
2. ¿De qué manera interpretaste el gráfico para responder la
pregunta anterior?
Tiempo (s)

Física • 2.° Medio 41


Lección 2

A partir de los datos obtenidos para el vehículo, también se puede construir el


gráfico de velocidad en función del tiempo.

Gráfico de velocidad en función del tiempo

Gráfico de velocidad-tiempo
1 Como la velocidad se mantiene constante, el gráfico v (m/s)
resulta ser una recta paralela al eje horizontal (que con-
5
tiene los valores del tiempo).

0 1 2 3 t (s)

2 A partir de este tipo de gráfico, podemos conocer la dis- Gráfico de velocidad-tiempo


tancia recorrida por el automóvil (en cualquier intervalo v (m/s)
de tiempo). Esto se realiza determinando el área bajo la
recta. En este caso, obtenemos que: 5

Distancia (d) = Área (figura) = base · altura


d = 3 s · 5 m/s = 15 m 0 1 2 3 t (s)

Actividad grupal
Interpretemos gráficos del MRU
Júntense en parejas y realicen las siguientes actividades.
Javiera y Fernando, dos estudiantes de 2° medio, para comprender mejor las ca- Gráfico de posición-tiempo
racterísticas de sus movimientos, deciden construir los gráficos que se muestran Posición (m) Javiera
al costado.
7
1. ¿Cuál es la posición inicial y la posición final de Javiera y de Fernando? 6
2. Determinen el desplazamiento de cada uno. 5

3. Determinen la velocidad media de Javiera y Fernando. 4

3
4. Construyan los gráficos de velocidad en función del tiempo para Javiera y
para Fernando. 2
1

0 1 2 3 Tiempo (s)

Gráfico de posición-tiempo

Posición (m) Fernando

2
A partir de los gráficos, respondan las preguntas a continuación:
1
a. ¿Qué distancia recorrió cada uno?
b. ¿Por qué la distancia recorrida tiene el mismo valor que el desplazamiento?
0 1 2 3 4 Tiempo (s)
Justifiquen.
c. ¿Qué ventajas tienen los gráficos en el estudio del movimiento? Argumenten.

42 Unidad 1 • Movimiento
Ecuación itinerario de un MRU
La posición de un cuerpo que describe un MRU no solo se puede representar me-
diante gráficos, sino que también con expresiones matemáticas, denominadas ecua-
ciones de itinerario, las cuales permiten conocer la posición de un cuerpo a partir de
su posición inicial, de la rapidez con la que se mueve y del tiempo que transcurre.

​​x→  ​​  f ​​- ​​x→  ​​  i ​​


Una de estas ecuaciones de itinerario se obtiene de la siguiente expresión: Importante

​​v→  ​​  = ___

​  = ​​ ______
​​  Δ​x  ​​   ​​   ⇒ ​​​x→  ​​  f ​​​ = ​​​x→  ​​  i ​​​ + ​​v→  ​​  ‧ Δt
Todas las expresiones que se
Δt Δt estudian en esta unidad pueden
ser trabajadas en su versión
​​x→  ​​  = ​​​x→  ​​  i ​​​ + ​​v→  ​​  ‧ Δt
Ahora, para cualquier tiempo, esta expresión matemática se escribe como:
escalar. De esta manera, la ecua-
ción itinerario se puede expresar:
Es importante mencionar que la expresión para determinar la posición correspon- x = xi + v ​​ ‧ Δt
de a la ecuación de una recta y que es consistente con los gráficos presentados
anteriormente.

Desarrollo de estrategias
Aprendiendo a interpretar gráficos y a aplicar modelos.

Situación problema

César y Andrea son atletas cuyo entrenamiento consiste en correr por un parque Gráfico de posición-tiempo
describiendo una trayectoria recta. Jimena, su entrenadora, les toma el tiempo duran- x (m)
te los primeros 8 segundos de su recorrido y, para analizar su rendimiento, construye
el gráfico que se muestra al costado. A partir de esto, determina: 60

a. La velocidad media de César.


b. La ecuación itinerario de César. sar

c. El tiempo que tarda César en llegar a los 100 metros, si continúa su carrera ea
dr
con velocidad constante. An
28
PASO 1 Identifico las variables del gráfico
20
La posición de César a los 0 s es de 28 m, y a los 8 s es de 60 m.
PASO 2 Aplico los modelos

​​x→  ​​  = ​​​x→  ​​  i ​​​ + ​​v→  ​​  ‧ Δt


a. Determino la velocidad media: b. Determino la ecuación itinerario:
→  ___________ 8 t (s)
Δ​ x ​
→ = ​​ ___
​​ v ​​  ​​ = ​ ​60
   m – 28
  m 
​​ = 4 m/s
Δt 8s
​​​ →​​    ​​​ = 28 + 4 ‧ t
x  César

c. Determino el tiempo.
Para ello, hay que despejar la ecuación itinerario 100 = 28 + 4 ‧ t
cuando César se encuentra en los 100 metros: 100 m – 28
     m 
t = ​​ ____________ ​​ = 18 s
4 m/s

PASO 3 Escribo la respuesta


En su carrera, César lleva una velocidad media de 4 m/s y tarda 18 s en llegar a los
100 m.
PASO 4 Aplico lo aprendido
Determina la velocidad media de Andrea. ¿Quién llega primero a los 100 metros:
César o Andrea? Fundamenta.

Física • 2.° Medio 43


A poner en práctica mediante un taller de habilid
ades científicas

Aprendiendo a organizar datos: en el MRU del agua


Habilidad: Organizar datos cuantita- Actitud: Sugerir soluciones y buscar
tivos y cualitativos con precisión. alternativas para resolver problemas.

Situación problema

La siguiente experiencia te permitirá


organizar datos para determinar la
rapidez con la que desciende el agua
en un Frasco de Mariotte. Este dispo-
sitivo consta de una botella que tiene
un orificio lateral cerca de su base y un
tubo recto pequeño en su interior. Al
llenar la botella con agua esta sale por
el orificio con rapidez constante por un
determinado tiempo.

Frasco de Mariotte. Montaje experimental.


Procedimiento experimental

Reunidos en parejas, consigan los materiales que se les solicitan. Luego, ✓ Una botella desecha- ✓ Un recipiente hondo
realicen el siguiente procedimiento: ble de 1,5 L con tapa. de 2 L de capacidad.
✓ Una bombilla o tubo ✓ Agua.
1. Perforen, con la supervisión de su profesor, la tapa de la botella de plástico delgado. ✓ Un cronómetro.
para introducir la bombilla. Sellen el orificio alrededor de la bom- ✓ Un marcador ✓ Plasticina.
billa con plasticina. permanente. ✓ Una regla.
✓ Cinta adhesiva para ✓ Un alfiler o aguja.
2. Realicen marcas en forma vertical cada 0,5 cm, desde el principio embalaje.
hasta el fondo de la botella.
3. Hagan un pequeño orificio en la última marca de la botella. Viertan una pe-
queña cantidad de agua y comprueben si, a través del orificio, fluye agua de Advertencia: Tengan precaución al
manipular los materiales corto pun-
forma permanente. Si el agua sale a borbotones, deberán agrandar un poco
zantes (alfiler o aguja).
más el orificio.
4. Cubran el orificio con cinta adhesiva y luego llenen la botella con agua por
encima del nivel marcado. Cierren la botella con la tapa que posee la bom-
billa incrustada. Reduce, Reutiliza, Recicla

5. Coloquen la botella sobre una mesa y en el suelo ubiquen el recipiente para ¿Qué harán con los materiales
contener el agua que saldrá por el orificio, tal como muestra el montaje cuando terminen la experiencia?
experimental. Al finalizar la actividad pueden
utilizar el agua para regar
6. Saquen la cinta adhesiva del orificio para que comience a salir el agua. Cuan- plantas y botar los materiales
do esta llegue a la primera marca inicien el cronómetro. desechables en contenedores
para reciclaje.
7. Dejen que el agua siga saliendo y registren el tiempo que indica el cronóme-
tro cuando el nivel del agua pase por cada una de las marcas en la botella.

44 Unidad 1 • Movimiento
Organización de los datos

a. Construyan una tabla con los valores registrados en cada intento. Pueden utilizar una como
la siguiente:
Distancia recorrida por el Tiempo que tarda el nivel Ayuda
nivel de agua (cm) de agua en descender (s)
← Para determinar el
0 0 tiempo solicitado, calculen
0,5 la diferencia entre el tiempo
inicial y el correspondiente
1,0 al paso por cada marca.
1,5
2,5

b. En un sistema de ejes coordenados representen la distancia recorrida por el nivel de agua


(OY) y el tiempo registrado (OX) y grafiquen los datos registrados en la tabla.

Análisis e interpretación de evidencias

c. ¿Qué característica posee el gráfico de posición en función del tiempo? ¿Qué pueden ob-
servar en él? Expliquen.
d. ¿Podrían determinar la rapidez aproximada con la que desciende el agua? De ser así, expli-
quen cómo lo harían.
e. Si el eje OY lo extienden hasta una posición de 30 cm, ¿cuánto tiempo tarda el agua en
alcanzar ese nivel? Describan dos maneras de hacerlo.
f. Elaboren una conclusión que relacione los datos obtenidos, la organización de los datos y lo
que saben respecto del MRU.
g. ¿Qué dificultades surgieron durante la obtención de los datos recopilados?, ¿cómo las
sobrellevaron?
h. ¿Cómo evaluarían su trabajo individual durante el trabajo colaborativo?

Física • 2.° Medio 45


Lección 2

Tema 2 Movimiento Rectilíneo Uniformemente Acelerado (MRUA)


En el mundo que nos rodea, es poco habitual que un movimiento permanezca
con velocidad constante, dado que la mayoría de ellos experimentan variaciones ¿Has escuchado hablar de la
aceleración? Si la tuvieras que
de rapidez, de dirección y/o de sentido. Por ejemplo, el solo hecho de iniciar una definir usando los conceptos de
caminata implica aumentar la velocidad cuando se deja el reposo inicial. velocidad y tiempo, ¿cómo lo
harías?
En este tema, aprenderás a utilizar modelos que te permitan describir movimien-
tos más complejos que posean velocidad variable. Para ello, realizarás diversas
actividades para buscar soluciones a problemáticas científicas manifestando in-
terés por trabajar de forma responsable y colaborativa.

Indaguemos a partir de nuestros aprendizajes previos Actividad grupal

Objetivo: Describir los cambios en el Reúnanse en parejas y consigan una bolita de cristal o de acero. Luego, utili-
estado de movimiento de un cuerpo. cen una superficie horizontal (como una mesa) para llevar a cabo el siguiente
Habilidad: Comparar las características procedimiento:
de dos sucesos.
1. Con su mano, impulsen la bolita de modo que se ponga en movimiento res-
Actitud: Ejecutar las tareas de manera pecto de la mesa (observen la imagen).
rigurosa.
2. Luego, déjenla caer desde unos 20 cm de altura. A partir de sus observacio-
Tiempo: 10 minutos.
nes, respondan las siguientes preguntas:
a. ¿Qué conceptos piensan que están involucrados en la actividad? Escríbanlos.

b. ¿Cuál era el estado de movimiento inicial de la bolita en ambas situaciones?


Expliquen.

c. ¿Cómo varió la velocidad de la bolita en la primera situación? Describan.

d. ¿Cómo varió la velocidad de la bolita en la segunda situación? Describan.

e. ¿Cómo pudieron determinar si la bolita cambió de velocidad? Fundamenten.

Montaje de la actividad. f. ¿Llegaron a acuerdos para desarrollar el procedimiento para alcanzar los
aprendizajes de forma colaborativa?

g. Fueron rigurosos al observar y describir cada una de las situaciones? De no


ser así, repitan la experiencia.

En la actividad anterior, pudiste observar que el movimiento de la bolita experimentó varia-


ciones en su velocidad porque pasó de estar en reposo a describir un movimiento con cierta
velocidad. ¿Qué magnitud física da cuenta del cambio de velocidad de un cuerpo?

46 Unidad 1 • Movimiento
La aceleración corresponde a la variación de la velocidad que experimenta un
cuerpo en un determinado tiempo. Para comprender mejor este concepto, ana-
licemos la siguiente situación.
La imagen muestra el movimiento que describe una atleta cuando comienza una
carrera desde el reposo.

​​​v ​​  i ​​​ = 0. ​v ​​f  (distinta de cero).


En el instante ti = 0, la atleta se encuentra En un instante tf, la velocidad de la atleta
en reposo, es decir, → es →
Después de la partida, la atleta intenta
incrementar su velocidad en el menor
tiempo posible.

​​→ )​​   que experimenta un cuerpo, sino que también del tiem-


Como puede apreciarse en la situación anterior, la aceleración no solo depende del
cambio de velocidad (Δv
po (Δt) en el cual este ocurre, lo que, matemáticamente, se puede expresar como:

​  = ​​ ______

  →

Donde:
​​​ →f​  ​​  es la velocidad final.
→ ​​ v   ​​  -
​​ v
​​   ​​  ​​
→ ​​  Δ​v   ​​ 
​​  m​  ​​ ​​  = ___ v
f i
​a  ​​  

​​​v→ ​​  i ​​​ es la velocidad inicial.
Δt ​tf​  ​​- ​ti​  ​​

Esta relación matemática se conoce como aceleración media. Entonces, la acele-


Importante
ración involucra cualquier cambio en la velocidad, ya sea un aumento o disminu-
ción, o un cambio en la dirección o sentido del movimiento. Si un cuerpo cambia La unidad de medida de la
de velocidad en la misma cantidad en intervalos de tiempos iguales, entonces se aceleración en el Sistema Interna-
dice que su aceleración es constante. Este tipo de movimiento se conoce como cional es m/​​s​​  2​​, y se obtiene de la

​  = ​ ​____
expresión:

Movimiento Rectilíneo Uniformemente Acelerado (MRUA).
​​  Δ​v   ​​ 
→​​    ​​​= ___ m/s
Como la aceleración depende de la variación de la velocidad, también es una mag- ​​​ a  ​​ = m/​​s​​  2​​
s   
m Δt
nitud vectorial, por lo que posee módulo, dirección y sentido. Este último, en un Esta unidad indica la cantidad
movimiento rectilíneo, se expresa con un signo positivo o negativo dependiendo de m/s que un móvil aumenta o
del sistema de referencia. disminuye en cada segundo.
Actividad individual
Aplico la ecuación de aceleración
Determina la aceleración de Usain Bolt en el campeonato mundial de Berlín, consi-
derando que alcanzó una rapidez de 0 a 12,2 m/s en tan solo 9,58 segundos.

Física • 2.° Medio 47


Lección 2

Desarrollo de estrategias
Aprendiendo a aplicar modelos para determinar la aceleración de un automóvil.

Situación problema

Un automóvil que parte del reposo con movimiento rectilíneo, alcanza una rapidez
de 72 km/h después de un minuto. ¿Cuál es el módulo de su aceleración media?

PASO 1 Identifico las variables


vi = 0
vf = 72 km/h
∆t = 1 min

PASO 2 Verifico las unidades de medida


Es importante recordar que todas las unidades de medida deben corresponder a
las utilizadas en el Sistema Internacional de unidades. En este caso, es necesario
expresar la rapidez en m/s y el tiempo en s.

​​vf​  ​​​ = 72 ​ ​___ km   ​​ ‧ ​​ ______


km   ​​ = 72 ​ ​___ 1000 m    ​​  1 h   
​​ ‧ ______
   ​​ = 20 m/s
h h 1 km 3600 s
60 s  ​​ 
Δt = 1 min = 1 ​​min​​ ‧ ​​ _____ = 60 s
1 min

PASO 3 Aplico el modelo


Es aquí, en donde se utiliza la expresión matemática para obtener el valor del módulo
de la aceleración media. En este caso:
​  m
20 __
​vf​  ​- ​vi​  ​ ________ s  ​– 0
am = ___​ Δv
Δt

  = _____


Δt
   ​
​ = ​ ​
60 s
   ​​ 
= 0,33 m/​​s​​  2​​
PASO 4 Escribo la respuesta
El módulo de la aceleración del automóvil es de 0,33 m/s2. Este resultado significa
que por cada segundo que pasa, la rapidez del automóvil aumenta en 0,33 m/s.

PASO 5 Aplico lo aprendido


En su clase de ciencias, Juan descubre que el guepardo es el animal terrestre más rá-
pido del mundo ya que puede alcanzar una rapidez de 0 a 96 km/h en tan solo 3 se-
gundos. Determina el módulo de la aceleración media que experimenta el guepardo.

Desarrollo de la misión
Considerando lo que has aprendido sobre los movimien-
tos con velocidad variable, confecciona la maqueta de la
misión junto con tu equipo de trabajo. ¿De qué manera se
distribuyeron las tareas?, ¿consideraron las habilidades de
cada uno? Fundamenten.

48 Unidad 1 • Movimiento
Representación gráfica de un MRUA las tic
Cuando la velocidad de un cuerpo en movimiento rectilíneo cambia de manera Ingresa el código  18TF2M049a
constante por unidad de tiempo, es decir, su aceleración es la misma, se dice que en la página web del texto. Allí
posee un movimiento rectilíneo uniformemente acelerado (MRUA). Por ejemplo, encontrarás una aplicación que te
en la siguiente imagen, se observan los efectos de una aceleración constante y permitirá graficar un MRUA.
positiva sobre un vehículo.
​​→  1​​  
v ​​→  2​​  
v ​​→  ​​3 
v ​​→  4​​  
v ​​→  ​​5 
v

​​ →  ​​  
a ​​ →  ​​  
a ​​ →  ​​  
a ​​ →  ​​  
a ​​ →  ​​  
a

Gráfico de posición en función del tiempo Gráfico de posición-tiempo


x (m)
En un MRUA, la distancia recorrida por un móvil se incre-
menta por cada unidad de tiempo. Por lo tanto, el gráfico de
posición en función del tiempo tiene la forma de una curva
(tal como se muestra en el gráfico). x3

Gráfico de velocidad en función del tiempo x2

El gráfico de la velocidad en función del tiempo corresponde


x1
a una recta, tal como se muestra en el gráfico de la derecha.
A partir del área bajo la recta, se puede determinar una 0 t1 t2 t3 t (s)
expresión que nos permite conocer la distancia recorrida en
un MRUA. En este caso, la distancia corresponderá a la suma
Gráfico de velocidad-tiempo
de las áreas A1 y A2 , es decir: v (m/s)

​​  1  ​​ ‧ (vf - vi) ‧ Δt


d = A1 + A2 = vi ‧ Δt + _
vf
2
Como el módulo de la aceleración es: ​  12 ​​ (vf - vi) ∆t
 ​__
A2
vf - vi
a = ______
​​   ​​ 
  vi
Δt
Entonces, la distancia resulta ser: A1 vi ∆t

​​  1  ​​‧ a ‧ Δt2


d = vi ‧ Δt + _
2 ti tf
t (s)
∆t
La pendiente del gráfico de velocidad en función del
tiempo corresponde a la aceleración media del móvil.

Gráfico de aceleración en función del tiempo


Gráfico de aceleración-tiempo
En un MRUA, la aceleración es constante, por lo tanto, su 2
a (m/s )
gráfico en función del tiempo corresponde a una línea rec-
ta, paralela al eje del tiempo. a

El área A limitada bajo la recta corresponde a la variación


del módulo de la velocidad (∆v). A

∆v = a · ∆t
t (s)

Física • 2.° Medio 49


Lección 2

Ecuación itinerario para un MRUA


En el siguiente recuadro, se presentan las expresiones matemáticas más repre-
sentativas del movimiento uniformemente acelerado. A partir de ellas y de su
combinación, es posible determinar cualquier variable involucrada en este tipo
de movimiento.

Variables relacionadas Ecuación

 ​​f  = ​​a→  ​​  ‧ Δt + ​​ v→i ​​  


Velocidad final, velocidad inicial, acelera- Velocidad:
ción y tiempo. ​​ →
v
Posición final, posición inicial, velocidad Itinerario:
​​  f ​​  = ​​ xi ​​ + ​​​v ​​  i ​​​ ‧ Δt + ​​ __
1 ​​​​  ‧ a →​​  ‧ Δt2
inicial, aceleración y tiempo. → → →
x
2

Desarrollo de estrategias
Aprendiendo a interpretar gráficos y aplicar modelos.
Gráfico de velocidad-tiempo
Situación problema
Velocidad (m/s)
En el gráfico del costado, se representa el movimiento de un
ciclista que viaja en línea recta y que parte desde el origen del 3

sistema de referencia. A partir de la información que se puede


extraer del gráfico, escribe la ecuación itinerario del ciclista y 2
determina su posición a los 6 segundos.
1
PASO 1 Identifico las variables del problema
xi = 0 vf = 3 m/s
0
vi = 2 m/s ∆t = 6 s 1 2 3 4 5 6 Tiempo (s)

PASO 2 Identifico las variables de la ecuación itinerario

xf = xi + vi ‧ Δt + _ ​​  1  ​​ ‧ a ‧ Δt2


2
PASO 3 Aplico los modelos
vf - vi
a = ______
​​  = _____
 ​​ 
  ​​  3 - ​​ 2   = ​​ _1  ​​ m/s2  ​​
Δt 6 6
PASO 4 Escribo la ecuación itinerario

xf = 0 + 2 ‧ t + _ ​​  1  ​​ ‧ _ ​​  1 ‧​​t2 → xf = 2 ‧ t + __ ​​  1  ​​ ‧ t2


2 6 12
La posición del ciclista en t = 6 (s) será:
​​  1  ​​ ‧ 62 = 15 m
xf = 0 + 2 + __
12
PASO 5 Aplico lo aprendido
Determina la ecuación de velocidad del ciclista. ¿Qué velocidad tendrá a los 10 s?

50 Unidad 1 • Movimiento
Ciencia, tecnología y sociedad

En tierra, el guepardo es el felino más veloz del mundo: alcanza valores de entre
112 y 120 km/h y puede acelerar de 0 a 96 km/h en 3 segundos. La velocidad
que alcanza este felino está dada por la forma de su cuerpo y su anatomía: posee
garras no retractiles que le dan una mayor adherencia a la superficie, sus fosas
nasales son de gran tamaño para favorecer el ingreso de más oxígeno durante
su carrera y su larga cola le permite maniobrar durante la carrera dando sorpren-
dentes giros para atrapar a su presa.
En el agua, el más veloz es el pez vela, el que puede alcanzar valores de hasta
110 km/h en distancias cortas. Este veloz representante del mundo acuático tiene
una cola rígida en forma de C que es idónea para la velocidad, además posee una
aleta dorsal que corta el agua como las hélices de los barcos y una prolongación
en su mandíbula superior que favorece su hidrodinámica al sortear las aguas con
mayor agilidad.
http://www.latam.discovery.com/animal-planet/los-animales-mas-veloces-del-planeta/

¿Qué les permite a estos animales ser los más rápidos en sus respectivos
ambientes?

ión
zac
b eti ífica
a t
¿Cómo crees que el estudio de la naturaleza permite desarrollas Alf cien

nuevos avances tecnológicos?

Cierre de la misión
Al inicio de esta lección, te propusimos la misión de con-
feccionar una maqueta que describiera un movimiento ¿Esta misión se incluirá en el proyecto del texto?
acelerado ¿De qué manera el equipo de trabajo buscó Fundamenta en la página 12.
soluciones para desarrollar este proyecto?
¿Cómo evaluarías los aportes realizados por cada inte-
grante del grupo? Fundamenta.

Reflexiono sobre lo que aprendí

Lee y comenta las siguientes preguntas con tus compañeros y compañeras para
saber si alcanzaste el propósito de la lección.
Contenido Habilidades/Estrategias Actitudes
¿Qué te pareció más difícil de comprender?, ¿Qué estrategias empleaste para ¿Cómo consideras que aportaste al traba-
¿a qué crees que se deba? ¿Qué te resultó utilizar correctamente los modelos jo en equipo?, ¿por qué? ¿Cómo podrías
más fácil de aprender? ¿De qué manera so- matemáticos de la lección? mejorar?
licitaste ayuda cuando tuviste dificultades?

Física • 2.° Medio 51


Integro lo que aprendí
Evaluación de proceso

Analiza
1 El siguiente gráfico de velocidad en función del tiempo se construyó con
información de un automóvil que transita en un camino rectilíneo.

v (m/s)

50
2
40
30 3
1 4
20
10 5

0 5 10 15 20 25 30 35 40 t (s)

Para cada uno de los tramos numerados, determina:


a. El tipo de movimiento descrito por el automóvil (MRU o MRUA).
Tramo 1 Tramo 2 Tramo 3 Tramo 4 Tramo 5
Tipo de
movimiento

b. La aceleración del automóvil.


Tramo 1 Tramo 2 Tramo 3 Tramo 4 Tramo 5
2
Aceleración (m/s )

c. La distancia recorrida.
Tramo 1 Tramo 2 Tramo 3 Tramo 4 Tramo 5
Distancia
recorrida (m)

d. ¿De qué manera el trabajo responsable te ha permitido buscar soluciones


a los problemas planteados en esta actividad?
Analiza
2 Un ciclista describe un MRUA de acuerdo a la siguiente ecuación itinerario:
xt = 3 + 5t + 2t2
A partir de la información que entrega esta ecuación, realiza lo siguiente:
e. Completa la tabla con los valores solicitados:

Posición inicial Velocidad media Aceleración media

f. Construye en tu cuaderno los gráficos solicitados:

Posición-tiempo Velocidad-tiempo Aceleración-tiempo

52 Unidad 1 • Movimiento
Aplica
3 Mariana desciende con su bicicleta por una pendiente que le otorga una
aceleración constante de 3 m/s2, durante 3 s. Si su velocidad inicial fue de
2 m/s, determina:
a. La velocidad final.
b. La distancia total recorrida.
c. ¿Qué medidas de seguridad debería considerar Mariana?
Aplica
4 Una moto viaja hacia la izquierda con una rapidez media constante de 25 m/s
cuando de pronto el vehículo que va delante se detiene bruscamente. Si la
moto alcanza a detenerse en 6 segundos, determina:
a. La aceleración de la moto cuando frena.
b. La distancia que alcanza a recorrer la moto mientras frena.
c. Dibuja el gráfico de velocidad en función del tiempo de la moto.
Sintetiza
5 A partir de los aprendizajes adquiridos en la lección, completa la siguiente
tabla indicando los contenidos, habilidades y actitudes desarrolladas.

Contenidos Habilidades Actitudes

¿Cómo voy?
Revisa tus respuestas y, según los resultados que hayas obtenido, marca con ✓ el
nivel de desempeño correspondiente. Si es necesario, pídele ayuda a tu profesor
o profesora.
Indicador Ítem Habilidad Nivel de desempeño
Analicé las características que describen 1, 2 y 5 Analizar y sintetizar. L: tres ítems correctos.
los MRU y MRUA.
ML: Dos ítems correctos.
PL: Uno o ningún ítem correcto.
Apliqué las ecuaciones que describen 3y4 Aplicar. L: Dos ítems correctos.
los MRUA.
ML: Un ítem correcto.
PL: Ningún ítem correcto.
L = Logrado; ML = Medianamente logrado; PL = Por lograr.

Reflexiono sobre mi desempeño


Según tu apreciación (1: en desacuerdo; 2: ni de acuerdo ni en desacuerdo; 3: de
acuerdo), marca con ✓ las siguientes afirmaciones:
1 2 3
He aplicado satisfactoriamente los modelos que permiten describir los movimientos.
He desarrollado nuevas habilidades para trabajar los aprendizajes de esta lección.
He trabajado responsablemente de forma proactiva y colaborativa.

Física • 2.° Medio 53


Lección
Análisis de los movimientos verticales
Propósito de la lección

Seguramente, en más de alguna oportunidad caída libre y el lanzamiento vertical. Para ello,
has lanzado una moneda para probar suerte necesitarás aplicar los aprendizajes adquiridos
o bien, se te ha caído un objeto de las manos. en las lecciones anteriores, para describir las
Pero ¿qué particularidades poseen estos características de cada tipo de movimiento.
movimientos? Además, deberás emplear las tecnologías de
En esta lección, analizarás diversas situa- la información y comunicación para procesar
ciones cotidianas en las que se describen la evidencias y formular explicaciones científicas.

Ciencia al día
La física en el parque de diversiones
Un parque de diversiones es un lugar para experimentar diferentes sen-
saciones, como emoción, temor o alegría, pero también es un labora-
torio de ciencia y tecnología, en donde se conjugan los principios de
la física con la entrega de entretención. Por ejemplo, en Chile, en
el primer parque de diversiones: Fantasilandia (fundado en el año
1978) es posible encontrar alrededor de 39 atracciones instaladas
dentro del Parque O’Higgins en Santiago. En la mayoría de ellas es
posible aplicar diferentes principios de la física, en particular, en el
llamado Xtreme fall, se puede observar el comportamiento de un
cuerpo en caída libre.
El Xtreme fall es un juego mecánico que entrega la emoción de pre-
cipitarse de manera abrupta desde la cima de una torre en caída libre.
Cuenta con dos versiones, una estructura grande para los adultos y una
versión más pequeña para los niños, y en ambas se puede experimentar
una gran rapidez en caída libre.
La estructura para los adultos consiste en una torre de 35 metros que al ascender
entrega una vista panorámica de la ciudad y que desciende en alrededor de 3 s, Fotografía del Xtreme fall en
por lo que se puede experimentar una rapidez cercana a los 26 m/s, lo que equi- Fantasilandia.
vale a unos 94 km/h de velocidad en caída libre.
¿Qué te parece que se pueda lograr este tipo de atracción a través del desarrollo
de la ciencia y la tecnología? Argumenta.

¿Qué importancia tienen las medidas de seguridad que poseen estos juegos?
Fundamenta.

ión
t i zac a
e
ab tífic
Alf cien

54 Unidad 1 • Movimiento
Ciencia, tecnología y sociedad

El PARACAÍDAS en contra de las caídas


¿Cómo podemos retrasar la caída de los objetos? Seguramente, esta fue la pre-
gunta que intentaron responder los científicos e inventores que elaboraron los
primeros paracaídas que, como su nombre lo dice, están diseñados para frenar
las caídas quizás, para cumplir el sueño más antiguo de la humanidad: volar. Pero,
¿cómo se desarrolló esta idea?
Se estima que los primeros intentos por confeccionar un paracaídas comenzaron
en China, donde se construyó una especie de paraguas para realizar saltos de
una torre. Pero no fue hasta el siglo XV cuando el pintor, escultor, matemático,
científico, ingeniero, diseñador e inventor Leonardo da Vinci ideó un aparato que Diseño de Leonardo da Vinci.
les sirviera a las personas para escapar de lo alto de un edificio que estuviera in-
cendiándose. Para esto, diseñó un “paracaídas” de forma piramidal, como el que
se muestra en la imagen. Aunque no se sabe si él probó este paracaídas, muchos
consideran a Leonardo da Vinci como el “padre el paracaidismo“.
En el año 1616 el italiano Fausto de Veranzio publicó un libro llamado Machinae
Nova (máquina nueva), en el cual aparecía un dibujo llamado “Homo Volans” (hom-
bre volando), que mostraba a un hombre saltando de una torre con un paracaídas
rectangular con cuatro líneas sujetas al cuerpo en forma de arnés, como se puede
ver en este dibujo adjunto. Este paracaídas es muy similar al que se usa actualmente
en el paracaidismo deportivo.
En 1779, el físico francés Sebastián Normand hizo una serie de estudios con para-
caídas, efectuando lanzamientos con animales. Dada la cantidad de experimentos
que realizó, se lo puede considerar como el primer constructor de paracaídas.
En 1785, Jean Pierre Blanchard, diseñó y construyó el primer paracaídas con cúpula
de seda que se podía envasar. Hasta esa fecha, todos los paracaídas eran construi-
dos con un armazón que mantenía la cúpula abierta.
¿Qué opinas de la historia del paracaídas?, ¿qué te pareció más interesante?

Homo Volans.

Inicio de la misión
En esta lección, tendrás la misión de crear un paracaídas
que permita frenar la caída de un huevo. Para ello, reúne-
te con dos o tres compañeros y confeccionen un modelo
de paracaídas que les permita frenar la caída del huevo,
desde un segundo piso, para que este no se quiebre al
caer. Al finalizar la lección, podrán poner a prueba su pro-
totipo, pero antes ¿cómo creen que esto sea posible?

Física • 2.° Medio 55


Lección 3

Tema 1 ¿Qué características posee la caída libre? ¿Qué tipo de movimiento se


Si se sueltan un paquete de arroz de 1 kg y otro de 3 kg simultáneamente desde describe en la caída libre: MRU
o MRUA? Argumenta.
la misma altura, ¿qué pasará en su caída? ¿Llegarán al suelo al mismo tiempo?
En este tema, analizarás las características de la caída libre de los objetos. Para
ello, manipularás herramientas tecnológicas que te permitan procesar evidencias
para formular explicaciones científicas.

Indaguemos a partir de nuestros aprendizajes previos Actividad grupal

Objetivo: Comprender las característi- En grupos de tres integrantes, consigan dos hojas de papel con las mismas carac-
cas de la caída libre. terísticas, una pelota maciza pequeña (como una pelota de taca taca o una bolita
Habilidad: Describir las características de cristal) y un celular con cámara. Luego, realicen el siguiente procedimiento:
de un suceso.
1. Uno de los integrantes del grupo, tome ambas hojas de papel y déjelas caer
Actitud: Manipular responsablemente simultáneamente desde la misma altura, mientras que otro integrante graba
herramientas tecnológicas. la caída de las hojas.
Tiempo: 15 minutos.
2. Ahora, arruguen y formen una esfera con una de las hojas de papel. Dejen
caer ambas hojas (una extendida y la otra arrugada) desde la misma altura y
simultáneamente. Graben la caída de las hojas.
3. Arruguen la otra hoja de papel y suelten ambas esferas simultáneamente
desde la misma altura. ¿Qué ocurre? Graben la caída de ambas esferas.
4. Finalmente, uno de ustedes tome la pelota maciza y una de las pelotas de
papel y suéltenlas simultáneamente desde la misma altura. Graben el movi-
miento descrito.
5. Observen las grabaciones de los 4 procedimientos y realicen un cuadro com-
parativo que describa las características del movimiento en cada caso.
Luego, respondan las siguientes preguntas:
Materiales de la actividad. a. Revisen las respuestas entregadas en la páginas 18 y 19 y compárenlas
con estos resultados. ¿Qué generó más curiosidad al revisar los resultados
obtenidos?

Reduce, Reutiliza, Recicla


b. ¿Por qué caen los objetos cuando se sueltan? ¿La masa influye en la caída
¿Qué harán con los materiales de los cuerpos? Argumenten.
luego de utilizarlos?, ¿por qué?

c. ¿Cómo definirían la caída libre?

d. ¿De qué manera la implementación de la cámara les permitió procesar los


resultados obtenidos?

Tal como pudiste experimentar en la actividad anterior, cuando los cuerpos caen
debido a la acción exclusiva de la fuerza de atracción gravitacional terrestre
experimentan una caída libre.

56 Unidad 1 • Movimiento
Para analizar el movimiento de caída de un cuerpo, observa la siguiente imagen
y lee la información asociada a ella.

Cuando se deja caer un cuerpo CONTEXTO HISTÓRICO


desde una determinada altura
Fue Galileo Galilei uno de los prime-
h0
(h 0), su rapidez inicial es igual a
ros científicos que estudió de manera
v0
cero (v0= 0).
formal la caída de los cuerpos. Cuan-
do Galileo realizó sus mediciones,
h1 no existían los cronómetros, por lo
v1
A medida que el cuerpo cae,
su rapidez se incrementa de que, para calcular el tiempo, usó un
forma constante. Esta variación recipiente con agua en el que dejaba
se debe a la acción de la fuerza caer de forma constante una gota.
de atracción gravitacional, que Este instrumento se denomina reloj
h2
acelera los cuerpos a una razón de agua. Para conocer más de él
v2
de 9,8 m/s2, es decir, por cada puedes ir a las páginas 66 y 67.
segundo que transcurre en la
caída del cuerpo, su rapidez se
incrementa en 9,8 m/s, siempre
y cuando el roce con el aire sea
despreciable. La aceleración
de gravedad se designa con
la letra g y su valor es aproxi-
hf
madamente constante, ya que
vf
depende del lugar de la Tierra Ecuaciones de movimiento
donde nos encontremos.
Como la caída libre es un ​​ →
v f
 ​​  = g​​ →  ​​  ‧ Δt + ​​ v→i ​​  

​​ →i ​​  + v
​​ →i ​​  ‧ Δt + ​​ __
1 ​​  ‧​​ g →​​  ‧ Δt2
ejemplo de un MRUA, las
​​ →
y  ​​  = y
ecuaciones de movimiento son f 2
las mismas pero en dirección
vertical por lo que cambiamos
el eje X por el eje Y.

Gráfico de altura en función del tiempo Gráfico de velocidad en función del tiempo

Velocidad (m/s)

Altura (m)

ho vo
Tiempo (s)

hf
Tiempo (s) vf

En este gráfico, la altura de un cuerpo que cae libremente, dis- En este gráfico, la velocidad se incrementa desde cero. La recta
minuye a medida que transcurre el tiempo. Dicha variación no es se encuentra bajo el eje horizontal, debido a que, por el sentido
constante, lo que se representa en el gráfico mediante una curva. del movimiento, la velocidad es negativa.

Física • 2.° Medio 57


A poner en práctica mediante un taller de habilid
ades científicas

Aprendiendo a analizar un experimento clásico:


Galileo y la caída libre
Habilidad: Explicar los resultados de Actitud: Usan tecnologías de la infor-
una investigación utilizando un lengua- mación y comunicación para expresar
je científico apropiado y pertinente. resultados y conclusiones

Reúnanse en grupos tres integrantes. Luego, lean y analicen el siguiente experimento:

Situación problema

En la época de Galileo Galilei, aún persistía la idea propuesta por Aristóteles (casi 18
siglos antes) acerca de la caída de los cuerpos: “mientras más pesado es un cuerpo,
más rápido cae”.
Sin embargo, Galileo desestimó dicha afirmación, ya que observó que todos los cuer-
pos caen exactamente igual, siempre que no haya obstáculos en su movimiento. Con
el fin de conocer en detalle el movimiento de caída de un cuerpo, realizó mediciones
(de tiempo y posición) en la caída vertical. Sin embargo, se percató de que el movi-
miento era demasiado rápido y que sus resultados no eran exactos. Galileo resolvió
este problema al hacer que el movimiento fuera más lento, por lo que, en lugar de
dejar caer un objeto verticalmente, lo hizo rodar por un plano inclinado, asumiendo
que, como en ambos casos la razón del movimiento era el peso del objeto, los dos
movimientos debían ser del mismo tipo.
El problema de investigación de Galileo era determinar una relación matemática que
diera cuenta de la caída de los cuerpos. Para ello diseñó el experimento que se des-
cribe a continuación.

Procedimiento experimental

Galileo hizo rodar una bola de bronce por una rampa, cuya inclinación fue modifican-
do. Luego, midió los tiempos en los que la bola alcanzaba determinadas posiciones.
25 cm
75 cm
125 cm
175 cm
225 cm

0 cm 25 cm 100 cm
225 cm
400 cm
625 cm

0s 1s 2s 3s 4s 5s

Los valores señalados en la imagen corresponden a medidas simuladas.

Después de modificar la inclinación de la rampa y de realizar varias mediciones, en-


contró que ciertas relaciones se repetían, validando de este modo su experimento.

58 Unidad 1 • Movimiento
Organización de los datos

Como las unidades de medida actuales son diferentes a las usadas en el tiempo de Galileo, la
tabla de resultados no contiene los valores originales, sino que es una simulación de algunos
datos basados en la experiencia de Galileo.
Tiempo (s) Posición (cm)
0 0
1 25
2 100
3 225
4 400
5 625

Análisis e interpretación de evidencias

a. Al analizar matemáticamente las variables, Galileo intentó conocer de qué manera se relacio-
naban. Completa la última columna de la tabla y descubre la relación que encontró Galileo.
t = Tiempo (s) t2 =Tiempo al cuadrado (s2) x = Posición (cm) x/t2 (cm/s2)
0 0 0 ---
1 1 25 25/1 = 25
2 4 100
3 9 225
4 16 400
5 25 625

b. ¿Cómo fue el valor obtenido para x/t2?, ¿qué se puede inferir de aquello?
c. ¿Cuál es la importancia que le asignas a la obtención de evidencias en el proceso científico?
d. ¿Qué importancia piensas que tienen investigaciones, como la realizada por Galileo, para la
evolución del conocimiento? Argumenta.

Elaboración de conclusiones

e. ¿Qúe conclusión se puede establecer a partir del experimento realizado por Galileo?

Comunicación de los resultados

Escojan una herramienta TIC para confeccionar un blog, utilizar una red social o la herramienta
tecnológica que más les acomode, para presentar la investigación realizada por Galileo. Para
ello, consideren secciones como título, resumen, introducción, materiales, métodos, resultados
representativos, discusión de los resultados, conclusiones, argumentos y referencias, entre otras.

Desarrollo de la misión
Ahora que conocen las características de la caída libre,
elaboren su prototipo de paracaídas y graben un video
que muestre la caída del huevo. Midan el tiempo que
tarda en caer. ¿Qué dificultades encontraron al mo-
mento de realizar esta misión?

Física • 2.° Medio 59


Lección 3

Tema 2 ¿Qué características posee el lanzamiento vertical?


La mayoría de nosotros, en algún momento, hemos lanzado un objeto, como
¿Cómo debe ser la trayectoria
una pelota en algún juego o deporte, un dado o incluso una moneda, pero ¿qué que sigue un cuerpo lanzado
características tiene este tipo de movimiento? verticalmente? Menciona un
ejemplo.
En este tema, analizarás las características del lanzamiento vertical: hacia arriba
y hacia abajo. Para ello, emplearás los aprendizajes adquiridos en las lecciones
anteriores para formular explicaciones científicas a diversas situaciones cotidianas
con la ayuda de herramientas tecnológicas.

Indaguemos a partir de nuestros aprendizajes previos Actividad grupal

Objetivo: Identificar las caracterís- En parejas, consigan una pelota pequeña y un celular con cámara. Luego, salgan
ticas del lanzamiento vertical. al patio del colegio y realicen la siguiente actividad:
Habilidad: Utilizar herramientas 1. Uno de ustedes lance la pelota de forma vertical hacia arriba, procurando que
tecnológicas para analizar las
características de un suceso.
describa una trayectoria vertical.
Actitud: Manipular responsable- 2. El otro miembro del grupo, grabe el movimiento que describe la pelota cuan-
mente herramientas tecnológicas. do se lanza hacia arriba.
Tiempo: 25 minutos. 3. Repitan los pasos anteriores procurando cambiar la velocidad con que lanzan
la pelota.
4. Analicen las características que describe el movimiento de la pelota en cada
caso, reproduciendo el video las veces que consideren necesarias. Luego,
respondan la siguientes preguntas:
a. ¿Qué ocurre con la velocidad de la pelota a medida que sube?
Fundamenten.

b. ¿Qué ocurre con la velocidad de la pelota cuando comienza a bajar?


Argumenten.

c. ¿Qué relación existe entre la velocidad inicial y la altura que alcanza la


pelota?

d. ¿Qué tipo de movimiento se describe en el lanzamiento vertical: MRU o


MRUA? Fundamenten.

e. ¿De qué manera el uso de la tecnología les permite analizar las caracterís-
ticas de los movimientos?
Representación de la actividad.

El lanzamiento vertical, ya sea hacia arriba o hacia abajo, se caracteriza por ser
un movimiento rectilíneo uniformemente acelerado, que siempre posee una ve-
locidad inicial distinta de cero, en el cual interactúa la aceleración de gravedad.

60 Unidad 1 • Movimiento
Para analizar el lanzamiento vertical de un cuerpo, observa la siguiente situación
y analiza la información asociada a cada imagen. ¿Cómo influye la aceleración de
gravedad en el movimiento de la
moneda?

Cuando se lanza verticalmente hacia arriba una moneda, podemos observar cla-
ramente que esta sube hasta llegar a su altura máxima y luego baja hasta volver
a su posición inicial, lo cual se puede representar en el siguiente gráfico.
Gráfico de posición en función del tiempo
y (cm)

y máx

t (s)
0 0,1 0,2 0,3 0,4 0,5 0,6 0,7

Pero, ¿qué ocurre con la velocidad durante el recorrido de la moneda? En este


caso, la moneda comienza su movimiento con una velocidad inicial (positiva) que
genera el movimiento hacia arriba. Producto de la acción de la gravedad, la mo-
neda comienza a disminuir su velocidad hasta que se detiene en el punto más
alto, instante en el que comienza a descender describiendo una caída libre, tal
como se representa en el siguiente gráfico.
Gráfico de velocidad en función del tiempo
v (m/s)

vi

0
0,1 0,2 0,3 0,4 0,5 0,6 t (s)

vf

¿Qué representa el área entre la curva y el


eje x de un gráfico de velocidad en función
del tiempo?

Física • 2.° Medio 61


Lección 3

Desarrollo de estrategias
Aprendiendo a elaborar gráficos.

Situación problema

En una prueba de balística, un grupo de investigadores desea determinar el tiem-


po que tarda un proyectil en impactar el suelo. Para ello, en su primera prueba
lanzan el proyectil desde una altura de 140 m y con una velocidad inicial de
-15 m/s. A partir de esta información, construye el gráfico de posición en función
del tiempo del proyectil y determina el tiempo que tarda en llegar al suelo.

PASO 1 Identifico los datos

yi = 140 m
vi = –15 m/s
g = –10 m/s2

PASO 2 Construyo una tabla de valores


Para determinar la posición del proyectil en cada instante de tiempo, es necesario
determinar la ecuación itinerario. En este caso:
y = 140 – 15t – 5t2
De esta manera, obtenemos los siguientes valores:
Tiempo (s) 0 1 2 3 4
Posición (m) 140 120 90 50 0

PASO 3 Construyo el gráfico


Para elaborar el gráfico, se debe construir un plano cartesiano, donde el eje x repre-
sente el tiempo y el eje y la posición del proyectil. Luego, se ubican los datos de la
tabla de valores.
Gráfico de posición en función del tiempo

160
140
120
Posición (m)

100
80
60
40
20
0
0 1 2 3 4 5
Tiempo (s)

PASO 4 Escribo la respuesta

El proyectil se demora 4 segundos en llegar al suelo.

PASO 5 Aplico lo aprendido


Construye el gráfico de velocidad en función del tiempo del proyectil. ¿Con qué
velocidad impacta el proyectil el suelo?

62 Unidad 1 • Movimiento
CIENCIA, TECNOLOGÍA Y SOCIEDAD

En la actualidad, existen diversos deportes extremos cuya


práctica de alto riesgo involucra la caída de los cuerpos con el
propósito de generar sensaciones y emociones que producen
adrenalina. Entre estas prácticas encontramos el paracaidismo
o el wingsuit.

¿Qué medidas de seguridad consideras


que hay que tener en cuenta para prac-
ticar estos deportes?

¿De qué manera el estudio de los mo-


vimientos ha permitido el desarrollo de
nuevas tecnologías?

ión
zac
b eti ífica
a t
Alf cien

Cierre de la misión
Al comienzo de la lección te planteamos confeccionar un
paracaídas para que un huevo descendiera sin quebrarse. ¿Esta misión se incluirá en el proyecto del texto?
¿Cumpliste la misión? ¿Qué ocurrió? ¿Cuánto tiempo tardó Fundamenta en la página 12.
en caer el huevo?
¿De qué manera el uso de las herramientas tecnológicas te
permitió analizar el trabajo realizado?

Reflexiono sobre lo que aprendí


Lee y comenta las siguientes preguntas con tus compañeros y compañeras para conocer si alcanzaste
el propósito de la lección.
Contenido Habilidades/Estrategias Actitudes
¿Consideras que podrías aplicar las ecuacio- ¿Qué estrategias utilizaste para in- ¿Cómo el uso de las TIC permite expresar ideas,
nes del movimiento para describir cualquier terpretar y confeccionar gráficos? resultados o conclusiones?
situación cotidiana? Fundamenta. ¿De qué manera las habilidades ¿Crees que las nuevas tecnologías contribuyen
¿Qué característica(s) de la aceleración de científicas se pueden potenciar con a la construcción de nuevos conocimientos o al
gravedad aprendiste en esta lección? el uso de la tecnología? perfeccionamiento de los ya existentes?

Física • 2.° Medio 63


Integro lo que aprendí
Evaluación de proceso

Interpreta
1 En un libro de física, Mariana encuentra los siguientes gráficos:
Gráfico de posición-tiempo Gráfico de posición-tiempo Gráfico de posición-tiempo
Posición (m)

Posición (m)

Posición (m)
Tiempo (s) Tiempo (s) Tiempo (s)

Describe el movimiento que se representa en cada uno de ellos, identifican-


do, además, el tipo de movimiento y sus características.
Aplica
2 Para un proyecto de una feria científica de su escuela, Cristian está diseñan-
do el modelo de un cohete a propulsión hidráulica que se eleve verticalmen-
te. Para ello, desea saber cuál debe ser su velocidad inicial de modo que
alcance una altura determinada.
a. ¿Qué modelo matemático debería emplear Cristian? Fundamenta.
b. Si la altura mínima que desea que alcance su cohete es de 45 metros,
¿cuál debe ser su velocidad inicial?
c. Construye los gráficos de posición y velocidad en función del tiempo para
el cohete cuando alcanza una altura de 45 metros.
Posición inicial Velocidad media

Aplica
2 Susana deja caer una pelota desde cierta altura. Si sabe que esta tarda exac-
tamente 1,5 s en llegar al suelo, ¿cuál será su velocidad al momento de im-
pactar sobre él?
Evalúa

3 A un estudiante se le cae la goma al piso desde una mesa de 1 metro de alto.


Al darse cuenta de esta situación, afirma que si la goma hubiera caído desde la
mitad de la altura (0,5 m), entonces esta tardaría la mitad del tiempo en caer. ¿Es
correcta esta afirmación? Argumenta.

64 Unidad 1 • Movimiento
Aplica
4 Javiera vive en un edificio que posee un ducto para la
basura (shaft de basura) y cada vez que deja caer una
bolsa, cuenta los segundos que esta tarda en impac-
tar el suelo, con el objetivo de determinar a la altura
en que se encuentra. A partir de esta información ¿qué
método le puedes proponer a Javiera para que deter-
mine la altura?, ¿qué variables debe considerar?

Sintetiza
5 Completa la siguiente tabla representando los graficos de posición, veloci-
dad y aceleración de la caída libre y del lanzamiento vertical.

Caída libre Lanzamiento vertical

¿Cómo voy?
Revisa tus respuestas y, según los resultados que hayas obtenido, marca con ✓ el nivel de desempeño
correspondiente. Si es necesario, pídele ayuda a tu profesor o profesora.
Indicador Ítem Habilidad Nivel de desempeño
Utilicé los distintos parámetros 1, 2 y 3 Interpretar y aplicar. L: Tres ítems correctos.
que describen los movimientos
verticales. ML: Dos ítems correctos.
PL: Uno o ningún ítem correcto.
Examiné diversas situaciones que 4y5 Evaluar y sintetizar. L: Dos ítems correctos.
describen un movimiento vertical y
su representación gráfica. ML: Un ítem correcto.
PL: Ningún ítem correcto.
L = Logrado; ML = Medianamente logrado; PL = Por lograr.

Reflexiono sobre mi desempeño


Según tu apreciación (1: en desacuerdo; 2: ni de acuerdo ni en desacuerdo; 3: de acuerdo), marca con
✓ las siguientes afirmaciones:
1 2 3
He descrito correctamente los sucesos relacionados con los movimientos verticales.
He aplicado nuevas estrategias para afrontar los nuevos desafíos.
He trabajado responsablemente en las actividades propuestas.

Física • 2.° Medio 65


La CIENCIA se construye

¿Cómo ha evolucionado el reloj a lo largo de la historia?


Cuando hablamos de movimiento, nos referimos a un cambio de lugar o de posición de
un cuerpo a lo largo del tiempo y respecto del un sistema de referencia. En todo mo-
mento, se entendía que la medición del tiempo se hacía mediante un reloj o cronóme-
tro. Sin embargo, en la Antigüedad los científicos no contaban con estos instrumentos.
Te invitamos a leer la siguiente información que te ayudará a conocer cómo se ha
realizado la medición del tiempo a lo largo de la historia.

Reloj de Sol Reloj despertador


(Siglo XXX a. de C.) (Siglo XVIII)
El reloj de Sol, también llamado Reloj de arena En el 1787, Levi Hutchins añadió
cuadrante, fue creado en el anti- (Siglo XVI) una campanilla a su reloj, la cual
guo Egipto y perfeccionado por los Este reloj posee dos recipientes se activaba cuando lle-
griegos. Está compuesto de una de cristal unidos por un estrecho gaba a una hora deter-
varilla que produce sombra sobre canal por donde pasa lentamen- minada, dando origen
una superficie plana. La proyección te la arena. Debido al reloj despertador.
de la sombra marca la posición del a su mecanismo, no
Sol en un determinado mo- es posible medir
mento, y así se puede tener lapsos de tiempo ¿Utilizas algún re
loj
una idea bastante aproxi- prolongados. despertador?, ¿c
uál?
mada de la
hora.

Reloj mecánico
(Siglo XVII)
Reloj de agua Este reloj está compuesto por una
(Siglo XV a. de C.) varilla metálica con un adorno en
su parte inferior que, con sus os-
También denominado clepsidra, cilaciones, regula el movimiento.
diseñado con el objetivo de saber
la hora durante la noche o los días
nublados. Constaba de dos reci-
pientes: a partir de la regularidad
lusiones
con que ascendía el nivel de agua ¿Recuerdas qué conc
rie ncia?
dentro de uno de los recipientes obtuvo en esa expe
se podía medir el transcurso del
tiempo. Este mecanismo le per-
mitió a Galileo Galilei (1564–1585)
registrar el tiempo de caída de un
objeto en un plano inclinado.

66 Unidad 1 • Movimiento
Reloj de pulsera
(Siglo XIX) Reloj atómico
El reloj de pulsera se creó para (Siglo XX) ¿Cuál h
abrá sid
de crea oe
r un relo l propósito
ser usado en aviación, debido a En 1955, en el Laboratorio Na-
que los pilotos de las aeronaves cional de Física (Reino Unido), se j atómic
o?
utilizaban un reloj de bolsillo construyó el primer reloj atómico
atado a sus piernas o brazos exacto, el más preciso del mundo.
para medir el tiempo de vuelo y Funciona con átomos que oscilan
realizar ciertos cálculos. En 1910 de forma natural a una determina-
este invento comenzó a venderse da frecuencia (expresada en hertz).
entre el público en general. Así
nació el primer cronómetro de
mano o reloj pulsera, tal como los
conocemos hoy.

Reloj inteligente
Reloj digital (Siglo XXI)
(Siglo XX) También conocidos como “smar-
Un fabricante de relojes suizo creó twatch”, funcionan de manera
y fabricó un modelo de reloj similar a un teléfono
mecánico-digital en 1956 inteligente, incluso su
tras descubrir la posibilidad funcionalidad se com-
de transferir las vibraciones para con las computa-
regulares del cristal de cuar- doras modernas.
zo a las manecillas del reloj.
Su difusión crece a media-
dos del siglo XX gracias a los
avances de la informática.
Este tipo de reloj permite
establecer con enorme
precisión las centésimas y
las milésimas de segundos.

Trabaja con la información

1 ¿Qué opinas sobre las contribuciones realizadas para la creación del reloj?
2 ¿Cuáles son los beneficios de disponer de un instrumento como el reloj? Fundamenta.
3 ¿Crees que se sigan realizando investigaciones científicas acerca de los relojes? Fundamenta.
4 ¿Te diste cuenta de que los personajes mencionados son todos hombres?, ¿qué piensas que
ocurre con el aporte de las mujeres en esta área?

Física • 2.° Medio 67


Síntesis
Utilizando un ÁRBOL GRÁFICO para
ordenar los aprendizajes

El árbol gráfico es un organizador que se utiliza para orde-


nar los contenidos y sus detalles a partir de un tema princi-
pal. En este organizador, se escribe el tema principal en el

Det
tronco del árbol y los subtemas y detalles en las ramas, tal

alle
como se muestra en la figura.

a
Sub
A continuación, te invitamos a conocer los pasos para cons-

em
bt
tem
truir un árbol grafico que te servirá como una entretenida Subt a

Su
e tem
Detall e ma
Sub

a
estrategia de estudio.
Tema
principal

le
tal

De
De

tal
le
Definir el tema principal
1
PASO Define el concepto central que engloba todos los contenidos tratados en la unidad:

Identificar los subtemas


2
PASO Pueden ser conceptos, definiciones, características, expresiones matemáticas o lo
que te parezca apropiado según el contexto. En este caso, los subtemas se ubican
en las ramas del árbol y pueden ser:

Posición Velocidad Aceleración MRU MRUA


(Página 25) (Página 29) (Página 47) (Página 40) (Página 46)

Relatividad Desplazamiento Distancia Lanzamiento vertical


(Página 30) (Página 27) (Página 27) (Página 60)

Caída libre Sistema de referencia Rapidez Trayectoria


(Página 56) (Página 25) (Página 28) (Página 27)

Relacionar los subtemas con ciertos detalles de manera gráfica


3
PASO Para concluir con la construcción del árbol, incluye esquemas, gráficos o detalles
que te permitan relacionar los subtemas con los contenidos, habilidades y acti-
tudes presentes en la unidad. Por ejemplo, si el subtema es la función itinerario,
podemos incluir una hoja del árbol en la cual aparezca la función que permite
determinar la posición de un cuerpo que se mueve con aceleración constante y,
además, el gráfico de posición en función del tiempo.

68 Unidad 1 • Movimiento
Construye el árbol gráfico
4
PASO Completa el siguiente árbol gráfico representando los aprendizajes adquiridos en la
unidad. Puedes incorporar más ramas si lo consideras necesario y ten presente que:
✓ El tema principal se ubica en el tronco del árbol.
✓ En las ramas del árbol deberán ir los subtemas o contenidos relevantes.
✓ En las hojas y las raíces del árbol se detallan los subtemas por medio de
esquemas o gráficos.

Gráfico de
aceleración-tiempo
Trabajo
riguroso
a (m/s2)

t (S)
Interés
Gráfico de
posición-tiempo
a=
x (m)

Pos MO
t (S) ición VIM
IEN
T
UN O RE
IFO C
Gráfico de
te RM TILIN
velocidad-tiempo
v=c E EO
v (m/s)

t (S)

s Ap
f ico lic
á ac
gr ió
de n
de
sis
áli m
od
An elo
s

Reflexiona
5
PASO ✓ ¿Por qué es importante mantener un orden para elaborar un árbol gráfico?
✓ ¿Qué herramienta tecnológica implementarías para confeccionar un árbol gráfico?
✓ ¿En qué situaciones te sería útil emplear este organizador gráfico?

Física • 2.° Medio 69


Evaluación final

Para que conozcas cómo va tu proceso de aprendizaje, te invitamos a realizar las siguientes actividades.

1 Para profundizar los conceptos estudiados en clases de Física, Loreto y Maximiliano decidieron
analizar el movimiento de un cuerpo que se desliza por un plano inclinado, tal como se representa
en la siguiente imagen:

Para ello, ubicaron un bloque de 1,5 kg de masa sobre una superficie de madera. Ambos ob-
servaron que este comenzó a deslizarse en línea recta sobre la superficie, con una rapidez que
aumentaba paulatinamente.
Explica
a. ¿Qué tipo de movimiento experimenta el bloque MRU o MRUA?
b. Menciona aquellos conceptos estudiados en la unidad y que se encuentran asociados a la
experiencia realizada por Loreto y Maximiliano.
2 De la experiencia anterior, Loreto y Maximiliano construyeron el siguiente gráfico de velocidad en
función del tiempo:
Gráfico de velocidad-tiempo

v (m/s)

2 t (s)

-5,5

Analiza
a. ¿Por qué la pendiente de la recta es negativa?
b. ¿Cómo debería ser el gráfico aceleración-tiempo? Haz un bosquejo.

70 Unidad 1 • Movimiento
Aplica
c. ¿Cuál es el módulo de la aceleración del bloque?
d. Considerando los valores del gráfico, ¿qué distancia recorre el bloque entre los 0 y los 2 s?
Evalúa
e. ¿Qué variable(s) debería(n) modificar Loreto y Maximiliano para que el bloque se mantenga
inmóvil? Explica.
f. Si se utilizara un bloque del mismo material pero de mayor masa, ¿qué variable(s) se vería(n)
afectada(s) al realizar nuevamente el experimento?
Aplica
3 Un ciclista que se encuentra detenido en la luz roja de un semáforo, se pone en movimiento al
cambiar la luz a verde. A continuación sigue una trayectoria rectilínea y tarda 10 s en alcanzar una
velocidad de 8 m/s. A partir de esta información:
a. Determina la aceleración media del ciclista durante los 10 s de movimiento.
b. Calcula la distancia que recorre el ciclista en los 10 s que acelera.
c. Si después de los primeros 10 s el ciclista continúa moviéndose con velocidad constante, ¿qué
distancia recorre en los siguientes 20 s?
d. ¿Cuál es la distancia total que recorrió en los 30 s de los cuales hay información?
e. Construye el gráfico de velocidad en función del tiempo del movimiento descrito por el ciclista
durante los 30 s.

Gráfico de velocidad en función del tiempo

f. ¿Qué estrategias empleaste para confeccionar el gráfico?

Aplica
4 Un transbordador de autos se desplaza de forma rectilínea sobre el agua calma a 20 km/h. Desde
la parte trasera del transbordador una persona camina, con una rapidez de 2 km/h respecto del
vehículo, en dirección contrario a la del movimiento del transbordador para hacerle una pregunta
al capitán.
a. ¿Cuál es la rapidez de la persona respecto del agua?
b. ¿Cuál es la rapidez del agua respecto de la persona?
c. ¿Cuál es la rapidez del transfer respecto a la persona?

Física • 2.° Medio 71


Evaluación final
Analiza
5 Un astronauta que se encuentra parado en un acantilado de un satélite natural lanza verticalmen-
te hacia arriba un objeto. El gráfico de dicho movimiento se representa a continuación:
Gráfico de velocidad-tiempo

v (m/s)

0
1 3 t (s)

-6

A partir de la información contenida en el gráfico, responde:


a. ¿Con qué rapidez salió el objeto de la mano del astronauta?
b. ¿En qué instante el objeto alcanzó la altura máxima?
c. ¿Qué distancia recorrió el objeto en los 3 s?
d. ¿Cuál fue el desplazamiento del objeto durante los 3 s?
e. ¿Qué herramienta tecnológica utilizarías para elaborar gráficos?
Aplica
6 Carolina se mueve en su patineta en línea recta hacia la derecha y sin cambiar su rapidez, alcan-
zando a recorrer 10 m en 8 s. A partir de esto, responde:
a. ¿Qué tipo de movimiento describe Carolina? Justifica.
b. ¿Cuál es la rapidez media de Carolina? ¿Cuál es su velocidad media?, ¿en qué se diferencian
ambas?
c. ¿Cuál es su ecuación itinerario si parte moviéndose a 7 metros del inicio del sistema coordenado?
d. ¿En qué posición se encontrará a los 24 segundos si continúa moviéndose de la misma manera?
e. ¿Qué medidas de seguridad debe considerar Carolina para utilizar la patineta?
Aplica
7 Un jugador de vóleibol golpea una pelota verticalmente hacia arriba y logra transmitirle una ve-
locidad inicial de 30 m/s. Con esta información, completa la siguiente tabla y luego responde las
preguntas asociadas.
yi ymáx vi g tsubida

a. ¿Cuál es la ecuación itinerario de la pelota?


b. ¿En qué instante de tiempo la pelota alcanza su altura máxima?
c. ¿Cómo son los gráficos de posición y velocidad en función del tiempo? Constrúyelos en tu
cuaderno.

72 Unidad 1 • Movimiento
Relaciona lo aprendido con Artes Visuales
La persistencia de la memoria, también
conocida como “Los relojes blandos” o
“Los relojes derretidos” es un famoso
cuadro del pintor español Salvador Dalí
pintado en 1931. ¿Qué crees que quiso
representar el artista con los relojes de-
rretidos? Elabora una hipótesis conside-
rando todo lo que aprendiste sobre el
movimiento y la medición del tiempo.

¿Cómo me fue?
Revisa tus respuestas y, según los resultados que hayas obtenido, marca con ✓ el nivel de desempeño
correspondiente. Pídele ayuda a tu profesor o profesora.

Indicador Ítem Habilidad Nivel de desempeño


Identifiqué los parámetros aso- 2 (c y d), 3, 4, Analizar y aplicar. L: Cinco o seis ítems correctos.
ciados al movimiento y apliqué 5, 6 y 7
las relaciones matemáticas que lo ML: Tres o cuatro ítems correctos.
describen. PL: Menos de tres ítems correctos.
Analicé las características de 1 y 2 (a, b, Explicar y evaluar. L: Dos ítems correctos.
los movimientos en situaciones e y f)
cotidianas. ML: Un ítem correcto.
PL: Ningún ítem correcto.
L = Logrado; ML = Medianamente logrado; PL = Por lograr.

Reflexiono sobre lo que aprendí


¿Recuerdas las metas y estrategias planteadas al inicio de la unidad? Si no las recuerdas, vuelve a
revisar las páginas 20 y 21. Luego, reflexiona en torno a las siguientes preguntas.
¿Por qué es importante analizar las ¿Cómo desarrollaste la estrategia planteada al ¿De qué manera lograste valorar
características de los movimientos? inicio de la unidad para el logro de tus metas? el conocimiento científico?

Física • 2.° Medio 73


Fuerzas
Propósito de la unidad
La presente unidad tiene como hilo conductor el concepto de evidencias a partir de experiencias prácticas que permiten,
fuerza, de manera que las actividades que se desarrollan en el entre otras cosas, desarrollar procesos científicos asociados
Texto se articulan de acuerdo con los contenidos, habilida- a las características de las fuerzas para explicar el comporta-
des y actitudes que se derivan del estudio de la dinámica del miento de algunos materiales, tanto en su deformación como
movimiento. La principal finalidad es que los y las estudiantes en su uso en diversos dispositivos tecnológicos.
analicen los efectos que provocan las fuerzas sobre los cuerpos Además, la unidad de la Guía didáctica tiene como propó-
en los que actúan, así como las leyes de la naturaleza que se sito apoyar, desde la labor docente, la adquisición de los
relacionan con ellas. Se espera que comprendan los principios aprendizajes, habilidades y actitudes por parte de los y las
70
de Newton y los apliquen para explicar diversas situaciones estudiantes. Para ello, se entregan una serie de orientacio-
cotidianas en las que participan las fuerzas. En particular, las nes didácticas, actividades complementarias e instancias de
y los estudiantes deberán utilizar correctamente un diagrama apoyo para la evaluación. Por ello, para la presente unidad
de cuerpo libre para obtener la fuerza neta que actúa en un del Texto del estudiante y de la Guía didáctica, se espera pro-
cuerpo. También se procura que analicen y apliquen la ley de mover y apoyar el desarrollo de los siguientes contenidos,
Hooke para explicar el comportamiento de materiales elásticos. habilidades, actitudes, Objetivos Fundamentales Transver-
Respecto de las habilidades de investigación científica, sales y grandes ideas de las ciencias.
en esta unidad se fortalece el análisis y procesamiento de

Contenidos Habilidades de Investigación Científicas (HIC)


• Características de las fuerzas. En esta unidad aplicarán las siguientes habilidades de inves-
• Fuerza neta como resultado de la suma de fuerzas tigación científicas:
sobre un cuerpo. • Observar y plantear preguntas. (HIC 1)
• Tipos de fuerza comunes, como peso, normal, tensión, • Planificar y conducir una investigación. (HIC 2)
fuerza de roce y fuerza elástica. • Procesar y analizar la evidencia. (HIC 3)
• Ley de Hooke. • Evaluar una investigación. (HIC 4)
• Diagrama de cuerpo libre. • Comunicar una investigación. (HI 5)
• Primera ley de Newton o principio de inercia.
• Segunda ley de Newton o principio de masa.
Guía Didáctica del Docente

• Tercera ley de Newton o principio de acción y reacción.


• Situaciones cotidianas que se explican a partir de los
principios de Newton.

Unidad 2 • Fuerzas
Objetivos Fundamentales Transversales (OFT)
Los OFT integran las actitudes y valores con el desarrollo de conocimientos y habilidades. En la unidad
se promueve el logro de los siguientes:
• Proactividad y trabajo. (OFT 2)
• Dimensión física y Dimensión moral. (OFT 6)
• Dimensión sociocultural y ciudadana. (OFT 8)

Actitudes
Los aprendizajes involucran, además de la dimensión cognitiva, actitudes que contemplan el desarrollo
en los ámbitos personal, social, ético y ciudadano que derivan de los OFT. En las actividades propuestas
se promueven las siguientes actitudes:
• Esforzarse y perseverar en el trabajo personal entendiendo que los logros se obtienen solo después
de un trabajo riguroso, y que los datos empíricamente confiables se obtienen si se trabaja con
precisión y orden. (OA B)
• Demostrar valoración y cuidado por la salud y la integridad de las personas, evitando conductas de
riesgo, considerando medidas de seguridad y tomando conciencia de las implicancias éticas de los
avances científicos y tecnológicos. (OA F)
• Demostrar valoración e interés por los aportes de hombres y mujeres al conocimiento científico y
reconocer que desde siempre los seres humanos han intentado comprender el mundo. (OA H)

Grandes ideas
• El movimiento de un objeto depende de las interacciones en que participa. (GI 7) 71

Organización de los contenidos de la unidad del Texto del estudiante


Los contenidos de la unidad se organizan en dos lecciones, con el fin de las y los estudiantes comprendan
las características de las fuerzas y los efectos que provocan en los cuerpos. El siguiente esquema muestra,
en una panorámica general, cómo se organizan los contenidos en la unidad del Texto del estudiante.

Fuerzas

Lección 1: Comprendiendo Lección 2: Análisis de los


las fuerzas y sus efectos movimientos verticales

Características Identificando las fuerzas Estableciendo la


generales de las fuerzas en la vida cotidiana primera ley de Newton

Aplicando la segunda
Guía Didáctica del Docente
ley de Newton

Estableciendo la tercera
ley de Newton

Física • 2.º medio


Planificación de la unidad Tiempo estimado: 18 horas pedagógicas

La siguiente propuesta de planificación considera las Actitudes, OFT, Aprendizajes Esperados (AE), Ob-
jetivos de Aprendizaje (OA) y los Indicadores de Evaluación (IE) asociados a cada uno de ellos, que se
desarrollan en cada lección del Texto del estudiante.

Lección Tiempo Actitudes OFT Aprendizajes Esperados

1 8 Esforzarse y perseverar en el Proactividad y trabajo. Comprender los efectos que tiene una
trabajo personal entendiendo fuerza neta sobre un objeto, utilizando
Dimensión física y Dimen-
que los logros se obtienen solo el diagrama de cuerpo libre.
sión moral.
después de un trabajo riguroso,
Explicar situaciones cotidianas por
y que los datos empíricamente Dimensión sociocultural y
medio de las leyes de Newton.
confiables se obtienen si se tra- ciudadana.
baja con precisión y orden.
Demostrar valoración y cuidado
por la salud y la integridad de
las personas, evitando conductas
2 10 de riesgo, considerando medi-
das de seguridad y tomando
conciencia de las implicancias
éticas de los avances científicos y
tecnológicos.
Demostrar valoración e interés
por los aportes de hombres y mu-
jeres al conocimiento científico
y reconocer que desde siempre
72 los seres humanos han intentado
comprender el mundo.

Notas:
Guía Didáctica del Docente

Unidad 2 • Fuerzas
ad

unid
2

Objetivos de Aprendizaje Indicadores de Evaluación

Explicar, por medio de investigaciones experimentales, los IE 1. Identifican una fuerza como la interacción entre dos cuerpos y
efectos que tiene una fuerza neta sobre un objeto, utilizando su carácter vectorial, entre otras características.
las leyes de Newton y el diagrama de cuerpo libre. (OA 10)
IE 2. Realizan investigaciones experimentales para obtener eviden-
cias de la presencia de fuerzas, como peso, roce y normal, que actúan
sobre un cuerpo, en situaciones cotidianas, describiéndolas cualitati-
va y cuantitativamente.
IE 3. Aplican las leyes de Newton en diversas situaciones cotidianas,
como cuando un vehículo frena, acelera o cambia de dirección su
movimiento, entre otras.
IE 4. Encuentran, con un diagrama de cuerpo libre, la fuerza neta o
resultante sobre un objeto en el que actúa más de una fuerza.
IE 5. Analizan el efecto que provoca la fuerza neta o resultante en el
movimiento de un objeto.
IE 6. Aplican la ley de Hooke en diversas investigaciones experi-
mentales y no experimentales en las que se utilizan resortes u otros
materiales elásticos.

73

Notas:

Guía Didáctica del Docente

Física • 2.º medio


Inicio de la unidad Páginas 74 a 79

¿Qué efectos provocan las fuerzas sobre los cuerpos? Motivación para el aprendizaje
Páginas 74 y 75
“La opinión generalizada de los
Para comenzar con el estudio de las fuerzas, en estas páginas se presenta una alumnos y alumnas es que lo
situación en la que se observan tres deportistas realizando diferentes actividades: aprendido en la escuela poco tiene
levantamiento de pesas, acrobacias en argollas y salto en caballete. A partir de la que ver con sus vidas, sus intereses,
conversación que entablan los espectadores de la ilustración, se espera activar los preocupaciones e inquietudes. Los
conocimientos previos de los y las estudiantes, de manera que sean parte del conocimientos que se enseñan en
diálogo. Para ello, solicite que, a partir de sus conocimientos sobre las fuerzas y la la escuela son, en la mayor parte
información que se obtiene de la imagen, respondan las preguntas de la actividad de los casos, muy teóricos, ale-
individual. jados de la realidad y con pocas
posibilidades de aplicación; es lo
que los expertos denominan "co-
¿Cómo podemos identificar las fuerzas? nocimiento inerte", y tiene efectos
Páginas 76 y 77 muy negativos sobre la motivación.
Por tanto, si queremos estimular en
Para seguir indagando en los aprendizajes previos de las y los estudiantes, en estas los alumnos y alumnas el deseo de
páginas se muestra una situación cotidiana, en la que un grupo de mujeres se en- aprender, lo primero que tenemos
cuentra practicando básquetbol. A partir de esta imagen, se espera que respondan que hacer es tratar de relacionar
las preguntas asociadas, de manera que logren identificar las características de las lo que enseñamos en las escuelas
fuerzas y sus efectos, como resultado de su aparición por la interacción entre dos con el mundo real, es decir, darle
o más cuerpos. un sentido, un significado y una
Es importante destacar que, en esta instancia inicial, todas las respuestas son váli- utilidad a lo que se enseña”.
das, ya que se trata de una etapa de exploración, de manera que más que centrarse A. Valle Arias (2007)
74
en lo correcto o incorrecto, las y los estudiantes deben expresar sus ideas en un
escenario favorable para el aprendizaje.
Guía Didáctica del Docente

Evaluación de conocimientos previos

Utilice estas actividades para evaluar los conocimientos previos de sus estudian-
tes. Para ello, solicite que respondan individualmente las preguntas planteadas;
luego, las y los estudiantes que lo deseen pueden exponer sus respuestas para
analizar las situaciones descritas.

Unidad 2 • Fuerzas
ad

unid
2
¿Qué aprenderás y descubrirás en la unidad? Metacognición
Página 78
Considere lo que propone Anna
Esta sección entrega a las y los estudiantes las principales metas que se espera Lucía Campos (AÑO) sobre el ce-
que alcancen en la unidad, las cuales se enfocan en los aprendizajes esperados rebro y el aprendizaje, para plani-
propuestos. Para ello, en ¿Cómo alcanzarlas?, se destacan las principales acciones ficar sus estrategias de enseñanza.
que deberán realizar para el logro de los aprendizajes, entre las cuales se encuen- El cerebro establece una ruta para
tran las siguientes: realizar actividades prácticas, aplicar modelos, diseñar afiches, el aprendizaje: “Si hacemos un re-
realizar diagramas. Por último, en ¿Para qué alcanzarlas?, se señala la importancia sumen sencillo de las principales
que tiene el logro de los aprendizajes propuestos, teniendo en cuenta las actitudes investigaciones relacionadas con
que se promueven en la unidad. el proceso de aprendizaje, pode-
Para trabajar esta sección, solicite a sus estudiantes que completen la información mos ver que el cerebro para apren-
solicitada estableciendo sus propias metas para la unidad, considerando las estra- der necesita percibir y codificar
tegias y habilidades que emplearán para lograrlas y el propósito que cada una de una información (input) y para
ellas estimula. ello utiliza sus recursos multisen-
soriales, el cuerpo, la motivación
y todos los conocimientos previos
¿Cómo te gustaría protagonizar tu propio aprendizaje? almacenados en un sistema de me-
Página 79 moria en especial. A partir de allí
se desencadena una serie de acon-
En esta instancia se espera que a partir de las preguntas planteadas, las y los es- tecimientos a nivel neurológico,
tudiantes planifiquen su aprendizaje en función de sus motivaciones personales. como por ejemplo, la activación
Para ello, en este caso particular, el enfoque del trabajo se centra en los aprendi- del mecanismo de atención, que
zajes relacionados con la fuerza y la dinámica del movimiento, de tal manera que permitirá que el alumno procese
la información más relevante ig- 75
se genere la instancia de que definan los contenidos, estrategias y actitudes que
desean desarrollar a lo largo de la unidad. Invite a sus estudiantes a compartir sus norando otros estímulos (externos
motivaciones con el resto del curso y a expresar de manera verbal sus intenciones o internos) y empiece a adquirir
dentro del estudio de las fuerzas. de manera directa o indirecta el
aprendizaje. Para ello, los recursos
manipulativos, los materiales con-
¿Cómo lograr mis metas? cretos, todas las estrategias, méto-
Página 79 dos, procedimientos y actividades
variadas van a permitir que el
En esta oportunidad se les propone a las y los estudiantes planificar el desarrollo nuevo aprendizaje sea adquirido y
de un glosario temático como estrategia de estudio para los aprendizajes que se se desarrollen nuevas conexiones
trabajarán en la unidad. Para ello, guíelos para establecer los pasos que les permi- sinápticas (y nuevas capacidades).
tirán desarrollar con éxito esta estrategia. Como el aprendizaje se caracteri-
za por la habilidad de adquirir
nuevas informaciones (Gazzani-
ga, 2002) es de fundamental im-
portancia que el educador no solo
propicie verdaderas oportunidades
de entendimiento de la propuesta
de aprendizaje, sino también que
certifique que el alumno la está in- Guía Didáctica del Docente
corporando de manera adecuada.
En esta etapa el maestro debe des-
empeñar un papel básico de me-
diador, marcando así la diferencia
entre los procesos de enseñanza y
aprendizaje”.

Física • 2.º medio


Desarrollo de la unidad Páginas 80 a 109

LECCIÓN 1: Comprendiendo las fuerzas y sus efectos

Las actividades propuestas en el Texto del estudiante tienen como propósito el logro de los Aprendizajes
Esperados (AE), abordando con diferentes estrategias los Indicadores de Evaluación (IE) e incorporando
en cada una de ellas el trabajo con habilidades, actitudes y Objetivos Fundamentales Transversales (OFT).
AE IE Actividad Habilidad Actitud/OFT
Comprender los IE 1 Inicio de la misión (P. 81) HIC 2 OA B-F/OFT 2-6
efectos que tiene IE 1 Indaguemos (P. 82) HIC 1-2-3 OA B-F/OFT 2-6
una fuerza neta
sobre un objeto, uti- IE 4-5 Reflexiono sobre la fuerza neta (P. 85) HIC 1 OA B-F/OFT 2-6
lizando el diagrama IE 2-6 Indaguemos (P. 86) HIC 1-2-3 OA B-F/OFT 2-6
de cuerpo libre.
IE 2 Aplico los tipos de fuerzas (P. 87) HIC 3 OA B-F/OFT 2-6
IE 2 Desarrollo de estrategias (P. 88) HIC 3 OA B-F/OFT 2-6
IE 2-6 A poner en práctica (P. 90 y 91) HIC 1-2-3-5 OA B-F/OFT 2-6
IE 1 Desarrollo de la misión (P. 911) HIC 2-5 OA B-F/OFT 2-6
IE 1-4-5 Elaboremos un diagrama de cuerpo libre (P. 92) HIC 3 OA B-F/OFT 2-6
IE 1 Cierre de la misión (P. 93) HIC 4 OA B-F/OFT 2-6
IE 1-2-4-5 Integro lo que aprendí (P. 94 y 95) HIC 1-3 OA B-F/OFT 2-6

Propósito de la lección A continuación, se presenta una serie de orientaciones para


trabajar los temas, actividades y secciones presentes en la
En esta lección se trabajan los conceptos básicos asociados lección del Texto del estudiante. Junto con ello, se incorpo-
76 a las fuerzas. Para ello, se hace un recorrido por los efectos ran actividades e información complementarias, entre otros
que generan las fuerzas; las características de su represen- recursos, en la Guía didáctica.
tación gráfica; la determinación de la fuerza neta y la identi-
ficación de los tipos de fuerza: peso, normal, tensión, roce y
elástica junto con el estudio de la ley de Hooke. Lo anterior Ciencia, tecnología y sociedad P. 80
se desarrolla a partir de los recursos y actividades presentes
en el Texto del estudiante en función de los Indicadores de Para seguir activando los conocimientos previos de las y los
Evaluación considerando los contenidos, habilidades y ac- estudiantes, en esta sección se presenta el texto “La FUERZA
titudes propuestos. de atracción de los agujeros negros”, para desarrollar un aná-
En el Texto del estudiante, las diferentes temáticas se desa- lisis de la ciencia y la tecnología, considerando su impacto en
rrollan con una propuesta didáctica que trabaja los conteni- la sociedad. Para ello, se intencionan preguntas asociadas a la
dos comenzando con actividades Indaguemos, que buscan la información que entrega el texto, de manera que argumenten
activación de los conocimientos previos para posteriormente sus ideas de forma escrita. Aproveche para destacar el rol de
formalizarlos en el contenido tratado en el texto. Luego, se científicos chilenos en los observatorios astronómicos. Explí-
presentan actividades que abarcan distintas habilidades, queles que se debe en parte a las condiciones favorables que
enfocándose principalmente en las de orden superior, ciclo tienen los cielos de Chile en algunos lugares de la Zona Norte.
que se repite a lo largo de la lección. De forma articulada al Muchos de los observatorios que hay en Chile son administra-
tratamiento del contenido, en los Desarrollo de estrategias se dos por científicos de la Comunidad Europea u otras entidades
entregan herramientas para resolver problemas y en A poner internacionales, sin embargo, son muchos los científicos chile-
en práctica se trabajan formalmente las habilidades de in- nos que trabajan en la mantención de instalaciones y equipos,
en investigaciones extranjeras y en investigaciones propias.
Guía Didáctica del Docente

vestigación científica. Asimismo, a lo largo de la lección se


incorpora el trabajo de las actitudes con los OFT. Sugiera a sus estudiantes realizar una investigación sobre el de-
En la Guía didáctica se presentan pautas para poder utilizar sarrollo de la astronomía en Chile. Luego, pídales que respon-
algunas de las actividades del Texto del estudiante como dan las siguientes preguntas: ¿qué impacto para el desarrollo
instrumentos de evaluación: fichas de trabajo para los dife- científico nacional puede tener el hecho de que la Comunidad
rentes ritmos de aprendizaje, actividades complementarias Europea u otras entidades extranjeras instalen observatorios en
y evaluaciones, con sus respectivos solucionarios. Chile?, ¿de qué manera influirá lo anterior en los ámbitos del
desarrollo tecnológico, social, económico y político?

Unidad 2 • Fuerzas
ad

unid
2
analizado en conjunto con todos los integrantes del curso,
Ciencia al día P. 81 rescatando así sus impresiones. Una vez que las y los estu-
diantes hayan analizado las características del video, solicite
Esta sección es parte de Ciencia, tecnología y sociedad, por lo que planifiquen su trabajo para la confección del afiche.
que también pretende dar valor a las investigaciones cien-
tíficas que nos ayudan a comprender mejor el mundo que
nos rodea. En este caso en particular, se dispone del texto Tema 1: ¿Cuándo nos movemos? Páginas 82 a 85
“Levantamiento de pesas en la NATURALEZA”, que expone
la impresionante capacidad de las hormigas para levantar Activación de conocimientos previos
grandes masas. Para analizar esta información, se sugiere
realizar algunas preguntas: ¿qué importancia tiene compren- Para activar los conocimientos previos de sus estudiantes
der el mundo animal en el desarrollo de la ciencia?, ¿de qué respecto de sus aprendizajes sobre el concepto de fuerza
manera este tipo de información puede ayudar a generar puede preguntarles: ¿Cuándo aplicamos fuerzas? Mencionen
nuevas tecnologías? un ejemplo. ¿Qué efectos provocan las fuerzas? ¿Qué carac-
terísticas presentan las fuerzas?
Centros de investigación en Chile Para abordar los diferentes ritmos de aprendizaje de sus
estudiantes, le proponemos trabajar con materiales concre-
En la Universidad de Chile, específicamente en uno de los tos para evidenciar los efectos de las fuerzas, por ejemplo,
laboratorios del Departamento de la Ciencia de los Mate- con un elástico, arcilla y un auto de juguete. A los y las es-
riales, se analizan distintos materiales con el propósito de tudiantes que lograron comprender estos conceptos, puede
determinar las propiedades mecánicas de estos: tensiones proponerles hacer un mapa conceptual a medida que vayan
de fracturas, módulos elásticos, deformaciones elásticas y adquiriendo los nuesvos aprendizajes.
plásticas, microdureza y tenacidad de la fractura. A partir
Actividad del texto (P. 82)
de estas propiedades, se evalúa la calidad de los mate-
Indaguemos
riales que se utilizan en ingeniería o construcción, por
ejemplo el cemento, la madera, las aleaciones metálicas Considerando el objetivo, las habilidades y las actitudes que 77
y los polímeros. se trabajan esta actividad exploratoria, considere ampliar los
materiales que pueden ser manipulados por sus estudiantes
En este laboratorio, se investigan materiales metálicos
y solicite que, a partir de los recursos que se encuentran en
y no metálicos, sometiéndolos a diversas pruebas de re-
su entorno, reconozcan materiales que se comporten como
sistencia a fuerzas de tracción, compresión y flexión. Lo
el elástico o la plastilina. La actividad propuesta busca que
anterior, con el fin de desarrollar nuevos materiales para
reconozcan sus ideas previas sobre los efectos de las fuerzas.
satisfacer las necesidades tecnológicas y sociales que los
materiales tradicionales no logran satisfacer. Puede evaluar las habilidades y actitudes científicas de la
actividad realizada, usando la siguiente rúbrica:
Para obtener mayor información, visite:
http://www.dcm.uchile.cl/materiales_avanzados.html Rúbrica de evaluación de habilidades y actitudes
Niveles de logro
Inicio de la misión (P. 81) Indicadores
¿Cómo conducir una investigación? L ML PL
Condujeron adecuadamente la experiencia.
Para llevar a cabo esta misión, las y los estudiantes deberán
aprender e investigar sobre las características de las fuer- Describieron detalladamente los efectos que
zas, para luego confeccionar un afiche que dé cuenta de una provocó la manipulación de cada material.
situación hipotética que muestre qué creen que ocurriría Establecieron que debieron aplicar una
si las fuerzas no existieran. Es importante destacar que, en fuerza para manipular los materiales.
ese caso, se requiere de un análisis mucho más profundo Describieron las características del procedi-
sobre este tipo de aprendizajes, ya que se solicita imaginar miento efectuado de acuerdo al análisis de Guía Didáctica del Docente
un mundo que no existe y que, además, es totalmente dife- las evidencias.
rente a la realidad que conocemos. Es por esto que se asocia L = Logrado; ML = Medianamente logrado; PL = Por lograr.
un recurso audiovisual, que si bien no puede dar cuenta del
escenario que se solicita, permite realizar una aproximación
sobre lo que ocurriría si se cambian las condiciones habitua-
les en las que realizamos nuestras funciones cotidianas. Si
posee los medios necesarios, proyecte el video para que sea

Física • 2.º medio


Desarrollo de la unidad

Ventana de profundización disciplinar Tema 2: Identificando las fuerzas en la vida


Superelasticidad cotidiana Páginas 86 a 92
Las aleaciones con memoria de forma (en inglés shape Actividad del texto (P. 86)
memory alloy) son aleaciones metálicas que, después de Indaguemos
experimentar una deformación aparentemente plástica,
Esta actividad le permitirá favorecer el trabajo colaborati-
vuelven a su forma original cuando son sometidos a un
vo, la comunicación y la valoración de las ideas entre los
cambio de temperatura. Estas propiedades son denomina-
integrantes. Propicie un clima de respeto por las ideas y el
das memoria de forma térmica y memoria de forma elásti-
trabajo de otros, con el fin de que sus estudiantes evidencien
ca (o superelasticidad).
la importancia de estas actitudes.
Puede evaluar las habilidades y actitudes científicas de la
actividad realizada, usando la siguiente rúbrica:
Rúbrica de evaluación de habilidades y actitudes
Niveles de logro
Indicadores
L ML PL
Conducen adecuadamente la experiencia.
Describen correctamente la relación entre la
masa y la elongación del resorte.
Anticipan posibles resultados a partir de lo
establecido en la experiencia.
Señalan mejoras concretas para realizar la
Cuando se habla de la “memoria de un material”, se hace misma experiencia para obtener resultados
78
referencia a su capacidad para recuperar su forma incluso más precisos.
después de severas deformaciones. Las aplicaciones po- L = Logrado; ML = Medianamente logrado; PL = Por lograr.
tenciales de estos materiales son enormes, desde la medi-
cina hasta la ingeniería, pudiendo ser usados para generar Actividad del texto (P. 87)
fuerza o movimiento (mediante efectos de memoria de for- Aplico los tipos de fuerzas
ma) o para almacenar energía (superelasticidad).
Esta actividad permite poner en práctica los aprendizajes
Fuente: Archivo editorial adquiridos sobre la tensión y el peso. Para ello, es preciso
Representación de las fuerzas (P. 84) que las y los estudiantes reconozcan que, al permanecer en
reposo la lámpara, necesariamente el valor de la fuerza neta
La representación abstracta de conceptos físicos, por ejem- es cero, por lo que el módulo de la tensión es igual al valor
plo, de las fuerzas como vectores, puede presentar dificul- del peso.
tades para sus estudiantes. Explíqueles que corresponden
a representaciones que permiten analizar los fenómenos Puede evaluar la actividad realizada, usando la siguiente
físicos, pero que no necesariamente representan la realidad. rúbrica:

Actividad del texto (P. 85) Rúbrica de evaluación


Reflexiono sobre la fuerza netas Niveles de logro
Indicadores
Guíe a sus estudiantes para que analicen correctamente el L ML PL
esquema de las fuerzas que actúan en la situación planteada Aplican correctamente el modelo matemáti-
a partir de la dirección y sentido de las flechas que represen- co para determinar el valor del peso.
Guía Didáctica del Docente

tan las fuerzas. Reconocen que el módulo del peso es igual


al módulo de la tensión.
Identifican las características de la fuerza
resultante en la situación planteada.
L = Logrado; ML = Medianamente logrado; PL = Por lograr.

Unidad 2 • Fuerzas
ad

unid
2
Desarrollo de estrategias (P. 88) Las TIC (P. 89)
Realice la secuencia del problema en la pizarra y resuélvalo Para trabajar de manera lúdica y dinámica la ley de Hooke, pue-
paso a paso con sus estudiantes, con el objetivo de incor- de utilizar el recurso digital, que se encuentra disponible en la
porar este tipo de estrategia para resolver problemas y que página de simulaciones virtuales https://phet.colorado.edu/.
ellos puedan aplicarla luego en el paso 5 Aplico lo aprendido.
La fuerza elástica y la ley de Hooke (P. 89)

Actividad complementaria
Capacidad de los materiales para recuperar su forma
Reúnanse en parejas y consigan un lápiz, una cuchara y
un palito de helado. Luego, apliquen sobre cada uno de
ellos una fuerza, de manera similar a como se muestra en
la imagen.

La interfaz de este recurso permite identificar las caracterís-


ticas de la ley de Hooke a partir de la manipulación de masas
y resortes. Además, en esta página encontrará el recurso Ley
de Hooke, que permite estirar y comprimir los resortes para
explorar las relaciones entre la fuerza, la constante del resor-
te, el desplazamiento y la energía potencial.

Procuren que la fuerza ejercida sea de baja magnitud. A


partir de la experiencia, respondan:
79
a. ¿Qué conceptos es necesario conocer para realizar la
actividad?
b. ¿Qué materiales experimentaron una deformación
momentánea?
c. Si aumentaran paulatinamente la magnitud de la
fuerza aplicada, ¿cuál de los materiales experimentaría
primero una deformación permanente? Ambos recursos favorecen considerablemente la compren-
sión de esta ley.
d. ¿Por qué un determinado material tiene la capacidad
para recuperar su forma? Propongan una explicación
que dé cuenta de la estructura interna del material. Ventana de profundización disciplinar
e. Plantéenle a su compañera o compañero de trabajo ¿Qué diferencia una teoría de una ley?
una pregunta relacionada con la actividad.
La ciencia busca explicar los fenómenos que ocurren en
nuestro entorno, basándose para ello en leyes o teorías. Una
teoría corresponde a un conjunto de hipótesis que no es-
tán completamente demostradas; en cambio, una ley es un
hecho observable o una proposición científica que muestra
la relación entre dos o más variables, donde cada ley repre-
senta una propiedad de sistemas concretos, que en general Guía Didáctica del Docente
se expresan matemáticamente. La ley también se considera
como una regla y norma invariante. Para el análisis de esta
ventana, pregunte a sus estudiantes: ¿qué teorías conocen?
¿En qué se diferencia una ley de una teoría? ¿Por qué la ley
de Hooke es una ley científica y no una teoría?
Fuente: Bunge, M. (2005). La ciencia. Su método y su filosofía.
Buenos Aires: Editorial De Bolsillo.

Física • 2.º medio


Desarrollo de la unidad
Alfabetización científica 1 2
Para que sus estudiantes puedan comprender su entorno,
pregúnteles: ¿qué es la arquitectura antisísmica?, ¿por qué
algunos edificios no se derrumban durante un terremoto?,
¿qué principio o ley física explican el funcionamiento de los
edificios antisísmicos? Coménteles que a partir de grandes
terremotos se ha investigado sobre las propiedades de los 2. Al retirar el alambre del lápiz notarán que tiene la for-
materiales, con el fin de disminuir daños en las edificaciones ma de un resorte. Estírenlo de forma gradual, de modo
frente a un sismo de gran magnitud. Para esto se han cons- que este pueda recuperar su forma (observen la imagen
truido edificios antisísmicos, cuya base corresponde a una 2). Finalmente, apliquen una fuerza hasta que el resor-
“almohada” de hormigón, permitiendo que el edificio oscile te no pueda recuperar su forma original.
con el movimiento sísmico, cumpliendo con la ley de Hooke. Respondan las siguientes preguntas:
A poner en práctica (P. 90 y 91) a. ¿Por qué, al enrollar el alambre, este adquirió propie-
Guíe a sus estudiantes en la construcción de su hipótesis dades elásticas? Expliquen.
de trabajo, indicándoles que deben considerar las variables b. ¿Cómo explicarían que, al aplicar una fuerza mayor so-
involucradas, en este caso, la fuerza aplicada y la elongación bre el resorte, este haya perdido la capacidad de recupe-
del resorte. Si no cuentan con un set de masas, pueden usar rar su forma? ¿Sucederá lo mismo en todos los resortes?
objetos como tornillos, tuercas o monedas y, a partir de ellos,
construir un set de masas, determinando su valor con una c. ¿Realizaron el procedimiento siguiendo la secuencia
balanza. Es importante que mencione a sus estudiantes las de pasos sugerida? De no ser así, ¿piensan que deben
exigencias formales que debe tener el afiche científico con el repetir la actividad?
cual comunicarán los resultados de la investigación. Desarrollo de la misión (P. 91)
Puede evaluar la actividad realizada, usando la siguiente ¿Cómo comunicar los resultados de una investigación?
80 rúbrica:
Para llevar a cabo el proyecto de esta lección, oriente a sus
Rúbrica de evaluación para la comunicación de estudiantes para que puedan elaborar el afiche de acuerdo a
resultados las características de la situación solicitada. Para establecer
Niveles de logro un trabajo interdisciplinar con la asignatura de Artes Visua-
Indicadores les, puede solicitar la orientación del profesor o profesora
L ML PL
de la asignatura.
El afiche responde al formato requerido.
Se responde el problema de investigación. Puede evaluar la actividad realizada, usando la siguiente
rúbrica:
Se formula una hipótesis que responde al
problema de investigación y es comprobable Rúbrica de evaluación de habilidades y actitudes
en el experimento.
Niveles de logro
Se registran los datos medidos en el Indicadores
L ML PL
experimento.
Seleccionan un plan de trabajo coherente
Se concluye a partir de las evidencias
con la misión encomendada.
recogidas.
Justifican el procedimiento realizado para
L = Logrado; ML = Medianamente logrado; PL = Por lograr.
llevar a cabo su trabajo.
Implementan adecuadamente la estrategia
Actividad complementaria para ejemplificar lo solicitado.
¿Hasta qué punto se puede estirar o comprimir un resorte? Trabajan de forma responsable y ordenada
Guía Didáctica del Docente

Reúnanse en grupos de tres integrantes y realicen el pro- siguiendo el plan establecido.


cedimiento que sigue: L = Logrado; ML = Medianamente logrado; PL = Por lograr.
1. Consigan un alambre fino de cobre, de unos 20 cm de
largo, y un lápiz. Luego, enrollen el alambre sobre este
último, tal como se muestra en la imagen 1. Procuren
que las espiras queden muy juntas.

Unidad 2 • Fuerzas
ad

unid
2
Actividad del texto (P. 92) Reflexiono sobre lo que aprendí (P. 35)
Elaboremos un diagrama de cuerpo libre
Para orientar esta sección, invite a sus estudiantes a que
Guíe a sus estudiantes en la ubicación de las fuerzas, ya que el analicen su proceso de aprendizaje en la lección, respon-
problema puede ser complejo al tener ruedas el carro. Puede diendo las preguntas plateadas. Es importante que, en esta
pedirles que primero realicen el diagrama de cuerpo libre para instancia, las y los estudiantes identifiquen además de los
una caja que se tira con una cuerda, al igual que el carro. contenidos abordados, las habilidades, estrategias y actitu-
des que se desarrollaron durante la lección.

Ciencia, tecnología y sociedad P. 93


Integro lo que aprendí P. 94 y 95
Al trabajar la sección, ponga énfasis en la relación que tie-
nen las lecturas con los conceptos estudiados durante la Invite a sus estudiantes a poner en práctica los aprendizajes
unidad y en el hecho de que la dinámica, a pesar de haber adquiridos en la lección, respondiendo las preguntas que se
sido desarrollada casi a cabalidad hace ya algunos siglos, proponen en esta evaluación de proceso. Una vez que fina-
sigue presentando nuevas aplicaciones, pues la fuerza es lice, pídales que respondan la sección ¿Cómo voy? y que regis-
uno de los fenómenos físicos más cotidianos con los que tren su nivel de desempeño según las respuestas correctas que
nos relacionamos. hayan logardo. De acuerdo al nivel de desempeño, se propone
el trabajo con las actividades complementarias propuestas en
Cierre de la misión (P. 93)
esta Guía. El propósito de estas actividades es trabajar con los y
¿Cómo evaluar una investigación? las estudiantes según sus diferentes ritmos de aprendizaje. Si
En esta instancia de la misión, se espera que las y los estu- sus estudiantes obtuvieron un nivel de desempeño logrado, pí-
diantes evalúen su desempeño en el trabajo realizado, ana- dales que desarrollen la Ficha de ampliación de la página 91. A
lizando su trabajo personal como colectivo, con el propósito quienes obtuvieron un desempeño correspondiente a Por lograr
de que puedan calificarlo a partir de la retroalimentación. y Medianamente logrado, que invítelos a trabajar en la Ficha de
Para ello, además de las preguntas planteadas en el Texto refuerzo de la página 90. 81
del estudiante, se sugiere implementar las siguientes pautas Durante el transcurso de la unidad, el Texto del estudiante
de evaluación: propone una serie de actividades que promueven la meta-
cognición del estudiantado, de manera que puedan ser parte
Pauta de auto-evaluación
activa de su proceso de aprendizaje, evaluando permanente-
Aspectos a evaluar Sí No mente sus estrategias y técnicas.
Realicé aportes para la ejemplificación del caso hipo-
tético planteado.
Motive a sus estudiantes para que respondan las preguntas pro-
puestas en la tabla de la sección Reflexiono sobre mi desempeño,
Fui responsable en mis tareas dentro del grupo de de la página 95, con el fin de que puedan analizar su proceso de
trabajo.
aprendizaje. Si quiere reforzar este trabajo, invítelos a completar
Trabajé con conciencia e interés en el desarrollo de el siguiente cuadro con la información solicitada.
la misión.
Respeté las decisiones del grupo con el propósito de Fortalezas
desarrollar satisfactoriamente el proyecto. Aspectos para mejorar
Acciones para llevar a cabo
Pauta de co-evaluación
Revise la información entregada en la tabla anterior; así po-
Aspectos a evaluar Sí No drá analizar cómo ellos y ellas reflexionan sobre su proceso
Hubo buena disposición para el trabajo colaborativo. de aprendizaje.
Los distintos miembros del grupo colaboraron con ideas.
Se respetaron distintas ideas para llevar a cabo el Guía Didáctica del Docente
procedimiento.
Se comunicaron de una manera clara y adecuada.

Física • 2.º medio


Desarrollo de la unidad

LECCIÓN 2: Explicando las leyes de Newton

Las actividades propuestas en el Texto del estudiante tienen como propósito el logro de los Aprendizajes
Esperados (AE), abordando con diferentes estrategias los Indicadores de Evaluación (IE) e incorporando
en cada una de ellas el trabajo con habilidades, actitudes y Objetivos Fundamentales Transversales (OFT).
AE IE Actividad Habilidad Actitud/OFT
Explicar situaciones IE 2 Inicio de la misión (P. 97) HIC 2 OA B-F/OFT 2-6
cotidianas por medio de IE 1-2-3 Indaguemos (P. 98) HIC 1-2-3 OA B-F-H/OFT 2-6-8
las leyes de Newton.
IE 3 Identifiquemos las características de la inercia (P.99) HIC 3 OA B-F/OFT 2-6
IE 1-2-3 Indaguemos (P. 100) HIC 1-2-3 OA B-F/OFT 2-6
IE 3 Desarrollo de estrategias (P. 102 y 103) HIC 3 OA B-F/OFT 2-6
IE 3 Indaguemos (P. 104) HIC 3 OA B-F/OFT 2-6
IE 3 Explico la tercera ley de Newton (P. 105) HIC 3 OA B-F/OFT 2-6
IE 3 Integro lo que aprendí (P. 108 y 109) HIC 3 OA B-F/OFT 2-6

Propósito de la lección Ciencia al día P. 96


En esta lección se trabajan las leyes de Newton de manera
que las y los estudiantes logren analizar diversas situacio- Con la intención de activar las ideas previas de las y los estu-
nes cotidianas que les permitan explicar en mundo que los diantes sobre los contenidos de esta lección, e identificar la im-
rodea, de acuerdo a lo propuesto en los Aprendizajes Es- portancia del desarrollo tecnológico a partir de la investigación
perados. Lo anterior se desarrolla a partir de los recursos científica y su impacto en la sociedad, en esta sección se presen-
82
y las actividades presentes en el Texto del estudiante y en ta el texto “La biomecánica deportiva”. En este, se menciona el
la Guía didáctica, en función del logro de los Indicadores trabajo realizado por el Dr. En Fisiología Mecánica Jorge Zúñiga,
de Evaluación considerando, además de los contenidos, las quien utilizó sus conocimientos para crear una prótesis de ma-
habilidades y actitudes. no para personas con discapacidades. Para destacar el trabajo
realizado por este científico chileno, realice preguntas como las
En el Texto del estudiante, las diferentes temáticas se desa-
siguientes: ¿Qué beneficios puede generar la implementación
rrollan con una propuesta didáctica que trabaja los conteni-
de esta tecnología? ¿Qué propiedades deben tener los materia-
dos comenzando con actividades Indaguemos, que buscan la
les para crear este tipo de tecnología? ¿De qué manera el estu-
activación de los conocimientos previos para posteriormente
dio científico ayuda a generar tecnologías como esta?
formalizarlos en el contenido tratado en el texto. Luego, se
presentan actividades que abarcan distintas habilidades,
enfocándose principalmente en las de orden superior, ciclo Ciencia, tecnología y sociedad P. 97
que se repite a lo largo de la lección. De forma articulada al
desarrollo del contenido, en los Desarrollo de estrategias se En esta sección se expone el texto “La fuerza G”, que describe
entregan herramientas para resolver problemas. Asimismo, los experimentos realizados por John Stapp para determinar
a lo largo de la lección se incorpora el trabajo de las actitudes la resistencia humana para soportar grandes aceleraciones.
con los OFT. Comente a sus estudiantes que las fuerzas G corresponden a
En la Guía didáctica se presentan pautas para utilizar algu- una forma intuitiva de medir la aceleración, que toma como
nas de las actividades del Texto del estudiante como instru- referencia la aceleración de gravedad en la superficie terres-
mentos de evaluación: fichas de trabajo para los diferentes tre y que está relacionada con las sensaciones que se perci-
ritmos de aprendizaje, actividades complementarias y eva- ben cuando se experimenta una aceleración. Para trabajar
Guía Didáctica del Docente

luaciones, con sus respectivos solucionarios. con esta información, puede formular esta pregunta: ¿De
qué manera creen que la investigación realizada por Stapp
A continuación se presenta una serie de orientaciones para
contribuyó a la ciencia y la tecnología?
trabajar los temas, actividades y secciones presentes en la
lección del Texto del estudiante. Junto con ello, se incorpo-
ran actividades e información complementarias, entre otros
recursos, en la Guía didáctica.

Unidad 2 • Fuerzas
ad

unid
2
Actividad complementaria
Ciencia, tecnología y sociedad
Ventana de profundización didáctica
Evaluación para el desarrollo de competencias
La telaraña, uno de los materiales más elásticos y resisten-
tes de la naturaleza Con el fin de que exista coherencia con los nuevos mode-
los constructivistas del aprendizaje, la evaluación es uno
de los principales focos para que el proceso de enseñan-
za aprendizaje sea efectivo. Desde las nuevas teorías del
aprendizaje, el contenido es entendido tridimensional-
mente. Esto quiere decir que en él subyacen los conceptos,
los procedimientos y las actitudes. En este sentido, la eva-
luación debe responder a esta triada, entendiendo que de
esta forma se estarán evaluando competencias entendidas
como un saber complejo que resulta de la movilización, la
integración y la adecuación de estos tres componentes en
diversas situaciones.
El rol del estudiante en este nuevo enfoque evaluativo es
activo y permite que pueda utilizar sus conocimientos de
manera creativa con el fin de resolver problemas reales. En
este sentido, la evaluación de los aprendizajes debe estar
enfocada en un amplio espectro de tareas significativas pa-
ra el desarrollo de competencias de las y los estudiantes,
que permitan demostrar sus conocimientos a través de la
Las fibras de seda de araña son mucho más resistentes que resolución de problemas en diversos contextos.
un cable de acero de similar grosor y muchísimo más elás-
ticas, ya que puede estirarse hasta 135 % de su longitud Fuente: Villardón, L. (2006). Evaluación del aprendizaje para pro- 83
original sin romperse. La elevada elasticidad y la altísima mover el desarrollo de competencias. Educatio siglo XXI, 24, 57-76.
(Adaptación).
resistencia a la tracción de la seda de araña natural no
tienen igual, ni siquiera con las fibras producidas a par-
tir de proteínas de seda de araña pura. Un equipo alemán Tema 1: Estableciendo la primera
está tratando de desarrollar una hilera artificial de seda
sintética. Las aplicaciones potenciales de este compuesto
ley de Newton Páginas 98 a 99
que imita la seda de las arañas son incontables, desde su
Mencione a sus estudiantes que estas leyes constituyen la
empleo como material de sutura quirúrgica reabsorbible,
base para el estudio del movimiento de los cuerpos en fun-
hasta el desarrollo de nuevas fibras para ropa o carrocerías
ción de las fuerzas que lo producen, por lo tanto, a partir de
de automóviles.
ellas podrán explicar la causa de los movimientos que se
Fuente: Revista Nature, 13 de mayo 2010. describen en la vida cotidiana. Invítelos a intentar explicar
cómo se producen algunos movimientos, antes de comen-
Respecto del texto, reflexiona y responde:
zar a estudiar las leyes de Newton y que luego, al finalizar
¿Qué otros ejemplos conoces en los cuales el conocimiento su estudio, las retomen con el fin de evidenciar la utilidad
y/o la tecnología desarrollada por el ser humano ha parti- de estas. Los movimientos que puede proponer a sus estu-
do desde la imitación de la naturaleza? Explica. diantes son por ejemplo: dos personas en los extremos de un
Inicio de la misión (P. 97) balancín, un vehículo que sube una pendiente, una bolita
¿Cómo conducir una investigación? dentro de un autobús que frena, las partículas de un gas o
los remos de un bote que avanza sobre el agua.
En esta lección, las y los estudiantes tendrán la misión de Guía Didáctica del Docente
escribir la letra de una canción que les permita explicar las Actividad del texto (P. 98)
tres leyes de Newton. Para la planificación de este proyecto, Indaguemos
considere que esta estrategia ya fue implementada en la Uni- Es importante que les indique a sus estudiantes que, antes
dad 1, por lo que ya se encuentran familiarizados con ella. de grabar, practiquen sacar el naipe con un golpe seco; pue-
de que esto no resulte en el primer intento. Aproveche esta
instancia para trabajar aspectos valóricos, por ejemplo el
trabajo colaborativo.

Física • 2.º medio


Desarrollo de la unidad
Puede usar la siguiente rúbrica para que sus estudiantes se Reconocen correctamente la relación entre
autoevalúen. la fuerza y la aceleración.
Pauta de auto-evaluación Señalan medidas de seguridad adecuadas
para implementar en la experiencia.
Aspectos a evaluar Sí No
Establecen la importancia de considerar me-
Hubo buena disposición para el trabajo colaborativo.
didas de seguridad en experiencia prácticas.
Los distintos miembros del grupo colaboraron con L = Logrado; ML = Medianamente logrado; PL = Por lograr.
ideas.
Se respetaron distintas ideas para llevar a cabo el Desarrollo de estrategias (P. 102 y 103)
procedimiento.
En el paso 5 aplico lo aprendido, indique a sus estudiantes
Se comunicaron los resultados de una manera clara que usen el mismo paso a paso desarrollado en el problema
y adecuada. propuesto, con el fin de que reconozcan la utilidad de aplicar
los modelos. Es importante que en esta instancia sus estu-
Alfabetización científica diantes puedan poner en práctica todos los aprendizajes de
La inercia es posible observarla en distintas situaciones co- la lección. El segundo ejercicio de la sección Aplico lo apren-
tidianas. Por ejemplo, cuando un vehículo toma una curva, dido puede presentar un grado de complejidad para sus es-
los pasajeros que viajan al interior, sentirán que una “fuerza tudiantes, ya que no es exactamente igual al desarrollado en
ficticia” los empuja en el sentido hacia donde originalmente el paso a paso. Guíelos a analizar la situación planteada con
se dirigía el vehículo. El término “fuerza ficticia” tiene rela- preguntas como las siguientes: ¿qué representa la cuerda?,
ción con el hecho de que se observa el efecto de una fuerza, ¿se transmite alguna fuerza por ella?, ¿cuál?
pero en realidad es la inercia la que actúa, tratando de man-
Las TIC (P. 103)
tener el estado de movimiento de los pasajeros al interior
del vehículo. En este recurso, se pueden encontrar cuatro tipos de activida-
des, en las que se pueden evidenciar los efectos de las fuer-
84 Contexto histórico zas. Para estudiar la segunda ley de Newton, las actividades
Utilice la información de esta cápsula para enfatizar el carác- permiten modificar las fuerzas que actúan cuando se empuja
ter dinámico del proceso de construcción de conocimiento un carro con diferentes masa (con un refrigerador, una caja,
científico. Puede mencionar que la evolución del conoci- o una persona) para observar cómo se mueven los objetos.
miento hasta modelos más completos es un fenómeno inhe- Dependiendo de la actividad seleccionada, se pueden dimen-
rente a otras ramas de la física, como el electromagnetismo, sionar las masas, la aceleración y la fuerza aplicada.
la termodinámica, la óptica, o la física atómica.

Tema 2: Aplicando la segunda ley de Newton


Páginas 100 a103

Actividad del texto (P. 100)


Indaguemos
Implemente esta actividad para comprender la relación que
establece la segunda ley de Newton. A demás, es importante
que en esta actividad se resalte la importancia de la habili-
dad y actitud declarada para el desarrollo de los aprendiza-
jes. Puede evaluar las habilidades y actitudes científicas de
la actividad realizada, usando la siguiente rúbrica: Tema 3: Estableciendo la tercera
ley de Newton Páginas 104 a106
Guía Didáctica del Docente

Rúbrica de evaluación de habilidades y actitudes


Niveles de logro Actividad del texto (P. 104)
Indicadores Indaguemos
L ML PL
Conducen adecuadamente la experiencia. En esta actividad, se espera que las y los estudiantes evalúen
una situación en la que se cumple la tercera ley de Newton.
Establecen las características del suceso
En este caso particular, se formula una actividad teórica, por
cuando se modifica la masa.
lo que es fundamental que se guíe su trabajo.

Unidad 2 • Fuerzas
ad

unid
2
Desarrollo de la misión (P. 105) Pauta de auto-evaluación
¿Cómo comunicar una investigación?
Aspectos a evaluar Sí No
Para llevar a cabo el proyecto de esta lección y de acuerdo al Realicé aportes para la confección de la maqueta.
trabajo interdisciplinar señalado al inicio del mismo, solicite
a sus estudiantes que trabajen en la canción que les permita Fui responsable en mis tareas dentro del grupo de
explicar las leyes de Newton. Para confirmar que las ideas trabajo.
planificadas por cada grupo de trabajo sean correctas, genere Trabajé con conciencia e interés en el desarrollo de
la instancia para que cada equipo enseñe su trabajo.Puede la misión.
evaluar la actividad realizada, usando la siguiente rúbrica:
Pauta de co-evaluación
Rúbrica de evaluación de habilidades y actitudes
Aspectos a evaluar Sí No
Niveles de logro
Indicadores Tuvieron una buena disposición para realizar el
L ML PL
trabajo de forma colaborativa.
Conducen adecuadamente la experiencia.
Respetaron los tiempos asignados para realizar cada
La letra de la canción ejemplifica correcta- una de las tareas.
mente las leyes de Newton.
Se respetaron distintas ideas para llevar a cabo el
Realizan un trabajo metódico y coherente procedimiento.
con el tiempo asignado.
Utilizan conocimientos científicos en solu-
ciones de problemas cotidianos.
Integro lo que aprendí P. 108 y 109
Consideran la importancia de comprender
el mundo que los rodean a partir del conoci- Explique a sus estudiantes que esta sección constituye una
miento científico. instancia evaluativa, mediante la cual pueden medir el logro
L = Logrado; ML = Medianamente logrado; PL = Por lograr. de sus aprendizajes. Pídales que completen la sección ¿Cómo 85
voy?, para identificar el logro de los aprendizajes propuestos.
Luego de revisar las tres leyes de Newton, le proponemos
que invite a sus estudiantes a realizar el Desafío complejo, Para trabajar los diferentes ritmos de aprendizaje de sus es-
propuesto en la página XX de la Guía didáctica, que corres- tudiantes, le proponemos que utilice las siguientes activida-
ponde a una aplicación de estas leyes a fenómenos coti- des complementarias: si el nivel de desempeño es Logrado,
dianos, en este caso en los trucos de magia. Esta actividad pídales que completen la Ficha de ampliación de la página
involucra de forma articulada el trabajo con los contenidos, 93, de la Guía didáctica. En el caso de que su desempeño
habilidades y actitudes declaradas, por lo que puede consti- fuese Medianamente logrado o Por lograr, solicíteles que de-
sarrollen la Ficha de refuerzo de la página 92.
tuir un trabajo de síntesis para este tema.
Puede complementar el trabajo metacognitivo de sus es-
tudiantes en la sección Reflexiono sobre mi desempeño con
Ciencia tecnología y sociedad P. 107 las siguientes preguntas: ¿qué conceptos que no conocías al
principio de la lección podrías definir con tus propias pala-
Analice esta información sobre el uso del cinturón de seguri- bras ahora?, ¿qué es lo que debes ejercitar para superarlo?
dad junto con sus estudiantes. Considere visualizar el video Plantea una pregunta relacionada con el movimiento que
propuesto en Las TIC para evidenciar la importancia del uso aún no puedas responder, e investiga qué conocimientos se-
del cinturón de seguridad. Solicite a sus estudiantes que re- ría necesario dominar para hacerlo.
conozcan las tres leyes de Newton para aplicar los aprendi-
zajes a esta situación.
Cierre de la misión (P. 107)
Guía Didáctica del Docente
¿Cómo evaluar una investigación?
Para que las y los estudiantes evalúen su desempeño en el
trabajo realizado, analizando su desempeño personal como
colectivo, pueden calificarlo a partir de la retroalimentación.
Para ello, además de las preguntas planteadas en el Texto
del estudiante, se sugiere implementar las siguientes pautas
de evaluación:

Física • 2.º medio


Cierre de la unidad Páginas 110 a 117

La CIENCIA se construye
Páginas 110 y 111

Esta sección tiene la finalidad de enfatizar el carácter di-


námico del proceso de construcción del conocimiento cien-
tífico. En este caso en particular, se muestra una línea de
tiempo respecto de la evolución del concepto de fuerza a lo
largo de la historia.
Trabajo con la información
Para indagar en esta sección, lea la información junto con
sus estudiantes de manera que puedan evidenciar esta evo-
lución considerando el contexto social e histórico. Para ello,
además de las preguntas que se proponen en el texto, puede
realizar otras preguntas: ¿qué importancia ha tenido el acce-
so igualitario a la educación para equiparar las posibilidades
de ambos sexos en el acceso al mundo del conocimiento y el
trabajo calificado? ¿Seguirán los científicos de la actualidad
realizando investigaciones sobre el concepto fuerza?

Síntesis
Páginas 112 y 113

86 ¿Cómo trabajar con los organizadores gráficos?


La actividad Síntesis de estas páginas promueve la confeccio-
nar de un organizador gráfico. Para ello, se guía el proceso de
construcción de una tela de araña en función de cinco pasos
de tal manera que, al ir siguiéndolos, las y los estudiantes lo-
gren su confección de forma independiente. De esta manera,
se espera que en la página 113 del Texto, el estudiante com-
plete la tela de araña propuesta representando los conteni-
dos, habilidades y actitudes desarrolladas en la unidad. Si
lo desea, puede solicitar previamente materiales para que
sus estudiantes puedan elaborar este mismo organizador de
forma más extendida y con mayores recursos en un pliego
de cartulina.
Para finalizar, mencione a sus estudiantes que también pue-
den utilizar diferentes herramientas tecnológicas disponi-
bles en Internet para realizar un organizador gráfico, como
por ejemplo: al introducir el código 18GF2M086a en el sitio
web codigos.auladigital.cl, encontrará una herramienta di-
gital en la que se pueden crear mapas conceptuales, expor-
tarlos como imagen y compartirlos a través de una dirección
Guía Didáctica del Docente

URL. Además, permite crear mapas de forma colaborativa.

Unidad 2 • Fuerzas
ad

unid
2
Evaluación final Evaluar para aprender
Páginas 114 a 117
Para integrar la evaluación al
El propósito de estas páginas es evaluar los contenidos, las habilidades y las proceso de planificación de ma-
actitudes trabajadas en la unidad. Sugiera a las y los estudiantes que trabajen nera efectiva se debe ampliar su
de manera individual, de modo que puedan identificar aquellos contenidos y/o sentido, valorando su función pe-
habilidades que aún no han alcanzado. dagógica. En este sentido, es fun-
Para complementar las preguntas planteadas en la sección Reflexiono sobre lo que damental que las y los estudiantes
aprendí, puede plantear las siguientes: ¿qué estrategias utilicé para abordar los tomen conciencia de que la eva-
contenidos de esta unidad?, ¿fueron efectivas?, ¿qué debería cambiar para ser más luación es una instancia más de
eficaz en el estudio de la próxima unidad? aprendizaje. Desde esta perspecti-
va, la evaluación debe poseer las
De forma adicional, le proponemos trabajar con la evaluación de las páginas 96 y siguientes características:
97 de la Guía didáctica. Esta evaluación presenta preguntas de selección múltiple,
cuyas respuestas las encuentra en la sección Solucionario de la Guía didáctica, » Comunicativa. El proceso de eva-
página 100. luación debe ser un diálogo entre
el profesor o profesora y sus estu-
diantes. En este se deben definir
criterios claros desde el comienzo,
considerando los puntos de vista
de ambas partes.
» Formativa. La evaluación debe
estar centrada en las dificultades
del proceso de aprendizaje de las y
los estudiantes a partir de una ade- 87
cuada regulación pedagógica, con-
siguiendo una gestión evaluativa
que deseche el concepto de error.
» Motivadora. El profesor o profe-
sora debe procurar resaltar los lo-
gros de sus estudiantes. Además,
debe utilizar recursos de evalua-
ción que respondan a los intereses
de ellos y que se relacionen con su
entorno cercano.
Fuente: Hernández, L. (2010). Evaluar
para aprender: hacia una dimensión
comunicativa, formativa y motivadora
de la evaluación. Enseñanza de las
Ciencias, 28 (2), 285-293 (Adaptación).

Pregunta de integración con otras asignaturas


Guía Didáctica del Docente
Para intencionar los aprendizajes adquiridos en esta unidad con otras asignatu-
ras, como Historia, Geografía y Ciencias Sociales, en esta pregunta se relaciona
el concepto de fuerza con el Dios Griego Ares. Para ello, se muestra la visión
de los griegos respecto de la “fuerza bruta” y la “fuerza inteligente”. Se espera
que en esta instancia, las y los estudiantes respondan las preguntas asociadas,
considerando los aprendizajes de la unidad y el contexto en el cual se desarrolla
la mitología griega.

Física • 2.º medio


ad

unid
2
Profundización disciplinar

¿Qué sería del mundo sin fricción?


La fricción es un fenómeno de mucha importancia en la vida del ser humano por su intervención en toda clase de
movimientos.
• Produce calor. Cuando nos frotamos las manos, ocu- contacto. Gracias a esa particularidad, podemos cami-
rre una fricción por deslizamiento que provoca calor: nar o detenernos. Caminamos debido a que la fricción
esta es una característica de la fricción. Por ejemplo, nos permite apoyarnos sobre el suelo. Sin ella, sería
el uso de los fósforos. como intentar caminar sobre el aire. Sin fricción cual-
• Disminuye la velocidad. Los paracaídas nos sirven de quier movimiento sería eterno.
frenos, ya que nos ayudan a detener nuestro movi- • Permite el fácil desplazamiento. Reducir la fricción
miento descendente. permite desplazarse con mayor facilidad. Un buen
• Permite pulir superficies. Cuando dos superficies se ejemplo son los patines.
encuentran en contacto, una sobre otra, se produce • Permite el sostenimiento de objetos. Esto se observa,
un desgaste. Esta ventaja se utiliza al pulir o lijar por ejemplo, en los cuadros colgados en las paredes.
superficies. • Permite unir metales. Sin necesidad de soldarlos, po-
• Nos ayuda a movilizarnos. La fricción es la resistencia demos unir los metales. Por ejemplo, tuercas, clavos,
al deslizamiento que se produce entre dos cuerpos en etc.

88 La formación de ciclones
La Tierra, en general, se observa a una escala en la cual fuerzas ficticias. Este modelo permite amplificar el uso del
no es perceptible su rotación. En esta escala, la segun- diagrama de las fuerzas a contextos no inerciales.
da ley de Newton, expresada matemáticamente como
→ →
​​F ​​  = m · ​​  a  ​​,  funciona sin muchas dificultades al ser válida Uno de estos contextos es el estudio del clima. Cuando
para sistemas de referencia inerciales. Pero para la Tierra, se presenta una baja presión atmosférica, las partículas
que tiene un movimiento de rotación en escala tal que su se agrupan en un punto donde la presión es menor. Ade-
movimiento es perceptible, no es válida esta formulación más, por las fuerzas ficticias, se genera un giro caracterís-
matemática. tico dependiendo del hemisferio. En el hemisferio sur los
vientos generados son en el sentido de las agujas del reloj,
Cuando se consideran sistemas de referencia no inerciales y en el hemisferio norte en el sentido contrario, dando
se deben añadir vectores al diagrama de fuerzas, las lla- la forma conocida al ciclón. La fuerza ficticia que gene-
madas fuerzas ficticias. Las sensaciones que uno percibe ra esto es conocida como fuerza de Coriolis, que también
al estar dentro de un vehículo, al ser este un sistema de está involucrada en la generación de corrientes marinas.
referencia no inercial, son producto de las denominadas

Fuentes: http://www.math.cmu.edu/~wn0g/Force.pdf y http://fisica.ciencias.uchile.cl/files/apuntes/fisica.pdf


Guía Didáctica del Docente

Unidad 2 • Fuerzas
ad

unid
2
Profundización didáctica

El uso de ANALOGÍAS en el aula


“Es cierto que las analogías nada deciden, pero pueden hacer que uno se sienta más a gusto”

Sigmund Freud

◾◾ Reflexione
¿Utiliza correctamente la transposición didáctica al hacer
analogías?
◾◾ Empatice
¿Aprendería mejor si le explicaran un fenómeno usando
analogías?
◾◾ Decida
¿Le gustaría usar analogías como apoyo al proceso de
aprendizaje?

En la vida cotidiana, estamos impregnados de modelos ◾◾ La trama, que establece las relaciones entre las caracte-
como una vía para entender el mundo que nos rodea. Es rísticas o atributos de determinadas partes del análogo
por ello que la modelización ha ido en progreso en la en- y el tópico. 89
señanza de las ciencias. Las analogías en la educación no
dejan de ser un intento más de modelizar, de acercar el Cuando utilice una analogía, debe poner especial cuidado
conocimiento científico a los y las estudiantes para lograr en:
su aprendizaje, una herramienta didáctica que busca re- ◾◾ Identificar claramente los atributos al comparar lo co-
lacionar conceptos y contenidos abstractos con la reali- nocido (el análogo) con lo que quiere dar a conocer (el
dad concreta aprovechando lo que los y las estudiantes tópico), ya que pueden surgir errores conceptuales.
ya conocen para aumentar su potencial imaginativo, su
creatividad y, sobre todo, su habilidad para realizar nue- ◾◾ Explicar claramente las limitaciones existentes en
vas conexiones entre sus conocimientos. la analogía, sin importar cuán simple sea, para que
los y las estudiantes no caigan en simplificaciones
La estructura de la analogía consta de tres elementos: reduccionistas.
◾◾ El tópico o dominio, que consiste en los conocimientos ◾◾ Utilizar un análogo que sea familiar para los y las es-
(conceptuales, procedimentales o actitudinales) que se tudiantes, ya que si el análogo es poco representativo,
pretende enseñar. puede producir una transferencia analógica incorrecta
◾◾ El análogo, es decir, el objeto o concepto central de la de las características.
analogía.
Fuentes: Fernández, J. Portela, L. González, B. y Elortegui, N. (2001).
Las analogías en aprendizaje de la Física en Secundaria. Actas I Congreso Nacional de Didácticas Específicas, Granada.
Guía Didáctica del Docente

Física • 2.º medio


ad

unid
Ficha de refuerzo
2
Lección 1

Para reforzar los aprendizajes de la lección, realiza las siguientes actividades en tu


cuaderno.

1. ¿En cuál de los siguientes casos la fuerza aplicada produce un cambio permanente y en
cuál uno momentáneo?
Material fotocopiable

2. Natalia empuja una caja, como se muestra en la siguiente imagen. Dibuja todas las fuerzas
que actúan sobre la caja.

90

3. Daniel dibuja las fuerzas que actúan sobre un carro, como se muestra a continuación.
→ → →
​​​  ​​   1​​​
F  ​​​ F ​​   2​​​ F 
​​​  ​​   3​​​

¿Cuáles de las fuerzas tienen igual dirección?, ¿cuáles igual magnitud?, ¿cuáles igual
sentido?
4. Marcelo realiza un gráfico en su cuaderno sobre la variación de la longitud de un resorte
en función de la fuerza aplicada sobre él, pero olvidó anotar el valor de una de las fuerzas.
Sin embargo, si el resorte cumple con la ley de Hooke, ¿cuál es el valor de F?
Fuerza (N)

0 9 12 Elongación (cm)
5. Amelia midió la elongación de un resorte al aplicarle una fuerza de 3 N, obteniendo un va-
lor de 12 cm. Si el resorte cumple con la ley de Hooke, ¿cómo se puede saber su elongación
al aplicarle una fuerza de 4 N? Explica.

Nombre:
ad

unid
Ficha de ampliación
2
Lección 1

Para reforzar los aprendizajes de la lección, realiza las siguientes actividades en tu


cuaderno.

1. Rodrigo afirma que, si un cuerpo no recupera su forma mientras una fuerza sigue actuando,
no podrá hacerlo cuando la fuerza deje de actuar. ¿Es correcta su afirmación? De no ser así,
corrígela.
2. Pon un libro sobre la mesa: ¿qué fuerzas actúan sobre el libro? Luego, empuja el libro con
tu mano: ¿qué fuerzas actúan en este caso sobre el libro? Elabora un diagrama de cuerpo
libre para ambas situaciones.
3. Cuando dos amigos juegan a tirar una cuerda para ver quién mueve al otro, ambos aplican
una fuerza sobre la cuerda. ¿Qué características tienen estas fuerzas en relación con su

Material fotocopiable
dirección y su sentido?
4. Cecilia midió la variación de la elongación de un resorte al ser sometido a diferentes fuerzas
y con los datos obtenidos realizó el siguiente gráfico:

Fuerza (N)

91
3

0
1 2 3 4 Δx (cm)

Si el resorte cumple con la ley de Hooke, ¿cuál es el valor de la constante de elasticidad del
resorte?
5. En un laboratorio se midió la elongación de un resorte al aplicarle diferentes fuerzas y los
datos se registraron en la siguiente tabla:
Fuerza neta (N) 2 3 4
Elongación (cm) 6 7 12

¿Cumplen con la ley de Hooke las medidas registradas? De no estar correctas, ¿a qué crees
que se debe?

Nombre:
ad

unid
Ficha de refuerzo
2
Lección 2

Para reforzar los aprendizajes de la lección, realiza las siguientes actividades.

1. Sebastián empuja un coche de 10 kg, con una fuerza de 5 N: ¿cuál es el módulo de la ace-
leración que experimenta el coche?

2. Marcelo empuja un macetero de 15 kg con una fuerza de 50 N: ¿con qué aceleración se


moverá el macetero si la fuerza de roce que se opone al movimiento es de 15 N?
Material fotocopiable

3. Si sobre tu mesa dejas un libro de 50 g: ¿qué valor tendrá la fuerza normal que la mesa
aplica sobre tu libro? Considera g =9,8 m/s2.

92

4. Soledad empuja una cajonera de 20 kg, con una fuerza de 60 N. Si logra moverla con una
aceleración de 2 m/s2, ¿cuál es el valor de la fuerza de roce?

5. Un mueble de 100 kg es empujado horizontalmente sobre una superficie horizontal, de ma-


nera que se desplaza aumentando su velocidad. Si la diferencia entre la fuerza de roce y la
fuerza que empuja al mueble es de 100 N, ¿cuál será la aceleración que adquiere el mueble?

6. Elabora un organizador gráfico que señale las características de las leyes de Newton. Para
ello, completa la siguiente tabla.
1° Ley de Newton 2° Ley de Newton 3° Ley de Newton
Características

Ejemplo

Nombre:
ad

unid
Ficha de ampliación
2
Lección 2

Para reforzar los aprendizajes de la lección, realiza las siguientes actividades en tu


cuaderno.

1. Sobre tres cajas se aplican las fuerzas que se muestran en la imagen: ¿en qué caso la caja
se moverá hacia la derecha con una aceleración de 2 m/s2? Considera que cada caja tiene
una masa de 10 kg.

A B B
30 N 10 N 30 N 10 N 30 N 30 N

Material fotocopiable
2. Un nadador de 60 kg, al llegar a un extremo de la piscina, empuja con sus piernas la pared
de esta. A partir de esta situación, responde:
a. ¿Cómo se explica que al empujar la pared de la piscina el nadador pueda impulsarse
hacia adelante?
b. ¿Qué fuerza actúa en contra del movimiento del nadador?
c. Si el nadador empuja la pared de la piscina con una fuerza de 75 N y la fuerza que se
opone al movimiento es de 35 N, ¿con qué aceleración se desplazará?
93
3. Juan ubica un cubo de 3 kg, de tres formas distintas, como se muestra en el diagrama. ¿En
qué caso la fuerza normal será mayor y en cuál menor? Justifica.

A B C

4. Joaquín tiene que empujar una caja de 100 kg de masa por el patio de su colegio: ¿con que
fuerza deberá empujarla para que se desplace, si el coeficiente de roce µe = 0,25?
5. Andrés se ubica sobre una patineta frente a un muro, aplicando con sus manos una fuerza
de 85 N sobre él. ¿Qué aceleración adquirirá Andrés luego de empujar el muro? Considera
que la masa de Andrés es de 60 kg.
6. Elabora un organizador gráfico que relacione los aprendizajes de la lección 2, considerando
contenidos, habilidades y actitudes.

Nombre:
ad

unid
Desafío complejo
2
Objetivo: Determinar la constante Habilidades: Procesamiento e Actitudes: Valora la perseve-
de elasticidad de un resorte usan- interpretación de datos y formula- rancia, el rigor, la flexibilidad y
do diferentes modelos. ción de explicaciones, apoyándose la originalidad al desarrollar las
en conceptos y modelos teóricos actividades de la unidad.
del nivel.

¿Podemos modelar los fenómenos naturales de varias formas?


INICIO
Lean atentamente la siguiente situación.
Material fotocopiable

Fernanda necesita calcular la fuerza que necesita aplicar para estirar 1cm un resorte, pero
no conoce la constante de elasticidad del resorte. ¿Cómo podrías determinar esta constante?

DESARROLLO el valor en el número de ciclos, para que su valor


Realicen la siguiente actividad, que les permitirá sea más exacto.
responder la pregunta inicial. Reúnanse en grupos de 3. Repitan el procedimiento para diferentes masas.
tres integrantes y consigan los siguientes materiales: Grafiquen el período al cuadrado (T2) en función
un resorte, masas de diferentes medidas, un soporte de la masa. Determinen la pendiente del gráfico.
94
para el resorte, un cronómetro y una regla. CIERRE
Método 1 A partir de la actividad experimental y lo aprendido
1. Cuelguen el resorte en el soporte y midan su lon- en la lección, respondan las siguientes preguntas.
gitud inicial sin haber colgado ninguna de las a. En el caso del método 1, la ecuación que modela
masas en él. Luego, cuelguen una de las masas el fenómeno es F = k · Δx y en el método 2
​​  4π   
en el resorte y midan su longitud mientras la ma-
es T2 = ___
2
​​ m ¿Qué representa la pendiente de
sa está colgada. k
2. Vuelvan a realizar este procedimiento usando di- cada uno de los gráficos realizados?
ferentes masas y registren los datos en una tabla. b. ¿El valor de la constante de elasticidad obtenido
Determinen la variación de la longitud del resor- con ambos métodos es el mismo? Si hubiesen
te, que corresponde al largo final menos el largo diferencias, ¿a qué creen que se deben?
inicial. Además, calculen la fuerza aplicada por la
c. ¿Qué etapas de la obtención de datos mejorarían
masa colgada en el resorte, usando g = 9,8 m/s2.
para que sus datos fuesen más precisos? ¿Qué
3. Realicen un gráfico de fuerza en función de la modificarían?
elongación y determinen su pendiente.
d. ¿Existe solo una forma de modelar los fenóme-
Método 2
nos naturales? Expliquen.
1. Cuelguen el resorte en el soporte y una de las ma- e. ¿Es importante ser riguroso al realizar este tipo
sas en su extremo. Registren la medida de la masa. de actividades? ¿Por qué?
2. Estiren el resorte a una cierta amplitud y mídan-
f. Qué otros fenómenos podrían observar para
la. Luego, suelten el resorte y midan su período
estudiar la ley de Hooke?
de oscilación. Midan varios ciclos y luego dividan

Nombre:
ad

unid
Desafío complejo
2
Objetivo: Diseñar situaciones Habilidades: Comprender las limitaciones Actitudes: Demostrar in-
experimentales para demostrar y la utilidad de modelos y teorías como re- terés, curiosidad, rigor y
los principios de Newton. presentaciones científicas de la realidad, perseverancia.
que permiten dar respuesta a diversos fe-
nómenos o situaciones problema.

¿Los principios de Newton explican los trucos de magia?


INICIO
Lee atentamente la siguiente situación.

Material fotocopiable
Un mago realiza muchos trucos en los que están involucrados infinidad de conceptos
físicos, entre ellos los principios de Newton. ¿Podrías demostrar que un truco de magia se
explica a partir de la física?

DESARROLLO 5. Diseñen un póster en que deben incluir lo siguiente:


Realicen la siguiente actividad experimental, que les • Título del truco: debe ser llamativo e interesante.
permitirá evidenciar algunos de los fenómenos que te • Imágenes del truco: ya sean fotos o dibujos.
ayudarán a responder la pregunta planteada al inicio. • Explicación del truco a partir de uno o más de 95

los principios de Newton.


Reúnanse en grupos de cinco integrantes y revisen
6. Organícense para realizar una presentación del
en Internet algunos trucos de magia.
truco de magia en el patio del colegio, con el fin de
1. Identifiquen un truco de magia que puedan expli- explicárselo a otros estudiantes del establecimien-
car a partir de algún principio de Newton. to. Haga énfasis en la explicación física del truco.
2. Reúnan los materiales necesarios para replicar el CIERRE
truco.
A partir de la actividad experimental y lo aprendido
3. Planifiquen un procedimiento para replicar el
en la lección, respondan las siguientes preguntas.
truco escogido, indicando lo siguiente:
• Los pasos que seguirán. a. ¿Cuál o cuáles de los principios de Newton están
• Las características que deben tener los presentes en el truco de magia escogido? Explica.
materiales. b. ¿Otras leyes de la física pueden explicar lo suce-
• El montaje que deben realizar. dido en otros trucos de magia?, ¿cuáles?
• Consideraciones que deben tener al realizar el c. En relación con el trabajo de exposición del truco
truco. de magia a los demás compañeros del colegio,
4. Realicen el truco de magia, pueden usar la cáma- ¿qué fue lo que les produjo más dificultad al expli-
ra de un celular para grabarlo y luego analizarlo. car el fenómeno? ¿Qué aspectos creen que po-
Si es necesario, repitan el procedimiento hasta drían mejorar para futuros trabajos de este tipo?
obtener el resultado esperado.
d. ¿Qué fue lo que más les llamó la atención al
desarrollar la actividad?

Nombre:
Evaluación de la unidad
I. Selección múltiple 3 3. ¿En cuál de las siguientes situaciones se
producirá una deformación permanente?
1 Un cuerpo se encuentra inmóvil en un plano
inclinado, tal como muestra la siguiente A. Al aplastar un globo inflado.
imagen: B. Al estirar suavemente un elástico.
C. Al comprimir un resorte.
D. Al presionar un trozo de greda.
E. Al apretar una pelota de goma.

4 Gabriel necesita encontrar un material que,


al aplicarle una fuerza, sufra una deforma-
ción momentánea. ¿Qué puede utilizar?
¿Cuál o cuáles de las siguientes fuerzas A. Un alambre de cobre.
actúan?
Material fotocopiable

B. Plastilina.
I. El peso.
C. Un elástico de billetes.
II. La normal.
D. Un papel.
III. La fuerza de roce estático.
E. Un plumón de pizarra.
A. Solo I.
B. Solo I y II. 5 Dos amigos quieren construir un dina-
mómetro utilizando un resorte, ¿qué
C. Solo I y III.
características debería tener este resorte?
96 D. Solo II y III.
I. Que al disminuir su longitud no
E. I, II y III.
se evidencie el efecto de la fuerza
2 ¿Qué fuerza permite que el columpio de la restauradora.
imagen permanezca a una cierta distancia II. Que al aumentar su longitud se observe
del suelo? la acción de la fuerza restauradora.
III. Que al disminuir su longitud se
manifieste la fuerza restauradora.
A. Solo I.
B. Solo II.
C. Solo III.
D. Solo II y III.
E. I, II y III.
A. La tensión. 6 ¿En cuál de las siguientes situaciones actúa
B. El peso. la fuerza de roce cinético?
C. El roce cinético. A. Cuando un libro está sobre una mesa.
D. La normal. B. Cuando se empuja una caja sin moverla.
Guía Didáctica del Docente

E. El roce estático. C. Cuando se cuelga un cuadro en la pared.


D. Cuando un mueble se desliza por una
alfombra.
E. Cuando un auto está detenido en un
semáforo.

Unidad 2 • Fuerzas
ad

unid
2
7 El resorte que se muestra en la imagen cum- 9 Un grupo de estudiantes realizó un expe-
ple con la ley de Hooke. Javiera le cuelga rimento, en el que aplicaron diferentes
una masa de 250 g y se estira 2 cm: ¿cuál fuerzas sobre un resorte y midieron su esti-
será su estiramiento si cuelga una masa de ramiento en cada caso, registrando los datos
100 g? Considera g = 10 m/s2. en la siguiente tabla:

Fuerza neta Elongación (cm)


10 2,5
x 5
30 y
40 10

Si el resorte cumple con la ley de Hooke y se

Material fotocopiable
encuentra en su rango de elasticidad, ¿cuá-
A. 0,8 cm les son los valores que faltan en la tabla?
B. 1,25 cm A. x = 20; y = 7,5 D. x = 7,5; y = 20
C. 1,6 cm B. x = 15; y = 7,5 E. x = 20; y = 20
D. 2,5 cm C. x = 7,5; y = 7,5
E. 4 cm
10 La velocidad de un automóvil con respecto
8 En una clase de Física un grupo de estu- a un bus es de −20 km/h. Si la velocidad
del bus con respecto a un observador que se 97
diantes realiza la comprobación de la ley de
Hooke, usando un resorte y colgando en él encuentra en reposo a la orilla del camino
masas de diferentes valores. A partir de los es de 60 km/h, ¿cuál es la velocidad del
datos obtenidos, construyeron el siguiente automóvil con respecto al observador?
gráfico: A. −40 km/h D. 80 km/h
B. −20 km/h E. 20 km/h
Fuerza (N)
40 C. 40 km/h
30 11 8. Marcelo escribe el siguiente texto en su
20 cuaderno:
10
Un cuerpo permanecerá moviéndose a
0 velocidad constante si no actúan fuerzas
4 8 12 16 Δx (cm)
exteriores.
Si el resorte cumple con la ley de Hooke,
¿Qué título debería ponerle Marcelo al texto?
¿cuál es el valor de la constante de elastici-
dad del resorte? A. El principio de acción y reacción.
A. 10 N/cm B. La fuerza de gravedad.
Guía Didáctica del Docente
B. 4 N/cm C. El segundo principio de Newton.
C. 2,5 N/cm D. El principio de inercia.
D. 0,4 N/cm E. El roce cinético.
E. 0,6 N/cm

Física • 2.º medio


Solucionario
Desarrollo de estrategias (P. 88)
Del Texto del estudiante
Frc = 136 N
A poner en práctica (P. 90 y 91)
Inicio de unidad (P. 75)
a. Gráfico fuerza-elongación
1. Por ejemplo: Que todos los deportistas se encuentran
aplicando las fuerzas.
2. Porque las fuerzas se aplican cuando dos o más cuerpos

Fuerza (N)
interactúan entre sí, por ello, se dice que la fuerza “no
se tiene”.
Inicio de unidad (P. 76 y 77)
Respuestas variadas. 0 Elongación (m)
Lección 1: Comprendiendo las fuerzas y sus efectos
b. Esta respuesta depende de los materiales y valores utili-
Tema 1: Características generales de las fuerzas
zados. Se determina utilizando la ley de Hooke.
Indaguemos (P. 82) c. Esta respuesta depende de los materiales y valores
a. Al manipular el elástico, este se estira. Luego, vuelve a utilizados.
su forma original. d. La fuerza es directamente proporcional a la elongación.
b. Se deforma completamente cuando se manipula. e. Representa el valor de la constante de elasticidad del
c. Para manipular cada material, se deben aplicar fuerzas. resorte utilizado.
d. Por ejemplo: Realizamos el procedimiento de forma or- f. Sí, ya que el resorte cumple con la ley de Hooke.
denada y responsable para analizar correctamente las g. Esta respuesta depende de la hipótesis planteada.
características de cada suceso.
h. Algunos errores de procedimiento comunes: errores de
e. Hubiera tenido un resultado similar al del elástico, ya medición, errores de registro y errores de cálculo.
que la pelota de goma recupera su forma original luego
i. Las evidencias son fundamentales al momento de vali-
98 de apretarla.
dar una ley. Estas dan cuenta de la veracidad y predicti-
Reflexiono sobre la fuerza neta (P. 85) bilidad del modelo matemático asociado a ella.
1. Ganó el equipo de la derecha. Elaboremos un diagrama de cuerpo libre (P. 92)
2. Que todos se muevan hacia la derecha. N
Tema 2: Identificando las fuerzas en la vida cotidiana
Indaguemos (P. 86) FR F
a. Algunos conceptos son: aceleración de gravedad, fuerza
de gravedad, desplazamiento.
b. Por ejemplo: Sí, ya que los resortes tiene la capacidad de P
estirarse cuando se les aplica una fuerza. Que el carro se mueva hacia la derecha.
c. A mayor masa, mayor es la elongación del resorte. Integro lo que aprendí (94 y 95)
d. La fuerza de gravedad. 1.
e. Recupera su forma original, debido a las propiedades del
material. Normal Fuerza
f. Por ejemplo: Utilizar masas graduadas y medir el largo aplicada
del resorte para establecer cuál es la elongación de él de Fuerza
a cuerdo a la magnitud de la masa colgada. de roce
Aplico los tipos de fuerzas (P. 87)

1. ​​p→ ​​ = –8 N; ​​T ​​ = 8 N
→ Peso
Guía Didáctica del Docente

2. F 
​​  ​​N  = 0. Significa que la lámpara se encuentra en reposo.
3. Por ejemplo: Analizando la información y aplicando los 2. Solo la dirección.
modelos matemáticos. Trabajando de manera ordenada. 3. a. FN = 5N.
b. Que el objeto se mueva hacia la derecha.
c. Actúan las fuerzas peso y normal. Sin embargo, al
tener la misma magnitud pero sentidos opuestos,
mantienen en equilibrio vertical al cuerpo.

Unidad 2 • Fuerzas
ad

unid
2
4. a. Aproximadamente 35 N. c. La fuerza que provoca el movimiento del auto es la
b. Aproximadamente 5 N. tensión.
5. a. La caja no se moverá, debido a que la fuerza de roce d. Por ejemplo: Trabajar ordenadamente y manipular con
estático máximo es mayor que la fuerza aplicada. precaución todos los materiales para que la experiencia
b. La fuerza de roce estático máximo es 274,4 N. se desarrolle de la manera correcta.
6. e. El auto hubiera alcanzado una gran velocidad, provo-
Peso cando un colapso en el montaje. Por ello, es importan-
te respetar medidas para asegurar la seguridad de la
Tensión experiencia.
Tipos Normal Desarrollo de estrategias (P. 103)
1. mA = 13 kg; FA = 123,5 N; FB = 66,5 N
Fuerza roce
2. a. a = 6 m/s
2

Fuerza Ley de 1. b. Bloque 1: F = 6 N; bloque 2: F= 42 N; bloque 3:


Fuerza

restauradora Hooke
F = 12 N.
Representación Vectorial c. La tensión de la cuerda es 6 N.
Indaguemos (P. 104)
Cambios en la forma
1. Respuestas variadas.
Efectos Cambios en el estado 2. Si bien, por acción y reacción la fuerza que aplica el bu-
del movimiento rro a la carreta es igual en magnitud y dirección pero con
Lección 2: Explicando las leyes de Newton sentido contrario a la que la carreta le aplica al burro, la
Tema 1: Estableciendo la primera ley de Newton carreta logra moverse debido a que posee ruedas y su
fuerza resultante es distinta de cero.
Indaguemos (P. 98)
3. En cualquier situación en la que un cuerpo aplique fuer-
a. Movimiento, peso, fuerza, roce, aceleración de gravedad. za sobre otro. Por ejemplo: cuando se golpea una pelota 99
b. Dos respuestas probables a esta pregunta son: con el pie o cuando se mueve una caja.
• Que la moneda se quede en su lugar. 4. Por ejemplo: Si las fuerzas de acción y reacción son de la
• Que la moneda salga disparada junto con el naipe. misma magnitud, ¿qué ocurre con el movimiento de los
c. La fuerza de roce entre el naipe y la moneda no logró cuerpos donde están presentes?
cambiar el estado de la moneda, por lo que esta cayó Aplico la tercera ley de Newton (P. 105)
producto de la gravedad. El burro de la paradoja se equivocó porque consideró que
d. Por ejemplo: Cuando frena un auto y los cuerpos siguen las fuerzas se anulaban. La tercera ley de Newton se aplica
en su estado de movimiento. sobre cuerpos diferentes, por esta razón, estas fuerzas no se
e. Por ejemplo: ¿Qué ocurre con el estado de movimiento anulan.
de la moneda cuando se saca repentinamente el naipe Integro lo que aprendí (P. 108 y 109)
que la sostiene?
1. –1,5 m/s2
Identifiquemos las características de la inercia (P. 99) 2. Cuando Paz aplica una fuerza f sobre una caja de masa
a. El padre, ya que posee más masa. m, la aceleración del cuerpo es a = F/m. Si se dupli-
b. Experimentan la inercia cuya tendencia es mantener su ca la fuerza, también se duplica la aceleración, ya que
estado de reposo. a = 2F/m.
c. Los cuerpos experimentan un movimiento hacia atrás 3. Respuestas variadas. Por ejemplo:
producto de la aceleración del bus. Primera ley de Newton.
d. Por ejemplo: Los postulados científicos pueden ser revo- Ejemplo: La carga mal instalada sobre la parrilla de un auto.
lucionarios para la época. El postulado se puede aplicar Fundamento: Cada vez que el auto acelere o frene, la
en la actualidad, ya que se cumple para todas las situa- Guía Didáctica del Docente
carga experimentará la inercia. Es decir, se moverá hacia
ciones, por ello se dice que es una ley científica. adelante si frena y se moverá hacia atrás cuando acelere.
Indaguemos (P. 100)
Segunda ley de Newton.
a. MRUA, ya que parte desde el reposo aumentando su
Ejemplo: La fuerza que se debe implementar para
velocidad.
acelerar un carro de bomberos cuando este disminuye
b. A mayor masa (de las piedras), mayor es la diferencia de a la mitad su masa por la pérdida del agua.
velocidad del auto.
Fundamento: Para aplicar la misma fuerza sobre el

Física • 2.º medio


Solucionario
carro, al disminuir a la mitad la masa, la aceleración 6. a. La fuerza neta es cero. Esto provoca que la cuerda se
debe aumentar al doble. encuentre en reposo.
Tercera ley de Newton. b. Los niños saldrían expulsados hacia atrás producto
Ejemplo: Pegarle a una pelota. de la inercia.
Fundamento: La fuerza que aplica el pie sobre la pelota 7. R. Fuerza normal; M: Fuera de roce estático; Q: Peso.
es la misma que la pelota ejerce sobre el pie pero en 8. a. En el caso 2 la mesa acelerará más. En el caso 1 la
sentido contrario. mesa no acelerará.
4. a. Por ejemplo: Es muy importante utilizar el cinturón b. En el primer caso, la mesa no se moverá: en el se-
de seguridad, ya que este evita que los cuerpos gundo caso, la mesa se moverá hacia la derecha; en
salgan eyectados del auto producto de la inercia. el tercer caso, la mesa se moverá en diagonal (en
b. La primera ley de Newton se observa cuando los dirección a su vértice superior izquierdo).
pasajeros de un automóvil tienden a mantener la 9. La afirmación de Juan es incorrecta ya que las fuerzas de
velocidad del vehículo cuando este frena brusca- acción y reacción no se anulan debido a que actúan en
mente. La segunda ley se aprecia cuando el pasajero cuerpos diferentes.
percibe una fuerza proporcional a la desaceleración
que experimenta el auto. La tercera ley de Newton
De la guía didáctica
se aplica cuando el cinturón ejerce una fuerza sobre
el pasajero que es igual a la del pasajero sobre el
Actividad complementaria (P. 79)
cinturón pero en sentido contrario.
a. Algunos conceptos necesarios son el de fuerza, el de
Evaluación final (P. 114 a 117)
elasticidad y el de flexibilidad.
1. a. Normal b. Para fuerzas de baja magnitud, todos los materiales ex-
perimentaron deformaciones momentáneas.
c. El palito de helado debería experimentar primero una de-
100 formación permanente (más precisamente una ruptura).
d. La capacidad que tiene un material para experimentar
una deformación elástica se relaciona con la energía po-
tencial elástica que enlaza a sus moléculas.
e. Respuestas variadas.
Peso (atleta + pesas) Actividad complementaria (P. 80)
b. Fuerza de roce estático, ya que no hay movimiento. a. Al enrollar el alambre, este se convirtió en un resorte.
c. La fuerza neta es 0, ya que las fuerzas están equili- b. Debido a que el resorte sobrepasó su límite de elastici-
bradas y no hay movimiento. dad. Dependiendo de la magnitud de la fuerza, este he-
2. a. a = 10 m/s2 cho puede suceder con todos los resortes.
b. La tercera ley de Newton cuando Susana se impulsa c. Respuestas variadas.
con el muro, ya que le aplica una fuerza que es de
Ficha de refuerzo (P. 90)
igual magnitud que la que la muralla le aplica a ella,
1. En la lata la deformación es permanente, en el globo es
lo que permite su movimiento; y la segunda ley de
momentánea.
Newton producto de la aceleración que experimenta →
al impulsarse con la fuerza que ejerce sobre el muro. 2. ​​N ​​  
c. Debería considerar utilizar casco, rodilleras y code-

ras, además de moverse con precaución. ​​F ​​R  →
​​F ​​  
3. Es correcto, ya que de acuerdo a la tercera ley de New-
ton, la Tierra atrae a José con una fuerza que es de igual
Guía Didáctica del Docente

magnitud pero en sentido contrario a la fuerza que apli- →


​​P ​​  
ca José sobre ella.
4. ​​p→ ​​ = –680 N. 3. F1 y F2 tienen igual dirección, F1, F2 y F3 tienen igual mag-
→ nitud y todas tienen diferentes sentidos.
5. a. ​​p  ​​ = –580 N.
→ 4. El valor de la fuerza F es 3 N.
b. ​​T ​​ = 290 N.
5. La elongación del resorte al aplicarle una fuerza de 4 N
c. Existe la fuerza de roce estático entre las manos del
es de 16 cm.
gimnasta y las argollas.

Unidad 2 • Fuerzas
ad

unid
2
Ficha de ampliación (P. 91) 2° ley de Newton
1. Es correcta la afirmación para cuerpos que no poseen Si sobre un cuerpo actúa una Ejemplo: Al aumentar la masa
propiedades elásticas. Sin embargo, sus estudiantes pue- fuerza neta, este adquirirá una de un cuerpo que se desea
den mencionar que cuando una fuerza deja de actuar aceleración directamente pro- mover, se deberá aplicar una
sobre un material con propiedades elásticas, este recu- porcional a la fuerza aplicada, mayor fuerza.
perará su forma, siempre que no se haya sobrepasado donde la masa del cuerpo es la
su límite elástico. constante de proporcionalidad.

2. El diagrama de cuerpo libre cuando el libro está sobre 3° ley de Newton


la mesa es: Si un cuerpo A ejerce una fuerza Ejemplo: Cuando se rema en un
→ sobre un cuerpo B, entonces, bote, cada remo ejerce una fuer-
​​N ​​   este último ejercerá una fuerza za sobre el agua (hacia atrás).
de igual magnitud sobre A, pero Como reacción, el agua empuja
→ en sentido opuesto. el remo hacia adelante, provo-
​​F ​​   cando que el bote se mueva.

En el caso de que el estudiante empuje el libro hacia la Ficha de ampliación (P. 93)
derecha, el diagrama de cuerpo libre cuando se empuja 1. La caja que se moverá hacia la derecha con una acelera-
el libro es: ción de 2 m/s2 es la B.
Normal
Fuerza de roce
Fuerza ejercida
por la persona
2. a. Lo sucedido se explica por el principio de acción y
reacción.
Peso b. La fuerza de roce.
c. El nadador se desplazará con una aceleración apro-
3. En esta situación ambas fuerzas tienen la misma direc-
ximada de 0,7 m/s2.
ción, pero sentidos opuestos.
3. En el caso C es mayor, en el A es menor y en el caso B
4. El valor de la constante de elasticidad del resorte es
no existe.
1 N/cm. 101
4. La fuerza que debe aplicar Joaquín tiene que ser mayor
5. El resorte no cumple con la ley de Hooke, ya que entre
que 245 N.
los datos no hay una relación lineal. Sus estudiantes
5. La aceleración que experimenta Andrés es de aproxima-
pueden mencionar que esto se puede deber a que el re-
damente 1,4 m/s2.
sorte ha sobrepasado su límite de elasticidad, a que no
cumple la ley de Hooke o a errores en las mediciones. 6. Respuestas variadas.
Ficha de refuerzo (P. 92) Desafío complejo (P. 94)
1. El módulo de la aceleración que experimenta el coche a. Método 1: La pendiente corresponde al valor de la cons-
es de 0,5 m/s2. tante de elasticidad.
2. El módulo de la aceleración con que se moverá el mace- Método 2: La pendiente corresponde a (4 π2)/k.
tero es de aproximadamente 2,3 m/s2. b. La constante de elasticidad en ambos casos deberían ser
3. El valor de la fuerza normal será de 0,49 N. iguales. Sin embargo, las posibles diferencias pueden
atribuirse a errores en la medición, en los cálculos o a
4. La fuerza de roce que se opone al movimiento de la ca-
haber sobrepasado el límite de elasticidad del resorte al
jonera es de −20 N, considerando positivo el sentido del
realizar las mediciones.
movimiento.
c. Las respuestas de sus estudiantes serán variadas y de-
5. La aceleración que adquiere el mueble es de 1 m/s2.
penderán de cómo se haya realizado la actividad.
6.
d. En esta experiencia se demuestra que existe más de una
1° ley de Newton forma para modelar el comportamiento de un resorte.
Un cuerpo permanecerá en esta- Ejemplo: Al estar dentro de un e. Es importante, ya que la falta de rigurosidad resta exac-
do de reposo o de movimiento vehículo que se pone en mar- titud a las mediciones, con lo que se obtienen resultados Guía Didáctica del Docente
rectilíneo uniforme si no actúa cha, los pasajeros experimentan
que no son fiables.
ninguna fuerza sobre él o si la un impulso hacia atrás.
resultante de las fuerzas que f. Las respuestas de sus estudiantes serán variadas. Po-
actúan es nula. drían mencionar el uso de otros materiales elásticos, por
ejemplo, un elástico.

Física • 2.º medio


Solucionario
Desafío complejo (P. 95) Evaluación de la unidad (P. 96 y 97)
a. Las respuestas dependerán de las características del 1. E
truco escogido por cada grupo. En la explicación sus es- 2. A
tudiantes deben demostrar el dominio del principio de 3. D
Newton involucrado. 4. C
b. Se dan argumentos válidos para la inclusión de otras 5. D
leyes y principios físicos.
6. D
c. Las respuestas dependerán de cada grupo, promueva
7. A
el análisis de la exposición del truco y de las posibles
8. C
mejoras.
9. A
d. Las respuestas dependerán de los intereses de cada
estudiante. 10. D
11. E

Notas:

102
Guía Didáctica del Docente

Unidad 2 • Fuerzas
Notas

103

Guía Didáctica del Docente

Física • 2.º medio


FUERZAS
¿Qué efectos provocan las
fuerzas sobre los cuerpos?

¿Cuál de los Yo recuerdo que el


deportistas tendrá profesor de física
más fuerza? decía que la fuerza
no se tiene.

Entonces, si la fuerza
no se tiene ¿cómo
pueden levantar esas
pesas?

74
Actividad individual

1 Si pudieras participar de la conversación, ¿qué aportes


realizarías?

2 ¿Por qué crees que el profesor de Ana dice que la fuerza


“no se tiene”?

75
Evaluación inicial
¿Cómo podemos identificar las fuerzas?

Para reconocer las características de las fuerzas y sus efectos, responde las siguientes
preguntas a partir de la situación que se muestra en la ilustración.

¿Qué debió hacer la jugadora para saltar?

¿Qué permite que la pelota describa


¿Qué hay que hacer para una trayectoria parabólica?
que la pelota se mueva?

76 Unidad 2 • Fuerzas
¿Estarán todos los jugadores
sometidos a fuerzas? Fundamenta.

¿Será correcto decir que ambas Propón medidas de seguridad que


jugadoras se aplican fuerzas?, ¿por qué? apunten a evitar conductas de
riesgo en este deporte.

Física • 2.° Medio 77


¿Qué aprenderás y descubrirás en la unidad?
Te presentamos las principales metas, estrategias y propósitos de la unidad. Luego, propón
las metas que te gustaría lograr, las estrategias que emplearías para alcanzarlas y el propósito
de estas.

Metas ¿Cómo alcanzarlas? ¿Para qué alcanzarlas?

Comprender los efectos que ✓✓ Realizando actividades Para perseverar en el trabajo


tiene una fuerza neta sobre un prácticas. personal, entendiendo que
objeto, utilizando un diagrama ✓✓ Aplicando modelos. los logros se obtienen por me-
de cuerpo libre. ✓✓ Desarrollando procesos dio de un trabajo riguroso y
científicos. preciso.
✓✓ Trabajando con precisión.
✓✓ Realizando diagramas.
✓✓ Diseñando afiches.

Explicar situaciones cotidia- ✓✓ Realizando actividades Para entender que los seres
nas por medio de las leyes de prácticas. humanos hemos intentado
Newton. ✓✓ Aplicando modelos. comprender el mundo, consi-
✓✓ Valorando los aportes derando las implicancias éti-
científicos. cas de los avances científicos y
✓✓ Proponiendo medidas de tecnológicos.
seguridad.
✓✓ Escribiendo la letra de una
canción.

Propón tus propias metas Establece las estrategias Identifica el propósito de


para esta unidad. que usarás para el logro de tus metas.
tus metas.
✓✓
✓✓
✓✓
✓✓
✓✓
✓✓
✓✓
✓✓
✓✓

78 Unidad 2 • Fuerzas
¿Cómo te gustaría protagonizar tu propio aprendizaje?

En esta unidad aprenderás las características de


las fuerzas y sus efectos, pero ¿qué te gustaría
aprender sobre ellas?
¿Existen situaciones, en las que interactúan las
fuerzas, que te parezcan interesantes y te gus-
taría comprender?

En la unidad anterior aplicaste modelos, inter-


pretaste gráficos sintetizaste contenidos, entre
otros acciones. ¿Qué estrategia(s) te gustaría
seguir utilizando?, ¿por qué?
¿De qué manera, el uso de estrategias te permi-
te comprender mejor los nuevos aprendizajes?

¿Qué valor tienen los aportes de hombres y


mujeres al conocimiento científico?, ¿conside-
ras que estos permiten comprender mejor el
mundo que nos rodea?

¿Cómo lograr mis metas?


Para que puedas alcanzar tus metas y puedas adquirir los aprendizajes que se trabajarán es esta
unidad, te proponemos la estrategia de escribir un glosario temático. Para ello, solicita la ayuda
de tu profesor o profesora de Lengua y Literatura para definir, en tres pasos, cómo desarrollarás
esta estrategia.

ESTRATEGIA
Escribir un glosario temático

Paso 1 Paso 2 Paso 3

Física • 2.° Medio 79


Lección
Comprendiendo las fuerzas y sus efectos
Propósito de la lección

Las fuerzas están presentes en todas las ac- racterísticas generales de las fuerzas y sus
ciones que realizamos a diario, como cuando efectos, como resultado de su aparición por
pateamos una pelota, aplastamos una lata o la interacción entre dos o más cuerpos. Para
empujamos una puerta para abrirla. Ahora ello, deberás realizar actividades que te per-
bien, ¿qué es una fuerza?, ¿y qué efectos mitirán comprender que los logros personales
provoca? se obtienen realizando un trabajo riguroso,
En esta lección, aprenderás sobre las ca- preciso y ordenado.

Ciencia, tecnología y sociedad CIENCIA en

CHILE

La FUERZA de atracción de los agujeros negros


Desde que se sabe que en el universo existen los agujeros negros y
que uno de ellos está localizado en el centro de nuestra galaxia,
ha sido de gran interés estudiarlos. Ahora bien, ¿qué son los
agujeros negros? Un agujero negro es un objeto con una
enorme masa concentrada en un lugar pequeño, que
posee una fuerza de gravedad muy elevada.
¿Cómo es posible estudiar los agujeros negros? El
doctor en Astronomía Ezequiel Treister, académico
del Departamento de Astronomía de la Univer-
sidad de Concepción, señala que estos cuerpos
no se pueden observar directamente, sino que
solo es posible ver sus constituyentes, y/o los
efectos gravitacionales que producen, por me-
dio de telescopios especializados. Haciendo
uso de estos instrumentos, el doctor Treister
y su equipo han realizado investigaciones re-
lacionadas con el crecimiento de los agujeros
negros súper masivos, es decir, de masas muy
grandes, y la relación de estos con la formación y
evolución de las galaxias.

Fuente: http://www.explora.cl
(Adaptación)
¿Sabes qué es la fuerza gravitacional? ¿Qué importancia
tendrá el estudio de los agujeros negros? ¿Qué efectos crees
que puede provocar la elevada fuerza gravitacional que posee
un agujero negro?

ión
ac
e tiz ca
ab tífi
Alf cien

80 Unidad 2 • Fuerzas
Ciencia al día
“Levantamiento de pesas”
en la NATURALEZA
Las hormigas corresponden a una familia de insectos so-
ciales que aparecieron en la Tierra a mediados del período
cretácico. Son uno de los grupos de más éxito en el planeta,
prosperando en la mayor parte de los ecosistemas terres-
tres. Se estima que hay entre mil y diez mil billones de hor-
migas en la Tierra, correspondiendo, aproximadamente, al
15 % de la biomasa de los animales terrestres. Este insecto
ha colonizado la mayoría de los ambientes terrestres: los
únicos lugares que no poseen hormigas (originarias) son la
Antártica y algunas islas.
Existen muchas curiosidades respecto de las hormigas. Una
de ellas es que algunas especies pueden levantar cerca de
50 veces su propio peso y hasta 30 veces su volumen. Es
decir, en relación con su masa y volumen, tienen una gran
capacidad para ejercer fuerza.
Si un ser humano de 70 kg tuviera la misma capacidad para
ejercer fuerza, podría levantar una masa de tres toneladas
y media, equivalente a tres automóviles pequeños. Esto
convierte a las hormigas en las campeonas olímpicas del
levantamiento de pesas en la naturaleza.
¿Qué crees que pasaría si los humanos tuviéramos la ca-
pacidad de ejercer tanta fuerza como las hormigas? Si tu
tuvieras esta capacidad, ¿cómo la utilizarías?, ¿por qué?

Inicio de la misión
Si constantemente estamos sometidos a muchas fuerzas,
¿imaginas cómo sería la vida si no existieran? Para hacerte
una idea, observa el video de la Estación Espacial Interna-
cional que se encuentra en tu aula digital. Para ello ingresa
el siguiente código 18TF2M081a e identifica las caracte-
rísticas de la situación que se muestra.
En esta lección, tendrás la misión de confeccionar un crea-
tivo afiche que muestre un caso hipotético que simule lo
que ocurriría si no existieran las fuerzas. Para ello, solicita
la ayuda de tu profesor o profesora de Artes Visuales y en
conjunto con tu equipo de trabajo planifica el desarrollo
de esta misión. Al finalizar el estudio de las fuerzas y sus
efectos, expondrás tu afiche al curso.

Física • 2.° Medio 81


Lección 1

Tema 1 Características generales de las fuerzas ¿A qué crees que hace


referencia la frase “que la fuerza
te acompañe”?, ¿cómo definirías
¡Qué la fuerza te acompañe! Es una frase que se hizo popular en las películas de el concepto de fuerza?
ciencia ficción de Star Wars, pero ¿es posible que las fuerzas nos acompañen?,
¿llevamos, acaso, una fuerza con nosotros? Para responder preguntas como esta,
en este tema, conocerás las características generales de las fuerzas y sus efectos
sobre los cuerpos. Para ello, realizarás diversas actividades que te permitirán tra-
bajar de forma ordenada y rigurosa.

Indaguemos a partir de nuestros aprendizajes previos Actividad grupal

Objetivo: Reconocer y registrar apren- En parejas, consigan una barrita de plastilina y un elástico (de pelo o de billete).
dizajes previos. Luego, realicen el siguiente procedimiento:
Habilidad: Analizar y relacionar las 1. Cada uno de ustedes deberá tomar uno de los ingredientes y manipularlo: es-
características de un suceso.
tiren y suelten el elástico de billete y amasen la plastilina, por ejemplo. Analicen
Actitud: Manipular materiales en forma lo que ocurre con cada uno de los materiales.
precisa, ordenada y segura.
2. Luego, intercambien los materiales y vuelvan a manipularlos como estimen
Tiempo: 15 minutos.
conveniente. Comparen los resultados obtenidos con ambos materiales. Re-
gistren diferencias y similitudes.
Finalmente, respondan las siguientes preguntas:
a. ¿Qué efectos reconocieron al manipular el elástico?, ¿qué características
tiene este material?

b. ¿Qué ocurre al manipular la plastilina?, ¿qué características posee este


material?

Imágenes del procedimiento de


c. ¿Qué hicieron para manipular cada material?, ¿aplicaron una fuerza?
la actividad. Fundamenten.

d. ¿De qué manera realizaron el procedimeinto para el análisis de las


evidencias?

e. Si hubieran manipulado una pelota de goma, ¿qué resultados habrían


obtenido?

Seguramente notaste en la actividad anterior que, para manipular el elástico o la


plastilina, debiste aplicar una fuerza que provocó la deformación (parcial o total) de
cada uno de los materiales. Pero, ¿qué es una fuerza?, ¿y qué efectos provoca?

82 Unidad 2 • Fuerzas
La fuerza es la manifestación de una interacción o acción mutua entre dos o más
cuerpos. Esta no es una propiedad intrínseca de ellos, ya que un cuerpo no posee
fuerza por sí solo. Generalmente, reconocemos una determinada fuerza por los
efectos que puede ocasionar, como veremos a continuación.

Cambios en la forma

Los cambios en la forma de un cuerpo originados por una


fuerza pueden ser clasificados en no permanentes, si la for-
ma del objeto vuelve a su estado original cuando la fuerza
deja de actuar (lo que se representa en la primera imagen
mediante la fuerza que se ejerce sobre el elástico); y en per-
manentes, si la alteración en la forma del objeto se mantie-
ne una vez desaparecida la fuerza (lo que se observa en la
imagen, cuando la fuerza que aplica la mano sobre la arcilla
deja una impresión permanente en ella).

¿Qué otros ejemplos puedes


mencionar de este tipo de
efectos de la fuerza?

Deformación no permanente. Deformación permanente.

Cambios en el estado del movimiento

Dependiendo de la manera en que una fuerza es aplicada sobre un cuerpo, esta puede oca-
sionar los siguientes efectos en su estado de movimiento.

F
v
F
F

v
v

Cuando una determinada fuerza Si una fuerza actúa en sentido con- Cuando la fuerza actúa en una direc-
actúa a favor del movimiento de un trario al movimiento de un cuerpo, ción diferente a aquella en la que se
cuerpo, producirá en este un incre- puede producir sobre este una dis- mueve el objeto, provoca cambios en
mento de su rapidez. minución de su rapidez. la dirección de su movimiento.

Física • 2.° Medio 83


Lección 1

Tirar la cuerda debe ser el juego más popular a la hora de aplicar fuerzas. La idea
de este es enfrentar a dos equipos y poner a prueba su fuerza uno contra otro,
pero ¿qué debe ocurrir para que un equipo gane?

¿Qué ocurre si todas las niñas y niños tiran la cuerda con la misma fuerza?

Importante

La unidad en la que se mide el


módulo de una fuerza en el Sis-
Representación de las fuerzas tema Internacional es el newton,

Tal como se puede inferir de la imagen, cuando aplicamos una fuerza (​​F ​​) , debe-
llamado así en honor al físico y
matemático inglés Isaac Newton.
mos considerar el sentido en que la aplicamos para obtener los efectos que de- Un newton representa la fuerza
seamos. Por esta razón, las fuerzas se representan mediante vectores los cuales necesaria para cambiar, en un
indican lo siguiente:


segundo, la rapidez de un cuerpo

​​F  ​​  
de 1 kg de masa en 1 m/s. Esta
Sentido Dirección unidad equivale a:
1 kg ‧ m
1 newton = 1 N = ​ ​________  ​​ 

​s​​  2​

Magnitud

84 Unidad 2 • Fuerzas
¿Cuántas fuerzas se están
aplicando en la cuerda?

Fuerza neta

  ). Para

Cuando las fuerzas se ejercen en conjunto, es como si hubiese una sola fuerza
actuando. Esta fuerza resultante recibe el nombre de fuerza neta (​​F ​​neta ¿Importa el sentido en el que
determinar la fuerza neta sobre un cuerpo, se debe obtener la suma vectorial de cada niño y niña aplica la fuerza?
Fundamenta.
todas las fuerzas que actúan sobre él.
Supongamos que los niños de la imagen, aplican las siguientes fuerzas:

​​  ​​

F 1  = –4 N ​​ F 

3
​​  = –1 N F 
​​ →​​  = 2 N
4

​​  ​​

F 2  = –2,5 N ​​  ​​

F 5  = 1 N ​​ F 

6
​​  = 5 N
En este caso, la fuerza neta será:


​​F  ​​neta = –4 N – 2,5 N - 1 N + 1 N + 2 N + 5 N = 0,5 N

Actividad individual
Reflexiono sobre la fuerza neta
1. Según el resultado de la fuerza neta del ejemplo anterior, ¿qué equipo ganó?
2. ¿Qué efecto provocó este resultado? Señala las características.

Física • 2.° Medio 85


Lección 1

Tema 2 Identificando las fuerzas en la vida cotidiana


Ten por seguro que, en este momento, sobre ti se ejercen a lo menos dos fuerzas. ¿Qué fuerzas conoces?
¿Podrías identificar cuáles son?
En este tema, estudiaremos las principales fuerzas que actúan en nuestro en-
torno. Para ello, ejecutarás diversas actividades prácticas que te permitirán lo-
grar exitosamente el aprendizaje de conceptos, realizando efectivamente sus
procedimientos.

Indaguemos a partir de nuestros aprendizajes previos Actividad grupal

Objetivo: Descubrir la relación En grupos de tres o cuatro integrantes, consigan un resorte, un soporte uni-
entre la masa y la elongación de versal y un par de masas de diferente magnitud. Luego, realicen el siguiente
un resorte. procedimiento:
Habilidad: Observar y relacionar
variables.
1. Anclen uno de los extremos del resorte al soporte universal, tal como se mues-
tra en la imagen del costado. Luego, suspendan de él la masa de menor
Actitud: Ejecutar correctamente un magnitud. Observen lo que ocurre con el resorte. ¿Qué creen que ocurra al
procedimiento.
cambiar la masa?
Tiempo: 20 minutos.
2. Repitan el procedimiento, pero esta vez utilicen la masa de mayor magnitud.
Observen lo que sucede con el resorte.
Luego, respondan las siguientes preguntas:
a. ¿Qué conceptos ya estudiados piensan que están presentes en la activi-
dad? Escríbanlos.

b. ¿Ocurrió aquello que esperaban?, ¿por qué?

c. ¿Cómo se relaciona la elongación del resorte con la magnitud de la masa


que se suspendió de él? Expliquen.

d. ¿Qué fuerza actúa sobre el sistema masa-resorte?

e. ¿Qué pasa con el resorte cuando se deja de aplicar la fuerza?, ¿a qué creen
que se debe?

f. ¿De qué manera podrían mejorar el procedimiento para obtener resultados


más precisos? Propongan un procedimiento alternativo.

Imagen del montaje de la actividad.

Tal como experimentaste en la actividad anterior, el resorte se estira debido a la


fuerza que ejerce cada masa sobre él. ¿Qué nombre recibe esta fuerza?
A continuación, te invitamos a estudiar los diversos tipos de fuerzas que podemos
encontrar en nuestro entorno.

86 Unidad 2 • Fuerzas

La fuerza de atracción gravitacional o peso (​​P ​​) 
Es la fuerza de atracción gravitacional que ejerce un cuerpo celeste sobre cual-
quier otro cuerpo cercano a su superficie. En la Tierra, los cuerpos caen debido a
esta atracción con una aceleración igual a la aceleración de gravedad. Esta fuerza
apunta hacia el centro de la Tierra y su magnitud es proporcional a la masa del
cuerpo y se calcula multiplicando la masa por la aceleración de gravedad.

Peso del cuerpo ​​P  ​​  = m ‧ g​​ →  ​​   Aceleración de gravedad

Masa del cuerpo



La fuerza normal (​​N ​​) 
Cuando nos encontramos de pie, acostados o sentados sobre una superficie, ¿qué
impide que la fuerza de gravedad nos lleve hacia el centro de la Tierra?
La fuerza que actúa en este caso es la denominada fuerza normal. Esta corres-
ponde a la fuerza que toda superficie ejerce sobre un cuerpo que se encuentra
apoyado en ella, y su dirección es siempre perpendicular a la superficie, de allí su
nombre (normal =perpendicular).

​__›
​__› N
N ​ ​  
​ ​  
​__›
N
​ ​  

Cuando la superficie es horizontal, Si la superficie de apoyo está Cuando la superficie es vertical,


la fuerza normal tiene la misma inclinada, la dirección del peso y la la fuerza normal tiene la misma
magnitud y dirección que el peso, normal son diferentes. Además, la magnitud que la fuerza aplicada, en
pero sus sentidos son opuestos. magnitud de la fuerza normal es forma perpendicular a la superficie.
menor que la del peso.


La tensión (​​T ​​) 
Cuando las fuerzas se transmiten a través de cuerdas, cables y estructuras de
diferente tipo, entonces estamos en presencia de las denominadas fuerzas de
tensión. Por ejemplo, la cuerda que sostiene una lámpara colgante está sometida
a una tensión, cuya magnitud es igual, en este caso, al peso de la lámpara.

Actividad individual
Aplico los tipos de fuerzas
Si la lámpara de la imagen del costado tiene una masa de 0,8 kg:
​__›
1. ¿Cuál es el valor del peso de la lámpara?, ¿cuál es el valor de la tensión? T
​ ​  
2. ¿Cuál es la fuerza neta sobre la lámpara?, ¿qué significa este resultado?
3. ¿De qué manera buscaste la solución de las preguntas anteriores?, ¿qué es-
trategias empleaste? ​__›
P
​ ​  

Física • 2.° Medio 87


Lección 1


La fuerza de roce por deslizamiento (​​F ​​r )

Es la fuerza que se opone al movimiento y surge por las imperfecciones
de las superficies en contacto. Hay dos tipos, el roce estático (​​F r​​ e), que


es la oposición al movimiento antes de que se produzca y el cinético
  ), que es la oposición cuando el cuerpo ya está en movimiento.
(​​F ​​rc
El roce estático máximo siempre es mayor cuando el cuerpo está en
reposo que cuando está en movimiento.
Estas fuerzas se calculan multiplicando la fuerza normal sobre el
cuerpo por un coeficiente de roce (μ) que depende de las caracterís-
ticas de las superficies en contacto.

Fuerza de roce Fuerza de


Fre = μe · N Fuerza normal Frc = μc · N Fuerza normal
estático máximo roce cinético

Coeficiente de roce estático Coeficiente de roce cinético

Desarrollo de estrategias
Aprendiendo a aplicar modelos para determinar la fuerza de roce.

Situación problema

Juan desea mover una caja de 60 kg, para lo cual, la desliza sobre una superficie
horizontal, tal como muestra la imagen. Si se considera que el coeficiente de roce
cinético es 0,18, ¿cuál es el valor de la fuerza de roce cinético?

PASO 1 Identifico la incógnita


Se desea determinar el valor de la fuerza de roce cinético Frc

PASO 2 Registro los datos


m = 60 kg μc= 0,18

PASO 3 Aplico los modelos


La fuerza de roce cinético se determina: Frc = N · μc
En este caso, el valor de la normal es igual al valor del peso, por lo tanto:
N = P = m · g = 60 kg · 10 m/s2 = 600 N
De esta manera:
Frc = 600 N · 0,18 = 108 N
PASO 4 Escribo la respuesta
La fuerza de roce cinético entre la superficie y la caja es de 108 N.
PASO 5 Aplico lo aprendido
Fernando desea mover una caja de 40 kg de masa sobre una superficie horizon-
tal. Si el coeficiente de roce estático máximo entre la caja y el piso es 0,34, ¿cuál
es el valor de la fuerza de roce?

88 Unidad 2 • Fuerzas
La fuerza elástica y ley de Hooke
Supongamos un resorte al cual se le cuelgan diferentes masas. Tal como se puede las tic
apreciar en la siguiente ilustración, el resorte se estira a medida que se le cuelgan
diferentes masas y cada una de estas, ejerce una fuerza sobre el resorte, provo- Ingresa el código  18TF2M089a en
cando un estiramiento proporcional. Así, cuando se cuelga una masa de 5 g, el tu aula digital y realiza el laboratorio
de resortes para comprender mejor
estiramiento del resorte es de 0,5 cm; cuando la masa es de 10 g, el estiramiento
la ley de Hooke.
es de 1,0 cm y es de esperar que para estirar 1,5 cm el resorte, se le deba colgar
una masa de 15 g.
10
20
30

0,5 cm
1,0 cm
1,5 cm
40

Cuando al resorte se le
50 cm

cuelga una masa de 5 g,


este se estira 0,5 cm. Si el resorte se estira
Cuando al se le cuelga 1,5 cm, ¿se puede determinar
una masa de 10 g, el la magnitud de la masa que se
resorte se estira 1,0 cm. colgó?, ¿por qué?

La relación entre la fuerza y la deformación de los materiales fue establecida por


el físico Robert Hooke, quien observo que el estiramiento del resorte es propor-
cional al peso que se cuelga de él. Así, estableció la siguiente relación matemática:

Constante de propor-
cionalidad elástica
Magnitud de la
F = k · ∆L
fuerza aplicada

Elongación del resorte

El valor de la constante k depende de las características del resorte y corresponde


a la medida de la resistencia que posee el resorte para elongarse. Así, mientras
mayor sea el valor de k, mayor es la resistencia a la deformación.
La capacidad de un resorte de volver a su forma original, se debe a la fuerza res-
tauradora o elástica. Esta fuerza es de igual módulo que la fuerza que se ejerce
sobre el resorte, pero en sentido opuesto, tal como se muestra en la ilustración
del costado. Así, si relacionamos la fuerza restauradora con la elongación del
resorte, obtenemos que:

Constante de
proporcionalidad elástica FR
Magnitud
de la fuerza FR = -k · ∆L
restauradora F

Elongación del resorte

Física • 2.° Medio 89


A poner en práctica mediante un taller de ha
bilidades científicas

Aprendiendo a desarrollar procesos científicos:


¿Cómo utilizar las propiedades elásticas de algunos
materiales para medir fuerzas?
Objetivos: Determinar la Habilidad: Procesar e inter- Actitud: Proponer distintas
constante de elasticidad de un pretar datos y aplicar ley de formas para mejorar la preci-
resorte para medir fuerzas. Hooke. sión y la calidad del trabajo.

Situación problema

Seguramente, cuando han ido a comprar frutas o pan, antes de pagar, deben determinar
su peso (en estricto rigor, su masa). Ahora bien, ¿bajo qué principio físico funcionan los
instrumentos que determinan la masa o el peso de los cuerpos? Para medir una fuerza,
como el peso, se le debe asignar un determinado valor numérico utilizando un instru-
mento apropiado para ello.
En el siguiente taller, determinaremos la contante de elasticidad de un resorte para
medir fuerzas. Para ello, reúnanse en grupos de cuatro o cinco integrantes y concideren
el siguiente problema que guiará la presente investigación: ¿Cómo se puede utilizar un
resorte para medir el peso de los cuerpos? Respecto a esta interrogante, propongan
una hipótesis.
Para ponerla a prueba su hipótesis, desarrollen el siguiente diseño experimental.

Procedimiento experimental

Reúnan los siguientes materiales: un Fijen uno de los extremos del resor- Suspendan del extremo libre del re-
resorte de tracción (su longitud debe te al soporte y midan su longitud, tal sorte la masa de menor magnitud.
aumentar al aplicarle una fuerza ex- como se representa en la imagen. Re- Midan nuevamente la longitud. Repi-
terna), cuatro masas graduadas, una gistren dicho valor como x 0 . tan este procedimiento con cada una
regla y un soporte. de las masas y registren sus medicio-
nes como x1, x2 , x3 y x4 .

90 Unidad 2 • Fuerzas
Organización de los datos

Registren los resultados obtenidos en la tabla siguiente, considerando las unidades de


medida del Sistema Internacional y determinen la constante de elasticidad del resorte.
Constante de
Masa (kg) Peso (N) Elongación (m)
elasticidad (N/m)
1
2
3
4

Análisis e interpretación de evidencias

a. Construyan un gráfico en el que se represente la relación entre la fuerza peso y la


elongación del resorte.
b. ¿Cuál es la contante del resorte?, ¿cómo lo determinaron?
c. Si se cuelga un quinto cuerpo de masa desconocida, que provoca un estiramiento
de 0,05 m en el resorte, ¿qué peso tendrá el objeto?

Elaboración de conclusiones

d. ¿Qué relación existe entre la fuerza y la elongación?


e. ¿Qué creen que representa la pendiente del gráfico fuerza-elongación?
f. Con la información que poseen, ¿consideran que pueden utilizar el resorte para
medir el peso y la masa de los cuerpos? Fundamente.
g. ¿Se validó o rechazó su hipótesis? Expliquen.
h. ¿Qué errores en el procedimiento podrían haber afectado sus resultados?
i. ¿Qué importancia piensan que tienen las evidencias experimentales en la funda-
mentación de un modelo o ley?

Comunicación de los resultados

Para comunicar los resultados de esta investigación, elaboren un afiche científico en el


que respondan preguntas como las siguientes: ¿cuál fue el problema de investigación?,
¿qué hipótesis se propuso?, ¿en qué consistió el diseño experimental?, ¿cuáles fueron
nuestras conclusiones?
Para su confección, consideren el modelo que se presenta al costado e incluyan imá-
genes, gráficos y tablas.

Desarrollo de la misión
Ahora que ya conocen todos los tipos de fuerza y sus efectos,
reunidos en sus grupos de trabajo, consideren un ejemplo
cotidiano de sus aplicaciones y elaboren el afiche imaginando
qué ocurriría si, en ese caso, no se pudieran utilizar las fuer-
zas. ¿Con qué dificultades creen que se encontrarán?

Física • 2.° Medio 91


Lección 1

Diagrama de cuerpo libre


Cuando sobre un cuerpo o sistema actúan varias fuerzas, un modelo que resulta
útil para estudiar la situación es el diagrama de cuerpo libre. Este corresponde a
una simplificación esquemática que permite analizar las fuerzas que interactúan
sobre un mismo cuerpo. Independiente de la forma del o los objetos en estudio,
las fuerzas se trasladan al centro de masa del sistema (lugar geométrico donde
actúa la fuerza neta), tal como se representa en las siguientes situaciones:

Situación Diagrama de cuerpo libre


​__›
N​
​   

_​ _›
Una caja apoyada P​
​   
sobre una superficie.

​__›
​N​   ​__›
​__›
F
​ r  ​ F​
​   

_​ _›
Una caja tirada por una cuerda P​
​   
en dirección oblicua.

​__› ​__›
F
​ r  ​ ​N​  

_​ _›
Un cuerpo inmóvil sobre P​
​   
un plano inclinado.

Actividad grupal
Elaboremos un diagrama de cuerpo libre
Esteban mueve un carro tirándolo de una cuerda tal como se muestra en la ima-
gen. Mediante un diagrama de cuerpo libre, dibuja las fuerzas que actúan sobre
el carro.

¿Qué efectos provoca la fuerza neta sobre el carro?

92 Unidad 2 • Fuerzas
Ciencia, tecnología y sociedad
Desde hace ya varias décadas se viene trabajando sobre la idea de crear
un exoesqueleto que le permita al ser humano realizar tareas que, de otro
modo, le resultarían imposibles. Un exoesqueleto es una estructura que
se adosa al cuerpo humano desde fuera, ; de hecho, el prefijo de origen
griego exo- significa “fuera”.
Hoy en día, existen muchos prototipos de exoesqueletos que, gracias a
mecanismos robóticos y computarizados, incrementan la fuerza y la resis-
tencia del cuerpo humano. Uno de los principales usos que está teniendo
este tipo de tecnología se da en el ámbito de la medicina, ya que permi-
te trabajar la motricidad de personas que se encuentran con movilidad
limitada.
¿Cuáles crees que son los beneficios de implementar este tipo de
tecnología?

¿De qué manera el desarrollo científico se puede lleva a cabo a partir de


la observación de la naturaleza?

ión
ac
e tiz ca
ab tífi
Alf cien

Cierre de la misión
Al comienzo de la lección te planteamos que imagi-
naras la vida sin la acción de las fuerzas y que crearas ¿Esta misión se incluirá en el proyecto del texto?
un afiche creativo que expusiera su punto de vista en Fundamenta en la página 12.
una situación particular. ¿Cómo les fue en la creación
de este afiche? ¿Cómo evaluarías los aportes reali-
zados por cada integrante del grupo? Fundamenta.

Reflexiono sobre lo que aprendí


Lee y comenta las siguientes preguntas con tus compañeros y compañeras
para saber si alcanzaste el propósito de la lección.

Contenido Habilidades/Estrategias Actitudes


¿Qué fuerzas están actuando en este mo- ¿Qué estrategias desarrollaste para llevar Al alcanzar los logros de esta lección, ¿qué
mento sobre ti? Fundamenta. a cabo la misión? sentiste?
¿Qué efectos provoca la fuerza neta que ¿De qué manera realizar diagramas te per- ¿Cómo evaluarías tu proceso de apren-
actúa sobre ti? mitió comprender mejor los aprendizajes dizaje en esta lección?, ¿cómo podrías
de esta lección? mejorar?

Física • 2.° Medio 93


Integro lo que aprendí
Evaluación de proceso

Representa
1 Verónica arrastra una caja sobre una superficie horizontal, tal como se mues-
tra en la imagen. A partir de lo anterior, realiza un diagrama de cuerpo libre,
que muestre las fuerzas que actúan sobre la caja. Señala a qué fuerza corres-
ponde cada una.

Compara
2 Para comprender las características de las fuerzas, Sebastián representa grá-
ficamente dos fuerzas que actúan sobre un cuerpo.
​__›
F​
​ 1  

​__›
F​
​ 2  
¿Qué elemento(s) de los vectores asociados a dichas fuerzas es (son) igual(es)?
Compara
3 Silvana y Juan aplican dos fuerzas sobre un objeto de 2 (kg) que se encuen-
tra apoyado en el suelo. Respecto de esta situación, elaboran el siguiente
diagrama de cuerpo libre:

__› __›
F​1  = -5 N F​2  = 10 N

a. ¿Cuál es la fuerza neta sobra el objeto?


b. ¿Qué efecto provoca la fuerza neta sobre el objeto? Fundamenta.
c. ¿Actúan fuerzas verticales sobre el objeto? Argumenten.

94 Unidad 2 • Fuerzas
Analiza
4 Sobre el suelo se ubica un mueble de madera de 3 kg de masa y sobre él una
planta, cuya masa es de 0,5 kg.
a. ¿Cuál es la magnitud de la fuerza normal ejercida por el piso sobre el
mueble?
b. ¿Cuál es el valor de la fuerza normal ejercida sobre la planta por el mueble?
Analiza
5 Isabel aplica una fuerza (horizontal) de módulo 250 N sobre una caja de
80 kg de masa que está en reposo sobre una superficie, tal como se repre-
senta en la imagen. Considera μc = 0,25 y μe = 0,35.
a. Determina si la caja se moverá o permane-
cerá en reposo.
b. Determina el valor de la fuerza de roce.
Sintetiza
6 Construye un organizador gráfico que relacio-
nes los aprendizajes de esta lección. Para ello,
considera los siguientes términos: fuerza, fuer-
za neta, peso, tensión normal, fuerza de roce,
fuerza restauradora, ley de Hooke, vector, efec-
tos, cambios de forma, cambios de estado del
movimiento.

¿Cómo voy?
Revisa tus respuestas y según los resultados que hayas obtenido, marca con ✓ el
nivel de desempeño correspondiente. Si es necesario, pídele ayuda a tu profesor
o profesora.
Indicador Ítem Habilidad Nivel de desempeño
Reconocí los elementos asociados a la 1y2 Representar y comparar. L: Dos ítems correctos.
representación de una fuerza.
ML: Un ítem correcto.
PL: Ningún ítem correcto.
Apliqué los modelos matemáticos para 3, 4, y 5 Aplicar, clasificar y analizar. L: Tres ítems correctos.
identificar los tipos de fuerzas en dife-
ML: Dos ítems correctos.
rentes situaciones.
PL: Uno o ningún ítem correcto.
L = Logrado; ML = Medianamente logrado; PL = Por lograr.

Reflexiono sobre mi desempeño


Según tu apreciación (1: en desacuerdo; 2: ni de acuerdo ni en desacuerdo; 3: de
acuerdo), marca con ✓ las siguientes afirmaciones:
1 2 3
He analizado satisfactoriamente situaciones en las que actúan las fuerzas.
Han sido efectivos mis métodos de estudio.
He trabajado de forma rigurosa y ordenada.

Física • 2.° Medio 95


Lección
Explicando las leyes de Newton
Propósito de la lección

Así como las fuerzas se encuentran presen- En esta lección aprenderás a dar explica-
tes en cada acción que realizamos, existen ciones a diversas situaciones que ocurren
leyes naturales que nos permiten explicar los cotidianamente valiéndote de las leyes de
fenómenos que ocurren en nuestro entorno. Newton. Ello, con la intención, por un lado, de
En forma particular, las leyes de Newton, nos que valores los aportes de mujeres y hombres
permiten entender los fenómenos que ocu- en el desarrollo de la ciencia y, por otro, de
rren producto de la acción de las fuerzas. que identifiques las implicancias éticas de los
avances científicos y tecnológicos.

CIENCIA en

Ciencia al día CHILE


La biomecánica deportiva
La biomecánica consiste en la investigación de los fenóme-
nos biológicos a través de procedimientos de la mecánica
y su objetivo es estudiar las fuerzas internas y externas
(cinética) y los movimientos asociados que afectan al ser
humano y a los animales (cinemática). Esta ciencia se ha
desarrollado durante los últimos 90 años y durante las
tres últimas décadas su enfoque y mayor impacto se ha
centrado en los deportes.
La biomecánica deportiva basa su atención en la me-
cánica del cuerpo humano en diferentes actividades,
como correr, saltar o andar en bicicleta; de tal manera
que los conocimientos mecánicos involucrados en estas
actividades proporcionen una base científica para obtener
un máximo rendimiento o indagar en los mejores imple-
mentos u objetos para el deportista.
En la actualidad, las aplicaciones derivadas de la biomecánica
han mejorado la calidad de vida e muchísimas personas: atletas o
no atletas, discapacitados o no discapacitados, niños y personas mayores,
etc. En Chile, por ejemplo, Jorge Zuñiga, doctor en fisiología mecánica, utilizó El Dr. Jorge Zúñiga junto a la
sus conocimientos para crear una prótesis de mano que es utilizada por muchos mano biomecánica.
niños.
¿Qué opinas sobre este tipo de tecnología? ¿Cómo crees que ayuda el desarrollo
de la biomecánica a las personas con discapacidad?

n
ció
za
b eti ífica
a t
Alf cien

Fuente: Izquierdo, M. (2008). Biomecánica y bases neuromusculares de la actividad física y el


deporte. Madrid: Médica Panamericana (Adaptación)

Conicyt. (Junio 16, 2016). Jorge Zúñiga, Dr. en fisiología mecánica: el científico inventor
chileno que decidió donar su creación al mundo. Marzo 9, 2017, de Explora Sitio web: http://
www.conicyt.cl/explora/2016/06/16/jorge-zuniga-dr-en-fisiologia-mecanica-el-cientifico-
inventor-chileno-que-decidio-donar-su-creacion-al-mundo/

96 Unidad 2 • Fuerzas
Ciencia, tecnología y sociedad

La fuerza G
La resistencia humana a las grandes aceleraciones es Gracias a las investigaciones de Stapp, en la actualidad
generalmente medida en fuerzas G y el límite humano existe una ley que obliga a los fabricantes de vehícu-
está determinado por la resistencia del cerebro y otros los a incluir cinturones de seguridad en ellos. Por otra
tejidos blandos a las aceleraciones o desaceleraciones parte, los pilotos y astronautas se someten a un duro
bruscas. Si bien hay personas que pueden desmayarse entrenamiento y deben pasar rigurosas baterías de
en una montaña rusa, siendo sometidas a alrededor pruebas, para demostrar que pueden soportar eleva-
de 3 o 4 G, hay otras, como el coronel John Stapp, das fuerzas G, aunque ninguna tan extrema como las
que soportó decenas de fuerzas G durante cortos pe- realizadas por Stapp.
ríodos de tiempo, aunque con graves riesgos para su
¿Qué medidas de seguridad debió
integridad física.
considerar Stapp en sus pruebas?
John Stapp era un doctor militar que estaba tan inte-
resado en averiguar los límites humanos que puso en
riesgo varias veces su vida al realizar misiones prácti-
camente suicidas en nombre de la ciencia. Uno de sus
experimentos fue probar los efectos de las desacele-
raciones repentinas en el cuerpo humano, su objetivo
era encontrar métodos para mantener a los pilotos con
vida en caso de que se estrellaran. La ciencia fijaba el
límite humano en 18 G, pero esto nunca había sido
probado rigurosamente.
El 30 de abril de 1947 Stapp puso a prueba un des-
acelerador humano similar a las que se realizan actual-
mente en automóbiles (ver imágen). Después de varias
pruebas con un muñeco de seguridad, Stapp decidió
ser él quien experimentara la desaceleración. Para junio
de 1951, había realizado decenas de pruebas, sufriendo
varias fracturas. En 1954, durante su prueba más extre-
ma, desaceleró desde los 1 017 km/h a 0 km/h en 1,4 s, Fuente: Stapp. J. (1957). Human tolerance to deceleration.
soportando 43 veces su propio peso, es decir 43 G. The American Journal of Surgery, 93 (4), 734-740.

Inicio de la misión
En esta lección, tendrás la misión de escribir la letra de
una canción que explique las leyes de Newton. Para ello,
solicita la colaboración de tu profesor o profesora de Músi-
ca para que te guíe en la construcción de la letra utilizando
la melodía de la canción que estimes conveniente.
Para llevar a cabo esta misión, reúnete con dos compañe-
ros. Luego, en el siguiente espacio, planifiquen su estrate-
gia de trabajo, considerando que, al finalizar esta lección,
deberán presentar su canción al curso.

Física • 2.° Medio 97


Lección 2

Tema 1 Estableciendo la primera ley de Newton


¿Sabes quién es Newton?, ¿qué
Seguramente en más de una ocasión has sentido esa sensación de ser impulsado sabes de sus leyes?
hacia adelante cuando el vehículo en que viajas se detiene bruscamente. ¿Por qué
crees que ocurre esto?
En este tema, podrás explicar diversas situaciones cotidianas en las que se experi-
menta esta ley, entendiendo la importancia de este aporte al conocimiento científico.

Indaguemos a partir de nuestros aprendizajes previos Actividad grupal

Objetivo: Observar la tendencia de En grupos de tres integrantes, consigan un vaso, una moneda, un naipe y un
los cuerpos a mantener su estado de celular. Luego, realicen el siguiente procedimiento:
movimiento.
1. Ubiquen el naipe sobre el vaso, y sobre este último la moneda. ¿Qué pasará
Habilidad: Explicar las características con la moneda si se saca el naipe? Elaboren una hipótesis.
de un suceso.
2. Un integrante tome con los dedos la punta del naipe y tire de él con un movi-
Actitud: Establecer preguntas para
comprender mejor el mundo que nos miento rápido, tal como se representa en la imagen. Observen lo que sucede
rodea. con la moneda.
Tiempo: 20 minutos. 3. Empleando sus celulares, graben un video de la experiencia y compártanlo
con el resto de su curso a través de las diferentes redes sociales.
Luego, respondan las siguientes preguntas:
a. ¿Qué conceptos ya estudiados están presentes en la actividad?

b. ¿Pasó lo que esperaban al sacar el naipe? Expliquen.

c. ¿Cómo explicarían el fenómeno producido? Recuerden que pueden utilizar


el video de la experiencia para observar y analizar el suceso.
Fotografía del procedimiento
de la actividad.

d. ¿Qué otros fenómenos cotidianos podrían explicar a partir de esta


experiencia?

e. ¿Cuál puede ser la pregunta de investigación de esta experiencia?, ¿de qué


manera consideras que preguntas como estas permiten realizar aportes al
conocimiento científico?

Tal como experimentaste en la actividad anterior, al quitar el naipe la moneda


permanece en su posición, ya que conserva su estado inicial de reposo, cayendo
dentro del vaso. Esta tendencia de los cuerpos de mantener su estado, ya sea de
reposo o movimiento, se conoce como inercia.

98 Unidad 2 • Fuerzas
Al tirar de manera horizontal del naipe, se puede observar que la fuerza de roce
entre el naipe y la moneda tiende a desplazarla levemente. Sin embargo, a pesar
del movimeinto del naipe, la moneda mantiene su inercia y esta cae al vaso por
efecto de la gravedad.

Representación de la inercia de la moneda.

Entonces, ¿Qué es la inercia? Es una propiedad que tienen los cuerpos para man- CONTEXTO HISTÓRICO
tener su estado de reposo o movimiento a no ser que una fuerza externa actúe
sobre él. Esta propiedad se establece en la primera ley de Newton o principio de En 1687, el físico y matemático
inercia, la cual postula lo siguiente: inglés Isaac Newton formuló tres
leyes fundamentales de la dinámica
Un cuerpo permanecerá en estado de reposo o de movimiento rectilí- (la rama de la física que estudia el
neo uniforme si no actúa ninguna fuerza sobre él o si la resultante de movimiento de los cuerpos en rela-
las fuerzas que actúan es nula. ción con las fuerzas que lo modifi-
can). La importancia de estas leyes
Es importante aclarar que la masa de un cuerpo es una medida de su inercia, es tal, que a partir de ellas se pudo
ya que mientras mayor sea su masa, más fuerza se necesitará para modificar su explicar un sinfín de fenómenos:
estado de movimiento. desde el movimiento de los cuerpos
presentes en nuestro entorno, hasta
el de las partículas subatómicas.

Actividad grupal
Identifiquemos las características de la inercia
Un padre y su hijo se encuentran en reposo dentro de un
bus de metro que se encuentra detenido en la calle. Al res-
pecto, respondan las siguientes preguntas:
a. ¿Cuál de los dos posee mayor inercia?, ¿por qué?
b. ¿Qué pasará con los cuerpos si el vehículo se pone en
marcha? Fundamenten.
c. ¿Cómo será la sensación de cada uno cuando el bus
acelera? Comparen.
d. Considerando la época de Newton, ¿de qué mane-
ra crees que influyó este postulado en la comunidad
científica? ¿Por qué este principio ha permanecido
hasta la actualidad?

Física • 2.° Medio 99


Lección 2

Tema 2 Aplicando la segunda ley de Newton


¿Has notado la diferencia entre trasladar una mochila con muchos libros y ¿Cómo crees que se relacionan la
una mochila vacía? Probablemente lo has tenido que hacer muchas veces: fuerza, la masa y la aceleración?
Menciona un ejemplo.
¿en qué caso resulta más fácil moverla?
En este tema, podrás establecer, por medio de la observación y la ex-
perimentación, la relación que encontró Newton entre los conceptos de
fuerza, masa y aceleración. Para ello, realizarás diversas actividades con-
siderando sus respectivas medidas de seguridad.

Indaguemos a partir de nuestros aprendizajes previos Actividad grupal

Objetivo: Comprender la relación que En grupos de tres integrantes, consigan un auto de juguete, un metro de lana,
existe entre la fuerza aplicada y la un vaso plástico, un cilindro (pueden utilizar un pegamento en barra) y piedras
masa de un cuerpo. pequeñas. Luego, realicen el siguiente procedimiento:
Habilidad: Formular explicaciones, 1. Armen el montaje que aparece en la imagen del costado y ubiquen el auto a
apoyándose en las observaciones y
un metro del borde de la mesa.
conceptos científicos.
Actitud: Establecer medidas de 2. Dentro del vaso introduzcan una a una las piedras hasta que observen que el
seguridad para un procedimiento auto comienza a moverse.
experimental.
3. Observen cómo se desplaza el auto. Si lo desean, pueden grabar con sus
Tiempo: 30 minutos. celulares la experiencia.
4. Repitan los pasos anteriores, pero esta vez aumenten la cantidad de piedras
que introducen en el vaso.
5. Reiteren este procedimiento las veces que consideren necesarias para esta-
blecer resultados confiables.
Luego, respondan las siguientes preguntas:
a. ¿Qué tipo de movimiento describe el auto? Fundamenten.

b. ¿Qué ocurre con el auto a medida que aumenta la masa?

Montaje de la actividad.
c. ¿Qué tipo de fuerza provoca el movimiento del auto?

Reduce, Reutiliza, Recicla


¿Qué harán con los materiales d. ¿Qué medidas de seguridad se pueden considerar en esta actividad?
al finalizar la experiencia?

e. ¿Qué hubiera ocurrido si en el montaje se hubieran ubicado todas las pie-


dras juntas de una vez? ¿Por qué es necesario adoptar medidas de segu-
ridad cuando se realizan actividades experimentales?

En la actividad anterior pudiste notar que al aumentar la cantidad de piedras que


se introducían en el vaso, el auto se movía más rápido; pero ¿cómo se relacionan
estos conceptos con la segunda ley de Newton?

100 Unidad 2 • Fuerzas


Para comprender la segunda ley de Newton, supongamos que, una persona
ejerce fuerzas de igual magnitud sobre dos bloques del mismo material y que se
encuentran sobre superficies similares (suponer sin roce), tal como se representa
en las siguientes imágenes:

m 1 = 20 kg
m 2 = 2 kg

Claramente, al aplicar una fuerza similar sobre ambos bloques, el de menor masa
acelerará más y, por lo tanto, recorrerá una distancia mayor.
Similar a la actividad anterior, Newton observó que el efecto de una fuerza de-
pende de las características del cuerpo sobre el cual se ejerce, en particular de
su masa. Enunció entonces la segunda ley de Newton, también conocida como
principio de las masas, la cual plantea lo siguiente:

Si sobre un cuerpo actúa una fuerza neta, este adquirirá una acelera-
ción directamente proporcional a la fuerza aplicada, donde la masa
del cuerpo es la constante de proporcionalidad.

La aceleración producida tiene la misma dirección y sentido que la fuerza neta,


por lo que este principio se puede escribir matemáticamente de la siguiente
forma:

Fuerza neta ​​ →  ​​  
​​F  ​​  = m ‧ a Aceleración

Masa
En el ejemplo anterior, si la fuerza aplicada por la persona tiene una magnitud
de 40 N y si se desprecia el roce, el módulo de la aceleración que adquirirá cada
bloque, de acuerdo a la segunda ley de Newton, será:

→ →
   
→ →
​F  ​neta 40 N  ​F  ​neta 40 N ​​ = 20 m/​​s​​  2​​
​​   = ​​ ____
a m1 ​​   = ​​  _____  ​​ = 2 m/​​s​​  2​​ ​​   = ​​ ____
a m2 ​​   = ​​  _____
20 kg 2 kg

Física • 2.° Medio 101


Lección 2

Desarrollo de estrategias
Aprendiendo a aplicar el segundo principio de Newton.

Situación problema

Para cambiarse de casa, Patricio ejerce una fuerza de 50 N so-


bre un sistema compuesto por dos cajas, A y B, de masas 8 kg
y 2 kg, respectivamente. ¿Cuál es la aceleración del sistema?, A
¿cuál es el valor de la fuerza que actúa sobre la caja A?, ¿cuál
sobre la caja B? (Supón que no hay roce).

​​F  ​​  = 50 N B
PASO 1 Identifico las incógnitas
Para determinar la aceleración del sistema, debemos consi-
derar la masa total y aplicar la expresión que da cuenta del
segundo principio de Newton:


​​F  ​​neta ​​→​​  
= m ‧ a 
La aceleración de cada caja es la misma que la del sistema.
Sin embargo, la fuerza que actúa sobre cada una de ellas
es distinta. Para determinar esta última, se debe conocer la
aceleración y la masa de cada una de las cajas.

PASO 2 Registro los datos


Masa caja A: mA = 8 kg
Masa caja B: mB = 2 kg
Módulo de la fuerza: F = 50 N

PASO 3 Aplico los modelos


En primer lugar, determinaremos el módulo de la aceleración del sistema mediante
la siguiente expresión:
Fneta = m · a
Considerando que el valor de la masa del sistema es mA + mB, resulta:
Fneta = (mA + mB) · a
Despejando la aceleración, se obtiene:
Fneta
a = _______
​​ m + m  ​​ 
A B

Al reemplazar los datos en la expresión anterior, resulta:


Fneta 50N  ​​ = 5 m/s2​ ​​
a = _______
​​  +
m m = ​​ _________
  ​​    
A B 8 kg + 2kg
Luego, para determinar el módulo de la fuerza que actúa sobre la caja A, nuevamen-
te empleamos la segunda ley de Newton.
FA = mA · a

102 Unidad 2 • Fuerzas


Remplazando los valores, se obtiene:
FA = mA · a = 8 kg ‧ 5 m/s2 = 40 N
Para calcular el módulo de la fuerza sobre la caja B, realizamos un procedimiento
similar al anterior:
FB = mB · a = 2 kg ‧ 5 m/s2​​ = 10 N

PASO 4 Escribo la respuesta


El módulo de la aceleración que experimenta el sistema formado por ambas cajas
las tic
es 5 m/s2. La magnitud de la fuerza que actúa sobre la caja A es de 40 N y la que
actúa sobre la caja B es de 10 N. Ingresa el código 18TF2M103a en
tu aula digital y utiliza el simulador
PASO 5 Aplico lo aprendido para identificar la acción de la
segunda ley de Newton.
En parejas, apliquen la segunda ley de Newton en las siguientes situaciones.
1. Jimena aplica una fuerza de 190 N sobre un sistema formado por dos cajas,
que produce que este se mueva con una aceleración de módulo 9,5 m/s2.
Determinen la masa de la caja A y la fuerza que actúa sobre cada una de las
cajas (supongan que no hay roce).

A 190 N
7 kg

2. Claudia tira, con una fuerza neta de módulo 60 N, un sistema formado por tres
cajas, tal como se representa en el siguiente esquema.

60 N
Cuerda
1 kg 7 kg 2 kg

Considerando que la masa de la cuerda y la fuerza de roce son despreciables,


determinen:
a. La aceleración que adquiere el sistema.
b. La fuerza que actúa sobre cada uno de los bloques.
c. La magnitud de la tensión de la cuerda.

Física • 2.° Medio 103


Lección 2

Tema 3 Estableciendo la tercera ley de Newton


¿Te ha pasado alguna vez que golpeas un objeto con tu mano e instantánea-
¿Podrías mencionar un ejemplo
mente sientes dolor producto del golpe? Algunas personas dirían que esto ocurre concreto de una acción y su
porque por cada acción que realizamos existe una reacción. Ahora bien, ¿cómo reacción?
se relaciona esto con las fuerzas?
En este tema, aprenderás sobre las fuerzas de acción y reacción a partir del aná-
lisis de diversas situaciones que se pueden explicar por medio de la tercera ley
de Newton, entendiendo la importancia de esta en el conocimiento científico.

Indaguemos a partir de nuestros aprendizajes previos Actividad grupal

Objetivo: Reconocer situaciones en En parejas lean la siguiente paradoja:


donde se apliquen fuerzas.
Un campesino debe trasladar una carreta bien cargada y, para ello,
Habilidad: Plantear explicaciones a le pide a su burro que lo ayude. Amarra la carreta al burro y le dice:
situaciones cotidianas.
“¡ya burro, vamos!” El burro, con toda su testarudez, le dice: “¡No
Actitud: Establecer preguntas para lo haré! He estudiado la tercera ley de Newton y descubrí que al
comprender mejor situaciones aplicarle una fuerza a la carreta, ella aplicará simultáneamente una
cotidianas.
fuerza de igual magnitud pero en sentido contrario, por lo tanto,
Tiempo: 20 minutos. ambas fuerzas se anularán haciendo imposible mover la carreta”. El
campesino, con la cara llena de sorpresa, le dice al burro: “yo no sé
de Newton, pero sí sé que llevarás mi carreta, así que camina”. Y el
burro caminó y la carreta lo siguió.
Debatan en torno a lo leído y respondan las siguientes preguntas:
1. ¿Han oído hablar sobre la tercera ley de Newton? De ser así, explíquenla
brevemente.

2. ¿Por qué el burro pudo mover la carreta?

3. ¿En qué situaciones se aplica este mismo problema? Describan otras dos
situaciones.

4. ¿Cuál puede ser la pregunta de investigación para establecer el error del burro?

La tercera ley de Newton, a la que hace referencia el burro en la paradoja, esta-


blece que si se aplica una fuerza sobre un cuerpo (fuerza de acción), el cuerpo
también aplicará una fuerza de igual módulo pero en sentido contrario (fuerza
de reacción). Pero ¿en qué se equivocó el burro?

104 Unidad 2 • Fuerzas


Son varias las situaciones en las que se puede observar un par de fuerzas. Por
ejemplo, cada vez que te apoyas sobre una mesa o un muro, ejerces una fuerza.
Sin embargo, la superficie sobre la que te apoyas también ejerce una fuerza
sobre ti. A partir de esto, Newton planteó que nunca una fuerza se ejerce sobre
“la nada”, es decir, en la naturaleza, toda fuerza o acción va acompañada de su
correspondiente reacción. Esta afirmación se recoge en la tercera ley de Newton
o principio de acción y reacción, que plantea lo siguiente:

Si un cuerpo A ejerce una fuerza sobre un cuerpo B, entonces, este


último ejercerá una fuerza de igual magnitud y dirección sobre A,
pero en sentido opuesto.

Lo anterior se expresa de la siguiente manera:


→ →
   
Fuerza ejercida Fuerza ejercida
​​F  ​​A/B = -​​F  ​​B/A
por A sobre B por B sobre A

El signo menos (−) indica que el sentido de una fuerza es


opuesto al de la otra. Se dice que estas fuerzas forman un
par acción-reacción y que actúan siempre de forma simul-
tánea y nunca se anulan, ya que se ejercen sobre cuerpos
distintos.
La aplicación más directa de la tercera ley de Newton se
puede apreciar con claridad en el lanzamiento de cohetes,
pues para despegar, el cohete ejerce una fuerza sobre los
gases que expulsa y los gases ejercen una fuerza igual y
opuesta sobre el cohete.
También es posible observar la tercera ley en situaciones
más simples y cotidianas, como caminar. En esta acción, una
persona puede avanzar porque, cuando un pie empuja ha-
cia atrás contra el suelo, este empuja hacia delante sobre
el pie.
Actividad individual
Explico la tercera ley de Newton
¿En qué se equivocó el burro? Ahora que ya sabes cómo se
aplica la tercera ley de Newton, explícale al burro de la para-
doja cuál era su error en el planteamiento de su argumento
para no mover la carreta.

Lanzamiento de un cohete.

Desarrollo de la misión
Ahora que ya conoces las leyes de Newton, reúne-
te con tu grupo de trabajo y, junto con la ayuda de
su profesor o profesora de Música, escriban la letra
de su canción de acuerdo a la melodía que escogie-
ron. Graben la canción para presentarla al resto del
curso. ¿Qué estrategia les resultó más efectiva para
llevar a cabo esta misión?

Física • 2.° Medio 105


Lección 2

Las leyes de Newton actuando en conjunto


Aunque se podría pensar que cada una de las leyes de Newton responde a una
situación en particular, en la realidad estas coexisten en cualquier fenómeno que
involucre la acción de fuerzas. Para constatar este hecho, analicemos el siguiente
ejemplo, en el que un joven quiere trasladar una caja.

Inicialmente, dado que no se ha ejercido una fuer-


za sobre el cuerpo, la fuerza de roce es nula. Acá es
evidente el primer principio de Newton en acción: la
fuerza neta sobre la caja es nula, por lo que esta se
mantiene en reposo.


La fuerza de roce irá aumentando conforme la fuerza
​​  F ​​  aplicada aumente, pero el cuerpo se mantendrá en re-

​​ F’ ​​  poso mientras esta sea menor que la fuerza de roce es-
tático máxima.
En este caso, es posible identificar cómo se van generan-
do los pares de fuerzas descritos por el tercer principio de
Newton: la persona tira de la caja y la caja “tira” de ella
con la misma intensidad. Además, dado que la fuerza se
transmite a través de la cuerda, se genera otro par de ac-
ción y reacción en la interfaz del cuerpo y la superficie de
apoyo: el cuerpo empuja la superficie hacia adelante, y la
superficie “empuja” el cuerpo hacia atrás (esta es la fuerza
de roce). El primer principio de Newton permite explicar el
→ reposo del cuerpo: las fuerzas que actúan sobre el cuerpo
​​R’ ​​   → →
(​​ F ​​ y ​​ R’ ​​) se equilibran, por lo que la fuerza neta sobre el
→ cuerpo es nula. Es importante aclarar que aunque la fuer-
​​R ​​  
za neta sea nula no implica que el cuerpo esté en reposo.


Una vez que la fuerza aplicada supera el valor máximo F 
​​  ​​  
de la fuerza de roce estático, se genera una fuerza neta
no nula, puesto que el roce del bloque con el suelo ya no
puede equiparar en magnitud a la fuerza ejercida por la
persona sobre la superficie.
El segundo principio de Newton plantea que la existen-
cia de una fuerza neta implica que el cuerpo adquiere
una aceleración en el mismo sentido de esta fuerza, la
que puede ser determinada conociendo la magnitud de →
​​ R’ ​​  
la fuerza neta y la masa del cuerpo. Por lo tanto, el cuer-
po sale del reposo.

​​​  ​​   neta​​​
F 
Los pares de acción-reacción siguen existiendo, pero en
este caso, a diferencia del caso equilibrado anterior, las
→ → → →
magnitudes de ​​ F ​​  y ​​ F’ ​​ son distintas a las de ​​ R ​​  y ​​ R’ ​​. 

106 Unidad 2 • Fuerzas


Ciencia, tecnología y sociedad

El funcionamiento del cinturón de seguridad puede ser explicado median-


te los principios de Newton. Cuando el automóvil en el que viajamos se
detiene bruscamente, nuestro cuerpo tiende a seguir en movimiento a la
misma velocidad que tenía el vehículo (principio de inercia). El cinturón de
seguridad evita que salgamos expulsados en la misma dirección y sentido
con los que viaja el automóvil.
Debate con tus compañeros sobre la importancia de usar cinturón de se-
guridad al viajar en un vehículo y cómo la investigación científica permite
realizar avances que ayudan a la sociedad.
Luego, propongan un slogan que incentive el uso del cinturón de seguri-
dad en los conductores.

las tic

n Ingresa el código  18TF2M107a en


ció tu aula digital y observa el video que
t i za a
e c
ab tífi ahí se muestra para identifiques la
Alf cien
importancia del uso del cinturón de
seguridad.

Cierre de la misión
Al comienzo de esta lección te propusimos escribir la
letra de una canción para explicar las leyes de New- ¿Esta misión se incluirá en el proyecto del texto?
ton. ¿Qué estrategias desarrollaste para llevar a cabo Fundamenta en la página 12.
esta misión?
¿Cuál fue la recepción del curso al presentar la can-
ción?, ¿cómo evaluarías el trabajo realizado?

Reflexiono sobre lo que aprendí


Lee y comenta las siguientes preguntas con tus compañeros y compañeras
para saber si alcanzaste el propósito de la lección.

Contenido Habilidades/Estrategias Actitudes


¿Qué situaciones cotidianas compren- ¿Consideras que escribir una can- ¿Qué implicancias éticas consideras que tienen los
diste gracias a las leyes de Newton? ción es una buena estrategia de avances científicos y tecnológicos?
Menciona un ejemplo en cada caso. aprendizaje?, ¿qué otras estrate- ¿Qué importancia tienen los aportes realizados por
¿Qué contenidos aprendiste en esta gias utilizaste satisfactoriamente científicos en la evolución del conocimiento y la com-
lección? en esta lección? prensión del mundo?

Física • 2.° Medio 107


Integro lo que aprendí
Evaluación de proceso

Aplica
1 Un cañón de 400 kg, dispuesto en una superficie hori-
zontal, dispara una bala de masa 20 kg, proporcionán-
dole una aceleración de 30 m/s2, tal como se representa
en la imagen. ¿Qué aceleración adquiere el cañón de-
bido a la fuerza ejercida por la bala? Supón que el roce ? 30 m/s2
entre las ruedas del cañón y el suelo es despreciable.

Analiza

2 Cuando Paz aplica una fuerza ​​F ​​  sobre una caja de masa m, esta se mueve
con una aceleración a. ¿Qué ocurrirá con la aceleración si se desprecia el
roce y Paz duplica la fuerza ejercida sobre la caja? Redacta una explicación y
fundaméntala con expresiones matemáticas.

Explica
3 Enuncia un ejemplo, distinto a los mencionados en el texto, en el que pue-
das evidenciar cada ley de Newton. Fundamenta cada ejemplo.

Primera ley de Newton Segunda ley de Newton Tercera ley de Newton


Situación

Fundamento

108 Unidad 2 • Fuerzas


Reflexiona
4 Lee la siguiente información y responde las preguntas.

El funcionamiento del cinturón de seguridad puede ser explicado mediante


los principios de Newton
Cuando viajamos en automóvil y este se detiene bruscamente, nosotros tende-
mos a seguir en movimiento, a la misma velocidad que tenía el vehículo.
El cinturón de seguridad evita que salgamos expulsados en la misma dirección
y sentido con los que viaja el automóvil.

a. ¿Por qué es importante el uso del cinturón de seguridad en los automóviles? Explica utilizando
conceptos científicos.

b. ¿Cómo se evidencian las tres leyes de Newton en el funcionamiento del cinturón de seguridad?
Si lo deseas puedes buscar información adicional en Internet sobre las características y el fun-
cionamiento del cinturón de seguridad y completa la siguiente tabla.
Primera ley de Newton Segunda ley de Newton Tercera ley de Newton

¿Cómo voy?
Revisa tus respuestas y, según los resultados que hayas obtenido, marca con ✓ el nivel de
desempeño correspondiente. Pídele ayuda de tu profesor o profesora.
Indicador Ítem Habilidad Nivel de desempeño
Explicar situaciones cotidianas por 1, 2, 3 y 4 Aplicar, analizar, explicar y L: cuatro ítems correctos.
medio de las leyes de Newton. reflexionar.
ML: dos o tres ítems correctos.
PL: uno o ningún ítem correcto.
L = Logrado; ML = Medianamente logrado; PL = Por lograr.

Reflexiono sobre mi desempeño


Según tu apreciación (1: en desacuerdo; 2: ni de acuerdo ni en desacuerdo; 3: de acuerdo),
marca con ✓ las siguientes afirmaciones:
1 2 3
He explicado satisfactoriamente las leyes de Newton por medio de ejemplos.
Han sido efectivas las estrategias utilizadas para aprender.
He reconocido la importancia de los aportes científicos en laevolución del conocimiento.

Física • 2.° Medio 109


La CIENCIA se construye

¿Cómo ha evolucionado el concepto de fuerza?


En la unidad aprendiste que no se puede producir ningún cambio, movimiento o ac-
tividad en un objeto dado sin una causa que lo genere, es decir, sin una fuerza que
sea la responsable. Esta conclusión se ha desarrollado a lo largo de la historia gracias
a la contribución de diferentes personajes que caracterizaron el concepto de fuerza.

Aristóteles William Gilbert Johannes Kepler


(384–322 a. C.) (1544–1603) (1571–1630)
Filósofo e investigador de la antigua Físico y médico inglés, dió luces del Astrónomo, matemático y físico
Grecia, estableció que la fuerza es concepto de fuerza eléctrica rela- alemán. En un comienzo planteó
la causa del movimiento (como ti- cionándolo con el fenómeno de que la fuerza es el espíritu que ani-
rar o empujar) y postuló que para atracción que se producía al frotar ma a los cuerpos celestes y dirige
que, un cuerpo permanezca en ciertos materiales. A partir de esta sus movimientos. Luego, en 1608,
movimiento, hay que aplicarle una idea, postuló que las fuerzas que describió la fuerza de atracción
fuerza. Por lo tanto, si se le deja mantienen a los planetas en su mo- ejercida por la Tierra sobre un
de aplicar la fuerza, el cuerpo se vimiento orbital alrededor del Sol se objeto como una línea o cadena
detiene. podían explicar como el resultado magnética.
de una atracción magnética.

Siglo IV a. C. Siglo XVI y XVII

En el mundo En el mundo
En esa época, Alejandro Magno Luego del descubrimiento de América, a finales del siglo XV, se sucedieron las
lanzó su ejército contra el poderoso grandes exploraciones españolas y portuguesas por el Nuevo Mundo, el Pacífico,
y extendido Imperio persa. En Me- Asia, etc.
soamérica se desarrolla la escritura En Chile
zapoteca.
Hubo dos grandes terremotos: el de Concepción en 1570 y el que afectó a toda
la Zona Sur en 1575, que destruyó las ciudades ubicadas al sur del Biobío.

Importante
te,
¿Qué ley, actualmen
e pe nsa miento?
contradice est El concepto de “ciencia” se acuña recién en el siglo XIX, por lo que an-
tes de aquella época los investigadores que trabajaron desarrollando
teorías en torno a ella, no se pueden denotar técnicamente científi-
cos. Sin embargo, entenderemos como tal a los personajes señalados
en esta línea de tiempo y a lo largo del texto.

110 Unidad 2 • Fuerzas


Hasta el momento no se define qué es ¿Por qué crees que la ima
la fuerza; sin embargo, se tiene claridad gen de
las mujeres en ciencias se
de sus efectos. Considerando la época, hace más
evidente después del sigl
¿podrías indicar algunos de ellos? o XIX?

Galileo Galilei ¿Quién es el científico? Averigua: Emmy Noether


(1564–1642) (1642–1727) (1882-1935)
Físico y astrónomo italiano. Investigador, físico y matemático Destacada matemática y física ale-
Construyó el concepto de fuerzas inglés que postuló que el movi- mana. Albert Einstein la consideró
a partir de varias ideas. En la miento de un cuerpo se caracteriza como la mujer más importante en
primera, planteó que la fuerza era por su velocidad y su masa. Ade- la historia de la matemática. En
como una presión o transición que más, basándose en los trabajos de física, trabajó en las leyes de con-
mueve al cuerpo en la dirección Galileo, reflexionó sobre el hecho servación, cuyos aportes permi-
contraria a la que se movería de de que los objetos “pesaban” en la tieron resolver ciertos aspectos de
manera natural. Luego, planteó Tierra y que los cuerpos celestes la teoría general de la relatividad,
una nueva idea en que la fuerza giraban en torno a otros cuerpos que, entre otras cosas, describe
era la causa del movimiento (tal celestes. Esto lo llevó a establecer cómo la fuerza de gravedad afecta
como Aristóteles), pero la formalizó las tres leyes de la dinámica. el espacio-tiempo.
matemáticamente a través de los
conceptos de masa y velocidad.

Siglo XVII Actualidad

La física del siglo XXI busca unificar las fuerzas fundamentales en la llamada “teoría
unificada de campos”. Sin embargo, aún persiste una serie de interrogantes que man-
tienen a esta ciencia en un camino de constante búsqueda.

Trabaja con la información

1 Averigua sobre las dificultades que tuvieron los científicos del Renacimiento (siglo XIV al XVI) para
presentar sus investigaciones y publicaciones. ¿Qué actitud hubieses tenido ante esas dificultades?
Puedes asesorarte por tu profesor o profesora de Historia, Geografía y Ciencias Sociales.
2 ¿De qué manera la curiosidad por explicar los fenómenos cotidianos ha influido en el avance cientí-
fico y tecnológico? Explica.
3 Si pudieras viajar al pasado, ¿a qué científico o científica te gustaría conocer?, ¿qué le dirías?

Física • 2.° Medio 111


Síntesis
Elaborando una TELA DE ARAÑA para ordenar
los aprendizajes
Detall
La tela de araña es un organizador gráfico que permite es- Detalle e
tablecer relaciones entre un concepto central y las ideas que Detall
e
se relacionan con él. Las telas de araña proporcionan una Subidea
Detall
e
estructura que prioriza la información desde lo más general, Subidea
con la idea central, a lo más específico con los detalles, tal e
Detall Detall
e
como se muestra en la figura. Subidea

A continuación, te invitamos a conocer los pasos para cons- Subidea Idea central
truir una tela de araña que te servirá para implementar Detalle
como estrategia de estudio. Deta
lle
Subidea
Subidea
Detalle
Detalle Detall
e e
Detall

Definir la idea central


1
PASO Define el concepto principal que engloba los contenidos tratados en la unidad:

Identificar las subideas


2
PASO Pueden ser conceptos, definiciones, características, expresiones matemáticas o lo que te
parezca apropiado según el contexto. En este caso, las subideas pueden ser:

Fuerza Fuerza neta Primera ley de Newton Segunda ley de Newton


(Página 83) (Página 85) (Página 98) (Página 100)

Tercera ley de Newton Ley de Hooke Normal Peso


(Página 104) (Página 89) (Página 87) (Página 87)

Tensión Efectos de las fuerzas Fuerza de roce Fuerza elástica


(Página 87) (Página 83) (Página 88) (Página 89)

Relacionar las subideas con ciertos detalles


3
PASO Para progresar con la construcción de la tela de araña, incluye detalles de las subideas que
te permitan incluir los contenidos, habilidades y actitudes presentes en la unidad.

112 Unidad 2 • Fuerzas


Construye la tela de araña
4
PASO Completa el siguiente esquema representando los aprendizajes de la unidad.

lo
o de
r m ico
l i ca mát
Ap ate
m
Fuerza elástica

ón Precisió
si Fu n
Ten erza
Peso de
roc
e

Tipos Ley de Hooke


Norma
l

Efectos Fuerza

Reflexiona
5
PASO ✓ ¿Qué beneficios consideras que tiene este tipo de organizador gráfico?
✓ ¿De qué manera implementarías este organizador como estrategia de aprendizaje en
otras asignaturas?
✓ ¿Qué software o herramienta tecnológica usarías para elaborar una tela de araña?

Física • 2.° Medio 113


Evaluación final

Para que conozcas cómo va tu proceso de aprendizaje, te invitamos a realizar las siguientes actividades.

Representa
1 Ángela es la levantadora de pesas que conociste en el
inicio de la unidad. A partir de su imagen:
a. Representa las fuerzas que interactúan en la
situación.
b. ¿Qué tipo de roce se ejerce entre el pie y el suelo?

c. ¿Cuál es el valor de la fuerza neta?, ¿cómo lo


determinaste?

Aplica
2 Susana se encuentra sobre una patineta inicialmente en reposo y ejerce una fuerza de 700  N
sobre un muro para impulsarse. Como resultado, adquiere un movimiento hacia atrás. Si la masa
conjunta de Susana y su patineta es 70 kg, y la fuerza de roce estático máximo es prácticamente
nulo, responde las siguientes preguntas.
a. ¿Cuál es la aceleración que experimentó al empujar el muro?

b. ¿Cuál(es) ley(es) de Newton se ejemplifica(n)? Explica.

c. ¿Qué medidas de seguridad debería considerar Susana para andar en su patineta?

Explica
3 Si José, en la imagen, cae producto de la fuerza que
ejerce la Tierra sobre él, ¿es correcto afirmar qué José
atrae a la Tierra? Fundamenta tu respuesta a partir de
la tercera ley de Newton.
Aplica
4 ¿Cuál es el peso de una persona que posee 68 kg de
masa?

114 Unidad 2 • Fuerzas


Aplica
5 Kevin, es uno de los gimnastas que conociste al inicio de la
unidad. Si él tiene una masa de 58 kg.
a. ¿Cuál es su peso?
b. ¿Cuál es la tensión de cada una de las cuerdas que sos-
tienen las argollas si se encuentran verticales?
c. ¿Hay fuerza de roce en el ejemplo? Fundamenta.

Analiza
6 Seis niños juegan a tirar la cuerda y cada grupo lo hace con una fuerza de magnitud 125 (N).

a. ¿Cuál es el valor de la fuerza neta sobre la cuerda? Explica los efectos de este valor.
b. ¿Qué pasaría si la cuerda se rompe? Fundamenta.
Analiza
7 Alejandra deja su libro de Física sobre una superficie inclinada y nota que este se queda inmóvil. A
partir de esta situación, Alejandra elabora un diagrama de cuerpo libre, que represente las fuerzas
que interactúan sobre el libro, tal como se observa en la siguiente imagen:
​__›
R
​ ​  
​__›
M
​  ​ 

​__›
Q
​ ​  

¿A qué tipo de fuerza corresponde cada uno de los vectores representados?


→ → →
​​R ​​   ​​M​​    ​​Q ​​  

Física • 2.° Medio 115


Evaluación final
Evalúa
8 Renato y Amelia empujan simultáneamente una mesa durante cuatro segundos y en tres configu-
raciones distintas, tal como se observa en las siguientes imágenes.

Se sabe, además, que las fuerzas ejercidas por ambos tienen igual magnitud (aproximadamente
60 N) y que el roce entre las patas de la mesa y el suelo es prácticamente cero.
a. ¿En cuál de los casos la mesa acelerará más?, ¿en cuál menos? Fundamenta.
b. ¿Hacia dónde se moverá la mesa en cada uno de los casos? Elabora un diagrama de cuerpo
libre en cada caso.
Evalúa
9 Juan observa que dos basquetbolistas se em-
pujan mientras juegan un partido. A partir de
esta situación, le explica a Ana que ambos se
mantienen en reposo producto de las fuerzas
de acción y reacción de acuerdo a la tercera
ley de Newton. A partir de esta situación, de-
termina si la explicación de Juan es correcta
y fundamenta utilizando los aprendizajes ad-
quiridos en la unidad.

116 Unidad 2 • Fuerzas


Relaciono lo aprendido con Historia, Geografía y Ciencias Sociales
Ares es uno de los dioses del olimpo. Estos eran llamados así
porque se creía que vivían, de acuerdo a los antiguos grie-
gos, en un palacio situado en la cima del Monte Olimpo, que
es el monte más alto de Grecia. Cada uno de ellos tenía sus
propios atributos y representaba una fuerza de la naturaleza
o una idea. Ares, hijo de Zeus y Hera, es el dios guerrero por
excelencia y representa el dominio de la fuerza bruta sobre
la inteligencia. Es muy impopular entre los dioses; incluso le
resulta antipático a su propio padre. Es la antítesis de Ate-
nea, diosa también guerrera, que encarna la fuerza inteli-
gente y la astucia. ¿A qué crees que se referían los griegos
con fuerza natural, fuerza bruta y fuerza inteligente? ¿Cómo
se relacionan estos con la fuerza estudiada en física?

¿Cómo me fue?
Revisa tus respuestas y, según los resultados que hayas obtenido, marca con ✓ el nivel de desempeño
correspondiente. Pídele ayuda a tu profesor o profesora.

Indicador Ítem Habilidad Nivel de desempeño


Analizar los efectos de las fuerzas 1, 4, 5, 6, 7 Representar, interpretar, L: seis ítems correctos.
en diversos contextos. y8 aplicar, analizar y evaluar.
ML: tres a cinco ítems correctos.
PL: dos o menos ítem correcto.
Explicar las leyes de Newton en 2, 3 y 9 Aplicar, explicar y evaluar. L: seis ítems correctos.
situaciones cotidinas.
ML: tres a cinco ítems correctos.
PL: dos o menos ítem correcto.
L = Logrado; ML = Medianamente logrado; PL = Por lograr.

Reflexiono sobre lo que aprendí


¿Recuerdas las metas y estrategias planteadas al inicio de la unidad? Si no las recuerdas, vuelve a
revisar las páginas 78 y 79. Luego, responde las siguientes preguntas.
¿Consideras que comprendes mejor las ¿Te resultó útil escribir un glosario con los ¿De qué manera aplicaste los protoco-
situaciones que ocurren a tu alrededor con el aprendizajes de la unidad?, ¿cómo mejo- los y normas de seguridad al ejecutar
estudio de las fuerzas? ¿Cómo explicarías a rarías esta estrategia? los procedimientos experimentales y
un niño la idea de que la fuerza no se tiene? ¿Cumpliste tus metas? en todas las actividades propuestas en
la unidad?

Física • 2.° Medio 117


Bibliografía
Disciplinar Didáctica
◗◗ Cromer, A. (1981). Física para las ciencias de la vida. ◗◗ Adúriz-Bravo, A. (2005). Una introducción a la
Barcelona: Reverté S. A. naturaleza de la ciencia. La epistemología en la
enseñanza de las ciencias naturales. Buenos Aires:
◗◗ Hewitt, P. (1999). Física conceptual. México: Pearson.
Fondo de Cultura Económica.
◗◗ Jones, E. y Childrers, R. (2001). Física Contemporánea.
México: McGraw-Hill.
◗◗ Aragón, M. (2004). La ciencia de lo cotidiano.
Revista Eureka sobre Enseñanza y Divulgación de las
◗◗ Martínez, V., Miralles, M. y Galadí-Enríquez, D. (2005). Ciencias, 1(2).
Astronomía fundamental. Valencia: Universitat de
València.
◗◗ Benlloch, M. y Abreu, G. (2002). La educación en
ciencias: ideas para mejorar su práctica. Barcelona:
◗◗ Máximo, A. y Alvarenga, B. (2001). Física general. Editorial Paidós.
México: Oxford University Press.
◗◗ Gribbin, J. (2005). Historia de la ciencia 1543-2001.
◗◗ Perelman, Y. (1975). Física recreativa I. Moscú: Barcelona: Crítica.
Editorial Mir.
◗◗ Harlen, W. (2007). Enseñanza y aprendizaje de las
◗◗ Perelman, Y. (1976). Física recreativa II. Moscú: ciencias. Madrid: Ministerio de Educación y Ciencia.
Editorial Mir.
◗◗ Pozo, J. y Gómez, C. (2009). Aprender y enseñar ciencia:
◗◗ Ruiz, M. (2007). Hijos de las estrellas. Santiago: Del conocimiento cotidiano al conocimiento científico.
Ediciones B. Madrid: Morata.
◗◗ Sagan, C. (1992). Cosmos. Barcelona: Planeta. ◗◗ Quintanilla, M. (2012). Las competencias de
◗◗ Serway, R. Física para Ciencias e Ingeniería.(Vol. 1). pensamiento científico desde las “voces del aula”.
Santa Fé: Cengage Learning. Santiago: Bellaterra.
104
◗◗ Tippens, P. (2001). Física, conceptos y aplicaciones. ◗◗ Solsona, N. (1997). Mujeres científicas en todos los
México: McGraw-Hill. tiempos. Madrid: Talasa.

◗◗ Wilson, J. y Buffa, A. (2003). Física. México: Pearson


Educación.
◗◗ Zitzewitz, P. & Neff, R. (1997). Física I, principios y
problemas. Santafé de Bogotá: Mc Graw-Hill.
Guía Didáctica del Docente

Física • 2.º medio


Notas

105

Guía Didáctica del Docente

Física • 2.º medio


EDICIÓN ESPECIAL PARA EL
MINISTERIO DE EDUCACIÓN
PROHIBIDA SU COMERCIALIZACIÓN

You might also like